You are on page 1of 414

SỞ GD – ĐT QUẢNG NAM KỲ THI HỌC SINH GIỎI CÁC TRƯỜNG THPT CHUYÊN

TRƯỜNG THPT CHUYÊN KHU VỰC DUYÊN HẢI VÀ ĐỒNG BẰNG BẮC BỘ
NGUYỄN BỈNH KHIÊM LẦN THỨ XIII, NĂM 2022
ĐỀ THI MÔN:TIẾNG ANH
ĐỀ ĐỀ NGHỊ Thời gian: 180 phút (không kể thời gian giao đề)
(Đề thi gồm ....... trang)

--------------------------------------------------------------------------------------------------------
I. LISTENING (50 pts)
Part 1: TRUE/FALSE Statement
1. Based on the results of each indicator, actions can then be taken to improve racer quality.
2. Indicators five through eight are based on data outside the NHS staff survey and concentrate
on people's experiences with bullying, harassment, and abuse at work.
3. The disparity ratios for each band and may propose talent management strategies to aid in the
reduction of the disparity ratio.
4. The following phase may not involve stakeholder groups mapping out the current recruitment
process.
5. During the final phase, all information and work from all organizations involved, including
HR, EDI, and various strategy teams, is compiled.

1. 2. 3. 4. 5.

Part 2: Listen and answer these 5 questions with NO MORE THAN FOUR WORDS:
6. What happened to an isolated area in Siberia in 2016?
_________________________________________________
7. What did the Russian officials soon find out?
_________________________________________________
8. What did these functionaries have to burn so as to control the spread?
_________________________________________________
9. What is the adverse effect of burning carbon on permafrost?
_________________________________________________
10. What are the other effects of climate change aside from droughts in some areas and floods in
others?
________________________________________________

PART 3: You'll hear an interview with a singer songwriter called madeleine martin who's
talking about her life and career for questions 11 to 15 choose the answer A B C or D
which fits best according to what you hear.
11. What does Madeleine say about having to adopt a professional name
A. the disadvantages have outweighed the benefits
B. it met with some resistance from some people
C. it has taken her along time to get used to it
D. the change continues to cause confusion
12. how did having a part in a musical help Madeleine?
A. it led to further offers of work on stage
B. it provided inspiration for her songwriting
C. it enabled her to make some useful contacts
D. it allowed her to re-establish a routine in her life
13. Madeleine thinks that stars who seems to be behaving badly
A. might just be expressing their creativity
B. may be unaware the public does not approve
C. may just be keen to get media exposure
D. might behave equally poorly in another profession
14. Madeleine thinks that she has not become a big star because she
A. is too honest to push her way to the top
B. has been too ready to listen to other people
C. has never had enough say in her own career
D. is quick to blame herself when things go wrong
15. compared to her earlier work, Madeleine thinks that her latest songs
A.reflect her pop music roots more
B. reveal more about her as a person
C. have more carefully written lyrics
D. owe a greater debt to her producer

11. 12. 13. 14. 15.

Part 4: Listen and fill in the gaps with NO MORE THAN 3 WORDS
Early anesthetics were all described to consist of alcohol, 16____________________and
mandrake fruit.
However, nowadays, a combination between regional, 17____________________ agents is
used for a surgery.
As 18___________________, pain and other messages travel through the nervous system as
regional anesthesia prevents them from approaching the brain.
Cocaine, whose effects were accidentally discovered by an 19___________________ ,
sometimes acts as an anesthetic.
In western medicine, 20_______________ was the first common anesthetic , best known as a
recreational drug.
Doctors in the 1840s began sedating patients during either 21________________or surgeries.
Even though other derivatives such as 22________________are more common, nitrous oxide is
still favored till today.
Common intravenous agents are sedatives inducing unconsciousness and
23____________which relieve pain.
As different parts of the brain communicate with one another, the electrical signals in the brain
form a 24_________________.
Negative charge builds up and serves as a 25_____________ ,blocking the neuron from
transmitting electrical signals.
II. LEXICO-GRAMMAR (30 points)
Part 1. For questions 26-45, choose the correct answer A, B, C or D to each of the following
questions. Write your answers in the corresponding numbered boxes provided.(20 pts)
26. It was a close _______ but we just made it to the airport on time for our flight.
A. run B. drive C. call D. go
27.It’s good I got interested in that bottle. Granny_________ the poison taking it for her asthma
medicine because the bottle was unmarked.
A. could swallow B. had swallowed
C. might have swallowed D. was able to swallow
28. The government is doing all it can to _______ war on dangerous driving.
A. launch B. wage C. stage D. battle
29.Not many people are good at assessing their own abilities and Mark must be _______ for
recognising that he would never become a great musician.
A. merited B. commended C. advocated D. talented
30. The country has few natural resources and its economy has been _______ for some time
now.
A. ailing B. diseased C. unwell D. sickening
31. The first runner came in. The second runner followed him closely.
A. The first runner who closely followed by the second came in.
B. The second runner closely followed the first one came in.
C. The second runner who was closely followed came in with the first one.
D. In came the first runner, closely followed by the second.
32. Anyone who lies under oath will be charged with _______ the course of justice.
A. perverting B. inverting C. converting D. diverting
33. My sister likes wearing tight tops and close-fitting dresses but I prefer _______; sweaters
and jeans.
A. floppy B. baggy C. flabby D. shabby
34.There is a very strong movement now against _______ sports like hunting or shooting.
A. death B. killing C. blood D. terminal
35. Would you be ______ my letter while I’m away?
A. too good as to forward B. As good as forward
C. so good as to forward D. So good as to forwarding
36. The new company had been________ with one problem after another and looked as if it
were about to go under.
A. glorified B. tainted C. fraught D. bewildered
37. One of the most powerful pressure groups in the country is the road and car_______,
which opposes any legislation that will increase taxes specifically for them.
A. branch B. folder C. segment D. lobby
38. In its pure state antimony has no important uses, but ________ with other substances, it is
extremely useful metal.
A. combined physically or mentally B. when combined physically or mentally
C. physically or mentally combined D. the physical and chemical combination
39. I was scared ____________ when I looked down from the top of the cliff.
A. tight B. stiff C. hard D. solid
40. I'm sure I was invited to join the committee as the _______ woman.
A. token B. sample C. representative D. coupon
41. Unfortunately, his report doesn't _______ with what we've learnt from other sources.
A. pally B. rally C. tally D. ally
42. The princess's nanny's autobiography really gives the _______ on life among the royals.
A. show-down B. know-how C. low-down D. look-out
43. It's always nice to see Bella- she's such a _______ of sunshine.
A. light B. ray C. glow D. beam
44. I don't think Paul will ever get married - he's the stereotypical _______ bachelor.
A. settled B. confirmed C. fixed D. determined
45. A new generation of performers,______ those who by now had become a household name,
honed their skills before following the same path onto television.
A. no less talented than B. along with talented as
C. together with talented as D. having been more talented than

26. 27. 28. 29. 30. 31. 32. 33. 34. 35.

36. 37. 38. 38. 40. 41. 42. 43. 44. 45.

Part 2. For questions 46-55, write the correct form of each bracketed word in the numbered
space provided in the column on the right. (10 pts)
46.Even the most _________ equipment cannot replace the role of the teachers in the future.
(ART)
47. Can you make up a(n) _________ for baseball tomorrow? (FOUR)
48. Proposals to protect the right of mentally _________ people to be involved in important
decisions about their life have been published by the government. (CAPACITY)
49.Membership talks were launched in 2005, but progress has been slow, as several EU states
have serious ________ about Turkish EU membership. (GIVE)
50. They’re concerned about the _________ of carbon dioxide in the atmosphere. (BUILD)
51. I cannot imagine why there are such _________ people in this world. Don’t they feel any
sympathy for the homeless? (HEART)
52. The dramatic decrease in the __________ of this factory can be a direct result of
underpayment. (PUT)
53. It is a ______ fact that the vegeterian sausages were invented during World War I. (KNOW)
54. I was _________ and fell in love with her immediately at first sight. (LOVE)
55. His recent ups and downs made the criminal more ___________ than ever. (BITE)
46. 47. 48. 49. 50.

51. 52. 53. 54. 55.

47. foursome
III. READING (60 points)
Part 1. For questions 56-65, read the text below and think of the word which best fits each
gap. Use only one word in each gap.(15 pts)

Obsessive-Compulsive Disorder in Adults


This description might give you some sense of the tormented and anxious world that people
with Obsessive-Compulsive Disorder (OCD) live in. It is a world (56) ___________ with
dangers from outside and from within. Often elaborate rituals and thoughts are used to
(57)___________ off feared events, but no amount of mental or physical activity seems
adequate, so doubt and anxiety are often present.
People who do not have OCD may perform (58) ____________ in a ritualistic way, repeating,
checking, or washing things out of habit or concern. Generally, this is done
(59)_____________ much, if any, worry. What distinguishes OCD as a psychiatric disorder is
that the experience of obsessions, and the performance of rituals, reaches such intensity or
frequency that it (60)____________ significant psychological distress and interferes in a
significant way with psycho-social functioning. The guideline of at (61)____________ one hour
spent on symptoms per day is often used as a measure of ‘significant interference’. However,
among patients (62)_____________ try to avoid situations that bring on anxiety and
compulsions, the actual symptoms may not consume an hour. Yet the situations would dearly
constitute interfering with (63)___________. Consider, for instance, a welfare mother who
throws out more (64)____________ $100 of groceries a week because of contamination fears.
Although this behaviour has a major (65)____________ on her functioning, it might not
consume one hour per day.
56. 57. 58. 59. 60.
61. 62. 63. 64. 65.

Part 2. For questions 66 - 75, read the passage and choose the best answer to each of the
questions that follow. Write your answer in the corresponding numbered boxes.(10 pts)
Long before Julia Child and Emeril Lagasse, Antonin Carême (66) __________ international
fame, by cooking for kings and writing cookbooks that brought haute cuisine into 19th-century
homes. From restaurant empires to countless cookbooks to cooking shows, celebrity chefs are
everywhere in the modern world. Many credit television (67) __________ its invention; while
TV may have (68) __________ the visibility of celebrity chefs, it did not invent them: 19th-
century France did. In the decades following the French Revolution, Antonin Carême (69)
__________the world’s first culinary empire—with shops, (70) __________for royalty, and
best-selling cookbooks. He published his first one in 1815, a combination of the encyclopedic
and practical that (71) __________ his organized approach to cooking, which was the first
comprehensive guide to the preparation of many classics of the French repertoire. Like modern-
day professional chefs, he combined the roles of artist, scholar, and scientist, all generously (72)
__________ with self-promotion. Carême is best (73) __________ today, however, for his
brilliant pastries in the form of buildings and exotic landscapes made of spun sugar and almond
paste, called pièces montées or extraordinaires. They served as the grandiose centerpieces that
were still required on the tables of the post-revolutionary French aristocracy. Despite such lavish
productions, Carême acted as a (74) __________ between the elaborate grand cuisine (75)
__________ by royalty and the more modern, simpler approach that he formulated for the
growing middle class in his cookbooks. Until Carême, no one had used the phrase “You can try
this at home.”
66. A. secured B. obtained C. gained D. procured
67. A. with B. for C. to D. on
68. A. increased B. enhanced C. boosted D. heightened
69. A. erected B. shaped C. created D. built
70. A. catering B. restricting C. dedicating D. categorizing
71. A. embodied B. exemplified C. epitomized D. elucidated
72. A. decorated B. associated C. garlanded D. garnished
73. A. honored B. recognized C. remembered D. recalled
74. A. bridge B. link C. joint D. connection
75. A. selected B. recommended C. favored D. beloved
66. 67. 68. 69. 70.
71. 72. 73. 74. 75.

Part 3. Read the following passage and do the tasks that follow. (13 pts)
The Developing World
A.THE DEVELOPING WORLD – the economically underdeveloped countries of Asia. Africa.
Oceania and Latin America – is considered as an entity with common characteristics, such as
poverty, high birth rates, and economic dependence on the advanced countries. Until recently,
the developing world was known as ‘the third world’. The French demographer Alfred Sauvy
coined the expression (in French) in 1952 by analogy with the ‘third estate’ – the commoners of
France before and during the French Revolution – as opposed to priests and nobles, comprising
the First and second estates respectively. ‘Like the third estate’, wrote Sauvy, ‘the third world is
nothing, and it wants to be something’. The term, therefore, implies that the third world is
exploited, much as the third estate was exploited and that, like the third estate, its destiny is a
revolutionary one. It conveys as well a second idea, also discussed by Sauvy – that of
nonalignment, for the developing world belongs neither to the industrialised capitalist world nor
to the industrialised former communist bloc. The expression ‘third world’ was used at the 1955
conference of Afro-Asian countries held in Bandung. Indonesia. In 1956 a group of social
scientists associated with Sauvy’s National Institute of Demographic Studies, in Paris,
published a book called ‘Le Tiers-Monde’. Three years later, the French economist Francois
Perroux launched a new journal, on problems of underdevelopment, with the same title. By the
end of the 1950s, the term was frequently employed in the French media to refer to the
underdeveloped countries of Asia. Africa, Oceania and Latin America. Present-day politicians
and social commentators, however, now use the term ‘developing world’ in a politically correct
effort to dispel the negative connotations of ‘third world’.

B. Countries in the developing world have a number of common traits: distorted and highly
dependent economies devoted to producing primary products for the developed world;
traditional, rural social structures; high population growth and widespread poverty.
Nevertheless, the developing world is sharply differentiated, for it includes countries on various
levels of economic development. And despite the poverty of the countryside and the urban
shantytowns, the ruling elites of most third world countries are wealthy.

C. This combination of conditions in Asia, Africa, Oceania and Latin America is linked to the
absorption of the developing world into the international capitalist economy, by way of
conquest or indirect domination. The main economic consequence of Western domination was
the creation, for the first time in history, of a world market. By setting up sub-economies linked
to the West throughout the developing world, and by introducing other modern institutions,
industrial capitalism disrupted traditional economies and, indeed, societies. This disruption led
to underdevelopment.

D. Because the economies of underdeveloped countries have been geared to the needs of
industrialised countries, they often comprise only a few modem economic activities, such as
mining or the cultivation of plantation crops. Control over these activities has often remained in
the hands of large foreign firms. The prices of developing world products are usually
determined by large buyers in the economically dominant countries of the West, and trade with
the West provides almost all the developing world’s income. Throughout the colonial period,
outright exploitation severely limited the accumulation of capital within the foreign-dominated
countries. Even after decolonisation (in the 1950s, 1960s, and 1970s), the economies of the
developing world grew slowly, or not at all, owing largely to the deterioration of the ‘terms of
trade’ – the relationship between the cost of the goods a nation must import from abroad and its
income from the exports it sends to foreign countries. Terms of trade are said to deteriorate
when the cost of imports rises faster than income from exports. Since buyers in the
industrialised countries determined the prices of most products involved in international trade,
the worsening position of the developing world was scarcely surprising. Only the oil-producing
countries – after 1973 – succeeded in escaping the effects of Western domination of the world
economy.

E. No study of the developing world could hope to assess its future prospects without taking
into account population growth. While the mortality rate from poverty-related diseases
continues to cause international concern, the birth rate continues to rise at unprecedented levels.
This population explosion in the developing world will surely prevent any substantial
improvements in living standards, as well as threaten people in stagnant economies with
worsening poverty and starvation levels.

Questions 76-80
Write the appropriate number i – viii in spaces 76-80 below.
Choose the most suitable heading for each paragraph from the list of headings below.

List of Headings
i. The great divide between rich and poor.
ii. The status and destiny of the developing’ world follow a European precedent.
iii. Economic progress in the developing world slowed down In political unrest.
iv. More people, less food.
v. Western countries refuse to acknowledge their history of colonisation.
vi. Open trade is the main reason these countries become impoverished.
vii. Rivalry in the developing world between capitalist and former communist countries.
viii. Prices and conditions set by outsiders

76. paragraph A_______________


77. Paragraph B_______________
78. Paragraph C_______________
79. Paragraph D_______________
80. Paragraph E_______________

76. 77. 78. 79. 80.

Questions 81-84
Do the following statements agree with the view of the writer in the Reading Passage?
In boxes (81-84) on your answer sheet, write
True - if the statement agrees with the information
False - if the statement contradicts the information
Not Given - if there is no information on this

81. Agriculture still plays a role in the economy of developing countries. ________
82. The population of the developing world increases at such a fast ________
rate because they constantly need to renew the labour force. ________
83. Countries that spend more on imports than came from exports can experience problems.
________
84. As the developing world, oil-rich countries are also victims of dominance by Western
powers. ________
81. 82. 83. 84.

Questions 85-88
Write the correct letter A – F in spaces 85 – 88 below.
A. economic dependence on developed countries.
B. that decolonisation took a long time to achieve.
C. dictate the needs of industrialised countries.
D. share common characteristics.
E. that many economies stagnated.
F. a society that wants something it does not have.

85. Countries in the developing world


86. The term ‘the third world’ implies
87. One factor that is prevalent in the developing world is
88. One consequence of the terms of trade was
85. 86. 87. 88.

Part 4: You are going to read a magazine article. Seven paragraphs have been removed
from the article. Choose from the paragraphs A-H the one which fits each gap (89-95).
There is one extra paragraph you do not need to use. Mark your answers on the separate
answer sheet. (7 pts)

The internet café is not such a new idea - something similar existed back in the seventeenth
century

Where do you go when you want to know the latest news, keep up with celebrity gossip, find
out what others think of a new book, or stay abreast of the latest scientific and technological
developments? Today, the answer is obvious, you log on to the internet, Three centuries ago, the
answer was just as easy: you went to your favourite coffee house.
89.
What's more, rumours, news and gossip were often carried between them by their patrons and
runners would flit from one to another to report major events. Each establishment was,
therefore, an integral part of quite a complex web of contacts. But of even greater importance
was their role as centres of scientific education, literary and philosophical speculation,
commercial innovation and, sometimes, political fermentation.
90.
This reputation accompanied its spread into Europe during the seventeenth century, at first as a
medicine, and then as a social beverage in the eastern tradition. It was reflected in the decor of
the dedicated coffee houses that began to appear in European cities, London in particular, where
they were often adorned with bookshelves, mirrors, gilt-framed pictures and good furniture.
91.
There was a new rationalism abroad in the spheres of both philosophy and commerce, and this
ethos struck exactly the right note, whilst coffee was the ideal accompaniment The popularity of
the beverage owed much to the growing middle class of information workers - clerks, merchants
and businessmen-who did mental work in offices rather than performing physical labour in the
open, and found that it sharpened their mental faculties.
92.
As with modern websites, the coffee houses an individual or group frequented reflected their
interests, for each coffee house attracted a particular clientele, usually by virtue of its location
Though coffee houses were also popular in Paris, Venice and Amsterdam, this characteristic
was particularly notable in London, where eighty-two coffee houses had been set up by 1663,
and more than five hundred by 1700. For many, coffee houses had become almost an extension
of the home.
93.
That said, most people frequented several houses for the purpose of furthering their commercial,
social or political interests. A merchant, for example, would generally oscillate between a
financial house and one specialising in shipping or trade with a particular region. The wide-
ranging interests of Robert Hooke, a scientist and polymath, were reflected in his visits to
around sixty houses during the 1670s. Not to visit one at all was to invite social exclusion.
94.
This is exactly the kind of threat that worries some people today about the power of social-
networking sites Interestingly, a proclamation of 1675 that sought to outlaw the coffee houses of
London was met by a public outcry, for they had become central to commercial as well as
political life When it became clear that the proclamation would be ignored, it was toned down
and then quietly dropped.
95.
But history also provides a cautionary tale for those operators who would charge for access.
Coffee houses used to charge for coffee, but gave away access to reading materials. Many
coffee shops are now following the same model, which could undermine the prospects for fee-
based hotspots. Information, both in the seventeenth century and today, wants to be free - and
coffee- drinking customers, it seems, expect it to be.

Paragraphs
A. Such kinship was soon underlined by the establishment of so-called 'hotspots’. What's more,
from the outset these often provided access in establishments where coffee was also on offer -
this can't have been a coincidence.
B. Lavish entertainment at home was beyond the means of this social stratum but a few pence a
day on coffee could be afforded. What's more, coffee houses provided a forum for education,
debate and self-improvement, and were nicknamed 'penny universities’ in a contemporary
English verse.
C. According to local custom, social differences were left at the door when you entered such a
scholarly space, each of those details contributing to an ambience that fostered sober, respectful
behaviour. Indeed, anyone who started a quarrel had to atone for it by buying a coffee for all
present. In short, these were calm, well-ordered establishments that promoted polite
conversation and discussion.
D. But that was a risk some were willing to take, for coffee houses did have their detractors.
Coffee itself was held by some to be a harmful substance, although this was never taken
particularly seriously. The real opposition came from those who were alarmed at the houses'
potential for facilitating political discussion and activity.
E. The quality of the coffee wasn't the only factor governing which one this would be, however,
for these lively and often unreliable sources of Information typically specialised in a particular
topic or political viewpoint. They also doubled as outlets for a stream of newsletters and
pamphlets that reflected the interests of their particular clientele.
F. In the days before formal addresses or regular postal services were introduced, for example, it
became a common practice to use one as a mailing address. Regulars could pop in once or twice
a day, hear the latest news, and check to see if any post awaited them.
G. Coffee, the drink that fuelled this vibrant network, originated in the highlands of Ethiopia,
where its beans were originally chewed rather than infused for their invigorating effects. Coffee
spread into the Islamic world during the fifteenth century, where it came to be regarded as
stimulating mental activity and heightening perception.
H. The parallels are certainly striking. Originally the province of scientists, the Net also soon
grew to become a nexus of commercial, journalistic and political interchange. In discussion
groups, gossip passes freely - a little too freely, according to some regulators and governments,
which have generally failed in their attempts to rein them in.
Part 5: You are going to read an article in which four readers suggest locations for watching
wildlife. For questions 96 -105, choose from the readers (A – D). The readers may be chosen
more than once and write your answers in the column on the right. (15 pts)
Which reader…
offered money in return for the chance to interact directly with some 96. ____
animals?
feels that visiting the location has been a life-changing experience? 97. ____
says the location may well become more renowned in the future? 98. ____
has a suggestion for the novice wildlife tourist? 99. ____
mentions a physical reaction to the excitement of spotting certain 100. ____
animals?
feels it unwise to bank on seeing one particular species? 101. ____
mentions an abundance of animals belonging to one particular species? 102. ____
mentions unpaid work being offered as part of a trip? 103. ____
suffered some discomfort in order to witness one wildlife event? 104. ____
mentions one particularly enjoyable form of transport? 105. ____
WILDLIFE ENCOUNTERS
Four readers suggest great locations where you can watch wildlife in its natural surroundings.
A. KEVIN: Hallo Bay, Alaska
The first time you see a bear, when you realise that it’s just you, the guide and that bear, your
mouth definitely goes dry. Unlike in other more frequently visited area, the bears at Hallo Bay
don’t associate human with food as nobody’s ever fed them, so they pose no risk to people. You
can watch the bears fish in the river, nurse their cubs, photograph them hunting for clams on the
beach or find them sleeping with their full bellies nestled in a hollow they’ve dug in the sand.
For me, Hallo Bay’s a magical place. I’ve always been a person who was structured and
organised, but I’ve said for years now that I lost my list in Alaska. One thing which makes Hallo
Bay so special is that the remote camp has just a dozen guests at a time, with guided groups of
no more than half that many heading out to search for the bears. And there’s no shortage of
them; Hallo Bay has one of the world’s healthiest populations of coastal browns, maybe because
of the plentiful food supply. It must be how the planet was several hundred years ago.
Admittedly, Hallo Bay would be a bit challenging if you’d never been wildlife watching before.
But for me, even without the bears it would be a gorgeous place to visit.
B. SARAH: Madikwe Game Reserve, South Africa
It’s so hard to recommend just one location in Africa to go in search of the big five! However, if
you’ve never been on safari before, then travel is straightforward in South Africa and its parks
are the cheapest if you’re short of money. Also, if you want to take children with you there are
parks, such as Madikwe Game Reserve, that are malaria-free. This doesn’t mean you can’t have
an adventure. The park have well-equipped campsites and good-quality roads, so it’s perfectly
possible to fly in, hire a four-by-four, fully equipped with everything you need for a forth-night
camping, and head off on your own. There’s also an impressive selection of volunteer projects
involving animals, particularly around the country’s biggest parks. I spent four weeks helping at
a veterinary practice with African Conservation Experience. I got the chance to work with lion,
cheetah, sable antelope, elephant and buffalo. The work’s extremely hands-on and you have to
be ready for anything, whether it’s taking a lion’s temperature or treating a dog for a snake bite!
C. RAY: Playa Grande Sanctuary, Costa Rica
With concerns mounting about the pressure on the Galápagos Islands, Costa Rica’s popularity
as a wildlife venue could be about to take off, and deservedly so. It boasts the world’s highest
biodiveristy according to some guidebooks and packs in 850 species of birds and a quarter of
the world’s species of butterfly. From cloud forest to Carribbean beaches and from dry tropical
forest to mangrove swamps, Costa Rica has its all: iguanas at your feet, capuchin monkey
overhead, sloths are to be seen, and if you’re really lucky you’ll catch sight of one of Costa
Rica’s jaguars. However, perhaps the most magical thing to do here is watch turtles lay their
eggs on a moonlight drenched beach. It does require patience; we waited two nights, napping on
hard benches at the Playa Grande sanctuary, before one of the wardens shook us awake to say a
female had been spotted laboriously making her way out of the surf. The turtle go into a sort of
trance and we were allowed quite close to watch her dig a hole with her flippers and deposit
hundreds of eggs, the size of gold balls. She then casually covered them up and headed off back
down the beach – the last she’ll see of her young. The eggs are then gathered by the wardens
and taken to their hatchery to protect them from predators.
D. AMY: Chitwan National Park, Nepal
With tigers, snow leopards are one-horned rhinoceros, Nepal certainly has its share of
endangered animals. The snow leopard is perhaps the most exotic of them all but, with only a
maximum of five hundred of these cats left in the country, they’re incredibly difficult to spot.
Snow leopard treks are organised regularly, but if you go on one you need to enjoy it for the
sheer magnificience of the scenery and not feel let down if you don’t spot your ultimate prey. It
could be a life-changing experience, but it’s not that likely to happen. I visited Chitwan at the
foot of the Himalayas. The park was set aside for wildlife in 1959 and is the place to see Indian
rhinoceros as well as being one of the last refuges of the Benga tigers. One of the best ways to
view both is from the back of an elephant – something that is rather fabulous in itself. We were
having breakfast one day when two elephants were being taken for their daily wash on the river
bank near our hotel. We made a small donation and asked to help – it was one of the most
amazing animal encounters possible, sitting on the backs of those huge elephants scrubbing their
backs whilst they knelt in the water and sprayed us from their trunk! All the more special as it
was so impromptu.
IV. WRITING (60 POINTS)
Part 1. Read the following extract and use your own words to summarize it. Your summary
should be between 120 and 140 words long. (15 pts)
Humans have struggled against weeds since the beginnings of agriculture. Marring our gardens
is one of the milder effects of weeds – any plants that thrive where they are unwanted. They
clog waterways, destroy wildlife habitats, and impede farming. Their spread eliminates grazing
areas and accounts for one-third of all crop loss. They compete for sunlight, nutrients, and water
with useful plants.
The global need for weed control had been answered mainly by the chemical industry. Its
herbicides are effective and sometimes necessary, but some pose serious problems, particularly
if misused. Toxic compounds threaten animal and public health when they accumulate in food
plants, groundwater, and drinking water. They also harm workers who apply them.
In recent years, the chemical industry has introduced several herbicides that are more
ecologically sound. Yet new chemicals alone cannot solve the world‟s weed problems. Hence,
scientists are exploring the innate weed-killing powers of living organisms, primarily insects
and microorganisms. The biological agents now in use are environmentally benign and are
harmless to humans. They can be chosen for their ability to attack selected targets and leave
crops and other plants untouched. In contrast, some of the most effective chemicals kill virtually
all the plants they come in contact with, sparing only those that are naturally resistant or have
been genetically modified for resistance. Furthermore, a number of biological agents can be
administered only once, after which no added applications are needed. Chemicals typically
must be used several times per growing season.

………………………………………………………………………………………………………….
………………………………………………………………………………………………………….
………………………………………………………………………………………………………….
………………………………………………………………………………………………………….
………………………………………………………………………………………………………….
………………………………………………………………………………………………………….
………………………………………………………………………………………………………….
………………………………………………………………………………………………………….
………………………………………………………………………………………………………….
………………………………………………………………………………………………………….
………………………………………………………………………………………………………….
………………………………………………………………………………………………………….
………………………………………………………………………………………………………….
………………………………………………………………………………………………………….
………………………………………………………………………………………………………….
………………………………………………………………………………………………………….
………………………………………………………………………………………………………….
………………………………………………………………………………………………………….
………………………………………………………………………………………………………….
………………………………………………………………………………………………………….
………………………………………………………………………………………………………….
PART II. Graph Writing (15 pts)
The chart below shows the Average Monthly Temperatures for three African cities.
Summarize the information by selecting and reporting the main features, and make comparisons
where relevant.
You should write about 150 words.

………………………………………………………………………………………………………….
………………………………………………………………………………………………………….
………………………………………………………………………………………………………….
………………………………………………………………………………………………………….
………………………………………………………………………………………………………….
………………………………………………………………………………………………………….
………………………………………………………………………………………………………….
………………………………………………………………………………………………………….
………………………………………………………………………………………………………….
………………………………………………………………………………………………………….
………………………………………………………………………………………………………….
………………………………………………………………………………………………………….
………………………………………………………………………………………………………….
………………………………………………………………………………………………………….
………………………………………………………………………………………………………….
………………………………………………………………………………………………………….
………………………………………………………………………………………………………….
………………………………………………………………………………………………………….
………………………………………………………………………………………………………….
………………………………………………………………………………………………………….
………………………………………………………………………………………………………….

III. Essay writing (30 pts)


Write an essay of about 350 words on the following topics
Some people prefer to spend their lives doing the same things and avoiding change. Others,
however, think that change is always a good thing.

Discuss both these views and give your own opinion.


………………………………………………………………………………………………………….
………………………………………………………………………………………………………….
………………………………………………………………………………………………………….
………………………………………………………………………………………………………….
………………………………………………………………………………………………………….
………………………………………………………………………………………………………….
………………………………………………………………………………………………………….
………………………………………………………………………………………………………….
………………………………………………………………………………………………………….
………………………………………………………………………………………………………….
………………………………………………………………………………………………………….
………………………………………………………………………………………………………….
………………………………………………………………………………………………………….
………………………………………………………………………………………………………….
………………………………………………………………………………………………………….
………………………………………………………………………………………………………….
………………………………………………………………………………………………………….
………………………………………………………………………………………………………….
………………………………………………………………………………………………………….
………………………………………………………………………………………………….………
…………………………………………………………………………………………………………
………………………………………………………………………………………………………….
………………………………………………………………………………………………………….
………………………………………………………………………………………………………….
………………………………………………………………………………………………………….
………………………………………………………………………………………………………….
………………………………………………………………………………………………………….
………………………………………………………………………………………………………….
………………………………………………………………………………………………………….
………………………………………………………………………………………………………….
………………………………………………………………………………………………………….
………………………………………………………………………………………………………….
………………………………………………………………………………………………………….
………………………………………………………………………………………………………….
………………………………………………………………………………………………………….
………………………………………………………………………………………………………….
………………………………………………………………………………………………………….
………………………………………………………………………………………………………….
………………………………………………………………………………………………………….
………………………………………………………………………………………………………….
………………………………………………………………………………………………………….
………………………………………………………………………………………………………….
………………………………………………………………………………………………………….
………………………………………………………………………………………………………….
………………………………………………………………………………………………………….
………………………………………………………………………………………………………….
………………………………………………………………………………………………………….
………………………………………………………………………………………………………….
………………………………………………………………………………………………………….
………………………………………………………………………………………………………….
………………………………………………………………………………………………………….
………………………………………………………………………………………………………….
………………………………………………………………………………………………………….
………………………………………………………………………………………………………….
………………………………………………………………………………………………………….
………………………………………………………………………………………………………….
………………………………………………………………………………………………………….
………………………………………………………………………………………………………….
………………………………………………………………………………………………………….
………………………………………………………………………………………………………….
………………………………………………………………………………………………………….
………………………………………………………………………………………………………….
………………………………………………………………………………………………………….
………………………………………………………………………………………………………….
………………………………………………………………………………………………………….
………………………………………………………………………………………………………….
………………………………………………………………………………………………………….
………………………………………………………………………………………………………….
………………………………………………………………………………………………………….
………………………………………………………………………………………………………….
………………………………………………………………………………………………………….
………………………………………………………………………………………………………….

______________******_________________

Người ra đề:

Nguyễn Hoàng Cường : Số điện thoại: 0932128400

Nguyễn Phước Hào : Số điện thoại: 0905540418


HỘI CÁC TRƯỜNG CHUYÊN
VÙNG DUYÊN HẢI & ĐỒNG BẰNG BẮC BỘ ĐỀ THI MÔN: TIẾNG ANH
TRƯỜNG THPT CHUYÊN LÊ QUÝ ĐÔN KHỐI 11 - LẦN THỨ XIII
ĐIỆN BIÊN NĂM 2022

ĐỀ THI ĐỀ XUẤT Thời gian làm bài: 180 phút

(Đề thi gồm 24 trang)

A. LISTENING (50 points)


HƯỚNG DẪN PHẦN THI NGHE HIỂU
 Bài nghe gồm 4 phần, mỗi phần được nghe 2 lần, mỗi lần cách nhau 15 giây, mở
đầu và kết thúc mỗi phần nghe có tín hiệu.
 Mở đầu và kết thúc bài nghe có tín hiệu nhạc. Thí sinh có 3 phút để hoàn chỉnh
bài trước tín hiệu nhạc kết thúc bài nghe.
 Mọi hướng dẫn cho thí sinh (bằng tiếng Anh) đã có trong bài nghe.
Part 1. For questions 1-5, listen to a report about how European countries are
dealing with the coronavirus pandemic and decide whether these statements are
True or False. (10 points)
Statements T/ F
1. Under the full lockdown in Italy, all grocery stores must be shut down.
2. There is a ban on entry for people from Austria into Italy.
3. Angela Merkel warned that Coronavirus could infect up to 17% of
Germany's population.
4. Public gatherings still take place in the UK.
5. According to the professor, although the UK is taking more drastic
measures than Italy, its effectiveness is open to question.

Part 2. You’ll hear an interview with physicist teacher, Kieran Shaw, who has taken
his students to a Science Fair. For questions 6-10, write answers in the space
provided. Write NO MORE THAN THREE WORDS for each question or sentence.
(10 points)
6. What does Kieran criticize about the previous Science Fair?
Page 1 of 24
___________________________________________
7. Kieran says the fall in the nulmber of participants at some science fairs might be
caused by a belief that science fairs are _____________________________.
8. Which does Kieran believe is a problem among his students?
___________________________________________
9. Kieran says the most important factor in choosing a topic is whether it is likely to
_________________________________________ throughout the project.
10. According to Kieran, what mistake do some students make during their
presentations?
__________________________________________
Part 3. You will hear part of a discussion between two well-known business people,
Sam Boland and Jimmy Glynn, and a radio current affairs show host about a recent
newspaper article on the subject of jobless graduates. For questions 11-15, choose
the correct answer A, B, C, or D which fits best according to what you hear. (10
points)
11. What does Jimmy say about the qualifications graduate jobseekers have?
A. they help them avoid having to take “survival jobs”
B. few graduates are sufficiently qualified for the jobs available
C. few degrees teach skills relevant in the workplace
D. they are not really an advantage in the present job market
12. Why is Sam wary of employing graduates in his own company?
A. their superior performance can lead to workplace hostility
B. they can have a damaging effect on the chemistry of their team
C. he prefers to hire unskilled workers and invest in training
D. they only accept positions above lesser-skilled employees
13. In what respect have things changed since the speakers were job-seeking
graduates?
A. jobseekers used not to lie about their qualifications
B. jobseekers used to pretend they had better qualifications
C. jobseekers used to be far fresher and more enthusiastic
D. jobseekers used not to mind dumbing down their CVs

Page 2 of 24
14. Why does Jimmy have some sympathy for graduates?
A. he admires their sense of entitlement
B. they were too young to understand what they voted for
C. they have accumulated huge debts at college
D. they are victims of other people’s incompetence
15. Jimmy believes that college-goers of the future should
A. not rule out the Arts and Humanities.
B. work for foreign in preference to local firms.
C. choose their course of study carefully
D. be selective about where they study
Part 4. For questions 16-25, listen to a talk about CRISPR technology and supply
the blanks with the missing information. Write NO MORE THAN THREE WORDS
or NUMBERS taken from the recording for each answer in the space provided.
Back in 2006, bacteria used in the (16) _____________ was found to possess
CRISPR, a tool that acts as a pair of (17) _____________ and thus defends them
against deadly viruses. Application of CRISPR may improve our lives in three ways: -
CRISPR application in Agriculture has resulted in crop yields that are more nutritious
and possess higher (18) ______________, as can be seen in China’s new breed of rice
that produces (19) _______________ more grains. - CRISPR-based treatments can
cure genetic disorders like (20) ______________ and cystic fibrosis. By fixing (21)
________________, CRISPR helps to alleviate the burden of genetic diseases. -
CRISPR can also be used to eradicate infections. Using the technology called (22)
______________, scientists have engineered (23) ______________ that are unaffected
by malaria-carrying parasites and can pass this resistance on to their offspring.
Numerous questions have been raised about the ethical issues of CRISPR, as to who
can use it as a (24) ______________, and how to ensure equitable access. The birth of
CRISPR babies has necessitated a comprehensive (25) _______________ on
modifying human embryos until its long-term effects are better understood.
B. LEXICOLOGY AND GRAMMAR (30 points)
Part 1: Choose the best answer to each of the following questions. Write your
answers in the corresponding numbered boxes (20 pts)

Page 3 of 24
1. The choreographer ________ his fingers in time to the music so that the dancers
could pick up the tempo.
A. clenched B. snapped C. nudged D. beckoned
2. The shop wasn’t making much of a profit so in order to attract more customers, the
owner ________ some of the prices.
A. shelled out B. sold out C. marked down D. stocked up
3. Dr. Walker has spent her entire career studying the impact of poaching on
_________ animals like the tiger.
A. ground B. terrestrial C. worldly D. landmass
4. Closure of schools took place _________ falling numbers of pupils.
A. in the context of B. with regard to
C. with a concern for D. in consideration of
5. I decided to lend my brother some money to _________ until he could find himself
a job.
A. pull him over B. tide him over C. get him by D. put him up
6. The football club decided to _______ the team with a couple of world-class players.
A. beef up B. butter up C. dredge up D. drum up
7. Unions accept free accommodation and travel, yet they would scream _______
murder if the same was received by politicians.
A. red B. blue C. yellow D. black
8. Professional sports have become so competitive that all athletes know, no matter
how good they are, that one day they will ________.
A. paddle their own canoe C. take the wind out of their sails
B. blow their own trumpet D. meet their match
9. That he was using unscrupulous research methods only came out because his
assistant ______ on him to the press.
A. ratted B. hounded C. fished D. bugged
10. I’m hoping that this work experience will stand me in _______in my future career.
A. good grounding B. good stead C. fine precedent D. stable footing
11. Most people feel a slight _______of nostalgia as they think back on their school
days.

Page 4 of 24
A. feeling B. surge C. pang D. chain
12. The cost of a new house in the UK has become _______high over the last few
years.
A. totally B. astronomically C. blatantly D. utterly
13. The entire staff was thrown off _______when the news of the takeover was
announced.
A. composure B. disarray C. stable D. balance
14. The matter has been left in _______until the legal ramifications have been
explored.
A. recess B. suspension C. abeyance D. criticism
15. I didn’t want to make a decision ____, so I said I’d like to think about it.
A. in one go B. there and then C. at a stroke D. on and off
16. Now that I’ve lost my job, I think we are lucky we don’t have children yet. We
______ to raise them on Joe’s salary alone.
A. won’t be able B. aren’t able C. wouldn’t be able D. weren’t able
17. - Lam: "It's a pity she had to pull out of the competition." - Hung: "Yes, especially
since she ______ such excellent progress."
A. is making B. made C. had been making D. has been making
18. Her hair ______ in a bun, the bride looks superficially attractive.
A. styling B. styled C. was styled D. has been styled
19. This project is _______ for an introductory-level class.
A. too much really hard B. much too really hard
C. really much too hard D. really hard too much
20. My sister says she spent the last month looking for a job, but she didn’t get any
replies to the CVs she sent. She ______________.
A. couldn’t try hard B. can’t try hard enough
C. couldn’t have been trying hard D. could have had tried hard
Your answers:
1. 2. 3. 4. 5. 6. 7. 8. 9. 10.
11. 12. 13. 14. 15. 16. 17. 18. 19. 20.

Page 5 of 24
Part 2: Give the correct form of the words in brackets in each of the following
sentences. Write your answers in the corresponding numbered boxes (10 points)
Example:
0. She couldn’t go on holiday with Jonathan; they had too much emotional ______
(BAG).
Answer: 0. baggage
1. Known for power, speed, and ______, the goshawk has short wings and a long tail,
enabling them to dodge branches in pursuit of prey. (MANEUVER)
2. This room was much smaller, though the ceiling was quite as high; it gave the
______ sense of being stuck at the bottom of a deep well. (PHOBIA)
3. She ______ the things we need to buy – sugar, tea, and sandwiches. (NUMBER)
4. This book is ______. You can find information about the varieties of plants around
the world here. (EMBRACE)
5. Her bare feet arch elegantly, and the ankle bracelets she wears ______ her graceful
movements. (ACCENT)
6. The remnants of an ancient penguin that stood as tall as a grown man have been
found ______ in rock on a beach in New Zealand. (CASE)
7. He was a modest and ______ man who never gave the impression that he knew all
the answers. (ASSUME)
8. Many in the working class tend to be ______ towards authority figures and those in
superior class positions. (DEFER)
9. With the great achievement of U23 Vietnam team, many international newspapers
wrote ______ praising our heroes. (COMIC)
10. Having made that morally terrible mistake, the kidnapper is now _______.
(CONSCIENCE)
Your answers:
1. 2. 3. 4. 5.
6. 7. 8. 9. 10.

C. READING (60pts)
Part 1: For questions 1–10, read the following passages and decide which answer
(A, B, C or D) best fits each gap. Write your answers (A, B, C, or D) in
corresponding numbered boxes. (15pts)
Page 6 of 24
In the north-west corner of the island paradise Isla Perlita, nestling in the
shadow of Mount Machu, lies the sleepy village of San Lorenzo. Off the beaten
(1)______, there is nothing out of the ordinary about this quaint little village –
nothing, that is apart from the magnificent five - (2)______ Bay Hotel. The Bay, as it
is known locally, is a recent development catering for (3)______ travellers who enjoy
luxury holiday-making. Famous throughout the island for the outstanding quality of its
accommodation and the excellence of its cuisine, the Bay (4)______ 30 guest suites,
each with a charm and character of its own. Each suite looks (5)______ Falmer
Beach, commanding breathtaking views of the four miles of white sand, which gently
shelves into the (6)______ clear waters of the Crepuscan Sea. At the heart of the Bay
Hotel is personal, efficient and unobtrusive service. (7)______ staff anticipate your
every need in an atmosphere of quiet professionalism and genuine friendliness.
(8)______, the Bay Hotel is a place to get (9)______ from the stresses of everyday
life, and whether it is (10)______ away the hours soaking up the sun or taking
advantage of the wide range of recreational activities that the hotel has on offer, you
can be sure that a holiday at the Bay truly is the holiday of a lifetime.
1. A. path B. track C. road D. way
2. A. starred B. stars C. star D. starring
3. A. disconcerting B. discerning C. Distinctive D. discriminated.
4. A. announces B. claims C. asserts D. boast
5. A. back on B. onto C. Into D. down
6. A. crystal B. sky C. diamond D. pearl
7. A. Preoccupied B. Attentive C. Concentrated D. Undivided
8. A. All over B. For all C. Above all D. All along
9. A. out B. over C. Off D. away
10. A. whiling B. wearing C. wending D. winding
Your answers:
1. 2. 3. 4. 5.
6. 7. 8. 9. 10.

Page 7 of 24
Part 2: Read the following passage and choose the best answer (A, B, C or D)
according to the text. Write your answers (A, B, C or D) in the corresponding
numbered boxes (10pts).
Super memorisers
There are people in this world who are innately possessed of an ability to remember
things with quite an extraordinary degree of detail and exactness. These super
memorisers, as they are known, typically possess a brain naturally and distinctively
wired to maximize its memorizing potential. They are gifted such that they require no
particular training or effort to sharpen their memories; they remember things just as
effortlessly as most people forget them. Few of us are born with such gifts, sadly, but
there is much cause for optimism yet for those looking to improve their brain’s
performance tangibly in this area; a fact which Boris Konrad is testament to.
Konrad is a champion memoriser who, in winning gold in the German Memory
Championships one year recognized and recalled the names of 195 people in just
fifteen minutes. [■] His powers of recollection, then, are as admirable as they are
undisputed, but Konrad is self-trained and started out with a recall capacity that was
unremarkable. [■] Instead, he spent years developing memory strategies and
employing those strategies to improve in the area through practice and dedication. [■]
He, then, is a living proof that the average Joe with a gift for forgetfulness can
reinvent himself in the area. [■]However, his example is as much a reminder of the
extent of devotion that is required to reach this level as it is of the possibilities if one is
prepared to put in the effort, and there are not many people prepared to expend a
similar level of effort to this end, which is what really makes Konrad unique.
That said, whilst you may not become a super memoriser overnight, new research
suggests that it is possible to tangibly improve your memory in a relatively short space
of time by devoting roughly half an hour of your every day to the process. It is
necessary to learn and employ memorizing strategies such as the Memory Palace
technique Konrad uses, though, to yield such results; otherwise, you might be as well
be doing something else. In one recent study, for example, participants spent one
month training their memories in the aforementioned technique for 30 minutes every
day, which more than doubled their ability to remember list of information after just

Page 8 of 24
40 days. More impressively, recall performance remained high whether or not training
continued at the end of one month, which suggests after rewiring of the brain can be
permanent.
Even innately gifted memorisers use such mnemonic techniques to enhance their
recall ability, and, of the recall methods which exist, the methods of loci, which has
already been referred to here as the Memory Palace method, is the most prevalent one
adopted as revealed by a recent study of 35 memory champions. Indeed, at a more
rudimentary level, this method has been employed by orators and others required to
remember long strings of interconnected information for some time, and it actually
dates back to Ancient Greece, where it was first conceived of, remaining prevalent
right through to the Middle Ages and the Renaissance. It is only a new method then,
today, in the sense that the skill had been lost as such to most people for a very long
time. Indeed, many of today’s super memorisers employed it intuitively rather than in
a conscious effort to enhance their recall powers.
Participants in the study was separated into three distinct groups, with one group
receiving no memory training whatsoever and making no notable recall gains during
the process. The second group dedicated time and effort to upping their recall
capacity, but employed an everyday technique. However, those using the Memory
Palate technique more than doubled their initial recall capacity by the process’s end.
What’s more, their brain functions actually changed and their brain patterns began to
bear more of a likeness to those of innate super memorisers, leading researchers to
speculate that a total rewire might well be possible over time, such that a normal
individual’s recall capacity could match that of any memory champion. The
conclusion, therefore, was that memory is not necessarily an innately bestowed gift
you either are possessed of or not. Most of us have the potential to hone and expand
our memories very meaningfully indeed.
1. According to the first paragraph, people with marvelous power of retention usually
A. undergo rigorous practice to become that way.
B. demonstrate brain functions resembling anyone else.
C. throw in little effort to commit things to memory.
D. be tangibly more intelligent than other people.

Page 9 of 24
2. Why are Konrad’s achievements mentioned in paragraph two?
A. To underline the excellence of inborn super memorisers
B. To demonstrate the disparity between laymen and super memorisers
C. To prove that almost every individual can make a significant enhancement in
their retention powers
D. To show how even innate super memorisers have to exert themselves
3. What does the writer mean in the phrase ‘you might as well be doing something
else’ in paragraph three?
A. It is futile to invest time in improving recall capacity.
B. People should concentrate on their strengths.
C. The Memory Palace technique is not universally suitable.
D. Progress is only visible if you train in the right way.
4. What did the study mentioned in the third paragraph conclude?
A. The implication of moderate brain exercises can be enduring.
B. The maintenance of a decent recall capacity requires continuous training.
C. Your recall ability can more than double within one month.
D. Progress is only noticeable with at least 30 minutes spent on brain training.
5. What do we learn about the method of loci?
A. It is superior to the Memory Palace method.
B. Most skillful memorisers purposefully adopt it.
C. It has evolved from methods first employed in Ancient Greece.
D. It has been widely used among super memorisers.
6. What did the results of the study mentioned in the final paragraph reveal?
A. How the brains of super memorisers function is inimitable.
B. The application of appropriate technique exerts a tremendous influence on the
brain.
C. Memory training is futile unless employing a confirmed technique.
D. Most super memorisers are not talented at memorizing at birth.
7. The word "mnemonic techniques” in paragraph 4 mostly means
A. gimmicks used to improve brain functions.
B. endowed talents of super memorisers.

Page 10 of 24
C. tips specifically designed to aid the process of retaining information.
D. skills adopted by super memorisers to avoid absent-mindedness.
8. Look at the four squares [■] that indicate where the following sentence could be
added to the passage.
Indeed, after just 30 seconds, examination, he is able to retain the order of an
entire deck of cards.
Where would the sentence best fit?
A. First square
B. Second square
C. Third square
D. Fourth square
9. The word "orators” in paragraph 4 mostly means
A. language experts
B. body language experts
C. oral students
D. proficient speakers
10. The paragraph following the passage would most probably discuss
A. further research into the workings of brains of super memorisers.
B. the defeat of a normal person over a super memoriser in terms of recalling
capacity.
C. potential ways to expand our power of retention.
D. the popularity of memory palace technique.
Your answers
1 2. 3. 4. 5.
6 7. 8. 9. 10.

Part 3. For questions 1-13, read the following passage and do the tasks that follow.
(13pts)
Bismarck: A master of political and diplomatic juggling?
A. Otto Von Bismarck’s rise up the political ladder was swift and relentless. Having
entered parliament in 1847, he always harboured lofty ambitions, chief among them

Page 11 of 24
perhaps being the reunification of Germany into one strong, centrally controlled state,
though his own personal thirst for power was arguably even stronger. On becoming
Prussian Chancellor, he set about fulfilling his ambitions and in doing so proved
himself to be a diplomat of some considerable skill. Victory in the Austro-Prussian
war effectively ended Austria as a factor in German affairs. His political and military
juggling was taken a step further when he orchestrated a situation where France
declared war on Germany in 1870, making the French seem responsible for a conflict
he had always intended to create. And following another swift military triumph, this
time over the French, the German empire was proclaimed in January 1871.
B. In little more than nine years, Bismarck realised his lifelong ambition, steering
Germany to reunification. And by defeating Austria and France in quick succession,
he also created a power vacuum on mainland Europe, which he was determined to
fulfill himself. This was another opportunity for Bismarck to demonstrate his political
and diplomatic cunning. He set about creating a dictatorial Germany in which he, as
head of the Prussian parliament, would automatically become chancellor of the
German empire. He drafted a new German constitution to suit his own purposes and,
despite maintaining a veneer of democracy, the German parliament was effectively
powerless to oppose him. Provinces that were slow to support him were enticed with
bribes and before long the German empire was his to command.
C. It is a testament to his political skill that Bismarck achieved so much so quickly. At
this point in his colourful political career, he did appear, for all intents and purposes, a
master of political and diplomatic juggling. But challenges lay ahead and Bismarck’s
next target was the Catholic church, which he deemed too powerful and a threat to his
political dominance. He proceeded to enact a series of laws that seriously eroded the
power of the church. However, his plans backfired and Bismarck was forced to make a
political U-turn. Though here again, he somehow managed to save face. The damage
to his reputation was limited and indeed by the late 1870s, he had even managed to
win over the church whose support he now needed.
D. Bismarck viewed the growing popularity of the Socialist Democratic Party as a
serious threat. He bided his time and used the attempted assassination of the Kaiser as
an excuse to attack the socialists in 1878, blaming them for the attempt on the Kaiser’s

Page 12 of 24
life. He immediately arrested the leaders, banned party meetings, and suppressed
socialist newspapers. But despite his efforts to destroy the socialist movement, its
popularity had trebled by 1890. Just as his interventions with the church had not gone
as planned, Bismarck once again failed to achieve his objective; though, to his credit,
he held on to power.
E. His domestic position was relatively secure after 1871, Bismarck devoted a lot of
his time to foreign policy. Having used war to unite Germany and make her great,
Bismarck now believed that his ambitions were best served by peace. His plan to
isolate a hostile France would require all his considerable diplomatic skills. The
Dreikaiserbund agreement of 1873 between Germany, Austria-Hungary, and Russia
was a first step towards doing just that. The Balkan crisis, a conflict involving Russia
and Austria-Hungary, severely tested his diplomatic credentials, but his answer was to
offer himself as an ‘honest broker’ to help resolve the dispute. The subsequent
Congress of Berlin which he hosted was an outstanding success and only served to
reinforce Bismarck’s reputation as a shrewd diplomat. Bismarck’s foreign policy
would continue in this vein throughout his reign as Chancellor. He built up strategic
alliances with the big powers, Russia, Italy, and Austria-Hungary, in the hope that he
could keep his main threats, France and Britain, isolated.
F. In truth, Bismarck’s reign as chancellor of the German empire does seem to
confirm him as a shrewd and wily diplomat and politician, one whose objectives were
broadly achieved. Does this mean his so-called juggling was a success? Perhaps, but
Bismarck left a less than perfect legacy. He created a Germany in which the Kaiser
had the ultimate say in domestic affairs and enjoyed far too much power should he
choose to wield it. This meant that the future of the empire largely depended on the
strength and character of just one man, the Kaiser. A weak Kaiser would be disastrous
for the country’s welfare, and so it would soon prove. In the final analysts, Bismarck
put Germany back on the map again as a great power during his reign, but we should
not forget that he created the political situation that would be the downfall of his
country in the end. His political and diplomatic juggling, therefore, simply cannot be
considered a total success.
Questions 1-6

Page 13 of 24
Reading passage has six paragraphs A-F.
Choose the correct heading, i - ix, from the list of headings below.
Write the correct number i-ix in the boxes 1-6.
i The pace at which Bismarck advanced his ambitions
ii Separating church and state
iii A change of mind
iv Unexpected ramifications of socialist threat
v Unremitting efforts to create a virtual dictatorship
vi External objectives and activities
vii Abortive attempts to curb a movement
viii Political power on the wane
ix A critical analysis
1. Paragraph A
2. Paragraph B
3. Paragraph C
4. Paragraph D
5. Paragraph E
6. Paragraph F
Your answers
1. 2. 3. 4. 5. 6.

Questions 7-13
Do the following statements agree with the information given in the Reading
Passage?
In boxes 7-13, write
TRUE if the statement agrees with the information
FALSE if the statement contradicts the information
NOT GIVEN if there is no information on this
7. Germany defeated France and Austria successively to claim the dominant position
in Europe.
8. The Congress of Berlin was a great personal success for Bismarck.
Page 14 of 24
9. After Bismarck enacted laws to weaken the Catholic Church, he was forced to
change his policy and made a church an ally.
10. The Socialist Democratic Party took power from Bismarck in 1890.
11. Bismarck’s foreign policy was to wage war with all countries that posed a military
threat.
12. Bismarck considered his reign as German chancellor a failure.
13. It can be concluded that Bismarck was not actually a master of political juggling.
Your answers
7. 8. 9. 10. 11. 12. 13.

Part 4. In the passage below, seven paragraphs have been removed. For questions 1-
7, read the passage and choose from the paragraphs A-H the one which fits each
gap. There is ONE extra paragraph which you do not need to use. Write your
answers in the corresponding numbered box provided. (7pts)
Blind to Change
How much of the world around you do you really see? You only take in tiny pieces of
information at a time and that can have unnerving consequences, says Laura Spinney.
Imagine you’re walking across a college campus when an unknown man asks you for
directions. While you’re talking, two men pass between you carrying a door. After an
irritating minute of interruption you carry on describing the route. When you’ve
finished you are informed that you’ve just taken part in a psychology experiment, and
asked if you noticed any changes after the two men passed with the door. “No”, you
reply uneasily. The unknown man then explains that the man who approached you
initially walked off behind the door, leaving this man in his place. You are stunned;
the two men are dressed differently and have different voices and haircuts.
1

Rather than logging every detail of the visual scene, we are actually highly selective
about what we take in. Our impression of seeing everything is just that – an
impression. In fact we extract a few details and rely on memory, or perhaps even our
imagination, for the rest.

Page 15 of 24
2

Yet in 1991, the controversial claim was made that our brains hold only a few salient
details about the world - and that this is the reason we are able to function at all. We
don’t store elaborate pictures in short-term memory, because it isn’t necessary and
would take up valuable computing power.
\
3

Just a year later, at a conference on perception in Vancouver, it was reported that


people shown computer-generated pictures of natural scenes were blind to changes
that were made during an eye movement. In a typical laboratory demonstration of this
you might be shown a picture on a computer screen of, say, a couple dining on a
terrace.

It’s an unnerving experience. But to some extent, such “change blindness” is artificial
because the change is masked in some way. In real life, there tends to be a visible
movement that signals the change. But not always. For instance, we have all had the
experience of not noticing a traffic signal change because we had briefly looked away.
5

For instance, an experiment was done at Harvard in which people were shown a
videotape of a basketball game and asked to count the passes made by one or other
team. After about 45 seconds a man dressed in a gorilla suit walked slowly across the
scene, passing between the players. Although he was visible for five seconds, an
amazing 40 per cent of the viewers failed to notice him.

Such lapses raise important questions about vision. For instance, how can we reconcile
these gross lapses with our subjective experience of having continuous access to a rich
visual scene? One researcher has actually shown that imagining a scene activates parts

Page 16 of 24
of the visual cortex in the same way as seeing it. He says that this supports the idea
that we take in just what information we consider important at the time, and fill in the
gaps where the details are less important. The illusion that we see “everything” is
partly a result of filling in the gaps using memory. Such memories can be created
based on beliefs and expectations.
7

This particular idea has not been generally accepted. Yet most researchers in the field
do agree that of all the myriad visual details of any scene that we could record, we
take only what is relevant to us at the time. This leads us to the uncomfortable
realization that, for all our subjective experience of a rich visual world, it may, in fact,
be impossible to tell what is real and what is imagined.

A. Now imagine that the task absorbing their attention had been driving a car, and the
distraction had been a pedestrian crossing their path. According to some estimates,
nearly half of all motor-vehicle accidents in the US can be attributed to driver error,
including momentary loss of attention. It is more than just academic interest that has
made both forms of cognitive error hot research topics.

B. The image would disappear, to be replaced for a fraction of a second by a blank


screen, before reappearing significantly altered - by the raising of a railing in the
background, perhaps. Many people search the screen for up to a minute before they
see the change. A few never spot it.

C. In contrast, other researchers argue that we can get the impression of visual
richness without holding any of that richness in our heads. For instance, the “grand
illusion” theory argues that we held no picture of the visual world in our brains at all.
Instead, we refer back to the external visual world as different aspects become
important. The illusion arises from the fact that as soon as you ask yourself “Am I
seeing this or that?” you turn your attention to it and see it.

D. It sounds impossible, but when this test was carried out, a full 50 per cent of those
who took part failed to notice the substitution. The subjects had succumbed to what is

Page 17 of 24
called change blindness. Taken with a glut of recent experimental results, this
phenomenon suggests we see far less than we think we do.

E. The relationships between attention, awareness and vision have yet to be clarified.
Because we have a less than complete picture of the world at any one time, there is the
potential for distortion and error. How that complete picture could be objectively
established is controversial, but there is one obvious way forward.

F. This flies in the face of what vision researchers have long believed: that seeing
really means making pictures in the brain. According to this theory, by building
detailed internal representations of the world, and comparing them over time, we
would be able to pick out anything that changed.

G. And there’s a related phenomenon called inattentional blindness, that doesn’t need
any experimental visual trick at all: if you are not paying attention to some feature of a
scene, you won’t see it.

H. Rather, we log what has changed and assume the rest has stayed the same. Of
course: this is bound to mean that we miss a few details. Experimenters had already
shown that we may ignore items in the visual field if they appear not to be significant -
a repeated word or line on a page of text for instance. But nobody realized quite how
little we really do “see”.
Your answers
1. 2. 3. 4. 5. 6. 7.

Part 5. The passage below consists of four sections marked A, B, C and D. For
questions 1-10, read the passage and do the task that follows. Write your answers
(A, B, C or D) in the corresponding numbered boxes provided. (15pts)
A NIGHT TO REMEMBER
Our reporter spent a night camping out inside a zoo in Australia
A. Scanning the breakfast menu, I found myself swiftly losing my appetite: on offer
were beetle larvae, maggots and frozen mice. Fortunately, spending the night at
Sydney’s Taronga Zoo needn’t involve sampling the animals’ meals, though it does
entail getting extremely close to all manner of creatures. The Roar and Snore

Page 18 of 24
programme, curiously little known in Australia let alone overseas, allows you to spend
a night camping in the heart of the zoo, long after the crowds of day trippers have
dispersed. And if your experience of zoos is creatures slumped motionless in their
enclosures, or skulking in their dens, then let me assure you that at dusk, the place
comes alive and echoes with the din of hundreds of creatures making their presence
known. A ferryboat dropped me off at the foot of the zoo, which sprawls across a
steeply sloping swathe of bushland across the bay from the city. Guides, Steve and
Nikki, awaited with bright, enthusiastic smiles. Ten of us had signed up and, in no
time at all, we were busily erecting tents on a rectangle of grass outside the zoo’s
education centre. An adjacent area of gum trees was inhabited by kangaroos, wallabies
and an alarmingly inquisitive emu – our first animal encounter.
B. After a quick briefing, we set off into the gathering gloom. Steve unlocked a huge
metal gate that then clanged ominously behind us. We were in, though not without
some trepidation – what had we let ourselves in for? It was just us, a handful of
security staff and 2,000 furred and feathered inmates. A gibbon let out a haunting,
liquid whoop and wild possums frolicked noisily through the eucalyptus trees above
us, their dark silhouettes framed against a full moon. The skyscrapers of the city centre
twinkled in the distance as a pair of zebras scuffled in a cloud of dust; farther along, a
giant Kodiak bear lumbered menacingly around a large, rock-strewn enclosure. An
awesome spectacle in the gloom, but the buzz that evoked was soon surpassed when a
pair of snow leopards came within a few feet of us as they paced around the rocky
ledges of their cage.
C. Not all the animals at Taronga are behind bars. Semi-tame African guinea fowl
nest in the gum trees and a water buffalo let out a surprisingly cow-like moo, whilst a
group of macaws screeched as we climbed back up to the education centre. It gets
chilly after dusk, but Steve had left nothing to chance and a warming barbecue was
already sizzling with sausages and chicken kebabs. Meanwhile, Nikki brought out
some animals that are used to being handled: first a diamond python called Little
Spots and then a koala. Even the Australians in the group were enthralled as Nikki
showed us the pad of hardened skin that enables the species to sit for hours at a time in

Page 19 of 24
the crook of a tree. As she explained, the gum trees they eat contain only four percent
protein, so they have little energy to do much else.
D. After a bitterly cold night under canvas, it was a relief to be woken at dawn for
breakfast. As the sun rose and bathed the zoo in a soft, orange light, we fed carrots to
the zoo’s small herd of giraffes. One of them, Hope, is blind, and sought out the
carrots, with her long, slobbering blue tongue. ‘If you touch it, you’ll notice it feels
rough, like a cat’s,’ said her keeper. As Hope wrapped hers around my outstretched
finger, it felt more like sandpaper. As the sun’s rays warmed our chilled bodies, we
were taken on a behind-the-scenes tour, down a corridor with doors leading to each
enclosure, and notices reminding keepers of the animals’ diet and welfare
requirements. In a scrubbed kitchen, the inmates’ food is prepared. Here another
cheery keeper, Kristy, showed me the grey-coloured gloop that is fed to small
marsupials. ‘Have a taste,’ she said. I hesitated, then dipped in my finger. It was
delicious, like honey yoghurt. We were at the end of our visit and the zoo was again
admitting the public. Within an hour it seemed as though every four-year-old in
Sydney was careering around the place, letting out squeals of excitement – a sharp
contrast to the peaceful, privileged glimpse we’d been given.
In which section does the writer
1.......... mention a noise that made him feel slightly uneasy?
2.......... acknowledge the forethought of one of his hosts?
3.......... allude to the cleanliness of a section of the zoo?
4.......... suggest that most visitors don’t see the animals at their best?
5.......... imply that the activity he attended would benefit from wider publicity?
6.......... admit to an initial reluctance to take up a suggestion?
7.......... give us an impression of the scale and extent of the whole site?
8.......... describe a physical sensation that more than lived up to expectations?
9.......... report on one way in which the well-being of the animals is ensured?
10.......... mention a thrilling encounter with one particular species?
Your answers
1. 2. 3. 4. 5.
6. 7. 8. 9. 10.
Page 20 of 24
D. WRITING (60 pts)
Part 1: Summary (15pts) Read the following extract and use your own words to
summarize it. Your summary should be between 100 and 120 words long:
Plants play a very important role in our surroundings. Trees provide us with
fresh air, shade in summers, food, and other benefits without which we cannot even
think of living.
One of the most principal organs of a tree is a leaf. The leaves are the organs
for photosynthesis - a process when carbon dioxide is turned into oxygen. The
structures of leaves are adapted for efficient photosynthesis.
Most leaves are broad and so have a large surface area allowing them to absorb
more light. Also, they are thin, which means a short distance for carbon dioxide to
diffuse in and oxygen to diffuse out easily. The blade is the broad, flat part of the leaf.
Photosynthesis occurs in the blade, which has many green food-making cells.
If you look closer at leaves, you will notice networks of thin threads. Those networks
are called veins: they support the structure of the leaf and transport substances to and
from the cells in the leaf. The main vein of a leaf, running down the centre of the leaf,
is called midrib.
The area of some plants that connects the plant's stem and leaf is called
the petiole. The petiole is the pipeline through which the products of photosynthesis
are moved from individual leaves to the rest of a plant and through which necessary
chemicals and nutrients from other parts of the plant are brought to individual leave.
………………………………………………………………………………………….
………………………………………………………………………………………….
………………………………………………………………………………………….
………………………………………………………………………………………….
………………………………………………………………………………………….
………………………………………………………………………………………….
………………………………………………………………………………………….
………………………………………………………………………………………….
………………………………………………………………………………………….
………………………………………………………………………………………….

Page 21 of 24
………………………………………………………………………………………….
………………………………………………………………………………………….
………………………………………………………………………………………….
………………………………………………………………………………………….
………………………………………………………………………………………….
………………………………………………………………………………………….
Part 2. Chart description (15pts). The table shows data about underground railway
system in six major cities with date opened, kilometres of route and passenger
number per year in millions.
Summarize the information by selecting and reporting the main features, making
comparisons where relevant. Write at least 150 words.

Kilometers of Passengers per year


City Date opened
route (in millions)
London 1863 394 775
Paris 1900 199 1191
Tokyo 1927 155 1928
Washington DC 1976 126 144
Kyoto 1981 11 45
Los Angeles 2001 28 50

……………………………………………………………………………………………………………………………………………

……………………………………………………………………………………………………………………………………………

……………………………………………………………………………………………………………………………………………

……………………………………………………………………………………………………………………………………………

……………………………………………………………………………………………………………………………………………

……………………………………………………………………………………………………………………………………………

……………………………………………………………………………………………………………………………………………

……………………………………………………………………………………………………………………………………………

……………………………………………………………………………………………………………………………………………

……………………………………………………………………………………………………………………………………………

……………………………………………………………………………………………………………………………………………

……………………………………………………………………………………………………………………………………………

……………………………………………………………………………………………………………………………………………

……………………………………………………………………………………………………………………………………………
Page 22 of 24
……………………………………………………………………………………………………………………………………………

……………………………………………………………………………………………………………………………………………

……………………………………………………………………………………………………………………………………………

……………………………………………………………………………………………………………………………………………

……………………………………………………………………………………………………………………………………………

……………………………………………………………………………………………………………………………………………

……………………………………………………………………………………………………………………………………………

……………………………………………………………………………………………………………………………………………

……………………………………………………………………………………………………………………………………………

……………………………………………………………………………………………………………………………………………

……………………………………………………………………………………………………………………………………………

……………………………………………………………………………………………………………………………………………

……………………………………………………………………………………………………………………………………………

……………………………………………………………………………………………………………………………………………

……………………………………………………………………………………………………………………………………………

……………………………………………………………………………………………………………………………………………

……………………………………………………………………………………………………………………………………………

……………………………………………………………………………………………………………………………………………

……………………………………………………………………………………………………………………………………………

……………………………………………………………………………………………………………………………………………

……………………………………………………………………………………………………………………………………………

……………………………………………………………………………………………………………………………………………

Part 3: Write an essay of about 350 words to express your opinion on the following
issue (30 pts)
Online education is becoming more and more popular. Some people claim
that e-learning has so many benefits that it will replace face-to-face education soon.
Others say that traditional education is irreplaceable.
Discuss both views and give your opinion.
……………………………………………………………………………………………………………………………………………

……………………………………………………………………………………………………………………………………………

……………………………………………………………………………………………………………………………………………

Page 23 of 24
……………………………………………………………………………………………………………………………………………

……………………………………………………………………………………………………………………………………………

……………………………………………………………………………………………………………………………………………

……………………………………………………………………………………………………………………………………………

……………………………………………………………………………………………………………………………………………

……………………………………………………………………………………………………………………………………………

……………………………………………………………………………………………………………………………………………

……………………………………………………………………………………………………………………………………………

……………………………………………………………………………………………………………………………………………

……………………………………………………………………………………………………………………………………………

……………………………………………………………………………………………………………………………………………

……………………………………………………………………………………………………………………………………………

……………………………………………………………………………………………………………………………………………

……………………………………………………………………………………………………………………………………………

……………………………………………………………………………………………………………………………………………

……………………………………………………………………………………………………………………………………………

……………………………………………………………………………………………………………………………………………

……………………………………………………………………………………………………………………………………………

……………………………………………………………………………………………………………………………………………

……………………………………………………………………………………………………………………………………………

……………………………………………………………………………………………………………………………………………

……………………………………………………………………………………………………………………………………………

……………………………………………………………………………………………………………………………………………

……………………………………………………………………………………………………………………………………………

------- THE END -------

Người ra đề: Nguyễn Hạnh Tuyết (0974866717)


Trần Thị Quỳnh (0985002950)

Page 24 of 24
SỞ GIÁO DỤC VÀ ĐÀO TẠO BÌNH ĐỊNH ĐỀ THI ĐỀ XUẤT KỲ THI HSG
TRƯỜNG THPT CHUYÊN DUYÊN HẢI VÀ ĐỒNG BẰNG BẮC BỘ
LÊ QUÝ ĐÔN LẦN THỨ XIII
MÔN: TIẾNG ANH - KHỐI 11
Thời gian: 180 phút
Đề thi gồm: 14 trang

SECTION A. LISTENING (50 POINTS)


Part 1.You will hear a discussion in which two marine biologists. Gina Kelso and Thomas Ludman, talk
about an award-winning television film they made about wildlife in Antarctica. Choose the answer (A, B,
C or D) which fits best according to what you hear. (10 points)
1. Gina’s interest in marine biology dates from ______.
A. her earliest recollections of life in Africa
B. one memorable experience in childhood
C. the years she spent studying in England
D. a postgraduate research project she led
2. The first wildlife TV series they both worked on ______.
A. made use of a previously untried format
B. was not filmed in a natural environment
C. was not intended to be taken too seriously
D. required them to do background research
3. How did Thomas feel when he was asked to produce the programs about Antarctica?
A. disappointed not to be presenting the series
B. surprised that people thought he was suitable
C. uncertain how well he would get on with the team
D. worried about having to spend the winter there
4. When they were in Antarctica, they would have appreciated ______.
A. a less demanding work schedule
B. more time to study certain animals
C. a close friend to share their feelings with
D. a chance to share their work with colleagues
5. What was most impressive about the whales theyfilmed?
A. The unusual sounds the whales made.
B. The number of whales feeding in a small bay.
C. How long the whales stayed feeding in one area.
D. how well the whales co-operated with each other.
Your answers:
1. 2. 3. 4. 5.

Part 2. You will hear the historian, George Davies, talking about society and the theatre in England in the
time of William Shakespeare. Decide whether the following statements are true (T) or false (F).Write your
answers in the corresponding numbered boxes. (10 points)
1. According to Professor Davies, the level of literacy in sixteen-century England matched his expectations.
2. In Professor Davies' opinion, the advantage of the usual method of communication in the sixteenth century
was that people absorbed more of what they heard.
3. Professor Davies believes that Shakespeare's company developed their basic acting skills by attending
special voice classes.
4. In Professor Davies' view, the advantage of sixteen-century theatres was that the performances were
complemented by everyday life.
Page | 1
5. Professor Davies thinks that sixteen-century plays were expected to deal with personal confessions.
Your answers:
1. 2. 3. 4. 5.

Part 3. You will hear a student called Tina asking Professor Van Diezen for advice on choosing courses.
Listen and answer the following questions, using NO MORE THAN FIVE WORDS for each answer. (10
points)
1. What is the defining characteristic of a specialised course?
……………………………………………………………………………………………
2. For whom the Microbiology courses are available?
……………………………………………………………………………………………
3. Who are interested in Microbiology courses?
……………………………………………………………………………………………
4. Why will a Medical Science course be opened next year?
……………………………………………………………………………………………
5. Which is the quickest increasing subject in enrolment?
……………………………………………………………………………………………
Part 4. You will hear a radio report about Erik Weihenmayer, an adventurer. Complete the summary,
using the word or phrases you hear. Use NO MORE THAN THREE WORDS for each blank. (20 points)
An American named Erik Weihenmayer standing out as an adventurer without the
(1)………………… explains how he faces those challenges in today’s “Great Big Story”. He said the most
exciting part for him is in fact the movement, not the (2)………………… .
At 4 or 5, he was diagnosed with an (3)………………… disease and he went blind, which he thought
was a (4)………………… as the worst thing had happened, so there’s nothing else to lose. Then there was a
(5)………………… taking blind kids rock climbing, which he thought he wouldn’t have as a blind person.
When he got on to a rock face, he learned to do with his hands the things that (6)…………………
learn to do with their eyes. When clipping a bolt to a carabineer, he felt it to make sure it was correctly
clipped or that carabineer was going to hold him. Unable to look up the rock to see the holds and plan a
route, he could only see as far as his hands, which he thought was (7)………………… exciting. He loved the
sound of emptiness, which was meditative, very much like an (8)………………… .
Being a blind climber is really hard and you just have to embrace that suffering. Blindness is just like
all (9)………………… which you got to use as a catalyst to push you in new directions. It’s the idea of
(10)………………… into good things, and it’s something he thinks we all could use.

Your answers:
1. 2. 3. 4. 5.
6. 7. 8. 9. 10.

SECTION B. LEXICO-GRAMMAR (30 pts)


Part 1. Choose the word or phrase (A, B, C or D) which best completes each sentence(20 pts)
1.I hardly think Mary has got a bicycle herself, …………….?
A. hasn’t she B. doesn’t she C. has she D. does she
2. The hospital owes …………… for the construction of the new wing.
A. the government twenty million dollars B. for the government twenty million dollars
C. to the government twenty million dollars D. twenty millions of dollars to the government
3. The chairman had a recommendation that ……………….
A. each member studied more carefully the problem
B. the problem was more carefully studied
C. with more carefulness the problem could be studied
D. each member study the problem more carefully
4. Those campers are really ………….. They have no idea how to set up a tent.
Page | 2
A. white B. blue C. green D. black
5. I'd like to do something to change the world but whatever I do seems like a drop in the …………...
A. bucket B. garbage C. rubbish D. river
6. How did you know that he was lying?” – “It was just a ________ feeling.”
A. faint B. gut C. slight D. vain
7.Mrs. Jackson was on the point of going out of her apartment when she was _______ short by a phone call
from her husband.
A. caught B. halted C. brought D. stopped
8. People turned out in __________ to watch the parade on the Independence Day.
A. volume B. mass C. force D. bulk
9. It took him a long time to come to __________ with his redundancy
A. rules B. words C. terms D. steps
10. No decision has been taken about the building of the new airport. The authorities are still ______.
A. beating about the bush B. comparing apples and oranges
C. sitting on the fence D. holding all the aces
11. The president was eventually _______ by a military coup.
A. disposed B. despised C. deposed D. dispersed
12. The collapse of the silver market left him financially _______.
A. desolate B. dejected C. destitute D. derelict
13. He _______ so much harm on the nation during his regime that it has never fully recovered.
A. indicted B. inferred C. induced D. inflicted
14. Union leaders called for_______ between themselves and the government.
A. speeches B. elections C. debates D. consultations
15. It was clear from the beginning of the meeting that Jack was _______ on causing trouble.
A. inclined B. predisposed C. bent D. obsessed
16. The Internet has led to the faster and more effective _______ of information.
A. expansion B. coverage C. spread D. dissemination
17. This is the kind of crime that all decent people in society _______.
A. appall B. frown C. deplore D. disallow
18. For many young people, driving cars at high speed seems to _______ a rather fatal fascination.
A. contain B. comprise C. weave D. hold
19. On the news there was some dramatic _______ of the earthquake that had been captured by an amateur
cameraman.
A. shooting B. scenery C. clipping D. footage
20. I did not mean to offend her but she took my comments _______ and now will not talk to me.
A. amiss B. awry C. apart D. aside

Your answers:
1. 2. 3. 4. 5.
6. 7. 8. 9. 10.
11. 12. 13. 14. 15.
16. 17. 18. 19. 20.

Part 2. Fill in the gaps in the following sentences with the correct form of words (10 pts)
1. Alternative medicine can be ______ (LEAD), even off-putting for some people.
2. Few ______ (PRACTICE) of homeopathy, acupuncture and the like regard therapies as complete
substitutes for modern medicine.
3.Juvenile delinquencyrefers to ____________(SOCIETY) or illegal behavior by children or adolescents
and is considered a serious problem all over the world.
Page | 3
4. This juvenile _______________(CRIME)is apparent in marginal sectors of urban areas where children
are exposed to violence in their immediate social environment, either as observers or as victims.

5. Because delinquent basic education, if they have any, is poor they have been
______________(MARGIN) from society and destitute of any dignity or self - esteem.
6. These preventive policies should be prioritized over any ____________ (COERCE) measures.
7. Focus on the importance of family should become a priority because it is the primary institution of
_______________(SOCIAL) of youth
8. Scientists have wondered whether the secrets of our thoughts, ________(PERCEIVE) and even
consciousness itself might be hidden in the patterns of our brain waves.
9. A _______ (CLASS) lawsuit was filed Wednesday on behalf of applicants who were denied admission to
several universities affected by the scandal
10. Court documents allege the man at the _______ (CENTER) of the scandal,
Your answers:
1. 2. 3. 4. 5.
6. 7. 8. 9. 10.

SECTION C. READING (60 PTS)


Part 1. Fill in each blank in the following passage with ONE suitable sentences. Write your answers in the
spaces below. (7 pts)

Travel journalist Richard Madden reports on his first trip with a camera crew.
It was books that first captured my imagination about faraway places. TV travelogues always seemed
the poor relation to the classic written accounts, although of course the pictures were rather better. And then
there was the issue of authenticity. (1)………………These days programme-makers know that the audience
is more sophisticated and the presence of the camera is acknowledged. But can a journey with filming
equipment ever be anything other than a cleverly constructed fiction?
I recently got the chance to find out, when I was asked to present two one-hour programmes for an
adventure travel series. (2)……………..My sole qualification was as a journalist specialising in 'adventure'
travel. However, I was thought to have 'on-screen' potential
The first programme was filmed in Costa Rica. Within 24 hours of my arrival, I realised that this was
going to be very different from my usual 'one man and his laptop' expeditions. For a start, there were five of
us - director, cameraman, sound recordist, producer and presenter. And then there was the small matter of
£100,000 worth of equipment. (3)…………….. In his case, the term 'adventure' meant precisely that 'Made a
film with X,' he would say (normally a famous mountaineer or skier), before describing a death-defying
sequence at the top of a glacier in Alaska or hang-gliding off the Angel Falls in Venezuela. Invariably, these
reminiscences would end with the words: 'Had a great deal of respect for X. Dead now, sadly...'
Part of the brief for the series was to put the presenter in unusual situations and see how he or she
coped. (4)………..I don't have a head for heights and would make a poor rock-climber, so my distress is real
enough as the camera catches me dangling on a rope some 30 metres up, well short of the canopy platform.
Ironically, it was the presence of the camera, looking down on me from above, that gave me the
impetus for the final push to the top. By this time, I'd learnt how 'sequences' were cut together and realised
that one last effort was required. I had to struggle to stay coherent while the camera swooped within a few
millimetres of my face for my reaction. (5)……………..
Learning how to establish a rapport with the camera is vital and it took me a while to think of it as a
friend rather than a judge and jury. (6)………… The brief was simple. It needed to be 30 seconds long, sum
up my feelings, be informative, well-structured and, most important of all, riveting to watch. 'Ready to go in
about five minutes?' he would say breezily.
Page | 4
I soon discovered that the effect of the camera on what was going on around us was far less intrusive
than I had imagined. After a first flurry of curiosity, people usually lost interest and let us get on with our
job. (7)…………..Our trip coincided with an 80 per cent solar eclipse, a rare event anywhere in the world.
We were in a village called Santa Elena and captured the whole event on camera. The carnival atmosphere
was infectious and made a welcome addition to our shooting schedule.
( Extracted from “ Succeed in Cambridge English, CAE”)

A. One such sequence was the night we spent in the rainforest canopy near the Rincón de la Vieja National
Park in Guanacaste province.
B. The most intimidating moments were when Peter strolled up to me, saying that the light would only be
right for another 10 minutes, and that he needed a 'link' from one sequence to another.
C. We were also flexible enough to be spontaneous.
D. The project was the brainchild of the production company Trans-Atlantic Films, which wanted the series
presented by writers and adventurers, as well as TV professionals.

E. In the end, it was a magical experience, heightened all the more by the sounds of the forest - a family of
howler monkeys in a nearby tree, amplified through the sound recordist's headphones.
F. The project was the by products of so many efforts made by a staff of qualified leading experts.
G. I soon realised that the director, Peter Macpherson, was a vastly experienced adventure film-maker.
H. All those pretentious theatrical types dying of thirst in the desert, as if we didn't realise there was a
camera crew on hand to cater for their every need.

I. I soon realised I could make use of what the project was aimed at achieving.
Your answers:
1. 2. 3. 4. 5. 6. 7.

Part 2. Fill in each blank in the following passage with ONE suitable word. Number 0 has been done as
an example. Write your answers in the spaces below. (15 pts)
A new study conducted for the World Bank by Murdoch University's Institute for Science and
Technology Policy (ISTP) (0)………… demonstrated that public transport is more efficient than cars. The
study compared the proportion of wealth poured into transport by thirty-seven cities around the world. This
included both the public and (1) …………. costs of building, maintaining and using a transport system.
The study found that the Western Australian city of Perth is a good example of a city with minimal
public transport. As a (2) …………, 17% of its wealth went into transport costs. Some European and Asian
cities, on the other hand, spent as (3) ………. as 5%. Professor Peter Newman, ISTP Director, pointed out
that these more efficient cities were able to put the difference into attracting industry and jobs or creating a
better place to live.
According to Professor Newman, the larger (4) ……… city of Melbourne is a rather unusual city in
this sort of comparison. He describes it (5) …….… two cities: ‘A European city surrounded by a car-
dependent one’. Melbourne's large tram network has made car use in the inner city much lower, but the outer
suburbs have the same car-based structure as most (6) ………. Australian cities. The explosion in demand
for accommodation in the inner suburbs of Melbourne suggests a recent change in many people's preferences
as to (7) ………. they live.
Newman says this is a new, broader way of considering public transport issues. In the past, the case
for public transport has (8) ………….. made on the basis of environmental and social justice considerations

Page | 5
rather than economics. Newman, however, believes the study demonstrates that ‘the auto-dependent city
model is inefficient and inadequate in economic as (9) ………. .. as environmental terms’.
Bicycle use was not included in the study but Newman noted that the two most ‘bicycle friendly’
cities considered - Amsterdam and Copenhagen - were very efficient, even though their public transport (10)
………….. were ‘reasonable but not special’.
( Extracted from “ Succeed in Cambridge English, CAE”)
Your answers:
1. 2. 3. 4. 5.
6. 7. 8. 9. 10.

Part 3.Read the following passages and answer the questions ( 10 pts)

You are going to read reviews of four science fiction films. For questions 1 – 10, choose from the reviews
(A – D). The reviews may be chosen more than once.

About which of the films is the following stated?


1. The storyline is largely irrelevant.
2. One of the leading actors gives an outstanding performance.
3. The true nature of a leading character is disputed.
4. Its characters have unremarkable lifestyles.
5. There is fierce debate about what it means.
6. It’s often voted one of the best science fiction movies in history.
7. Key features of the film are scientifically unconvincing.
8. It poses questions that humans have always asked themselves.
9. One scene still shocks viewers today.
10. Some of the dialogue makes little sense.
Great science fiction films
Terry Stevens reviews four great science fiction films.
A. Blade Runner (1982)
Whether you prefer the original, rather theatrical release or the director’s cut of a few years later, Blade
Runner is perennially placed in opinion polls among the top five movies ever made in the genre. Directed by
Ridley Scott, the film revolves around Harrison Ford’s policeman, Rick Deckard, and his hunt for four
cloned humans, known as replicants, in an authoritarian city state sometime in the future. Replicants have
been declared illegal and Deckard is a ‘blade runner’, a specialist in exterminating them. Adding to the
interest is the issue of whether Deckard himself is a replicant. This is never clearly resolved in the film, and
fans continue to disagree over this point. When it first came out, the reception was muted, but it has grown in
popularity and critics now lavish praise on it. ‘It was groundbreaking in some ways,’ says one prominent
American writer on film, ‘but what it’s really about is something we’ve been interested in since the
beginning of history: What is it to be human? That’s what makes it truly great.’
B. 001: A Space Odyssey (1968)
One of the most controversial films of any genre, 2001: A Space Odyssey came from a collaboration
between the director, Stanley Kubrick, and the science fiction writer, Arthur C Clarke. It’s not an easy film to
sum up. The plot is mostly beside the point. It involves a government cover-up of something called the
Monolith, and a malfunctioning computer’s efforts to preserve the integrity of a space mission. Almost
independently of this are separate strands dealing with human evolution from prehistoric times to the space
age. Many have attempted to try and pin down this work with explanations about its deeper significance and
purpose, and to this day there are heated exchanges about this. What stays in the mind, though, is the impact
the film has on the senses. It has a soundtrack of often dissonant classical music played so loud that it often
interferes with what characters say. It is its astonishing visual style, however, that has probably had more
lasting influence than anything else about it.
C. Star Wars (1977)
Page | 6
It is almost impossible to argue against the inclusion of Star Wars or its rather darker sequel Empire Strikes
Back, in any list of top science fiction movies. Essentially westerns set in space, they cover the universal
themes of good versus evil, while making the leading actors Harrison Ford, Mark Hamill and Carrie Fisher
deliver lines of mind-boggling absurdity on a regular basis. The epic saga revolves around a battle between
an authoritarian Empire led by the Emperor and his part-human, partmachine henchman Darth Vader on one
side, and a small group of rebels on the other. The emphasis, however, is not on exploring deeper problems
of the human condition. Nor, unlike some film-makers, do the creators of Star Wars trouble themselves with
rooting their creations in the normal laws of physics; the force-wielding Jedi fight with theoretically
impossible light sabers and light-speed travel takes place in an implausible ‘hyperspace’. But the first two
Star Wars films have been the supreme blockbusters and paved the way in creating franchises for toys,
games and replicas that no major science fiction film can do without nowadays.
D. Alien (1979)
Alien is often remembered for the moment when an alien creature bursts out through the chest of one of the
crew members on the spaceship. This iconic moment has the power to unnerve even the most cynical of
contemporary audiences. The film has a lot more to it than that however. It is essentially an expertly made
horror story set on board a spaceship. The alien life form which invades the spaceship is very sinister but it is
made all the more so by the contrast with the portrayal of the ship’s crew. They are a bunch of very average
people who sit around eating pizza, playing cards and getting bored. This contrast between the crew’s very
mundane existence and the sheer awfulness of the alien is a very powerful one. Very striking too is
Sigourney Weaver’s portrayal of the reluctant hero Ellen Ripley. She is a compelling screen presence in this
movie and it established her as one of the top film actresses of her time.
(Extracted from “Cambridge English, CPE”)
Your answers:
1. 2. 3. 4. 5.
6. 7. 8. 9. 10.

Part 4. Choose the correct heading for each paragraph (1-5) from the list of headings below (i-x). There
are more headings than paragraphs so you will not use all of them.
Paragraphs B and G have been done for you as examples. ( 10 pts)

List of Headings
i. Disobeying FAA regulations
ii. Aviation disaster prompts action
iii. Two coincidental developments
iv. Setting altitude zones
v. An oversimplified view
vi. Controlling pilots’ licences
vii. Defining airspace categories
viii. Setting rules to weather conditions
ix. Taking off safely
x. First steps towards ATC
Example 1: Answer
Paragraph B x
1. Paragraph A
2. Paragraph C
3. Paragraph D
4. Paragraph E
5. Paragraph F
Example 2: Answer
Paragraph G vii

Page | 7
AIR TRAFFIC CONTROL IN THE USA
A. An accident that occurred in the skies over the Grand Canyon in 1956 resulted in the establishment of the
Federal Aviation Administration (FAA) to regulate and oversee the operation of aircraft in the skies over the
United States, which were becoming quite congested. The resulting structure of air traffic control has greatly
increased the safety of flight in the United States, and similar air traffic control procedures are also in place
over much of the rest of the world.
B. Rudimentary air traffic control (ATC) existed well before the Grand Canyon disaster. As early as the
1920s, the earliest air traffic controllers manually guided aircraft in the vicinity of the airports, using lights
and flags, white beacons and flashing lights were placed along cross-country routes to establish the earliest
airways. However, this purely visual system was useless in bad weather, and, by the 1930s, radio
communication was coming into use for ATC. The first region to have something approximating today’s
ATC was New York City, with other major metropolitan areas following soon after.
C. In the 1940s, ATC centres could and did take advantage of the newly developed radar and improved radio
communication brought about by the Second World War, but the system remained rudimentary. It was only
after the creation of the FAA that full-scale regulation of America’s airspace took place, and this was
fortuitous, for the advent of the jet engine suddenly resulted in a large number of very fast planes, reducing
pilots’ margin of error and practically demanding some set of rules to keep everyone well separated and
operating safely in the air.
D. Many people think that ATC consists of a row of controllers sitting in front of their radar screens at the
nation’s airports, telling arriving and departing traffic what to do. This is a very incomplete part of the
picture. The FAA realised that the airspace over the United States would at any time have many different
kinds of planes, flying for many different purposes, in a variety of weather conditions, and the same kind of
structure was needed to accommodate all of them.
E. To meet this challenge, the following elements were put into effect. First, ATC extends over virtually the
entire United States. In general, from 365m above the ground and higher, the entire country is blanketed by
controlled airspace. In certain areas, mainly near airports, controlled airspace extends down to 215m around
the ground, and, in the immediate vicinity of an airport, all the way down to the surface. Controlled airspace
is that airspace in which FAA regulations apply. Elsewhere, in uncontrolled airspace, pilots are bound by
fewer regulations. In this way, the recreational pilot who simply wishes to go flying for a while without all
the restrictions imposed by the FAA has only to stay in uncontrolled airspace, below 365m, while the pilot
who does want the protection afforded by ATC can easily enter the controlled airspace.
F. The FAA then recognized two types of operating environments. In good meteorological conditions, flying
would be permitted under Visual Flight Rules (VFR), which suggests a strong reliance on visual cues to
maintain an acceptable level of safety. Poor visibility necessitated a set of Instrumental Flight Rules (IFR),
under which the pilot relied on altitude and navigational information provided by the plane’s instrument
panel to fly safely. On a clear day, a pilot in controlled airspace can choose a VFR or IFR flight plan, and the
FAA regulations were devised in a way which accommodates both VFR and IFR operations in the same
airspace. However, a pilot can only choose to fly IFR if they possess an instrument rating which is above and
beyond the basic pilot’s license that must also be held.
G. Controlled airspace is divided into several different types, designated by letters of the alphabet.
Uncontrolled airspace is designated Class F, while controlled airspace below 5,490m is designated Class A.
The reason for the division of Class E and Class A airspace is where one finds general aviation aircraft (few
of which can climb above 5,490m anyway), and commercial turboprop aircraft. Above 5,490m is the realm
of the heavy jets, since jet engines operate more efficiently at higher altitudes. The difference between Class
E and A airspace is that in Class A, all operations are IFR, and pilots must be instrument-rated, that is,
skilled and licensed in aircraft instrumentation. This is because ATC control of the entire space is essential.
Three other types of airspace, Classes D, C and B, govern the vicinity of airports. These correspond roughly
to small municipal, medium-sized metropolitan and major metropolitan airports respectively, and encompass
an increasingly rigorous set of regulations. For example, all a VFR pilot has to do to enter Class C airspace is
establish two-way radio contact with ATC. No explicit permission from ATC to enter is needed, although the
pilot must continue to obey all regulations governing VFR flight. To enter Class B airspace, such as on
Page | 8
approach to a major metropolitan airport, an explicit ATC clearance is required. The private pilot who
cruises without permission into this airspace risks losing their license.
(Extracted from “Expert on Cambridge IELTS 3”)
Your answers:
1. 2. 3. 4. 5.

Task 2. Do the following statements agree with the views of the writer in the reading passage?
Write in the corresponding numbered boxes (3 pts)
YES (Y) if the statement agrees with the information given in the passage
NO (N) if the statement contradicts the information given in the passage
NO INFORMATION (NI) if there is no information on this in the passage
1. The FAA was created as a result of the introduction of the jet engine.
2. Beacons and flashing lights are still used by ATC today.
3. Some improvements were made in radio communication during World War II.

Your answers:
1. 2. 3.

Part 5: Read the following text and then choose from the list A - K the best phrase given below to fill each
of the spaces. Write one letter (A - K) in the spaces below. Each correct phrase may only be used once.
(1,0 m )
It is not easy to be systematic and objective about language study. Popular linguistic debate regularly
deteriorates into invective and polemic. Language belongs to everyone, so most people feel they have a right
to hold an opinion about it. And when opinions differ, emotions can run high. Arguments (1)_____ over
major policies of linguistic education.
Language, moreover, is a very public behavior, so it is easy for different usages to be noted and
criticized. No part of society or social behavior is exempt: linguistic factors (2)_____ personality,
intelligence, social status, educational standards, job attitude, and many other areas of identity and social
survival. As a result, it is easy to hurt, and to be hurt, when language use is unfeelingly attacked.
In its most general sense, prescriptivism is the view that one variety of language has an inherently higher
value than others, and that this (3)_____. The variety which is favoured, in this account, is usually a version
of the ‘standard’ written language, especially as encountered in literature, or in the formal spoken language
which most closely reflects this style. The view is pronounced especially in relation to grammar and
vocabulary, and frequently with reference to pronunciation. Adherents to this variety (4)_____; deviations
from it are said to be ‘incorrect’.
All the main languages (5)_____, especially in the 18th century approach to the writing of grammar and
dictionaries. The aims of these early grammarians were three fold: (i) they wanted to codify the principles of
their languages, to show that there was a system beneath the apparent chaos of usage, (ii) they wanted a
means of settling disputes over usage, and (iii) they wanted to point out what they felt to be common errors,
in order to ‘improve’ the language. The authoritarian nature of the approach (6)_____. Some usages are
‘prescribed’, to be learnt and followed accurately; others are ‘proscribed’, to be avoided. In this early period,
there were no half-measures: usage was either right or wrong, and it was the task of the grammarian not
simply to record alternatives, but to pronounce judgement upon them.
These attitudes are still with us, and they motivate a widespread concern that linguistic standards should be
maintained. Nevertheless, there is an alternative point of view that (7)_____ the facts of linguistic usage. This
approach is summarized in the statement that it is the task of the grammarian to describe, not prescribe_ to
record the facts of linguistic diversity, and not to attempt the impossible tasks of evaluating language variation
or halting language change. In the second half of the 18th century, we already find advocates of this view, such
as Joseph Priesley, whose Rudiments of English Grammar (1976) insists that ‘the custom of speaking is the

Page | 9
original and only just standard of any language’. Linguistic issues, it is argued, (8)_____. And this view has
become the tenet of the modern linguistic approach to grammatical analysis.
In our own time, the opposition between ‘descriptivists’ and ‘prescriptivists’ has often become extreme,
with both sides painting unreal pictures of the other. Descriptive grammarians (9)_____ care about standards,
because of the way they see all forms of usage as equally valid. Prescriptive grammarians have been
presented as bind adherents to a historical tradition. The opposition (10)_____ in quasi-political terms of
radical liberalism vs elitist conservatism.
( Extracted from “15 Days’ Practice for IELTS Reading”)

A. cannot be solved by logic and legislation

B. is best characterized by its reliance on ‘rules’ on grammar


C. is concerned less with standards than with
D. has even been presented
E. have been studied prescriptively
F. can start as easily over minor points of usage as
G. have been presented as people who do not

H. ought to be imposed on the whole of the speech community


I. are said to speak or write ‘correctly’
K. influence how we judge
Your answers:
1. 2. 3. 4. 5.
6. 7. 8. 9. 10.

SECTION D. WRITING (60 points)


Part 1. Read the following passage and use your own words to summarize it. Your summary should be
about 120 words long. (15 points)
Excessive demands on young people
Being able to multitask is hailed by most people as a welcome skill, but not according to a recent
study which claims that young people between the ages of eight and eighteen of the so-called “Generation
M” are spending a considerable amount of their time in fruitless efforts as they multitask. It argues that, in
fact, these young people are frittering away as much as half of their time again as they would if they
performed the very same tasks one after the other.
Some young people are juggling an ever larger number of electronic devices as they study. At the
same time that they are working, young adults are also surfing on the Internet, or sending out emails to their
friends, and/or answering the telephone and listening to music on their iPods or on another computer. As
some new device comes along it too is added to the list rather than replacing one of the existing devices.
Other research has indicated that this multitasking is even affecting the way families themselves
function as young people are too wrapped up in their own isolated worlds to interact with the other people
around them. They can no longer greet family members when they enter the house nor can they eat at the
family table.
All this electronic wizardry is supposedly also seriously affecting young people’s performance at
university and in the workplace. When asked about their perception of the impact of modern gadgets on their
performance of tasks, the overwhelming majority of young people gave a favorable response.

Page | 10
The response from the academic and business worlds was not quite as positive. The former feel that
multitasking with electronic gadgets by children affects later development of study skills, resulting in a
decline in the quality of writing, for example, because of the lack of concentration on task completion. They
feel that many undergraduates now urgently need remedial help with study skills. Similarly, employers feel
that young people entering the workforce need to be taught all over again, as they have become deskilled.
While all this may be true, it must be borne in mind that more and more is expected of young people
nowadays; in fact, too much. Praise rather than criticism is due in respect of the way today’s youth are able
to cope despite what the older generation throw at them.
...................................................................................................................................................................
...................................................................................................................................................................
...................................................................................................................................................................
...................................................................................................................................................................
...................................................................................................................................................................
...................................................................................................................................................................
...................................................................................................................................................................
...................................................................................................................................................................
...................................................................................................................................................................
...................................................................................................................................................................
...................................................................................................................................................................
...................................................................................................................................................................
...................................................................................................................................................................
...................................................................................................................................................................
...................................................................................................................................................................
...................................................................................................................................................................
...................................................................................................................................................................
...................................................................................................................................................................
...................................................................................................................................................................
...................................................................................................................................................................
...................................................................................................................................................................
...................................................................................................................................................................
...................................................................................................................................................................
...................................................................................................................................................................
...................................................................................................................................................................
...................................................................................................................................................................
Part 2. The pie chart below shows the main reasons why agricultural land becomes less productive. The
table shows how these causes affected three regions of the world during the 1990s. (15 points)

Summarize the information by selecting and reporting the main features, and make comparisons
where relevant.

Page | 11
1 2 3 4

1
3

...................................................................................................................................................................
...................................................................................................................................................................
...................................................................................................................................................................
...................................................................................................................................................................
...................................................................................................................................................................
...................................................................................................................................................................
...................................................................................................................................................................
...................................................................................................................................................................
...................................................................................................................................................................
...................................................................................................................................................................
...................................................................................................................................................................
...................................................................................................................................................................
...................................................................................................................................................................
...................................................................................................................................................................
...................................................................................................................................................................
...................................................................................................................................................................
Part 3.Write an essay of about 350 words to express your opinion on the following issue (30 points)
Schools are no longer necessary because children can get so much information available through the
Internet, and they can study just as well at home.
Page | 12
What is your own opinion?
Give reasons for your answer, and include any relevant examples from your knowledge or experience. You
may continue your writing on the back page if you need more space.
...................................................................................................................................................................
...................................................................................................................................................................
...................................................................................................................................................................
...................................................................................................................................................................
...................................................................................................................................................................
...................................................................................................................................................................
...................................................................................................................................................................
...................................................................................................................................................................
...................................................................................................................................................................
...................................................................................................................................................................
...................................................................................................................................................................
...................................................................................................................................................................
...................................................................................................................................................................
...................................................................................................................................................................
...................................................................................................................................................................
...................................................................................................................................................................
...................................................................................................................................................................
...................................................................................................................................................................
...................................................................................................................................................................
...................................................................................................................................................................
...................................................................................................................................................................
...................................................................................................................................................................
...................................................................................................................................................................
...................................................................................................................................................................
...................................................................................................................................................................
...................................................................................................................................................................
...................................................................................................................................................................
...................................................................................................................................................................
...................................................................................................................................................................
...................................................................................................................................................................
...................................................................................................................................................................
...................................................................................................................................................................
...................................................................................................................................................................
...................................................................................................................................................................
...................................................................................................................................................................
...................................................................................................................................................................
...................................................................................................................................................................
...................................................................................................................................................................
...................................................................................................................................................................
...................................................................................................................................................................
...................................................................................................................................................................
...................................................................................................................................................................
...................................................................................................................................................................
...................................................................................................................................................................
...................................................................................................................................................................
...................................................................................................................................................................
...................................................................................................................................................................
...................................................................................................................................................................
Page | 13
...................................................................................................................................................................
...................................................................................................................................................................
...................................................................................................................................................................
...................................................................................................................................................................
...................................................................................................................................................................
...................................................................................................................................................................
...................................................................................................................................................................
...................................................................................................................................................................
...................................................................................................................................................................
...................................................................................................................................................................
...................................................................................................................................................................
...................................................................................................................................................................
...................................................................................................................................................................
...................................................................................................................................................................
...................................................................................................................................................................
...................................................................................................................................................................
...................................................................................................................................................................
...................................................................................................................................................................
...................................................................................................................................................................
...................................................................................................................................................................
...................................................................................................................................................................
...................................................................................................................................................................

THE END

Gv ra đề: Chế Thị Hồng Thoa (0935002124) +Tống Mỹ Linh( 0987261135)

Page | 14
KỲ THI HỌC SINH GIỎI CÁC TRƯỜNG THPT CHUYÊN
KHU VỰC DUYÊN HẢI VÀ ĐỒNG BẰNG BẮC BỘ
LẦN THỨ XI, NĂM 2022

TRƯỜNG THPT CHUYÊN LÊ KHIẾT – QUẢNG NGÃI


ĐỀ ĐỀ XUẤT MÔN: TIẾNG ANH – KHỐI 11
Thời gian: 180 phút (Không kể thời gian giao đề)
ĐỀ ĐỀ XUẤT Ngày thi:14/7/2022
(Đề thi gồm 13 trang)
Điểm của bài thi: Họ, tên và chữ kí của 2 giám khảo: Số phách
Ghi bằng số: Giám khảo 1:
…………..… ......................................................................
Ghi bằng chữ: Giám khảo 2:
…………… ...........................................................................

Lưu ý: - Thí sinh làm bài vào đề thi này.


- Thí sinh không được sử dụng bất cứ tài liệu gì, kể cả từ điển
A. LISTENING (5.0 pts)
HƯỚNG DẪN PHẦN THI NGHE HIỂU
 Bài nghe gồm 4 phần, mỗi phần được nghe 2 lần, mỗi lần cách nhau 15 giây, mở đầu và
kết thúc mỗi phần nghe có tín hiệu.
 Mở đầu và kết thúc bài nghe có tín hiệu nhạc. Thí sinh có 3 phút để hoàn chỉnh bài trước
tín hiệu nhạc kết thúc bài nghe.
 Mọi hướng dẫn cho thí sinh (bằng tiếng Anh) đã có trong bài nghe.
HƯỚNG DẪN PHẦN THI NGHE HIỂU
 Bài nghe gồm 4 phần, mỗi phần được nghe 2 lần, mỗi lần cách nhau 15 giây, mở đầu và
kết thúc mỗi phần nghe có tín hiệu.
 Mở đầu và kết thúc bài nghe có tín hiệu nhạc. Thí sinh có 3 phút để hoàn chỉnh bài trước
tín hiệu nhạc kết thúc bài nghe.
 Mọi hướng dẫn cho thí sinh (bằng tiếng Anh) đã có trong bài nghe.
I. Listen and decide whether the following sentences are true (T) or false (F).
1. In a previous lecture, Dr North talked about the humankind’s relationship with sailing.
2. The focus of today’s talk will be on European fishing problems.
3. During the last century, the world population has grown very fast.
4. As well as over-fishing, the fact that oceans are more polluted is a reason for fishing stock
being on the point of collapse in the Pacific.
5. In the UK, fish used to be seen as a luxury.
II. Listen and complete the sentences below with NO MORE THAN TWO WORDS for each
answer.
(ROAD TO IELTS – LISTENING TEST 2 – SECTION 3 )
6. Students must follow ______ to prevent accidents in the lab.
7. The students have not been using ______ while in the lab.
8. Students cannot eat or drink until ______ is finished and they have washed their hands.
9. Tessa should tie her hair back to avoid danger when she is woking with a ______or
chemicals.
10. Students must wear long sleeves and shoes made of ______ in the lab.

Trang 1/13
III. You will hear a discussion in which two marine biologists. Gina Kelso and Thomas
Ludman, talk about an award-winning television film they made about wildlife in
Antarctica. Choose the answer (A, B, C or D) which fits best according to what you hear.
(CPE, Specifications and Sample Papers for Examinations from March 2013)
11.Gina’s interest in marine biology dates from ______.
A. her earliest recollections of life in Africa
B. one memorable experience in childhood
C. the years she spent studying in England
D. a postgraduate research project she led
12.The first wildlife TV series they both worked on ______.
A. made use of a previously untried format
B. was not filmed in a natural environment
C. was not intended to be taken too seriously
D. required them to do background research
13.How did Thomas feel when he was asked to produce the programs about Antarctica?
A. disappointed not to be presenting the series
B. surprised that people thought he was suitable
C. uncertain how well he would get on with the team
D. worried about having to spend the winter there
14.When they were in Antarctica, they would have appreciated ______.
A. a less demanding work schedule
B. more time to study certain animals
C. a close friend to share their feelings with
D. a chance to share their work with colleagues
15.What was most impressive about the whales they filmed?
A. The unusual sounds the whales made.
B. The number of whales feeding in a small bay.
C. How long the whales stayed feeding in one area.
D. how well the whales co-operated with each other.
IV. Listen to a piece of BBC news about the World War One centenary and fill in the
missing information. Write NO MORE THAN THREE WORDS taken from the recording
for each answer in the spaces provided.
People have criticised Germany's government for not doing enough to (1) ______ the World
War One centenary.
Germany hasn’t spent as much on events as some other (2) ______ countries. And the events
which have taken place have been seen as (3) ______by critics.
Traditionally in Germany the First World War is (4) ______the Second World War.
History teaching in German schools tends to focus on (5) ______ of the Nazis rather than what
happened (6) ______ earlier.
And since 1945 there's been a strong (7) ______ in Germany to anything that might be seen as
(8) ______ . So many people here are uncomfortable with any anniversary of a war or a battle.
There's still (9) ______ among historians about who was responsible for World War One. But
having spent the last 70 years atoning for Nazi guilt, many Germans have (10) ______to now
take on the blame for the First World War, too.
B. LEXICO-GRAMMAR
I. Choose the word/ phrase that best completes each of the following sentences.
1. Jack has egg ______ because he couldn't remember how to spell "Batman"!
A. on his teeth B. on his faceC. on his shirt D. on his fingers
2. As the maestro lifted his baton the theater was so still you could hear ______.
A. his heartbeat B. a pin drop C. bird wings D. the drum beating
Trang 2/13
3. I don't agree with you, but your idea certainly gives me food _____.
A. for fun B. for consider C. for thought D. for thinking
4. I think I understand the nuts and ______ of the operation.
A. screws B. hammer C. nails D. bolts
5. I'll be back in the twinkling of ______ .
A. an eye B. a lighting bolt C. a smile D. a laugh
6. Look, I will pay you back. Would you please call the ______?!
A. hunters off B. tigers off C. dogs off D. apes off
7. Ever since he married Jane, he's had a new ______ on life.
A. rent B. lease C. title D. loan
8. Oh, I'm sorry. I shouldn't have said that. I guess I really put my ______ in my mouth.
A. foot B. hand C. elbow D. knee
9. You need to keep a stiff ______ no matter what! Don't get nervous!
A. nose B. right arm C. eyebrow D. upperlip
10. I think you can take what he says at ______ value.
A. real B. face C. true D. straight
11. I’m afraid I’ve completely lost the ………… of the argument.
A. stream B. thread C. trace D. idea
12. When the teacher asked Mary what the chemical formula was, her mind was a total ______
A. void B. empty C. blank D. nothing
13. If you can win his attention ______for you.
A. the so much better B. the better so much
C. so much the better D. so the much better
14. No one can function properly if he or she is ______adequate sleep.
A. took away B. deprived of C. derived from D. got rid of
15. I was so exhausted that I went out like ______
A. a baby B. a candle C. a light D. a log
16. Most of ______ archaeologists know about prehistoric cultures is based on studies of
material remains.
A. what B. these C. which D. the
17. She ______ fainted when she heard that her child died.
A. rather than B. nothing but C. all but D. near
18. She had no chance to defend herself : the dog ______ for me as soon as I opened the door.
A. ran B. went C. fell D. stood
19. John ______ from social activity because he felt embarrassed with people.
A. held on B. held off C. held back D. held out
20. I’m afraid Tim doesn’t take much care over his homework. He usually does it ______
A. any how B. any old how C. how on earth D. how come
II. Provide the correct word form for each of the spaces below.
Complete each sentence, using the correct form of the word in parentheses
1. Hunger and a________meal did not sit happily side by side. (SLAP)
2. Britain’s nuclear power program began with a lie: it was a________ for the nuclear
weapons program. (SCREEN)
3. The_____vigor of man-made things, as suggested by these metaphors, is at the same time
ungainly and unnatural. (HUMAN)
4. The______woman did not utter a single word when her husband abused her. (SUBMIT)
5. Blinking to________herself to the darkness, Lexa already had her hands over her ears when
the deafening blast of thunder reached her. (CUSTOM)
6. In my heart of hearts, I think________laws such as these am abominations that bring the
entire legal system into disrepute.
Trang 3/13
7. He gained ______ for being difficult to work with as an actor. (NOTORIOUS)
8. Her latest novel is a _________ thriller, set some time in the late 21 sty century. (FUTURE)
9. The new policy only serves to _________ the inadequacy of help for the homeless.
(ACCENT)
10. Zombies are archetypal monsters from the bottom of the uncanny valley, with their dead
eyes and _______ faces. (EXPRESS)
C. READING
I. Read and fill in one suitable word.
(Mark Lester- Child Star CPE Reading & Use of English (2013)- Grivas p. 282)
Should you be over a certain age, my friend Mark Lester will need (1)___________
introduction, being probably the finest and most talented child actor the UK has ever
produced. His list of co-stars is proof of his standing and ability- Kirk Douglas, Shelley
Winters, Charlton Heston, Gordon Jackson, Oliver Reed and Britt Ekland are just a
(2)__________ of the major star with (3)__________ Mark acted. Although most people
remember Mark for his stunning performance in Sir Carol Reed’s Oscar-winning musical of
Lionel Bart’s Oliver he (4)____________, of course, star in many other world-class movies
such as Run Wild, Run Free, Eyewitness, Who slew Aunty Roo to name (5)____________ a
few. One of his most stunning performances was with Dirk Bogarde in Our Mother’s House
made when Mark was only six years old. In (6)_________, he had to play an autistic child. He
won a top acting award for this effort, which brought him to the notice of Sir Carol Reed, who
cast him perfectly as Oliver. I consider myself privileged to count him as a close friend. He
was a lovely unaffected boy (7)___________ being the world’s top child actor for nearly ten
years and he is just as wonderful (8)___________ adult. He has long since left the screen and
now (9)___________ his own highly successful osteopathy and acupuncture
(10)___________.
II. Read and choose the best answer.
(Succeed in CPE p.105)
Livestrong – but will the legacy?
In the early- to mid-1990s, Lance Armstrong was on the up-and-up. Success seemed to be
written in his stars; he notched up a stage win at the '93 Tour de France, then another in '95.
This cyclist was clearly coming of age in the sport, and he was, at 24 on registering his second
tour win, still a relative baby in cycling terms - most of his career lay ahead of him. Then, just
when it looked like he would conquer all before him, his '96 tour was cut disappointingly short
due to illness. And, as it would soon emerge, this was no ordinary illness; Armstrong had
testicular cancer. Fans were aghast and there was an out-pouring of sympathy for him.
But Armstrong would need more than goodwill to get through this. The cancer had
metastasized to the lungs and the brain. The prognosis was not at all good. Months of spirit-
and body-breaking chemotherapy followed and a delicate surgical procedure to remove the
malignancies on his brain was performed. Cycling mourned the surely permanent loss to the
sport of one of its most promising young disciples. But Armstrong wasn't finished yet.
In 1998, he made a remarkable, defiant and inspirational return to cycling and competed in the
Tour de France again the following year. But surely his would now only be a cameo role; after
all, what could one expect from a cancer survivor with a compromised liver and the other
familiar scars of cancer therapy? Except Armstrong had other ideas.
Four stage wins later, the legend of Armstrong was born; he had claimed the Tour and defied
the odds in the most emphatic of manners. His victory represented not just his announcement
as a force in cycling, but as a force for hope for millions of cancer sufferers the world over.
Indeed, Armstrong threw himself into campaigning for his newly-established cancer
foundation, Livestrong - so much so that he metamorphosed into a sort of human-embodiment
of the cause — he became the cause, and his annual battle with the French Alps came to
Trang 4/13
represent the struggle against the deadly disease. So long as Lance could succeed, there was
hope.
And succeed he did, beyond the wildest expectations of even the most optimistic of his
supporters, amassing a further six titles — so seven in consecution —before he retired in 2005.
His achievements were simply remarkable; his story absorbing; his book a must-read for all
cancer sufferers — their ray of hope; proof that hopefulness should never fade and that
sanguinity can and does make light of the odds - the tunnel, though long and at times
excruciating to pass through, has an end, and it is a happy one —the light is in sight.
After his seventh victory, he retired and the sporting world entered congratulatory mode,
writing his eulogies. But Armstrong had one more surprise for us; he wasn't finished yet.
There were whispers of a comeback, confirmed in 2009, and so it was that the legend would
ride again.
But the renewed focus on him wasn't all good; there were whispers of another kind, too;
sources, some credible, were claiming he had had an illicit ally all through his exploits; he
was, they claimed, in bed with the syringe. Our champion laughed off and dismissed these
claims but the rumours persisted and a cloud began to form over his legacy. Surely Armstrong
could not have earned his victories clean, some said.
We may never know for sure. Fast-forward to 2012 and despite an abandoned federal case,
those sharpening their knives for Armstrong seem to have finally nabbed him; ASADA, the
U.S. body tasked with cracking down on drug offenders charged Armstrong with doping and
the trafficking of drugs - and some say his failure to contest is indicative of his guilt. At any
rate, because he pleaded no-contest, he will now be stripped of all his titles; his legacy has
been pulled from under him.
And yet he has not, and now may never be tried, so we have not seen the evidence against him.
We do not know if he is guilty or innocent, and it still remains fact that he never failed an
official drugs test. Did he cheat? Does it matter? Does anyone care? Time may tell, but for
now, though his legacy is tainted, his legend, in the eyes of many of his loyal supporters, lives
on.
11. What does the writer mean when he says in the first paragraph that Lance Amstrong was
‘coming of age in the sport’
A. he was of the right age to be a competitive cyclist
B. he was nearly at the age at which it is expected that a cyclist should win
C. he was of a mature age for a cyclist and had few years left in the sport
D. he was beginning to figure as a real contender in his sport
12. What does “it” in paragraph 1 refer to?
A. Armstrong’s illness
B. the ’96 Tour de France
C. Amstrong’s career
D. none of the above
13. Which of the following statements is true about the cancer Armstrong had?
A. he recovered remarkably quickly from it, suffering little
B. it started in the lungs and spread to the brain
C. doctors were optimistic about his chances of survival
D. the generally held view was that it would prevent him from cycling professionally ever
again
14. What was implied when the writer said ‘but surely his would now only be a cameo role’ in
the third paragraph?
A. the author did not believe Armstrong would come back to the Tour de France at the time
B. the certainty about Armstrong’s desire to play a cameo role in the Tour de France
C. the general lack of confidence in Armstrong’s competitiveness
Trang 5/13
D. the popular demand for Armstrong to be back
15. Why does the writer say, 'Except Armstrong had other ideas', at the end of the third
paragraph?
A. Armstrong was determined to play some role in the Tour de France again
B. Armstrong's idea of victory had changed since he'd had cancer.
C. Armstrong was determined to defy the odds and become a real contender in the Tour de
France.
D. Armstrong didn't want to race for victory, he just wanted to represent cancer victims.
16. What does the writer compare Armstrong's Tour de France campaign struggle each year
after his return to the sport with?
A. the general fight against cancer
B. a cancer organisation
C. his fundraising for cancer
D. Armstrong's own personal cancer experience
17. What is one of the ways in which his story became about more than just cycling?
A. his published biography became a source of inspiration for cancer sufferers
B. cycling through a tunnel was like fighting cancer
C. he gave people hope that they could one day be professional athletes, too
D. he gave people the belief to fight the disease that is drug-taking in sport
18. What does Lance Armstrong’s ‘illicit ally’ represent?
A. the person who helped him win illegally
B. the drugs he used
C. the person with whom he cheated
D. the person who sold him drugs
19. What can be inferred about the rumours of Armstrong's drug-taking?
A. they were disproved in a state court case
B. they have not caused Armstrong's reputation and record any harm
C. they were eventually proved true beyond doubt
D. he had, but passed up, an opportunity to disprove them
20. What is the author’s purpose in the last paragraph?
A. informing readers of his legacy
B. denying his scandals
C. approving of his exploits
D. confirming his guilt
III. Read and complete the tasks required.
(Road to IELTS 4 Reading)
Choose the correct heading for paragraphs A–H from the list of headings below. Write the
correct number, i–xi, in boxes 21-27
List of Headings
i Gender bias in televised sport
ii More money-making opportunities
iii Mixed views on TV’s role in sports
iv Tickets to top matches too expensive
v A common misperception
vi Personal stories become the focus
vii Sports people become stars
viii Rules changed to please viewers
ix Lower-level teams lose out
x Skill levels improve
xi TV appeal influences sports’ success
Trang 6/13
Example: Paragraph A v
21 Paragraph B
22 Paragraph C
23 Paragraph D
24 Paragraph E
25 Paragraph F
26 Paragraph G
27 Paragraph H
Television and Sport
when the medium becomes the stadium
A The relationship between television and sports is not widely thought of as
problematic. For many people, television is a simple medium through which sports can be
played, replayed, slowed down, and of course conveniently transmitted live to homes across
the planet. What is often overlooked, however, is how television networks have reshaped the
very foundations of an industry that they claim only to document. Major television stations
immediately seized the revenue-generating prospects of televising sports and this has
changed everything, from how they are played to who has a chance to watch them.

B Before television, for example, live matches could only be viewed in person. For the
majority of fans, who were unable to afford tickets to the top-flight matches, or to travel the
long distances required to see them, the only option was to attend a local game instead,
where the stakes were much lower. As a result, thriving social networks and sporting
communities formed around the efforts of teams in the third and fourth divisions and below.
With the advent of live TV, however, premier matches suddenly became affordable and
accessible to hundreds of millions of new viewers. This shift in viewing patterns vacuumed
out the support base of local clubs, many of which ultimately folded.

C For those on the more prosperous side of this shift in viewing behaviour, however,
the financial rewards are substantial. Television assisted in derailing long-held concerns in
many sports about whether athletes should remain amateurs or ‘go pro’, and replaced this
system with a new paradigm where nearly all athletes are free to pursue stardom and to
make money from their sporting prowess. For the last few decades, top-level sports men and
women have signed lucrative endorsement deals and sponsorship contracts, turning many
into multi-millionaires and also allowing them to focus full-time on what really drives them.
That they can do all this without harming their prospects at the Olympic Games and other
major competitions is a significant benefit for these athletes.

D The effects of television extend further, however, and in many instances have led to
changes in sporting codes themselves. Prior to televised coverage of the Winter Olympics,
for example, figure skating involved a component in which skaters drew ‘figures’ in the ice,
which were later evaluated for the precision of their shapes. This component translated
poorly to the small screen, as viewers found the whole procedure, including the judging of
minute scratches on ice, to be monotonous and dull. Ultimately, figures were scrapped in
favour of a short programme featuring more telegenic twists and jumps. Other sports are
awash with similar regulatory shifts – passing the ball back to the goalkeeper was banned in
football after gameplay at the 1990 World Cup was deemed overly defensive by television
viewers.

E In addition to insinuating changes into sporting regulation, television also tends to


favour some individual sports over others. Some events, such as the Tour de France,
Trang 7/13
appear to benefit: on television it can be viewed in its entirety, whereas on-site enthusiasts
will only witness a tiny part of the spectacle. Wrestling, perhaps due to an image problem
that repelled younger (and highly prized) television viewers, was scheduled for removal from
the 2020 Olympic Games despite being a founding sport and a fixture of the Olympics since
708 BC. Only after a fervent outcry from supporters was that decision overturned.

F Another change in the sporting landscape that television has triggered is the framing
of sports not merely in terms of the level of skill and athleticism involved, but as personal
narratives of triumph, shame and redemption on the part of individual competitors. This is
made easier and more convincing through the power of close-up camera shots, profiles and
commentary shown during extended build-ups to live events. It also attracts television
audiences – particularly women – who may be less interested in the intricacies of the sport
than they are in broader ‘human interest’ stories. As a result, many viewers are now more
familiar with the private agonies of famous athletes than with their record scores or match-
day tactics.
G And what about the effects of male television viewership? Certainly, men have
always been willing to watch male athletes at the top of their game, but female athletes
participating in the same sports have typically attracted far less interest and, as a result,
have suffered greatly reduced exposure on television. Those sports where women can draw
the crowds – beach volleyball, for example – are often those where female participants are
encouraged to dress and behave in ways oriented specifically toward a male demographic.
H Does all this suggest the influence of television on sports has been overwhelmingly
negative? The answer will almost certainly depend on who among the various stakeholders
is asked. For all those who have lost out – lower-league teams, athletes whose sports lack a
certain visual appeal – there are numerous others who have benefitted enormously from the
partnership between television and sports, and whose livelihoods now depend on it.

Do the following statements agree with the claims of the writer? In boxes 28–32, write YES
if the statement agrees with the claims of the writer NO if the statement contradicts the claims
of the writer NOT GIVEN if it is impossible to say what the writer thinks about this
28 Television networks were slow to recognise opportunities to make money from televised
sport.
29 The average sports fan travelled a long way to watch matches before live television
broadcasts.
30 Television has reduced the significance of an athlete’s amateur status.
31 The best athletes are now more interested in financial success rather than sporting
achievement.
Complete the notes below. Choose NO MORE THAN TWO WORDS from the passage for
each answer. Write your answers in.
Effect of television on individual sports
• Ice skating – viewers find ‘figures’ boring so they are replaced with a 32 ______
• Back-passing banned in football.
• Tour de France great for TV, but wrestling initially dropped from Olympic Games due to 33
______

IV. Gapped text


(P 184 CPE Reading & Use of English (2013)- Grivas)
The Deepest Scar
I knew the mountains by name, by shape, by size; I felt I could even close my eyes and know
them through their presence. Observation Post One was near the peak of the highest of them
Trang 8/13
all and from it they spread out to all horizons; massive, craggy, their tops bare of snow in this
fierce midsummer heat, the dark rock gradually giving way to thicker and thicker pine down to
the valley floor, which was as perfect as a green carpet, fitted carefully around the foot of each
mountain.
34 ______
Inside me was a scar that mirrored it exactly, that I had come up here to allow to heal. I had
always come into the mountains to heal; the wilderness, the solitude, soothed me, calmed a
heart torn by my parents’ break-up, the disappointments of youth and adulthood. I just wasn’t
sure they were working their magic this time.
35______
I was starting to think that was the problem. The sadness was still laced with bitterness, and a
need to blame. And what else was there to blame but those huge silent giants, who did not
mourn or shed tears, did not sing funeral songs, but sat, eternal, stone-hearted. For them a
human death was nothing. A mountain took thousands of years to be born and thousands more
to die, and in that time would witness thousands of deaths both great and small.
36______
This was when the fire crews were summoned, the planes and helicopters dispatched. I, too,
had seen many of them, fought in the smoky thick of them to stop their spread, emerged
exhausted and hoarse from the heat but triumphant, as the flames subsided and the fire planes
doused the last of the smouldering embers.
37______
We’d been called up while on standby in Larch, my hometown. Twenty of us veterans and
some new bloods; five college kids, my little brother Steve one of them. He’d always wanted
to fight fires and was finally old enough to - he’d signed up for when it term at college ended,
and come to join me.
38______
What we hadn’t reckoned on was the wind change, and the strength of the westerly that caused
it. The moment we felt it, we veterans froze; it came roaring through the trees with another
deeper roaring adding to its voice. This last was our old enemy’s war cry; the fire sprang up
fifty feet into the air and raced towards us like a viking possessed, with a bloodlust to be sated.
39______
I was his elder brother by seven years – he had followed me for as long as I could remember.
It was often nice, sometimes annoying, but it was a fact. Why I had not realised it would
happen then is something I will never fathom and never forgive myself for.
40______
But physical scars, though painful and unsightly, do not hurt for long, and they show the world
at least that you have risked yourself, have fought. It was the scar inside that never seemed to
stop bothering me, the dark scar so much like the one down on the forest floor, so like the bare
earthen grave in which we laid him, my divorced and distant parents and the last and desolate
son.

A. I had wanted to fight fires for as long E. The other two were fine - had gone off
as I could remember; I waited for my a slant but reached the river safely,
schooling to end with impatience, itching plunged into its cool sanctuary. I was fine
to become part of what was, for me, the too, though I had to spend some weeks in
real world of nature and its forces, hospital, and the burns I received trying to
creative and destructive. Steve was pull Steve out from under the burning tree
different, studious and quiet, but I always will never properly heal, and will always
felt that my burning ambition had put a make people look twice.
spark in his heart that, like a forest fire,
Trang 9/13
had smouldered a long time before
bursting into life.
B. It was a big fire but the winds weren’t F. Likewise we fled, fear our demon, as
high and the position was good; we the trees popped into flame all around as
concentrated on cutting a firebreak from easily as candles on a birthday cake, and
Cawdor’s Ridge to the river, cutting it off smoke made our eyes sting. We headed
on two sides. This done, we looped around for the river and I, missing two men,
and started doing much the same on the turned back to make sure they were safe.
other side, from the river up to the ridge. I It was what I always did, trusting my luck,
remember being unconcerned, grinning at my instinct, my knowledge. My brother,
Steve’s ash-blackened face and flashing trusting only me, turned back too.
white teeth and eyes.
C. Except for that dark slash that I G. Tony Morgan had been doubtful when
couldn’t keep my eyes off, as hard as I I volunteered. He said it was too early to
tried. From the open side of Cawdor’s be alone. He was one of the new breed of
Ridge it cut a swathe of black burnt wood forest rangers, who’d learnt the skills in
and ash-thick earth down into the heart of college, knew countless facts and theories
the forest, stopped only by the river, that I lacked. But I insisted, knowing that I
glinting here and there in the sunshine as too had something he did not; a sense of
it wended its slow way to the sea. being part of it all, a feeling of kinship
with the mountain.
D. I had loved it, loved the danger, loved H. And as many fires. There were fires
the satisfaction of doing a hard and every summer, when the dead wood and
dangerous job well. They had called me grass were bone dry and summer storms
‘The Charm’ because I put myself at such snaked lightning into the peaks and
risk and always emerged unscathed. It had valleys. That was my job here at
seemed like a good thing, being lucky, but Observation Post One; any smoke was to
it didn’t anymore. The scars, one inside be logged, its position noted and observed
me and one down in the valley, catching for days after. Most fires went out alone,
my eyes; all that remained of the last fire I some gently smouldered and then sprang
had fought. up, sudden monsters, armed with swords
of flame and intent on destruction.

V. You are going to read an article about the Royal Society, a British scientific institution.
For questions 41-50, choose from the sections of the article (A—E). The sections may be
chosen more than once.
(CAE Practice Test Plus 2 p.106)
In the exam, mark your answers on the separate answer sheet.
In which section of the article are the following mentioned?

41 a belief that a certain development has been of particular use to scientists


42 the variety of ways in which the Royal Society encourages people who are not scientists to
consider scientific issues
43 a rapid reaction to research being made public
44 a particular development that requires urgent action to improve it
45 a resource for information on past scientific discoveries
46 a lack of understanding of scientific matters among people in general
47 a system that the Royal Society introduced
48 the fact that scientists do not always reach firm conclusions
49 a problem that is not limited to the world of science
Trang 10/13
50 the belief that certain things that are possible are not desirable

The unstoppable spirit of inquiry


The president of the Royal Society, Martin Rees, celebrates the long history of one of
Britain's greatest institutions.

A The Royal Society began in 1660. From the beginning, the wide dissemination of
scientific ideas was deemed important. The Society started to publish Philosophical
Transaction, the first scientific journal, which continues to this day. The Society's journals
pioneered what is still the accepted procedure whereby scientific ideas are subject to peer
review — criticised, refined and codified into 'public knowledge'. Over the centuries, they
published Isaac Newton's researches on light, Benjamin Franklin's experiments on lightning,
Volta's first battery and many of the triumphs of twentieth-century science. Those who want to
celebrate this glorious history should visit the Royal Society's archives via our Trailblazing
website.
B The founders of the Society enjoyed speculation, but they were also intensely engaged
with the problems of their era, such as improvements to timekeeping and navigation. After 350
years, our horizons have expanded, but the same engagement is imperative in the 21' century.
Knowledge has advanced hugely, but it must be deployed for the benefit of the ever-growing
population of our planet, all empowered by ever more powerful technology. The silicon chip
was perhaps the most transformative single invention of the past century; it has allowed
miniaturisation and spawned the worldwide reach of mobile phones and the internet. It was
physicists who developed the World Wide Web and, though it impacts us all, scientists have
benefited especially.
C Traditional journals survive as guarantors of quality, but they are supplemented by a
blogosphere of widely varying quality. The latter cries out for an informal system of quality
control. The internet levels the playing fields between researchers in major centres and those
in relative isolation. It has transformed the way science is communicated and debated. In 2002,
three young Indian mathematicians invented a faster scheme for factoring large numbers —
something that would be crucial for code-breaking. They posted their results on the web.
Within a day, 20,000 people had downloaded the work, which was the topic of hastily
convened discussions in many centres of mathematical research around the world. The internet
also allows new styles of research. For example, in the old days, astronomical research was
stored on delicate photographic plates; these were not easily accessible and tiresome to
analyse. Now such data (and large datasets in genetics and particle physics) can be accessed
and downloaded anywhere. Experiments and natural events can be followed in real time.
D We recently asked our members what they saw as the most important questions facing
us in the years ahead and we are holding discussion meetings on the 'Top Ten'. Whatever
breakthroughs are in store, we can be sure of one thing: the widening gulf between what
science enables us to do and what it's prudent or ethical actually to do. In respect of certain
developments, regulation will be called for, on ethical as well as prudential grounds. The way
science is applied is a matter not just for scientists. All citizens need to address these
questions. Public decisions should be made, after the widest possible discussion, in the light of
the best scientific evidence available. That is one of the key roles of the Society. Whether it is
the work of our Science Policy Centre, our journals, our discussion meetings, our work in
education or our public events, we must be at the heart of helping policy makers and citizens
make informed decisions.
E But science isn't dogma. Its assertions are sometimes tentative, sometimes compelling;
noisy controversy doesn't always connote balanced arguments; risks are never absolutely zero,
even if they are hugely outweighed by potential benefits. In promoting an informed debate, the
Trang 11/13
media are crucial. When reporting a scientific controversy, the aim should be neither to
exaggerate risks and uncertainties, nor to gloss over them. This is indeed a challenge,
particularly when institutional, political or commercial pressures distort the debate. Scientists
often bemoan the public's weak grasp of science — without some 'feel' for the issues, public
debate can't get beyond sloganising. But they protest too much: there are other issues where
public debate is, to an equally disquieting degree, inhibited by ignorance. The Royal Society
aims to sustain Britain's traditional strength in science, but also to ensure that wherever science
impacts on people's lives, it is openly debated.
D. WRITING
I. Read the following extract and use your own words to summarise it. Your summary
should be between 100 and 120 words long.
Have you ever wondered why soldiers are always clad in green? This is to enable them to
camouflage themselves during wartime. Hiding in the jungles, their green attire blend into the
surrounding trees and shrubs, making it difficult for the enemies to spot them.
Long before man make use of camouflaging, insects have already adopted the tactic of
disguise to escape from the clutches of their predators. By having body colors close to those of
the rocks and dried leaves, they catch less attention from the predators and hence escape from
being pursued. However, this kind of disguise works only if the insects remain still in the
presence of their predators.
Butterflies and moths have developed a variety of camouflage strategies since they are quite
defenceless and their predators - birds are abundant in supply. Many moth caterpillars
resemble dead twigs while the young of certain species of butterflies appear like bird
droppings. Adult butterflies and moths camouflage themselves too, in attempts to escape from
their hunters -- birds who are superior gliders. Possessing wings which resemble dried leaves
help certain butterflies and moths to hide among heaps of dried leaves when predators are
around.
Fortunately, not all insects choose the art of disguise to escape from their predators; otherwise,
the world would be so dull and colorless. There are insects which assimilate the bright body
colors of bees and wasps to escape from being pursued by their predators. The concept of
mimicry was derived, owing to the bees and wasps. Long ago, birds have already learnt to
avoid brilliantly colored wasps and bees in fear of their painful stings. Hence, over millions of
years, many harmless insects have assimilated the bees and wasps by imitating their bright
body colors and shapes. In this way, they appear dangerous to their predators and hence ward
them off.
Mimics of the wasps and bees are most commonly found in the gardens. The furry, plump bee-
fly not only appears like the bumble bee in terms of body colors, even its hums sound similar
too. The only difference is that the bee-fly does not have a sting and is hence harmless. The
hoverfly is another insect which imitates the body colors of the wasps. Their bodies are striped
yellow and black. The only deviations are that hoverflies do not have stings and they have only
one pair of wings each while wasps have two pairs each. These variations are hardly noticed
by the predators and hence help them to escape.

Trang 12/13
II. The charts below show reasons for travel and the main issues for the travelling public in
the US in 2009. Summarise the information by selecting and reporting the main features,
and make comparisons where relevant. You should write about 150 words.

III. Write an essay of 350 words on the following topic:


“Some educational systems emphasize the development of student's capacity for
reasoning and logical thinking, but students would benefit more from an education that
also taught them to explore their own emotions."
Present your perspective on this issue, using relevant reasons and examples to support your views.

Trang 13/13
SỞ GD&ĐT NAM ĐỊNH KỲ THI HỌC SINH GIỎI CÁC TRƯỜNG THPT CHUYÊN
TRƯỜNG THPT CHUYÊN KHU VỰC DUYÊN HẢI VÀ ĐỒNG BẰNG BẮC BỘ
LÊ HỒNG PHONG LẦN THỨ XIV, NĂM 2022

ĐỀ ĐỀ XUẤT ĐỀ THI MÔN: TIẾNG ANH – KHỐI 11


Thời gian: 180 phút (Không kể thời gian giao đề)
(Đề thi gồm 23 trang)

A. LISTENING (50 pts)


HƯỚNG DẪN PHẦN THI NGHE HIỂU
 Bài nghe gồm 4 phần, mỗi phần được nghe 2 lần, mỗi lần cách nhau 20 giây, mở
đầu và kết thúc mỗi phần nghe có tín hiệu.
 Mở đầu và kết thúc bài nghe có tín hiệu nhạc. Thí sinh có 3 phút để hoàn chỉnh bài
trước tín hiệu nhạc kết thúc bài nghe.
 Mọi hướng dẫn cho thí sinh (bằng tiếng Anh) đã có trong bài nghe.

Part 1. For question 1-5, listen to a recording about the way to become a successful
entrepreneur and decide whether the following statements are true (T) or false (F).
Write your answers in the corresponding numbered boxes provided. (10 pts)
1. To-be-entrepreneurs need to examine what makes people unhappy in the hope of
success.
2. Consumer society has solved every problem of modern life.
3. Frustration is a limited source of inspiration for entrepreneurs.
4. Taking the initiative to understand and master human distress can result in substantial
profits.
5. An effective handling of practicalities will definitely lead to a successful business.
Your answers
1. 2. 3. 4. 5.

Part 2. For questions 6-10, you will listen to a lecture about the British Isles.
Answer the following questions with NO MORE THAN THREE WORDS taken
from the recording. Write your answers in the space provided. (10 pts)
6. According to the lecturer, what is the British Isles in geographical terms?
………………………………………………………………………………………………………….…….

Page 1/23
7. What is the ‘United Kingdom of Great Britain and Northern Ireland’ in political
terms?
………………………………………………………………………………………………………….…….

8. What phrase is used to describe the political role of the four countries within the
United Kingdom?
………………………………………………………………………………………………………….…….

9. Besides its baffling geography and politics, what aspect of the British Isles is also
discussed?
………………………………………………………………………………………………………….…….

10. What presumably leads to the fact that the name “the British Commonwealth” fell
into disuse?
………………………………………………………………………………………………………….…….

Part 3. For questions 11–15, you will a conversation between two friends who have
just attended a workshop for people who want to increase their self-esteem.
Choose the answer (A, B, C or D) which fits best according to what you hear.
Write your answers in the corresponding boxes provided. (10 pts)
11. Following the workshop, both speakers seem to agree that _______.
A. they are looking forward to applying what they learnt
B. they’re likely to become unrecognizable to their family and friends
C. the woman’s therapist would have disagreed with a lot of what they heard
D. improving self-esteem isn’t something you can learn to do
12. What view is expressed about fear?
A. that some forms of fear are easier to overcome than others
B. that to develop self-esteem you must first be fearless
C. that being afraid forces you to take control of situations
D. that it is something we must learn to face up to
13. In the man’s opinion, _______.
A. being unduly apologetic may let others take advantage of you
B. being fair to others will boost your feelings of self-worth
C. apologizing always increases your feelings of self-doubt
D. there’s no point in saying sorry to people that you don’t respect
Page 2/23
14. What does the woman suggest about self-criticism?
A. It’s always harmful even if you rephrase it in a more positive way.
B. It can have a debilitating effect on your self-image.
C. Being self-critical is a habit she feels confident she can get out of.
D. It’s beneficial to be aware of our faults before others point them out.
15. According to the speakers, what did the workshops leader imply about our
accomplished friends?
A. Comparing ourselves to others is bound to leave us feeling worthless.
B. We should end our relationship with people who we think are too good for us.
C. Our relationship with them is proof of our own merit.
D. Only when we are confident will we see our friends’ real faults.
Your answers
11. 12. 13. 14. 15.
Part 4. For questions 16-25, listen to a piece of news about lithium-ion batteries
and complete the summary using NO MORE THAN THREE WORDS for each
gap. Write your answers in the space provided. (20 pts)
1991 marked the debut of lithium-ion batteries on the market as a component of
a(n) (16) _____________________. Due to their lightness, power and rechargeability,
they have become a fixture in several everyday devices and are now deemed (17)
_____________________.
These batteries were once overshadowed by (18) _____________________ in
the 1880s before catching on again with the present-day trend of decarbonization.
Indeed, lithium-ion batteries play an integral part in a low-carbon future, as they can be
a storage of (19) _____________________ regardless of weather conditions, not to
mention their durability and dependability. They have been serving their role in some
organizations and residential areas, and possibly in future homes where they take the
form of (20) _____________________.
Nevertheless, challenges are abundant. Millions of (21)
_____________________ will be necessitated for a rising number of electric vehicles,
while the extraction of lithium, which requires a substantial supply of groundwater and
subsequent evaporation, can cause water shortage and (22) _____________________.

Page 3/23
Sourcing cobalt, the major material for lithium-ion batteries, will also be challenging,
especially when most of this substance is located in the Democratic Republic of Congo,
a country with poor (23) _____________________ conditions. A low battery-recycling
rate is another concern.
Despite such difficulties, the field of battery research has been witnessing
exciting developments. Alternatives to cobalt are now being developed. In Singapore,
(24) _____________________ can bolster the battery recycling rate. Solid-state lithium
batteries will also become more (25) _____________________ with shorter charging
time and numerous charge cycles.
B. LEXICO AND GRAMMAR (30 pts)
Part 1. Choose the answer A, B, C, or D that best completes each of the following
sentences. Write your answers in the corresponding numbered boxes. (20 pts)
1. Judges already have substantial latitude to limit ______ arguments that might
mislead jurors; they could use it more often.
A. extraneous B. precarious C. fortuitous D. convivial
2. The athlete expects to return to the track after a two-month ______ of injuries.
A. sabbatical B. space C. hiatus D. interval
3. There may be an announcement about this tomorrow - or not, ______.
A. to all intents and purposes B. as the case maybe
C. by its nature D. by its own account
4. Many people no longer trust their own ______ memories and commit every detail of
their lives to some digital device or other and are completely lost without it.
A. controllable B. susceptible C. vulnerable D. fallible
5. ______ by the press for his buffoonish behavior, the candidate continued his run for
the presidency, undeterred.
A. Libeled B. Slandered C. Lampooned D. Hounded
6. Owing and living in a freestanding house is still a goal of young adults, ______
earlier generations.
A. as did B. as it was of
C. like that of D. so have
7. The nuclear bomb producing radioactive wastes was activated as someone had
______ with the buttons.

Page 4/23
A. tampered B. soothed C. disembarked D. aggravated
8. We do not have a secretary ______, but we do have a student who comes in to do a
bit of filing.
A. as such B. the least bit
C. whatsoever D. little more
9. It is ______ difficult to reverse the damage done to the ozone layer.
A. stupendously B. loquaciously C. coaxingly D. fiendishly
10. Paradoxically, Harry’s journey to tour around Vietnam did not ______ due to floods
in the central regions.
A. pull off B. pan out C. zip through D. knuckle down
11. In ______ of cultural legacy, more museums have been erected and several
campaigns have been launched into raising citizen’s awareness.
A. facilitation B. furtherance C. promotion D. keeping
12. These graduates are considered to be the ______ of the crop and can get jobs
wherever they want to.
A. wheat B. milk C. cream D. caviar
13. A new generation of performers, ______ those who by now had become household
name, honed their skills before following the same path onto television.
A. no less talented than B. together with talented with
C. along with talented with D. having been more talented with
14. A new government report delivers ______ warnings about climate change and its
impacts on the planet.
A. ominous B. dire C. fateful D. acute
15. Despite the harsh flow of the stream, she ______ her way through the water.
A. powered B. struck C. directed D. scrambled
16. A large proportion of the households in this area is ______ to the internet thanks to
a generous foreign donor.
A. linked with B. wired up C. hooked up D. crossed with
17. Facebook's shares are ______ after the company launched a new Instagram feature
that will compete with TikTok in the US.
A. holding her thumb B. on high skies
Page 5/23
C. Coming up roses D. on a tear
18. He ______ safety goggles, but he wasn’t and, as a result, the hot steel badly
damaged his eyes.
A. could have been wearing B. must have been wearing
C. should be wearing D. ought to have been wearing
19. Unlike his friends who also rose to stardom when they were still teenagers, Andy
didn’t have any ______ but became even more modest.
A. airs and graces B. beer and skittles
C. cock and bull D. nudge and wink
20. The spokesperson said the information campaign was a ______ to hide the most
regressive tax in history.
A. red tape B. fig leaf C. book cover D. witch cloak
Your answers
1. 2. 3. 4. 5. 6. 7. 8. 9. 10.
11. 12. 13. 14. 15. 16. 17. 18. 19. 20.

Part 2. Give the correct form of each bracketed word in each sentence. Write your
answers in the space provided. (10 pts)
1. Draconian measures to contain the pandemic notwithstanding, the
…………………………………… of Covid-19 ensued, battering the economy.
(SURGE)
2. This infamous enterprise was built of thousands of bodies of workers who were
…………………………………… fired when they asked for even the fundamentals
like a shelter or food. (CEREMONY)
3. Fearing that the recent changes will have ……………………………………
implications for the food industry, the company spared no effort to forestall any
problems. (REACH)
4. The majority of infected people are …………………………………… and unaware
of their condition. (SYMPTOM)
5. Thanks to months of …………………………………… research by the film
producers, the enthronement ceremony of Qianlong in “The story of Ruyi” was highly
commended for being historically accurate. (PAIN)
Page 6/23
6. The death of Bruno and Shmuel in “The boy in the striped pajamas” really tugs at
your …………………………………… (HEART)
7. Border …………………………………… between the two countries is one of the
issues that receive the most public attention. (LIMIT)
8. When I first met your mother, I just thought she might be 40 or
…………………………………… because she looked so young. (THERE)
9. When demand for a product …………………………………… its supply, the price
of that product will increase. (STRIP)
10. Rats must be …………………………………… from a building or they will cause
disease. (TERM)
III. READING (60 pts)
Part 1. Read the text below and think of the word which best fits each space. Use
only ONE WORD in each space. Write your answers in the space provided.
The first post-lockdown crops of the land army have been harvested. The food –
chard, spinach, lettuce and radish – is being parceled out to the local shops, market
stalls and those in (1) __________. Now the volunteer labor force has its (2)
__________ on a new goal: a land-use revolution that will make UK farming more
nature friendly, plant-based and resilient to (3) __________ shocks.
At Machynlleth, a bucolic town on the southern fringe of Snowdonia, the
recently formed Planna Fwyd! (Plant Food!) movement is encouraging sheep farmers to
diversify (4) __________ vegetable production as their ancestors did. Teams of
volunteers have sown (5) __________ of potatoes and, once or twice a week, they now
fan across the slopes to tend gooseberry bushes, peas and squash. Others distribute seed
packets to (6) __________ families and run online classes on how to grow plants at
home.
“If the whole coronavirus experience has taught us anything, it is (7) __________
we should be more self-sufficient. It was terrifying seeing the (8) __________ shop
shelves,” said Chris Higgins, a retired academic who gets as much (9) __________ as
he gives from the voluntary work. “It’s very enriching. Growing and cooking food and
working together is a great way of (10) __________ with the local community and
nature at the same time.”

Page 7/23
(adapted from “Britain beyond lockdown: can we make more space for nature?” by
Jonathan Watts, the Guardian)
Part 2. Read the following passage and choose the best answer. Write your
answers in the corresponding numbered boxes.
Using video gaming in education
It has become conventional wisdom that spending too much time playing video
games has a detrimental effect on children’s studies and their social development.
However, some educationalists are now questioning this theory and are using video
games as effective educational tools thus bridging the gap between recreational and
educational activities.
Due to the sophisticated nature of today’s games, teachers are able to justify the
inclusion of video and online games for many pedagogical reasons. There may, for
example, be sociological, psychological, and ethical implications built into the
gameplay. Harvey Edwards, who teaches IT classes in London, was one such educator
who decided to use video games in his lessons. To do this, he chose Minecraft, an
online game in which players create and develop imaginary worlds. He was somewhat
uneasy about attempting such an unconventional approach, not because of some
students’ unfamiliarity with the game but rather due to them not being able to make
sense of what he was trying to do with it. He worried that it might interfere with his
learners’ focus, but he couldn’t have been more surprised by the results.
Minecraft is an example of a ‘sandbox game’, in which gamers roam around and
change a virtual world at will. Instead of having to pass through numbered levels to
reach certain places, there’s full access from start to finish. The original version can be
adapted to control which characters and content are left in. Each student can then be
allocated tasks – such as house-building, locating items or problem-solving – which
they must complete within the game. Elements of more general skills can be subtly
incorporated into the lessons, such as online politeness and safety, teamwork and
resolving differences. Edwards feels that presenting such lessons in the context of a
game students probably already know and enjoy enables him to connect with them at
greater depth, and in more motivational ways.

Page 8/23
Bolstered by his success, Edwards introduced his approach to another school
nearby. He recalls that the first couple of sessions didn’t live up to his expectations.
Those who had played Minecraft before were keen for others to adopt their own style of
play. Unsurprisingly, this assortment of styles and opinions as to how the game should
proceed were far from harmonious. However, the sessions rapidly transformed into
something more cohesive, with the learners driving the change. With minimal teacher
input, they set about choosing leaders and established several teams, each with its own
clearly-defined role. These teams, now party to clear common goals, willingly
cooperated to ensure that their newborn world flourished, even when faced with the
toughest of challenges.
‘Human’ inhabitants in a Minecraft ‘society’ are very primitive and wander
around the imaginary world, waiting for guidance from players. [A] This dynamic bears
a resemblance to traditional education, an observation highlighted by Martina Williams,
one of the leaders of the group. [B] ‘Through the game, we were no longer passive
learners in the classroom, being told what and how to learn, but active participants in
our own society. [C] The leaders, meanwhile, had a vision for their virtual world as a
whole, encouraging everyone to play their part in achieving the group’s goals. [D]
Through creating their own characters and using these to build their own ‘world’,
students will have gained some experiential understanding of societal structure and how
communities work.
But not everyone is convinced by video games’ potential academic value. While
many progressive commentators cite extensive evidence to maintain that video games
encourage collaboration and build problem-solving skills, more traditional factions
continue to insist they are a distraction that do not merit inclusion in any curriculum.
Even less evangelical cynics, who may grudgingly acknowledge games have some
educational benefit, assert that this is only the case in the hands of creative educators.
However, the accusation most often levelled at video games is that they detract from the
social aspect of the classroom, particularly taking part in discussions. Dr Helen
Conway, an educational researcher, argues that video games can be used to promote
social activities. ‘Students become animated talking about the game and how to
improve their gameplaying and problem-solving skills,’ she says. ‘I find it strange, this
Page 9/23
image that many people have,’ Conway says. ‘Children are often totally detached from
their peers when undertaking more traditional activities, like reading books, but we
never suggest that books are harmful because they’re a solitary experience.
1. The first time Edwards used a game in his classes, he was ______.
A. convinced that learners would realise why he wanted them to play it
B. convinced that learners would see the reasons for playing it
C. anxious that he had chosen the wrong one for learners to play
D. sure that his reasons for getting learners to play it were valid
2. The writer suggests that Minecraft is a good choice of educational game because
______.
A. any number of learners can use it simultaneously
B. teachers can remove any inappropriate material
C. gamers can create educative tasks whilst playing it
D. players can develop their skills in a step-by-step way
3. Which of the following words in the fourth paragraph is used to convey a feeling of
approval?
A. keen B. harmonious C. driving D. newborn
4. In the fifth paragraph, the writer draws a comparison between a Minecraft ‘society’
and ______.
A. relationships within the group as they played
B. the way in which countries organise themselves
C. typical students in a school environment
D. how leadership operates in different situations
5. In the sixth paragraph, the writer feels that critics of video games in education
______.
A. are unwilling to admit that using them in class has benefits
B. make accurate observations about teachers who use them
C. use flawed research to support their objections to using them
D. acknowledge the drawbacks of more traditional teaching methods
6. The phrase ‘this image’ in the sixth paragraph refers to ______.
A. people who criticise gaming in education
Page 10/23
B. students discussing a game in a group
C. a group of students reading individually
D. a solitary player absorbed in a game
7. Where does this sentence belong to in the fifth paragraph?
Each group member had ideas as to how their function should develop.
A. [A] B. [B] C. [C] D. [D]
8. The word ‘subtly’ in the third paragraph is closest in meaning to ______.
A. intricately B. ingeniously C. ingenuously D. haphazardly
9. The word ‘grudgingly’ in the sixth paragraph is closest in meaning to ______.
A. gleefully B. vivaciously C. genially D. reluctantly
10. Which of the following best describes the author's attitude towards the application
of gaming into education?
A. supportive B. neutral C. cynical D. satirical
Your answers
1. 2. 3. 4. 5. 6. 7. 8. 9. 10.

Part 3. Read the passage and do the tasks that follow.


Trends in the Indian fashion and textile industries
During the 1950s, the Indian fashion scene was exciting, stylish and very
graceful. There were no celebrity designers or models, nor were there any labels that
were widely recognized. The value of a garment was judged by its style and fabric
rather than by who made it. It was regarded as perfectly acceptable, even for high-
society women, to approach an unknown tailor who could make a garment for a few
rupees, providing the perfect fit, finish and style. They were proud of getting a bargain,
and of giving their own name to the end result.
The 1960s was an era full of mischievousness and celebration in the arts, music
and cinema. The period was characterized by freedom from restrictions and, in the
fashion world, an acceptance of innovative types of material such as plastic and coated
polyester. Tight-fitting kurtas and churidars and high coiffures were a trend among
women.
The following decade witnessed an increase in the export of traditional materials,

Page 11/23
and the arrival in India of international fashion. Synthetics became trendy, and the disco
culture affected the fashion scene.
It was in the early 80s when the first fashion store ‘Ravissant’ opened in
Mumbai. At that time garments were retailed for a four-figure price tag. American
designers like Calvin Klein became popular. In India too, contours became more
masculine, and even the salwar kameez was designed with shoulder pads.
With the evolution of designer stores came the culture of designer fashion, along
with its hefty price tags. Whatever a garment was like, consumers were convinced that a
higher price tag signified elegant designer fashion, so garments were sold at
unbelievable prices. Meanwhile, designers decided to get themselves noticed by making
showy outfits and associating with the right celebrities. Soon, fashion shows became
competitive, each designer attempting to out-do the other in theme, guest list and media
coverage.
In the last decade of the millennium, the market shrank and ethnic wear made a
comeback. During the recession, there was a push to sell at any cost. With fierce
competition, the inevitable occurred: the once hefty price tags began their downward
journey, and the fashion-show industry followed suit. However, the liveliness of the
Indian fashion scene had not ended - it had merely reached a stable level.
At the beginning of the 21st century, with new designers and models, and more
sensible designs, the fashion industry accelerated once again. As far as the global
fashion industry is concerned, Indian ethnic designs and materials are currently in
demand from fashion houses and garment manufacturers. India is the third largest
producer of cotton, the second largest producer of silk, and the fifth largest producer of
man-made fibers in the world.
The Indian garment and fabric industries have many fundamental advantages, in
terms of a cheaper, skilled work force, cost-effective production, raw materials,
flexibility, and a wide range of designs with sequins, beadwork, and embroidery. In
addition, that India provides garments to international fashion houses at competitive
prices, with a shorter lead time, and an effective monopoly on certain designs, is
accepted the whole world over. India has always been regarded as the default source in
the embroidered garments segment, but changes in the rate of exchange between the
Page 12/23
rupee and the dollar have further depressed prices, thereby attracting more buyers. So
the international fashion houses walk away with customized goods, and craftwork is
sold at very low rates.
As far as the fabric market is concerned, the range available in India can attract
as well as confuse the buyer. Much of the production takes place in the small town of
Chapa in the eastern state of Bihar, a name one might never have heard of. Here fabric-
making is a family industry; the range and quality of raw silks churned out here belie
the crude production methods and equipment. Surat in Gujarat, is the supplier of an
amazing set of jacquards, moss crepes and georgette sheers - all fabrics in high demand.
Another Indian fabric design that has been adopted by the fashion industry is the
‘Madras check’, originally utilized for the universal lungi, a simple lower-body wrap
worn in southern India. This design has now found its way on to bandannas, blouses,
home furnishings and almost anything one can think of.
Ethnic Indian designs with batik and hand-embroidered motifs have also become
popular across the world. Decorative bead work is another product in demand in the
international market. Beads are used to prepare accessory items like belts and bags, and
beadwork is now available for haute couture evening wear too.
Questions 1-7: Complete the notes below. Choose ONE WORD ONLY from the
passage for each answer. Write your answers in the corresponding numbered
boxes provided.
Indian fashion: 1950-2000
1950s
 No well-known designers, models or (1) ...................
 Elegant clothing cost little
 Women were pleased to get clothes for a (2) ................... price
1960s
 New materials, e.g. (3) ................... and polyester
 Fitted clothing and tall hairstyles
1970s
 Overseas sales of (4) ................... fabrics rose
 Influence of international fashion
Page 13/23
1980s
 Opening of fashion store in Mumbai
 Popularity of American designers
 Clothing had a (5) ................... shape
 Designers tried to attract attention by presenting (6) ................... clothes and
mixing with stars
1990s
 Fall in demand for expensive fashion wear
 Return to (7) ................... clothing
Questions 8-13: Do the following statements agree with the views of the writer in
the passage? Write your answers in the corresponding numbered box provided.
TRUE (T) if the statement agrees with the writer
FALSE (F) if the statement contradicts the writer
NOT GIVEN (NG) if it is impossible to say what the writer thinks about this
8. At the start of the 21st century, key elements in the Indian fashion industry changed.
9. India now exports more than half of the cotton it produces.
10. Conditions in India are generally well suited to the manufacture of clothing.
11. Indian clothing exports have suffered from changes in the value of its currency.
12. Modern machinery accounts for the high quality of Chapa’s silk.
13. Some types of Indian craftwork which are internationally popular had humble
origins.
Your answers
1. 2. 3. 4. 5.

6. 7. 8. 9. 10.

11. 12. 13.


Part 4. In the passage below, seven paragraphs have been removed. Read the
passage and choose from paragraphs A-H the one which fits each gap. There is
ONE extra paragraph which you do not need to use. Write your answers in the
corresponding numbered boxes provided.
Page 14/23
IRIDOLOGY
Iridologists claim that there is a ‘map’ of the whole body in the iris of each eye and that
it provides a guide to various disorders and their treatments. The eye has long been
regarded as the ‘mirror of the soul’. From earliest antiquity, it has been believed that it
is one of the best indicators of spiritual condition, personality, temperament and
romantic feelings.
1.
For this reason, its condition is significant in assessing the general health of a patient.
However, long before examination of the retina was recognised as important, iridology
existed. That is, the science of diagnosing bodily conditions through study of the iris,
the coloured part of the eye. The history and origins of iridology have often been
disputed. It is difficult to substantiate its roots in ancient Asia and Africa since almost
the entire population is brown-eyed. It stands, however, as a European phenomenon,
dating back to the mid-nineteenth century.
2.
Iridology was born in von Peczely's garden in Budapest when he was just eleven.
During a fight with a wild owl, the bird's leg was broken. While glaring at its captor, the
bird developed a black line in its iris. The boy was able to notice developments in the
bird's eye as its leg healed. Slowly, the black line shrank, and when the leg had
completely healed, there was the merest trace of a mark left in the eye.
3.
As a medical student in Vienna, he was imprisoned, but found an outlet at this time by
studying the eyes. of fellow prisoners. Time and again, he noticed a correlation between
markings of the iris and known medical problems. He then firmly decided to devote
himself to serious study of the eye, once free.
4.
Meanwhile, a Lutheran minister, Pastor Nils Liljequist, was also examining irises in
Sweden. While suffering from a fever, he was treated with quinine, which led to a
change in the colour of his iris from blue to yellowish-green. It was this that led to his

Page 15/23
study of the relationship between eye colour and the use of medical drugs, particularly
the heavy metals which were so widely used then.
5.
It is the first decades of the twentieth century that are particularly interesting. American
Henry Lindlahr, working before iris photography was perfected, used his own eyes as a
basis for his drawings. He experimented with diet, fasting and various medicines. He
used the iris as a means of diagnosis before anyone else, cataloguing inflammations and
organic dysfunctions both in himself and in his patients.
6.
Iridological research has emphasized the fact that the human being is a whole
interrelated organism, which must be viewed as such when undergoing treatment. The
iridologist has the entire medical state and a great deal of the patient's medical history in
front of him when looking at the patient's iris, so he cannot view the patient in terms of
a single condition.
7.
Nevertheless, iris diagnosis remains a scientifically based tool despite the indifference
shown by orthodox practitioners. Through increased funding into research or even
greater public awareness, it could well be used as an inexpensive, yet accurate form of
diagnosis that could save time, money and perhaps lives.
MISSING PARAGRAPHS
A. Upon release, von Peczely quickly became convinced that he would be able to
demonstrate a relationship between the organs and limbs of the body and several parts
of the iris. The iris chart used by iridologists today is essentially the same as the one on
which he painstakingly mapped his findings.
B. One man at this time was outstanding in his work in this field. Hungarian
physician, Dr Ignatz von Peczely. deserves credit as the father of iridology. He was a
man with remarkable powers of observation, original thought and bravery. Throughout
his life, he faced opposition, dying in comparative obscurity. It is only now that he is
beginning to be appreciated.

Page 16/23
C. This led him to the realization that the iris could reveal the internal workings of
the body and so there would be no need for painful, dangerous exploratory operations.
In his conclusions, he noted that beneficial changes in the body were brought about by
homeopathic preparations which were not trapped in the body's tissues. Unlike
conventional drugs, colour traces were not left in the iris after use.
D. The youth later took up the controversial study of homeopathy and was so
successful at treating neighbours that there was great hostility from the medical
profession. He was forbidden to continue practicing and only did so under the guise of
orthodox medicine after he had qualified as a doctor.
E. Iridology was practically forgotten at this time until revived by homeopaths and
naturopaths, who managed to re-establish the science. The cold shoulder that von
Peczely was given when he first started practicing homeopathy was no less demeaning
than the one it received from orthodox medicine in the twentieth century.
F. He published and circulated a book on his findings. Most of Europe ignored this
and iridology in general. This may well have been because of its background and von
Peczely's interest in homeopathy. However, this was not the case everywhere and
towards the end of the century naturopathic workers in Germany and the USA
incorporated the findings of von Peczely with Liljequist's observations.
G. There has been scientific study of the eye as a guide to physical well-being quite
apart from this interest in a as an indicator of spiritual state. The ophthalmologist
examines the back of the eye, the retina, through the pupil, the black hole at the front of
the eye. The only place where nerve endings and blood vessels can be viewed directly is
through the retina.
H. It is for this reason that iridology is growing in popularity, and practitioners of
alternative therapies find this holistic attitude particularly encouraging. However, it
could be that this emphasis is the reason why iris diagnosis has yet to be taken more
seriously by the orthodox doctor.
Part 5. Read the article about being a film and television drama extra, identify which
section A–D each of the following is mentioned. Write ONE letter A–D in the
corresponding numbered space provided. Each letter may be used more than once.
THE LIFE OF AN EXTRA
Page 17/23
Most films and TV dramas require extras, those people we glimpse in the background
behind the main actors. Mike Jones describes what it’s like to be an extra.
A. During a break in the filming of a TV drama, I gravitated towards the table laden
with hot coffee and biscuits. As I reached it, however, I was duly informed that is was
reserved for the “talent” – the real actors – and was directed towards a rickety table, on
which sat an urn of hot water, some sugar packets, and nothing else. I tell this tale, not
just to grumble, but because it sums up the stark divide between the cast and the little
people in the background. Referring to us on set by our technical name of “supporting
artistes” is meant to make us feel more important, I suppose. “Walking background”
and “human props” are common, faintly humorous labels for us, but they’re fitting.
Extras aren’t supposed to say anything during a take; we aren’t paid to talk. Nor are we
allowed to talk between takes when everything is being reset. A gentle murmur of
conversation will inevitably well up among some groups, at which point one of the
assistant directors will immediately bellow for silence.
B. There are other non-negotiables, and failing to obey them could result in you
being fired and blacklisted from the industry. Your contract specifically orders you not
to talk to any of the actors. On one production, I was introduced to the lead actor and
told what my role would be in that particular scene. I smiled at him in a mild attempt at
camaraderie and he started right through me. Then, the expression on the face of the
lead actress made it clear she had decided not to make an effort with me. I knew I was
only an extra, but they might have at least feigned some interest. So why go through
this? Well, when I first signed up, I, like many others I know, saw it as an interesting
way of earning a second income. The far from generous fees, however, soon meant I
regarded it as little more than a hobby, and had I had a family, I probably wouldn’t have
been able to do it.
C. Occasionally, you’re picked out to play a more substantial part in one scene, and
you feel as if maybe the professional/supporting artist divide isn’t so insurmountable
after all. Then, when you’re finally released to go home at 2 a.m., after walking up and
down some stretch of pavement 50 times, and you realize that the actors still have hours
more to go, their job no longer seems quite so glamorous or privileged. Actually, night
shoots tend to be the worst, although the extra money you receive almost makes up for
Page 18/23
it. Once on the set of a blockbuster, a mix-up in the costume department resulted in me
spending two nights in an aircraft hangar, drinking terrible coffee and reading books –
and getting paid for it. Meanwhile, the other extras all ran around outside in the freezing
drizzle for an action scene. Every time they trudged back in from a take, their very
visible fatigue and discomfort made me feel somewhat fraudulent, aware as I was that
we were all being remunerated at the same rate for our efforts.
D. One of the bonuses of being an extra is the free catering. Getting up at 4 a.m.
isn’t so bad when you can go straight in for a large breakfast, and a filling lunch is
always provided, though obviously aren’t allowed to eat anything before the “talent”
and the crew. For period dramas, you will also get a free haircut, although this can
sometimes make things awkward. On one shoot, I had to phone in “stick” to my day
job, and then had trouble explaining to my boss the next morning why I now had a
1950s-style haircut. Another plus may be seeing yourself on screen, usually as a blurry
outline to one side or a tiny figure in the distance. That’s what extras do: blend into the
background and not divert attention from the main characters. Would I encourage
anyone to sample this life? Hardly, if the aim is to make money or get into the industry.
If, however, you like the idea of dressing up, with the possibility of glimpsing yourself
on the big screen for a few seconds, well, maybe.
In which section does the writer Your answers
describe how one advantage of the job didn’t go according to plan? 1. ________
mention experiencing a sense of guilt? 2. ________
point out the possible consequences of breaking rules? 3. ________
offer advice to people thinking of applying to work as an extra? 4. ________
refer to times when the gap between actors and extras appears to narrow? 5. ________
acknowledge how appropriate certain terms are? 6. ________
explain a common motivation for taking on work as an extra? 7. ________
mention gaining insights into hardships that actors experience? 8. ________
explain the purpose of an anecdote? 9. ________
refer to an incident when he was disappointed by people’s behavior? 10. ________

Page 19/23
D. WRITING (60 pts)
Part 1. Read the following extract and use your own words to summarize it. Your
summary should be about 120 words long. (15 pts)
Teleworking – working remotely from an office – is said to have many benefits
for the environment. Whether you like it or not, it is true to say that work is no longer
dependent on geography and this opens up a range of opportunities for working in new
ways and environments.
Half-time telecommuting could reduce carbon emissions by over 51 million
metric tons a year – the equivalent of taking all of New York’s commuters off the road.
Additional carbon footprint savings will come from reduced office energy, roadway
repairs, urban heating, office construction, business travel, paper usage (as electronic
documents replace paper). Although energy utilization will continue to grow as we
expand our industry and improve our standard of living, efficient use of energy will
always be of prime importance. By telecommuting to work instead of using more
conventional methods, there is a great potential to save energy. The three major areas
where energy can be conserved are vehicle-related materials and resources, highway-
related materials and resources, and work-related materials and resources.
A tremendous amount of energy is required to produce transportation equipment
such as automobiles, buses, trains and jet aircraft. If telecommuting is promoted, there
will be less use of this equipment and less energy will be required for production,
maintenance and repair of this equipment. Fuel resources and gasses needed to operate
this equipment will be reduced as well. The building and repair of highways and
maintenance requires a large consumption of energy, not only in the operation of the
highway construction and repair equipment but also in the manufacture and
transportation of the required materials. An increase in the percentage of people
telecommuting to work will decrease the need for expanded highways and associated
road maintenance. The first two areas are related to getting to work.
.......................................................................................................................................................................................................................................................

.......................................................................................................................................................................................................................................................

................................................................................................................................................................................. ......................................................................

.......................................................................................................................................................................................................................................................

.......................................................................................................................................................................................................................................................

Page 20/23
.......................................................................................................................................................................................................................................................

.......................................................................................................................................................................................................................................................

.......................................................................................................................................................................................................................................................

................................................................................................................................................................................................................................. ......................

.......................................................................................................................................................................................................................................................

.......................................................................................................................................................................................................................................................

.......................................................................................................................................................................................................................................................

.......................................................................................................................................................................................................................................................

.......................................................................................................................................................................................................................................................

.......................................................................................................................................................................................................................................................

.......................................................................................................................................................................................................................................................

.......................................................................................................................................................................................................................................................

Part 2. The table below shows the number of motorbikes sold by four motorbike
manufacturers in Fantasia between 2018 and 2021.
Summarize the information by selecting and reporting the main features, and
make comparisons where relevant. You should write about 150 words. (15 pts)
Year
2018 2019 2020 2021
Brand
Honda 15,250 13,570 14,000 16,150
Yamaha 5,300 6,700 7,050 6,100
SYM 1,200 1,580 1,600 1,840
Suzuki 2,300 3,400 4,000 4,500

.......................................................................................................................................................................................................................................................

.......................................................................................................................................................................................................................................................

.......................................................................................................................................................................................................................................................

.......................................................................................................................................................................................................................................................

.......................................................................................................................................................................................................................................................

.......................................................................................................................................................................................................................................................

.......................................................................................................................................................................................................................................................

.......................................................................................................................................................................................................................................................

.......................................................................................................................................................................................................................................................

........................................................................................................................................................................... ............................................................................

.......................................................................................................................................................................................................................................................

Page 21/23
.......................................................................................................................................................................................................................................................

................................................................................................................................................................................................... ....................................................

.......................................................................................................................................................................................................................................................

.......................................................................................................................................................................................................................................................

.......................................................................................................................................................................................................................................................

.......................................................................................................................................................................................................................................................

.......................................................................................................................................................................................................................................................

................................................................................................................................................................................................................................................... ....

.......................................................................................................................................................................................................................................................

Part 3. Essay writing (30 pts)


Some people say that all citizens should be given complete freedom to express their
personal opinions and concerns about every social problem on the Internet, while
others say that this may worsen the situation in reality. Discuss and give your own
opinions.
Give reasons for your answer and include any relevant examples from your own
experience or knowledge. Write at least 350 words.
.......................................................................................................................................................................................................................................................

.......................................................................................................................................................................................................................................................

.......................................................................................................................................................................................................................................................

.......................................................................................................................................................................................................................................................

.......................................................................................................................................................................................................................................................

.......................................................................................................................................................................................................................................................

.......................................................................................................................................................................................................................................................

.......................................................................................................................................................................................................................................................

.......................................................................................................................................................................................................................................................

.......................................................................................................................................................................................................................................................

.......................................................................................................................................................................................................................................................

.......................................................................................................................................................................................................................................................

.......................................................................................................................................................................................................................................................

.......................................................................................................................................................................................................................................................

.......................................................................................................................................................................................................................................................

.......................................................................................................................................................................................................................................................

.......................................................................................................................................................................................................................................................

.......................................................................................................................................................................................................................................................

.......................................................................................................................................................................................................................................................

Page 22/23
.......................................................................................................................................................................................................................................................

.......................................................................................................................................................................................................................................................

.......................................................................................................................................................................................................................................................

.......................................................................................................................................................................................................................................................

.......................................................................................................................................................................................................................................................

.......................................................................................................................................................................................................................................................

.......................................................................................................................................................................................................................................................

.......................................................................................................................................................................................................................................................

.......................................................................................................................................................................................................................................................

.......................................................................................................................................................................................................................................................

.......................................................................................................................................................................................................................................................

.......................................................................................................................................................................................................................................................

.......................................................................................................................................................................................................................................................

.......................................................................................................................................................................................................................................................

.......................................................................................................................................................................................................................................................

.......................................................................................................................................................................................................................................................

.......................................................................................................................................................................................................................................................

.......................................................................................................................................................................................................................................................

.......................................................................................................................................................................................................................................................

.......................................................................................................................................................................................................................................................

.......................................................................................................................................................................................................................................................

.......................................................................................................................................................................................................................................................

.......................................................................................................................................................................................................................................................

.......................................................................................................................................................................................................................................................

.......................................................................................................................................................................................................................................................

.......................................................................................................................................................................................................................................................

-THE END-

Page 23/23
TRƯỜNG THPT CHUYÊN HUỲNH MẪN ĐẠT
ĐỀ THI ĐỀ XUẤT DUYÊN HẢI VÀ ĐỒNG BẰNG BẮC BỘ LẦN THỨ XIII
MÔN: TIẾNG ANH 11
(Đề thi gồm 23 trang)
A. LISTENING (50 points)
Part 1. For questions 1-5, listen to a talk about office life and decide whether these
statements are True (T), False (F) or Not Given (NG). Write your answers in the
corresponding numbered boxes provided. (10 points)
1. Antony Slumbers believed that employee productivity was the reason for the
establishment of the office.
2. Open-plan offices were characterized by constant distraction and work inefficiency.
3. Unexpected meetings have yet to be proven to foster sudden inspiration or recognition.
4. Lack of monitoring from bosses is assumed to obstruct straightforward exchange.
5. Some research has pointed out that firms opting out of rigidity tend to draw the best
workforce.
Your answers:
1. 2. 3. 4. 5.

Part 2. For questions 6-10, listen to a report on Black Fungus in India and answer the
questions. Write NO MORE THAN FOUR WORDS taken from the recording for each
answer in the corresponding numbered boxes provided. (10 points)
6. Which form do the moles causing black fungal infection take?
7. Among Covid 19 patients, who are most vulnerable to black fungal infection?
8. Which problem with the nose may a patient experience when he or she suffers from black
fungal infection?
9. According to top Indian doctors, what is the culprit behind the rise of infections?
10. Which treatment do patients receive when the infection is identified on time?
Your answers:
6. 7. 8. 9. 10.

Page 1 of 23
Part 3. For questions 11-15, listen to part of a radio interview with a member of a
language revival action group and choose the answer (A, B, C or D) which fits best
according to what you hear. Write your answers in the corresponding numbered boxes
provided. (10 points)
11. The bill that was recently passed by the Scottish Parliament _______
A. states that Gaelic is the official language of Scotland.
B. enables Gaelic to be taught in Scottish schools.
C. has benefited teaching in Scottish schools.
D. represents a victory for Philip McNair's action group.
12. One of the action group's aims is to _______
A. encourage family members to speak Gaelic at home.
B. provide teachers with the incentive and the means to teach in Gaelic.
C. recruit and train more language teachers in secondary schools.
D. provide Scottish schools with equipment for language learning.
13. McNair attributes the revival of interest in Gaelic to _______
A. the success of a similar language revival campaign in Wales.
B. fears that Scottish Gaelic would suffer the same fate as Manx.
C. the fact that the Isle of Man now has several hundred Gaelic speakers.
D. the fact that Gaelic had been officially declared a dead language.
14. According to McNair, an important factor in achieving the group's goal is _______
A. the view that Gaelic is part of Scotland's cultural heritage.
B. the political autonomy that Scotland has finally won for itself.
C. the necessary backing to put ideas into practice.
D. the pride people take in having a strong cultural identity.
15. How does he regard the latest developments?
A. optimistically
B. stoically
C. impassively
D. fanatically
Your answers:
11. 12. 13. 14. 15.

Page 2 of 23
Part 4. For questions 16-25, listen to a talk about Asian unicorn start-ups and supply the
blanks with the missing information. Write NO MORE THAN THREE WORDS taken
from the recording for each answer in the space provided. (20 points)
UNICORN STARTUPS
16. Bain & Company report has attributed the boom of unicorns in the region to the increase
in private equity investment and _______.
17. _______ is the term used to describe a startup company valued at over $100 billion.
18. August 2020 marks the milestone of Grab in introducing financial services, such as
loans, _______, health insurance, and a pay-later program.
19. Besides being a service provider for mobile payments, logistics, and food delivery, Go
Jek is a _______.
20. Hyal Route owns the largest independent _______, shared fiber network platform in
Myanmar and Cambodia.
21. Traveloka is an online-based company, mainly dealing with flight tickets, _______, and
so on.
22. In the ecosystem of OVO, _______, business partners and members are able to conduct
digital payment and smart financial services.
23. Store images have been convertible into shelf insights thanks to the combination of
artificial intelligence, machine learning engines and detailed _______ from Trax.
24. Revolution Recrafted allows customers to live in aesthetically adorned properties by
creating a wide range of customized, _______ spaces.
25. Singapore and Indonesia will no longer be the exclusive _______ for unicorns.
Your answers:
16. 17. 18. 19. 20.

21. 22. 23. 24. 25.

B. LEXICO AND GRAMMAR (30 points)


Part 1. Choose the answer A, B, C, or D that best completes each of the following
sentences. Write your answers in the corresponding numbered boxes. (20 points)
1. Fear of media runs through it like a broad _______ streak.
Page 3 of 23
A. yellow B. green C. blue D. pale
2. The difficulty we face is _______ our intentions into concrete action.
A. changing B. converting C. translating D. interpreting
3.This great song has always _______ memories of Daisy’s schooldays.
A. tracked down B. conjured up C. brought in D. coupled with
4. The official record for the fastest _______ of a Rubik’s cube belongs to a Japanese student
who completed it in just under 15 seconds.
A. undoing B. unraveling C. unscrambling D. unwinding
5. _______, all the students couldn’t do it well.
A. At short notice did the teacher give the test
B. The teacher gave the test at short notice
C. Giving the test at short notice
D. The teacher giving the test at short notice
6. Compared to conventional heating, underfloor heating systems _______ far less fuel and
are therefore also more eco-friendly.
A. consume B. devour C. gulp D. gobble
7. There are enough facts and figures in this book to keep even the most obsessive _______
fascinated for hours.
A. nut B. anorak C. dog D. armchair
8. In the past, schoolteachers regularly _______ physical punishment to their pupils.
A. battened on B. shoved around C. meted out D. bailed out
9. She denied the accusation _______ true. She felt innocent.
A. to be B. being C. to being D. of being
10. He had to retire from the match suffering from a _______ ligament.
A. torn B. broken C. slipped D. sprained
11. Being a fertile ground for film production, Los Angeles is _______ with would-be
actors.
A. bulging at the seams B. flying by the seat of its pants
C. wearing the trousers D. burning a hole in its pocket
12. _______ are considered humorous is mainly due to his characters’ use of slang.
A. That Damon Ruyan’s stories B. Damon Ruyan’s stories, which
C. Damon Ruyan’s stories D. Because Damon Ruyan’s stories
Page 4 of 23
13. A(n) _______ love of country lays the foundation for sustainable development and
complete harmony.
A. ingrained B. indelible C. abiding D. established
14. It is necessary that they _______ a bit and examine the history of the problem.
A. backfire B. backtrack C. backlash D. backwash
15. Demand for the product is expected to peak five years from now and then to _______.
A. taper off B. fall down C. set back D. drift away
16. Her comments cast a _______ on the integrity of his employees.
A. blot B. slur C. stain D. drag
17. To apply for this position, each candidate has to submit a _______ photo besides other
required documents.
A. full-bodied B. full-scale C. full-length D. full-fledged
18. During the early period of ocean navigation, _______ any need for sophisticated
instruments and techniques.
A. so that hardly B. when there hardly was
C. hardly was D. there was hardly
19. Although the task is basically a no-brainer, she’s making such a _______ weather of it.
A. heavy B. weighty C. stormy D. rough
20. Although colouring books for adults may raise a few eyebrows, more and more people
are seeking them _______ as a way to relax and de-stress.
A. up B. on C. by D. out
Your answers:
1. 2. 3. 4. 5.
6. 7. 8. 9. 10.
11. 12. 13. 14. 15.
16. 17. 18. 19. 20.

Part 2. Use the correct form of the word given in brackets to fill in each blank. Write your
answers in the corresponding numbered boxes. (10 points)
1. It is vital that we _______ this realm if we ever want to get anything done effective in
securing it. (MYSTERY)

Page 5 of 23
2. The protagonist in the latest movie by James Wan seems _______ on self-destruction,
considering her serious alcohol problems. (BENT)
3. The Mole Antonelliana towers was completed in 1889, and today it is the home of the
National Museum of Cinema, housing equipment and _______ from the earliest days of film
production to the present. (MEMORY)
4. At best the _______ are patronised and promoted on their master's terms; at worst they
are treated with contempt as pariahs. (DOG)
5. Everyone was taken aback finding out how generous and _______ he could be even when
harmed. (ANIMUS)
6. Polar bears need sea ice to hunt, and warming melts sea ice. So the connection between
human-caused global warming and the _______ of the polar bears is crystal clear.
(DANGER)
7. He’s the _______ guy for questions about spreadsheets. (GO)
8. At the dawn of the Internet, many believed that it would enable a more _______, pluralist,
and really personalized platform, particularly with politics. (PARTICIPATE)
9. In the early medieval world, time past and present was meaningless because the vast
majority of people were _______, they did not even know their own age. (NUMBER)
10. An epic might have seemed like a great idea when he was young, but by the time he gets
actually around to writing it, it’s an entirely _______, utterly outdated form. (ANNUAL)
Your answers:
1. 2. 3. 4. 5.

6. 7. 8. 9. 10.

C. READING (60 points)


Part 1. Read the text below and think of the word which best fits each space. Use only one
word in each space. Write your answers in the space provided. (15 points)
Hats were once necessary to human life. They provided (1) _______ from both enemies
and the elements and were also ornaments, with ceremonial functions. The head is the (2)
_______ of intelligence, the source of speech, and the object of our glances. By wearing a
hat, you (3) _______ a frame around your personality and so cross the threshold from private
Page 6 of 23
to public, engaging with strangers on conventional (4) _______. The wearing of hats was,
therefore, an important part of our ancestors' attempt to create a public realm in which people
could be correctly dressed and part of the social fabric. The hat was a form of good manners,
a way of recognising others by putting a lid on the self. In many cultures, hats, and the
conventions associated with them, are words in a complex (5) _______, and learning their
grammar is a part of growing up. But in common with most uniforms, such conventions are
(6) _______ and, therefore, also divisive. The language of hats is, rather like the foreign
tongue, something which confirms the strangeness of the invading tribe. One (7) _______
may be the ubiquitous baseball cap. Although it often (8) _______ the logo of some
multinational firm, this globalised head (9) _______ appears strictly meaningless, a fashion
item. But is it'? The baseball cap, along with fast food and modern architecture, is now
conquering the world. What's more, it forms part of the (10) _______ of youth, and
increasingly these days only a courageous young person would choose any other form of
headgear, wouldn't they?
Your answers:
1. 2. 3. 4. 5.
6. 7. 8. 9. 10.

Part 2. For questions 1-13, read the following passage and do the tasks that follow. (13
points)
Bismarck: A master of political and diplomatic juggling?
A. Otto Von Bismarck’s rise up the political ladder was swift and relentless. Having entered
parliament in 1847, he always harboured lofty ambitions, chief among them perhaps being
the reunification of Germany into one strong, centrally controlled state, though his own
personal thirst for power was arguably even stronger. On becoming Prussian Chancellor, he
set about fulfilling his ambitions and in doing so proved himself to be a diplomat of some
considerable skill. Victory in the Austro-Prussian war effectively ended Austria as a factor
in German affairs. His political and military juggling was taken a step further when he
orchestrated a situation where France declared war on Germany in 1870, making the French
seem responsible for a conflict he had always intended to create. And following another
swift military triumph, this time over the French, the German empire was proclaimed in
January 1871.
Page 7 of 23
B. In little more than nine years, Bismarck realised his lifelong ambition, steering Germany
to reunification. And by defeating Austria and France in quick succession, he also created a
power vacuum on mainland Europe, which he was determined to fulfill himself. This was
another opportunity for Bismarck to demonstrate his political and diplomatic cunning. He
set about creating a dictatorial Germany in which he, as head of the Prussian parliament,
would automatically become chancellor of the German empire. He drafted a new German
constitution to suit his own purposes and, despite maintaining a veneer of democracy, the
German parliament was effectively powerless to oppose him. Provinces that were slow to
support him were enticed with bribes and before long the German empire was his to
command.
C. It is a testament to his political skill that Bismarck achieved so much so quickly. At this
point in his colourful political career, he did appear, for all intents and purposes, a master of
political and diplomatic juggling. But challenges lay ahead and Bismarck’s next target was
the Catholic church, which he deemed too powerful and a threat to his political dominance.
He proceeded to enact a series of laws that seriously eroded the power of the church.
However, his plans backfired and Bismarck was forced to make a political U-turn. Though
here again, he somehow managed to save face. The damage to his reputation was limited
and indeed by the late 1870s, he had even managed to win over the church whose support
he now needed.
D. Bismarck viewed the growing popularity of the Socialist Democratic Party as a serious
threat. He bided his time and used the attempted assassination of the Kaiser as an excuse to
attack the socialists in 1878, blaming them for the attempt on the Kaiser’s life. He
immediately arrested the leaders, banned party meetings, and suppressed socialist
newspapers. But despite his efforts to destroy the socialist movement, its popularity had
trebled by 1890. Just as his interventions with the church had not gone as planned, Bismarck
once again failed to achieve his objective; though, to his credit, he held on to power.
E. His domestic position was relatively secure after 1871, Bismarck devoted a lot of his time
to foreign policy. Having used war to unite Germany and make her great, Bismarck now
believed that his ambitions were best served by peace. His plan to isolate a hostile France
would require all his considerable diplomatic skills. The Dreikaiserbund agreement of 1873
between Germany, Austria-Hungary, and Russia was a first step towards doing just that.
The Balkan crisis, a conflict involving Russia and Austria-Hungary, severely tested his
Page 8 of 23
diplomatic credentials, but his answer was to offer himself as an ‘honest broker’ to help
resolve the dispute. The subsequent Congress of Berlin which he hosted was an outstanding
success and only served to reinforce Bismarck’s reputation as a shrewd diplomat.
Bismarck’s foreign policy would continue in this vein throughout his reign as Chancellor.
He built up strategic alliances with the big powers, Russia, Italy, and Austria-Hungary, in
the hope that he could keep his main threats, France and Britain, isolated.
F. In truth, Bismarck’s reign as chancellor of the German empire does seem to confirm him
as a shrewd and wily diplomat and politician, one whose objectives were broadly achieved.
Does this mean his so-called juggling was a success? Perhaps, but Bismarck left a less than
perfect legacy. He created a Germany in which the Kaiser had the ultimate say in domestic
affairs and enjoyed far too much power should he choose to wield it. This meant that the
future of the empire largely depended on the strength and character of just one man, the
Kaiser. A weak Kaiser would be disastrous for the country’s welfare, and so it would soon
prove. In the final analysts, Bismarck put Germany back on the map again as a great power
during his reign, but we should not forget that he created the political situation that would
be the downfall of his country in the end. His political and diplomatic juggling, therefore,
simply cannot be considered a total success.
Questions 1-6
Reading passage has six paragraphs A-F.
Choose the correct heading, i - ix, from the list of headings below.
Write the correct number i-ix in the boxes 1-6.
List of Headings
i. The pace at which Bismarck advanced his ambitions
ii. Separating church and state
iii. A change of mind
iv. Unexpected ramifications of socialist threat
v. Unremitting efforts to create a virtual dictatorship
vi. External objectives and activities
vii. Abortive attempts to curb a movement
viii. Political power on the wane
ix. A critical analysis

Page 9 of 23
1. Paragraph A _______
2. Paragraph B _______
3. Paragraph C _______
4. Paragraph D _______
5. Paragraph E _______
6. Paragraph F _______
Your answers:
1. 2. 3. 4. 5. 6.

Questions 7-13
Do the following statements agree with the information given in the Reading Passage?
In boxes 7-13, write
TRUE if the statement agrees with the information
FALSE if the statement contradicts the information
NOT GIVEN if there is no information on this
7. Germany defeated France and Austria successively to claim the dominant position in
Europe.
8. The Congress of Berlin was a great personal success for Bismarck.
9. After Bismarck enacted laws to weaken the Catholic Church, he was forced to change his
policy and made a church an ally.
10. The Socialist Democratic Party took power from Bismarck in 1890.
11. Bismarck’s foreign policy was to wage war with all countries that posed a military threat.
12. Bismarck considered his reign as German chancellor a failure.
13. It can be concluded that Bismarck was not actually a master of political juggling.
Your answers:
7. 8. 9. 10. 11. 12. 13.

Part 3. For questions 1-10, read an extract from an article and choose the answer A, B,
C or D that fits best according to the text. Write your answers in the corresponding
numbered boxes provided. (10 points)
Super memorisers

Page 10 of 23
There are people in this world who are innately possessed of an ability to remember
things with quite an extraordinary degree of detail and exactness. These super memorisers,
as they are known, typically possess a brain naturally and distinctively wired to maximize
its memorizing potential. They are gifted such that they require no particular training or
effort to sharpen their memories; they remember things just as effortlessly as most people
forget them. Few of us are born with such gifts, sadly, but there is much cause for optimism
yet for those looking to improve their brain’s performance tangibly in this area; a fact which
Boris Konrad is testament to.
Konrad is a champion memoriser who, in winning gold in the German Memory
Championships one year recognized and recalled the names of 195 people in just fifteen
minutes. [■] His powers of recollection, then, are as admirable as they are undisputed, but
Konrad is self-trained and started out with a recall capacity that was unremarkable. [■]
Instead, he spent years developing memory strategies and employing those strategies to
improve in the area through practice and dedication. [■] He, then, is a living proof that the
average Joe with a gift for forgetfulness can reinvent himself in the area. [■]However, his
example is as much a reminder of the extent of devotion that is required to reach this level
as it is of the possibilities if one is prepared to put in the effort, and there are not many
people prepared to expend a similar level of effort to this end, which is what really makes
Konrad unique.
That said, whilst you may not become a super memoriser overnight, new research
suggests that it is possible to tangibly improve your memory in a relatively short space of
time by devoting roughly half an hour of your every day to the process. It is necessary to
learn and employ memorizing strategies such as the Memory Palace technique Konrad uses,
though, to yield such results; otherwise, you might be as well be doing something else. In
one recent study, for example, participants spent one month training their memories in the
aforementioned technique for 30 minutes every day, which more than doubled their ability
to remember list of information after just 40 days. More impressively, recall performance
remained high whether or not training continued at the end of one month, which suggests
after rewiring of the brain can be permanent.
Even innately gifted memorisers use such mnemonic techniques to enhance their
recall ability, and, of the recall methods which exist, the methods of loci, which has already
been referred to here as the Memory Palace method, is the most prevalent one adopted as
Page 11 of 23
revealed by a recent study of 35 memory champions. Indeed, at a more rudimentary level,
this method has been employed by orators and others required to remember long strings of
interconnected information for some time, and it actually dates back to Ancient Greece,
where it was first conceived of, remaining prevalent right through to the Middle Ages and
the Renaissance. It is only a new method then, today, in the sense that the skill had been lost
as such to most people for a very long time. Indeed, many of today’s super memorisers
employed it intuitively rather than in a conscious effort to enhance their recall powers.
Participants in the study was separated into three distinct groups, with one group
receiving no memory training whatsoever and making no notable recall gains during the
process. The second group dedicated time and effort to upping their recall capacity, but
employed an everyday technique. However, those using the Memory Palate technique more
than doubled their initial recall capacity by the process’s end. What’s more, their brain
functions actually changed and their brain patterns began to bear more of a likeness to those
of innate super memorisers, leading researchers to speculate that a total rewire might well
be possible over time, such that a normal individual’s recall capacity could match that of
any memory champion. The conclusion, therefore, was that memory is not necessarily an
innately bestowed gift you either are possessed of or not. Most of us have the potential to
hone and expand our memories very meaningfully indeed.
1. According to the first paragraph, people with marvelous power of retention usually
_______
A. undergo rigorous practice to become that way.
B. demonstrate brain functions resembling anyone else.
C. throw in little effort to commit things to memory.
D. be tangibly more intelligent than other people.
2. Why are Konrad’s achievements mentioned in paragraph two?
A. To underline the excellence of inborn super memorisers
B. To demonstrate the disparity between laymen and super memorisers
C. To prove that almost every individual can make a significant enhancement in their
retention powers
D. To show how even innate super memorisers have to exert themselves
3. What does the writer mean in the phrase ‘you might as well be doing something else’
in paragraph three?
Page 12 of 23
A. It is futile to invest time in improving recall capacity.
B. People should concentrate on their strengths.
C. The Memory Palace technique is not universally suitable.
D. Progress is only visible if you train in the right way.
4. What did the study mentioned in the third paragraph conclude?
A. The implication of moderate brain exercises can be enduring.
B. The maintenance of a decent recall capacity requires continuous training.
C. Your recall ability can more than double within one month.
D. Progress is only noticeable with at least 30 minutes spent on brain training.
5. What do we learn about the method of loci?
A. It is superior to the Memory Palace method.
B. Most skillful memorisers purposefully adopt it.
C. It has evolved from methods first employed in Ancient Greece.
D. It has been widely used among super memorisers.
6. What did the results of the study mentioned in the final paragraph reveal?
A. How the brains of super memorisers function is inimitable.
B. The application of appropriate technique exerts a tremendous influence on the brain.
C. Memory training is futile unless employing a confirmed technique.
D. Most super memorisers are not talented at memorizing at birth.
7. The word "mnemonic techniques” in paragraph 4 mostly means _______
A. gimmicks used to improve brain functions.
B. endowed talents of super memorisers.
C. tips specifically designed to aid the process of retaining information.
D. skills adopted by super memorisers to avoid absent-mindedness.
8. Look at the four squares [■] that indicate where the following sentence could be added to
the passage.
Indeed, after just 30 seconds, examination, he is able to retain the order of an entire
deck of cards.
Where would the sentence best fit?
A. First square
B. Second square
C. Third square
Page 13 of 23
D. Fourth square
9. The word "orators” in paragraph 4 mostly means _______
A. language experts. B. body language experts.
C. oral students. D. proficient speakers.
10. The paragraph following the passage would most probably discuss _______
A. further research into the workings of brains of super memorisers.
B. the defeat of a normal person over a super memoriser in terms of recalling capacity.
C. potential ways to expand our power of retention.
D. the popularity of memory palace technique.
Your answers:
1. 2. 3. 4. 5.
6. 7. 8. 9. 10.

Part 4. In the passage below, seven paragraphs have been removed. For questions 1-7,
read the passage and choose from the paragraphs A-H the one which fits each gap. There
is ONE extra paragraph which you do not need to use. Write your answers in the
corresponding numbered box provided. (7 points)
Blind to Change
How much of the world around you do you really see? You only take in tiny pieces of
information at a time and that can have unnerving consequences, says Laura Spinney.
Imagine you’re walking across a college campus when an unknown man asks you for
directions. While you’re talking, two men pass between you carrying a door. After an
irritating minute of interruption you carry on describing the route. When you’ve finished
you are informed that you’ve just taken part in a psychology experiment, and asked if you
noticed any changes after the two men passed with the door. “No”, you reply uneasily. The
unknown man then explains that the man who approached you initially walked off behind
the door, leaving this man in his place. You are stunned; the two men are dressed differently
and have different voices and haircuts.
1

Page 14 of 23
Rather than logging every detail of the visual scene, we are actually highly selective about
what we take in. Our impression of seeing everything is just that – an impression. In fact we
extract a few details and rely on memory, or perhaps even our imagination, for the rest.
2

Yet in 1991, the controversial claim was made that our brains hold only a few salient details
about the world - and that this is the reason we are able to function at all. We don’t store
elaborate pictures in short-term memory, because it isn’t necessary and would take up
valuable computing power.
3

Just a year later, at a conference on perception in Vancouver, it was reported that people
shown computer-generated pictures of natural scenes were blind to changes that were made
during an eye movement. In a typical laboratory demonstration of this you might be shown
a picture on a computer screen of, say, a couple dining on a terrace.
4

It’s an unnerving experience. But to some extent, such “change blindness” is artificial
because the change is masked in some way. In real life, there tends to be a visible movement
that signals the change. But not always. For instance, we have all had the experience of not
noticing a traffic signal change because we had briefly looked away.
5

For instance, an experiment was done at Harvard in which people were shown a videotape
of a basketball game and asked to count the passes made by one or other team. After about
45 seconds a man dressed in a gorilla suit walked slowly across the scene, passing between
the players. Although he was visible for five seconds, an amazing 40 per cent of the viewers
failed to notice him.
6

Such lapses raise important questions about vision. For instance, how can we reconcile these
gross lapses with our subjective experience of having continuous access to a rich visual
Page 15 of 23
scene? One researcher has actually shown that imagining a scene activates parts of the visual
cortex in the same way as seeing it. He says that this supports the idea that we take in just
what information we consider important at the time, and fill in the gaps where the details
are less important. The illusion that we see “everything” is partly a result of filling in the
gaps using memory. Such memories can be created based on beliefs and expectations.
7

This particular idea has not been generally accepted. Yet most researchers in the field do
agree that of all the myriad visual details of any scene that we could record, we take only
what is relevant to us at the time. This leads us to the uncomfortable realization that, for all
our subjective experience of a rich visual world, it may, in fact, be impossible to tell what
is real and what is imagined.
A. Now imagine that the task absorbing their attention had been driving a car, and the
distraction had been a pedestrian crossing their path. According to some estimates, nearly
half of all motor-vehicle accidents in the US can be attributed to driver error, including
momentary loss of attention. It is more than just academic interest that has made both forms
of cognitive error hot research topics.
B. The image would disappear, to be replaced for a fraction of a second by a blank screen,
before reappearing significantly altered - by the raising of a railing in the background,
perhaps. Many people search the screen for up to a minute before they see the change. A
few never spot it.
C. In contrast, other researchers argue that we can get the impression of visual richness
without holding any of that richness in our heads. For instance, the “grand illusion” theory
argues that we held no picture of the visual world in our brains at all. Instead, we refer back
to the external visual world as different aspects become important. The illusion arises from
the fact that as soon as you ask yourself “Am I seeing this or that?” you turn your attention
to it and see it.
D. It sounds impossible, but when this test was carried out, a full 50 per cent of those who
took part failed to notice the substitution. The subjects had succumbed to what is called
change blindness. Taken with a glut of recent experimental results, this phenomenon
suggests we see far less than we think we do.

Page 16 of 23
E. The relationships between attention, awareness and vision have yet to be clarified.
Because we have a less than complete picture of the world at any one time, there is the
potential for distortion and error. How that complete picture could be objectively established
is controversial, but there is one obvious way forward.
F. This flies in the face of what vision researchers have long believed: that seeing really
means making pictures in the brain. According to this theory, by building detailed internal
representations of the world, and comparing them over time, we would be able to pick out
anything that changed.
G. And there’s a related phenomenon called in attentional blindness, that doesn’t need any
experimental visual trick at all: if you are not paying attention to some feature of a scene,
you won’t see it.
H. Rather, we log what has changed and assume the rest has stayed the same. Of course:
this is bound to mean that we miss a few details. Experimenters had already shown that we
may ignore items in the visual field if they appear not to be significant - a repeated word or
line on a page of text for instance. But nobody realized quite how little we really do “see”.
Your answers:
1. 2. 3. 4. 5. 6. 7.

Part 5. The passage below consists of five sections marked A-E. For questions 1-10, read
the passage and do the task that follows. Write your answers (A-E) in the corresponding
numbered boxes provided. (15 points)
Notorious Art 'Collector' Goes Free
Last week, Brian Barnes was acquitted of stealing a valuable ancient artefact. We
asked some of the key players about the trial.
A. Amelia Ashford (journalist)
I've been covering this case since the night of the robbery. A source from the museum
contacted me and told me that a priceless and irreplaceable ancient Egyptian gold statuette
had been stolen. This was not an opportunistic crime. The perpetrator knew in advance
exactly where the guards would be and had a meticulously crafted scheme for bypassing the
museum's security system.
The police investigation soon focused on Mr. Barnes, an art dealer with a shady past. They
had him under surveillance and an undercover officer, posing as a rich art collector, was
Page 17 of 23
negotiating to buy the statuette. Mr. Barnes must have been tipped off, or realised what was
going on, and tried to flee the country. He was arrested at the airport, but this abrupt end to
the police operation meant that, when the case came to court, they didn't have enough
evidence to secure a conviction.
B. Brian Barnes (the defendant)
Justice was done. It's as simple as that. The jury delivered the only possible verdict. To be
honest, I take my hat off to whoever stole that statuette. It was an audacious crime, and the
thieves clearly baffled the police, but I had nothing to do with it. I'm just an honest
businessman, and when the robbery took place, I was at the theatre. Surely, if the police
were unable to find anyone who could corroborate that, that's their fault, not mine. Now, I
just want to put this whole thing behind me. I'm going to take a well-earned holiday, and
then I'm looking forward to getting back to work.
C. Cristina Calviano (prosecution lawyer)
Obviously I'm very disappointed. I felt that we had a solid case against Mr. Barnes, and
enough persuasive arguments to get a conviction. The defendant clearly had the financial
means to mount an extravagant defence, and he took full advantage of that. What really hurt
us, though, was the judge's ruling that some of our key evidence could not be introduced in
court. The jury never heard that Mr. Barnes practically admitted stealing the statuette to an
undercover police officer or that we found a partial fingerprint at the scene of the crime that
we believe is Mr. Barnes'. We had to rely on a witness who saw the defendant with an item
resembling the statuette on the night in question, and sadly that wasn't enough to sway the
jury in our favour. Unfortunately, this means that the statuette is now unlikely to ever be
recovered.
D. Daniel Dawson (defence barrister)
To begin with, I don't think it befits someone in Ms. Calviano's position to criticise in public
the way the court handled the case. There were good legal reasons why she wasn't allowed
to present the evidence that she mentioned. It's also disingenuous of herto claim that, had
this evidence been included, it would have altered the outcome of the trial. Take, for
example, the partial fingerprint that the police found (which, by the way, was the only
tangible evidence that the prosecution had). The experts disagreed about it, and even the
police's forensic scientist admitted that she wasn't able to state with 100% certainty that it
was my client's print. Quite frankly, this is a case that should never have come to court.
Page 18 of 23
E. Ed Ellis (police detective)
Of course it's a shame, but that's the way it goes. You win some and you lose some. We'll
have to reopen our file on the case now but, after so much time and with no other suspects,
I doubt that my superiors will commit too many resources to it. We think that Barnes
managed to pass the statuette off to an accomplice before he was arrested, but after that the
trail goes cold.
The only consolation is that Mr. Barnes doesn't seem like the sort of person who will be able
to keep a low profile. He's on our radar now, and I have a hunch that this won't be his last
brush with the law.
Which person (A—E) does the following?
1. states that the physical evidence in the case was inconclusive
2. believes that reinvestigating the case won’t be the police’s priorities
3. suggests Mr. Barnes may have been warned about the police investigation
4. suspects Mr. Barnes will be in trouble with the police again
5. mentions Mr. Barnes' affluence as his assisting factor
6. expresses admiration for the thieves
7. says that the robbery had been carefully planned
8. complains about a decision made by a court official
9. mentions that Mr. Barnes' alibi could not be confirmed
10. accuses somebody of acting in an unprofessional manner
Your answers:
1. 2. 3. 4. 5.
6. 7. 8. 9. 10.

D. WRITING (60 points)


Part 1. Read the following extract and use your own words to summarise it. Your summary
should be between 100 and 120 words long. (15 points)
Man is forever changing the face of nature. He has been doing so since he first appeared
on the earth. Yet, all that man has done is not always to the ultimate advantage of the earth
or himself. Man has, in fact, destroyed more than necessary.
In his struggle to live and extract the most out of life, man has destroyed many species
of wildlife; directly by sheer physical destruction, and indirectly by the destruction or
Page 19 of 23
alteration of habitats. Some species may be able to withstand disruptions to their habitat
while others may not be able to cope.
Take the simple act of farming. When a farmer tills a rough ground, he makes it
unsuitable for the survival of certain species. Every change in land use brings about a change
in the types of plant and animals found on that land.
When man builds a new town, this means the total destruction of vast areas of farmland
or woodland. Here, you have the complete destruction of entire habitats and it is inevitable.
It follows therefore, that every form of human activity unavoidably upsets or changes
the wildlife complex of the area. Man has destroyed many forms of wildlife for no
reasonable purpose. They have also made many great blunders in land use, habitat
destruction and the extermination of many forms of wildlife.
Man's attitude towards animals depends on the degree to which his own survival is
affected. He sets aside protection for animals that he hunts for sport and wages a war on any
other creature that may pose a danger or inconvenience to him. This creates many problems
and man has made irreversible, serious errors in his destruction of predators. He has
destroyed animals and birds which are useful to farmers as pest controllers. The tragedy that
emerges is that all the killing of predators did not in any way increase the number of game
birds.
Broadly speaking, man wages war against the creatures which he considers harmful,
even when his warfare makes little or no difference to the numbers of those he encourages.
There is a delicate predator and prey equilibrium involving also the vegetation of any area,
which man can upset by thoughtless intervention.
Therefore, there is a need for the implementation of checks and balances. The
continued existence of these animals depends entirely on man and his attitude towards his
own future.
.............................................................................................................................................
.............................................................................................................................................
.............................................................................................................................................
.............................................................................................................................................
.............................................................................................................................................
.............................................................................................................................................
.............................................................................................................................................
Page 20 of 23
.............................................................................................................................................
.............................................................................................................................................
.............................................................................................................................................
.............................................................................................................................................
.............................................................................................................................................
.............................................................................................................................................
.............................................................................................................................................
.............................................................................................................................................

Part 2. Chart description (15 points)


The table and pie chart gave information about the population in Australia in 2020
according to different nationalities and areas.
Summarize the information by selecting and reporting the main features, and make
comparisons where relevant. You should write about 150 words.

.............................................................................................................................................
.............................................................................................................................................
.............................................................................................................................................
.............................................................................................................................................
.............................................................................................................................................
.............................................................................................................................................
.............................................................................................................................................
.............................................................................................................................................
.............................................................................................................................................
.............................................................................................................................................

Page 21 of 23
.............................................................................................................................................
.............................................................................................................................................
.............................................................................................................................................
.............................................................................................................................................
.............................................................................................................................................

Part 3. Essay writing (30 points)


Some people think that success should be measured by the material possessions a person
has acquired while others think it should be judged by the knowledge he or she has.
Write your essay about 350 words to discuss both views, and give your opinion.
.............................................................................................................................................
.............................................................................................................................................
.............................................................................................................................................
.............................................................................................................................................
.............................................................................................................................................
.............................................................................................................................................
.............................................................................................................................................
.............................................................................................................................................
.............................................................................................................................................
.............................................................................................................................................
.............................................................................................................................................
.............................................................................................................................................
.............................................................................................................................................
.............................................................................................................................................
.............................................................................................................................................
.............................................................................................................................................
.............................................................................................................................................
.............................................................................................................................................
.............................................................................................................................................
.............................................................................................................................................
.............................................................................................................................................
.............................................................................................................................................
Page 22 of 23
.............................................................................................................................................
.............................................................................................................................................
.............................................................................................................................................
.............................................................................................................................................
.............................................................................................................................................
.............................................................................................................................................
.............................................................................................................................................
.............................................................................................................................................
.............................................................................................................................................
.............................................................................................................................................
.............................................................................................................................................
.............................................................................................................................................
.............................................................................................................................................
.............................................................................................................................................
.............................................................................................................................................

------- THE END -------

GIÁO VIÊN RA ĐỀ

Nguyễn Bửu Trung


Tel: 0914499397

Page 23 of 23
KỲ THI HỌC SINH GIỎI CÁC TRƯỜNG THPT CHUYÊN
KHU VỰC DUYÊN HẢI VÀ ĐỒNG BẰNG BẮC BỘ
LẦN THỨ , NĂM 2022

ĐỀ THI MÔN: TIẾNG ANH 11


Thời gian: 180 phút (không kể thời gian giao đề)
Ngày thi:
ĐỀ XUẤT
(Thí sinh làm bài trực tiếp vào đề)

(Đề thi gồm 17 trang)

Điểm
Giám khảo 1 Giám khảo 2 Số phách
Bằng số Bằng chữ

A. LISTENING (50 points):


HƯỚNG DẪN PHẦN THI NGHE HIỂU
 Bài nghe gồm 4 phần; mỗi phần được nghe 2 lần, mỗi lần cách nhau 05 giây; mở đầu và
kết thúc mỗi phần nghe có tín hiệu. Thí sinh có 20 giây để đọc mỗi phần câu hỏi.
 Mở đầu và kết thúc bài nghe có tín hiệu nhạc. Thí sinh có 03 phút để hoàn chỉnh bài trước
tín hiệu nhạc kết thúc bài nghe.
 Mọi hướng dẫn cho thí sinh (bằng tiếng Anh) đã có trong bài nghe

Part 1. You will hear an interview with a woman called Barbara Darby, who works as a
casting director in the film industry. Choose the answer (A, B, C or D) that fits best
according to what you hear. (10 pts)
1. According to Barbara, a casting director needs above all _______
A. to learn from experience.
B. to be a good communicator.
C. to have a relevant qualification.
D. to have a natural feel for the job.
2. Barbara says that she looks for actors who _______
A. can play a variety of roles.
B. complement each other.
C. accept her way of working.
D. think deeply about a part.
3. At which stage in the casting process does Barbara meet the actors?
A. before she goes to see them performing live
B. once the director has approved them
C. before a final short list is drawn up
D. as soon as a final selection is made
4. Barbara explains that what motivates her now is a need for _______
A. personal satisfaction.
B. professional recognition.
C. a glamorous lifestyle.
Page 1 of 17
D. financial security.
5. What made Barbara give up her job for a while?
A. She’d become tired of travelling.
B. She was ready to try something new.
C. She felt she’d been put under too much pressure.
D. She found that she was no longer as committed to it.

Your answers:
1. 2. 3. 4. 5.

Part 2. Listen to an introduction of a new invention and answer the following questions.
Use NO MORE THAN THREE WORDS AND/OR A NUMBER for each answer. (10 pts)
1. How much was the watch used by Fossett sold for?
……………………………………………………………………………
2. Where were the two men near when the accident happened?
……………………………………………………………………………
3. Who might benefit a lot from this watch?
……………………………………………………………………………
4. What is the maximum range of the watch at sea?
……………………………………………………………………………
5. When was the self-winding watch invented?
……………………………………………………………………………
Your answers:
1. 2. 3. 4. 5.

Part 3. You will hear an interview with a comedian, Lenny Henry. Listen carefully and decide
the following statements are TRUE or FALSE according to what you hear. (10pts)
1. Lenny decided to do a degree because he was impressed by other actors who had been to the
university.
2. Studying for a degree has made Lenny think more seriously about his career.
3. According to Lenny, comedy makes people more sensitive.
4. Lenny says when he visited Debre Zeit, he was moved by the way people there handled their
situation.
5. Lenny hopes to write something in collaboration with other people.
Your answers:
1. 2. 3. 4. 5.

Part 4: Listen to the VOA news and fill in each gap with NO MORE THAN THREE
WORDS according to what you hear. (20pts)
Australian weapons developers have been inspired by the unfailing capacity of insects to spot
and zero in on food and the ability of bees to avoid colliding with each other in a (1)_______ .
Australia's Defense Science and Technology Organization has been looking at how the existing
research into insects' navigation and sight could make (2) _____ weapons more reliable and
improve their ability to hit moving targets.
Through its collaboration with several electronics companies, a new system known as
'Bioseeker' has been developed. Project Manager Philip Henschke says the study of insects has
been vital.
"A variety of insects have a unique capability to find the moving target and that's the particular
(3) ______ of what we're interested in from a weapons application in defense. What we've
Page 2 of 17
actually done is looked at the mathematics of how an insect sees and we've taken that
mathematics and from that we've looked at an (4)______ that will enable us to do what we call
a (5) _____, a map of the movement within a scene."
This information was then analyzed in special software to create a system designed to find, track
and destroy moving targets.
The Bioseeker technology is (6) _______ to undergo final testing, later this year. Its architects
believe that, if it is eventually used in battle, it will make soldiers safer by taking them further
away from the enemy.
Researchers aim to produce a (7) ______-and-guidance system that could eventually be reduced
to the size of a coffee cup. Possible applications include placing the technology inside rockets
used on the Australian Army's Tiger Attack helicopters.
The Australian military is relatively small, with about 50,000 personnel. However, the defense
force has a reputation for technological innovation.
The government in Canberra has said that, by 2020, it hopes to bring into service a (8) ______
of Super Hornet jet fighters and an (9) ______ aircraft, as well as a range of new helicopters and
airborne refueling airplanes.
The Australian military is involved in peacekeeping missions in (10) ________, Sudan and the
Solomon Islands and with the US-led campaign in Afghanistan.
Your answers:
1. 2. 3. 4. 5.
6. 7. 8. 9. 10.
B. LEXICO - GRAMMAR (30pts)
Part 1. Choose the correct answer A, B, C, or D to each of the following questions and
write your answers in the corresponding numbered boxes provided. (20pts)
1. Gorillas have long been the subject of _____ interest among zoologists for their uncanny
similarities to human beings.
A. firm B. utter C. keen D. sharp
2. The first thing Inspector Highgate noticed on entering his office was the ______ of the
cigarette.
A. glow B. sparkle C. glimmer D. flicker
3. Austen Grove, whose new novel, A Dublin Childhood, took the publishing world by
______ last week, is intensely wary of publicity.
A. force B. tempest C. storm D. thunder
4. What is particularly interesting about hypnosis is that it is capable of activating the
________ psychologist Carl Jung called the “creative imagination”.
A. grandiose B. notorious C. eminent D. prestigious
5. Those around her was especially supportive, _______ to her every whim and going out of
their way to be nice to her.
A. indulging B. pampering C. catering D. providing
6. She rocked the baby in her arms and watched his little face as he ______ to sleep.
A. drifted off B. carried off C. slipped off D. popped off
7. ______ retrospect, that first awful dream was probably the best.
A. At B. In C. To D. On
8. Do you ______ a cold sweat every time you see a snake?
A. give off B. break into C. set up D. turn on
9. Hundreds of people have visited our clinic over the years and found that the expert help
and advice they have received have given them a new ______ of life.
A. extension B. period C. lease D. length
10. The Egyptian government has pledged to _______ on the number of sightseers to protect
the ancient sight from being loved to death.
Page 3 of 17
A. clamp down B. push down C. put down D. weight down
11. I hadn’t met her before, but my cousin had been _______ to everyone about her
wonderful, warm and caring personality.
A. acclaiming B. plugging C. promoting D. raving
12. Bradley has ______ himself as the top scorer on the team.
A. messed B. screwed C. bungled D. distinguished
13. Many people feel the newspaper is too soft on the government and never _______when
Asking questions directly to the politicians.
A. shakes the water B. rocks the boat C. steers the engine D. moves the ship
14. She claims to be in favor of training, but so far she's only paid ______ service to the idea.
A. lip B. mouth C. tongue D. tooth
15. Anne is rather childish and tends to _______ if she can’t have what she wants.
A. ponder B. sulk C. hush D. muse
16. I didn’t want to make a decision _______, so I said I’d like to think about it.
A. in one go B. There and then C. At a stroke D. On and off
17. Not even losing all the time could _______ his enthusiasm for tennis.
A. recede B. Dampen C. Erode D. Belittle
18. We took such a great _______ to the place that we decided to go and live there.
A. affection B. Fondness C. Liking D. Attraction
19. He was highly knowledgeable on the areas. Many would say he was something of a
_______.
A. novice B. Probationer C. Tenderfoot D. Veteran
20. Materialism traps us _______ in a world of possessions hag-ridden by irrational fears of
likely loss and lurking dangers.
A. impromptu B. off the cuff C. on a whim D. unawares
Part 2. For questions 1-10, write the correct form of each bracketed word in the numbered
space provided. (10pts)
In an effort to escape from their hectic and (1. MATERIAL) ____________________ city lives,
more and more Northern Europeans are buying houses in rural areas of France, Spain, Italy and
Greece. Some relocate permanently in search of a more meaningful existence. Those who
cannot afford to give up their jobs seek a (2. THERAPY) ____________________ respite from
their stressful lifestyle by relaxing for a few weeks each year in their second home in the sun.
However, many of those who relocate permanently find that life in the country is not as
quiet and (3. EVENT) ____________________ as they had anticipated. Aspects of village life
which seemed delightfully (4. ATMOSPHERE) ____________________ in the context of a
two-week holiday can grate on the nerves when you love with them on a daily basis. Recently a
group of British residents in an Italian village took local farmers to court because they found the
smell of the villagers’ pigs (5. TASTE) ____________________. In other cases, foreigners have
complained to neighbors about the enthusiastic early-morning crowing of their cockerels, or to
village priests about the regular tolling of church bells.
(6. UNDERSTAND) ____________________, the local inhabitants are somewhat (7.
RESENT) ___________________ of these attitudes. They argue that the foreigners have an (8.
REAL) ____________________ view of what country life is like and that, since no one forced
them to come and live in a village, they are being (9. CRITIC) ____________________ by now
complaining about the (10. CONVENIENT) ___________________ of rural life.
Answer:
1. ……………………….. 2. ……………………….. 3. ………………………..
4. ……………………….. 5. ……………………….. 6. ………………………..
7. ……………………….. 8. ……………………….. 9. ………………………..
10. ………………………..
Page 4 of 17
C. READING (60pts)
Part 1: You are going to read a magazine article. Seven paragraphs have been removed
from the article. Chose from the paragraphs A-H the one which fits each gap (1-7). There
is one extra paragraph you do not need to use. (7pts)
The Important of being Dad
New dad, Rowan Daniel Foote, is in far from familiar territory here, and discovers,
while researching this piece, that being a father is anything but straight-forward.

What separates the good dad from the bad? By what parameters do we define ‘good dad’
anyway? What is the baseline? Is it a case of good dad changes the nappies while bad dad
watches TV? If only it were this simple.
1.______________________________________________________________________
Only recently, an Oxford research team linked the absence of an early-years father figure with a
predisposition in adolescent girls towards the development of mental health problems, likely to
significantly limit the affected teens’ ability to meet their full potential later in life. Similarly,
boys who do not have a credible father figure in their lives are more likely to get on the wrong
side of the law, and, for both sexes, the presence of a ‘good dad’ provides for greater motivation
to perform academically, so kids are likely to excel more at school when dad is an active rather
than passive participant.
2.______________________________________________________________________
Whether it is biological dad or stepdad is, according to the findings of the Oxford team, neither
here nor there, suggesting that a biological attachment and innate sense of affinity and desire to
bond is the exclusive domain of the mother. The key to a child’s level of happiness is the extent
of its involvement with whichever manifestation of dad circumstances conspire to present.
3. _____________________________________________________________________
So, we have established that dad needs to be hands on in his approach, but from day one, in year
one, during the terrible twos, from three and up, in the early years, during the teenage years…
basically when exactly?
4. ______________________________________________________________________
Now, that is not to say that ‘the ouchies’ (ages 1-4 – think first steps, little spills, big spills…
lots of tears), ‘the naughties’ (ages 5-12, when the mischief-making starts in earnest…) and the
‘do-the-exact-opposite-of-whatever-mummy-and-daddy-tell-me-ies’(the,ehem,teenageyears…)
aren’t important times for which daddy should be around too, but the findings do suggest that an
extra effort should be made to be present and involved as much as possible in those first few
months of life.
5. ____________________________________________________________________
For dad can be hard too – and frustrating to see how easily the bond between mother and baby is
formed. But there are things he can do to forge a meaningful bond of his own. Simple acts like
nappy-changing and bathing can make all the difference, and even a possessive mother will soon
tire of having to do these chores by herself, and will eventually welcome the extra pairs of hands
with her arms wide open. Even feeding does not have to be monopolized by mum, who can
facilitate here by expressing her milk to afford dad the opportunity to get involved, too. Besides,
many babies, whether for reasons of personal preference of the parents or out of necessity, are
bottle fed, which present far fewer complications for the willing would-be feeder-dad.
6. ______________________________________________________________________
Mind-altering fluffy chemicals aside though, there’s no doubt that dads have it tough today.
Theirs is a constantly changing role, and many new fathers undergo an identity crisis of sorts
just trying to define for themselves exactly what their purpose should be. Gone are the days of
dad simply being the breadwinner, but stay-at-home dads are equally rare. Most fathers have to

Page 5 of 17
juggle a full-time job with their strong desire to contribute at home, often fighting off tiredness
and frustration, and pushing their hobbies to the limit just to be there for their children.
7. ______________________________________________________________________
This can be a source of frustration for mums, too, who are often left feeling isolated and literally
‘holding the baby’. They often crave adult company and will naturally turn to dad to take over
when he arrives home in the evening, whether dad is of a mind to help out or not after his long
day. Communication is therefore an essential part of the equation for new parents; they must
speak with one another and develop a plan which affords both of them at least some break time
to recharge their batteries and keep them functioning well. Failure to do this will result not only
in their own health suffering, but, potential, that of their baby also.

A. The last point is interesting because it also has implications for children from broken home;
basically, what the research suggests is that whether dad is ‘real dad’, ‘new dad’, or ‘visit-three-
times-a-week-as-part-of-the-divorce-conditions –dad’ is of little relevance; so long as ‘whatever
dad’ is properly involved in his child’s upbringing and is a stable and constant influence, ‘any
dad’ will do just fine, once he is prepared to carry out his role and take on the responsibility of
parenthood. On the other hand, ‘shirker dads’ of any ilk, be they biological, step, occasional or
who knows, as a direct result of their lack of engagement with the parenting role, cause
permanent damage to the children in their care.
B. Apart from making a concerted effort to get involved, dad should also try to understand that
bonds take time to form, and, oftentimes, it is only a matter of persevering though, thankfully,
most men do not have to wait too long before their feigned or conscious effort to show interest
in the baby becomes far more genuine and instinctive. Increased levels of the prolactin hormone
will usually see this. As another study revealed, levels of the so-called ‘cuddle chemical’ usually
increase dramatically in new dads, which is nature’s way of rewiring pop’s brain for its new role
in fatherhood.
C. Well, just as is the case with mother and child, the Oxford study found that it is during the
first year of life – ‘the noughties’ (the 12 months it takes to get from zero to one) that it is the
most important to develop the bond between father and baby. The offspring of a father actively
involved in personal care in that period is less likely to develop behavioral problems as a teen,
and has the best chance of going on to become a well-rounded individual in later life.
D. Ultimately, what you give your child will be returned with interests in later years; you get out
of parenting what you put in. If you want your child to grow up into a well-balanced and
successful adult, then it is vital to afford them as much of your attention as possible, especially
in the teenage years, which are undoubtedly the most important of all when it comes to the
extent and effects of a father’s influence.
E. Their plight is not much helped by our archaic and heartless employment laws either, which
afford male workers little paid leave to spend time with their new-born kids, forcing them to
work long hours during the day only to have to come home to do another shift looking after
the baby, and to perhaps then have to endure a less-than-perfect night’s sleep into the bargain.
F. We place huge importance on the role of motherhood during the formative years of a child’s
life, heaping bucket loads of unnecessary pressure on an already dazed mum who is wondering
just what she has let herself in for, and who doesn’t need grandma and aunties Maureen and Peg
adding their tuppence worth of advice into the bargain. But, in doing so, we very often overlook
the father’s role. This despite the fact that research has consistently shown us that daddy’s
influence, or lack thereof, has just as pivotal a role to play in the development, for better or for
worse, of his child.
G. The study even found a link between the prospect of marital success and the degree of
involvement of the father of the newlywed during childhood; the more heightened this degree, it
seems, the more likely one’s marriage will succeed. In short, dad is critical from day one, and he
Page 6 of 17
has to be prepared to make time to spend time with his children no matter how much sacrifice
that entails.
H. But, truth told, that is easier said than done. After all, mummy has been carrying her little
package around with her for nine months by the time it pops out onto the delivery table, so she
has had nearly a year’s head start in having the sensation – and constant reminder – of
parenthood. Besides, given the strength of the maternal instinct in most women, an
unwillingness to share the experience, even with the person with whom the package was so
intimately created, is not atypical. It is up to mum, then, to guard against being overprotective
and to encourage her partner to play a meaningful role from as early as possible, preferably from
the very first moment little bundle is brought into its new home.

Part 2: Fill each of the following numbered blanks with ONE suitable word. Write your
answers in the corresponding numbered boxes provided. (15pts)
Celebrity Crossover
It is not surprising that actors want to be pop stars, and vice versa. (1) ___________________
that is deep in a part of our brain that most of us manage to keep under control, we all want to be
pop stars and actors.
Sadly, there’s nothing about the former profession that automatically qualifies you for the other,
(2) ___________________, of course, for the fact that famous actors and singers are already
surrounded by people who never (3) ___________________ no to them. on the whole, pop stars
tend to fare better on screen than their (4) ___________________ numbers do on CD. Let’s (5)
___________________ it: not being able to act is no big drawback in Hollywood, whereas not
being able to play or sing still tends to count against you in the recording studio.
Some stars do display a genuine proficiency in both disciplines, and a few even maintain
successful careers in both fields, but this just (6) __________________ a bad example for all the
others. (7) ___________________ every success, there are two dozen failures. And most of
them have no idea how terrible they are. (8) ___________________ as power tends to corrupt,
so celebrity tends to destroy the ability to gauge whether or not you’re making a fool of (9)
___________________.
But perhaps we shouldn’t criticize celebrities for trying to expand their horizons in this way.
(10) ___________________ there is one good thing about actors trying to sing and singers
trying to act, it is that it keeps them all too busy to write books.
PART 3: For questions 1–10, read the following passage about the inventor of Facebook,
Mark Zuckerberg and choose the best answer (A, B, C, or D) according to the text. Write
your answers (A, B, C, or D) in the corresponding numbered boxes. (10pts)
Mark Zuckerberg was born in 1984, in New York, into a prosperous, professional family. He
developed an interest in computers at an early age and when he was twelve, created a messaging
program which he named “Zucknet”. His dentist father used it in his surgery so that his
receptionist could inform him of the arrival of a patient without yelling across the room.
Together with his friends, Mark spent much of his spare time as a boy designing and making
computer games just for fun.
In 2002, Mark enrolled at Harvard University. By his second year at the institution, he had
acquired a reputation on the campus as a developer of software. It was at that time he invented
“Facemash” which compared pictures of female students and allowed users to vote on which
one was the most attractive. The program became very popular on campus, but was later shut
down by the university authorities who deemed it ‘inappropriate’.

Based on the buzz of “Facemash”, three of Mark’s fellow students, Divya Narendra, and
twins, Cameron and Tyler Winklevoss, asked him to work with them on an idea for a social
networking site called “Harvard Connection”. This site was designed to use information from
Page 7 of 17
Harvard’s student networks to create a dating site for the university’s elite. Mark was initially
enthusiastic about the complex project but boredom soon set in and he backed out of it to work
on his own social networking site. He created a site that allowed users to produce their own
profiles, upload photos, and communicate with other people beyond the confines of the
university. He called the site “Facebook” and he operated from one small college room until
June 2004 when he finally abandoned his studies to devote himself fully to Facebook and moved
the company to California.

By the end of 2004, Facebook had one million users and Mark was climbing high. However,
in 2006, the multi-millionaire business mogul faced his first big hurdle. The creators of Harvard
Connection claimed that he had used their idea, and insisted that he should compensate them for
their business losses. Mark maintained that the ideas were based on two very different types of
social networks but, after lawyers searched his records, incriminating emails revealed that he
may have intentionally stolen the intellectual property of his former partners. Although an initial
settlement of $65 million was reached between the two parties, the legal dispute over the matter
continued well into 2011, after the Harvard Connection creators claimed they were misled with
regards to the value of the Facebook stock.

Since amassing his sizeable fortune, Mark has used his millions to fund a variety of
philanthropic causes. He has put lots of money into the deteriorating schools system in
deprived parts on New York and he actively encourages other wealthy, young entrepreneurs to
follow his example and put back something into society.
1. As a child, Mark Zuckerberg ______.
A. created a software package for dentists
B. developed a communications program
C. sold computer programs to his friends
D. was mainly interested in playing computer games
2. According to the passage, during his time at university, Mark ______.
A. was known for developing unsuitable software
B. made a program that was disliked by the authorities
C. developed an unsuccessful program
D. produced education programs.
3. The Harvard Connection network was ______.
A. originally Mark's idea B. a site for all university students
C. for exchanging academic information D. not interesting enough for Mark
4. The Facebook social network developed at Harvard ______.
A. was a collaborative project with other students
B. could only be accessed by Harvard students
C. became Mark's full-time job after completing university
D. became really successful after June 2004
5. What does the word “mogul” in paragraph 4 is closest in meaning to?
A. tycoon B. thief C. manager D. inventor
6. The creators of Harvard Connection were unhappy because ______.
A. Mark was more successful than they were
B. they had lost a lot of business
C. they thought Mark had used their original concept
D. Mark had stopped working for them
7. Which of the following statement is TRUE according to the passage?
A. Mark did not copy the idea behind Harvard Connection
B. The two parties in dispute were unable to come to an agreement
Page 8 of 17
C. Mark stated that the two websites were not in any way alike
D. The Harvard Connection team were unaware of the value of Mark's company
8. The word “philanthropic” in paragraph 5 can best be replaced by ______.
A. failing B. moral C. problematic D. humanitarian
9. Which of the following is NOT MENTIONED in the passage?
A. Some of the creators of Harvard Connection used to be Mark’s fellow students.
B. Mark has deliberately taken the ideas of the creators of Harvard Connection.
C. Mark has been investing in improving education in less developed areas of New York.
D. The legal dispute that Mark faced started in 2006 and continued well into 2011.
10. Which of the following best serves as the title for the passage?
A. The Inventor of Facebook B. The Legal Dispute over Facebook
C. The Harvard Connection Creators D. The Success of Facebook
Your answers:
1. 2. 3. 4. 5.
6. 7. 8. 9. 10.

PART 4: Read the following text and answer questions. (13pts)


JARGON
A
Jargon is a loaded word. One dictionary defined it, neatly and neutrally, as ‘the technical
vocabulary or idiom of a special activity or group’, but this sense is almost completely
overshadowed by another: ‘obscure and often pretentious language marked by a roundabout way
of expression and use of long words’. For most people, it is this second sense which is at the
front of their minds when they think about jargon. Jargon is said to be a bad use of language,
something to be avoided at all costs. No one ever describes it in positive terms (‘that was a
delightful piece of rousing jargon’). Nor does one usually admit to using it oneself: the myth is
that jargon is something only other people employ.
B
The reality, however, is that everyone uses jargon. It is an essential part of the network of
occupations and pursuits that make up society. All jobs present an element of jargon, which
workers learn as they develop their expertise. All hobbies require mastery of jargon. Each
society grouping has its jargon. The phenomenon turns out to be universal - and valuable. It is
the jargon element which, in a job, can promote economy and precision of expression, and thus
help make life easier for the workers. It is also the chief linguistic element which shows
professional awareness (‘know-how’) and social togetherness (‘shoptalk’).
C
When we have learned to command it, jargon is something we readily take pleasure in, whether
the subject area is motorcycles, knitting, cricket, baseball or computers. It can add pace, variety
and humour to speech - as when, with an important event approaching, we might slip into
NASA-speak, and talk about countdown, all systems go, and lift-off. We enjoy the mutual
showing-off which stems from a fluent use of terminology, and we enjoy the in-jokes which
shared linguistic experience permits. Moreover, we are jealous of this knowledge. We are quick
to demean anyone who tries to be part of our group without being prepared to take on its jargon.
D
If jargon is so essential a part of our lives, why then has it had such a bad press? The most
important reason stems from the way jargon can exclude as well as include. We may not be too
concerned if we find ourselves faced with an impenetrable wall of jargon when the subject
matter has little perceived relevance to our everyday lives, as in the case of hydrology, say, or
linguistics. But when the subject matter is one where we feel implicated, and think we have a

Page 9 of 17
right to know, and the speaker uses words which make it hard for us to understand, then we start
to complain; and if we suspect that the obfuscation is deliberate policy, we unreservedly
condemn, labeling it gobbledegook and calling down public derision upon it.
E
No area is exempt, but the fields of advertising, politics and defence have been especially
criticized in recent years by the various campaigns for Plain English. In these domains, the
extent to which people are prepared to use jargon to hide realities is a ready source of
amusement, disbelief and horror. A lie is a lie, which can be only temporarily hidden by calling
it an ‘inoperative statement’ or ‘an instance of plausible deniability’. Nor can a nuclear plant
explosion be suppressed for long behind such phrases as ‘energetic disassembly’, ‘abnormal
evolution’ or ‘plant transient’.
F
While condemning unnecessary or obscuring jargon in others, we should not forget to look out
for it in ourselves. It is so easy to ‘slip into’ jargon, without realizing that our own listeners/
readers do not understand. It is also tempting easy to slip some jargon into our expression, to
ensure that others do not understand. And it is just as easy to begin using jargon which we
ourselves do not understand. The motivation to do such apparently perverse things is not
difficult to grasp. People like to be ‘in’, to be part of an intellectual or technical elite; and the use
of jargon, whether understood or not, is a badge of membership. Jargon, also, can provide a lazy
way into a group or an easy way of hiding uncertainties and inadequacies: when terminology
slips plausibly from the tongue, it is not essential for the brain to keep up. Indeed some people
have developed this skill to professional levels. And certainly, faced with a telling or awkward
question, and the need to say something acceptable in public, slipping into jargon becomes a
simple way out, and can soon become a bad habit.
Questions 1-6
The Reading has six paragraphs, A-F. Choose the correct heading for each paragraph from the
list of headings below.
LIST OF HEADINGS
i The benefits of simple language
ii A necessary tool
iii A lasting way of concealing disasters
iv The worst offenders
v A deceptively attractive option
vi Differing interpretations
vii Publicising new words
viii Feeling shut out
ix Playing with words
1. Paragraph A ……………
2. Paragraph B ……………
3. Paragraph C …………
4. Paragraph D ……………
5. Paragraph E ……………

Page 10 of 17
6. Paragraph F ……………
Questions 7-12
Complete the summary using the list of words A-H below.
THE UP-SIDE OF JARGON
Jargon plays a useful part in many aspects of life including leisure. For example, when people
take up pastimes, they need to develop a good (7)…………… of the relevant jargon. During
discussion of these or other areas of interest, conversation can become more exciting and an
element of (8)……… can be introduced by the use of shared jargon.
Jargon is particularly helpful in the workplace. It leads to more (9)…………… in the way
colleagues communicate during work hours. Taking part in (10)………… during moments of
relaxation can also help them to bond better.
It is interesting that members of a group, whether social or professional, often demonstrate a
certain (11)…………… towards the particular linguistic characteristics of their subject area and
tend to regard new people who do not wish to learn the jargon with (12)……….
A. Judgement B. jokes C. shop-talk
D. efficiency E. know-how F. command
G. contempt H. feeling I. possessiveness
J. pleasure K. fear L. humor
Do the following statements agree with the views of the writer in Reading Passage ?
YES if the statement agrees with the views of the writer
NO if the statement contradicts the views of the writer
NOT GIVEN if it is impossible to say what the writer thinks about this
Question 13: Jargon should not be used if the intention is to exclude others.
PART 5. You are going to read the transcript of a series of interviews with ordinary people
conducted for the Have you say feature of a daily newspaper. Choose from the people (A-
D). The people may be chosen more than once. Which person gives each of these opinions
about the education system? (15pts)
1………… - The testing of very young people to determine their academic pedigree is unethical.
2………… - The education system is designed in such a way as to unfairly favour people who
have the money to invest in private education.
3…………- Students in countries where extra evening classes have to be attended are more
prone to feeling the effects of extremely tiredness.
4…………- The policy of giving out higher grades is making a mockery of the entire testing
system.
5…………- The degree of difficulty of modern examinations is a lot lower than was the case for
students sitting the same exams in the past.
6…………- Children are more likely to engage with what they are learning if technology is
incorporated into the teaching methods used.
7…………- While our third level education system is well funded and very modern, we have
neglected the primary level, where more investment is required.
8…………- Students should not have to overly exert themselves study-wise in the evenings as
they must also have the opportunity to enjoy their youth.
9…………- While we are often quick to criticize the education system, most of us appreciate
how fortunate we are in this country to have such a good one.
10…………- Many capable people are prevented from pursuing a third level education due to
the financial constraints they are under.

Page 11 of 17
Have you say…on Education
Four ordinary people give their views on the country’s education system

Edward A
The British Education system is fundamentally flawed in numerous ways. For a start, I
believe the 11-Plus exam is morally reprehensive. Children develop at different speeds, so to
promote a situation where we divide up our young so early based on their performance in
one stupid test seems to me ridiculous. Those who pass the 11-Plus are classed as success
stories and they are expected to go on to do great things academically speaking in what
essentially becomes a self-fulfilling prophecy. Meanwhile at such a young age, those who
don’t manage to pass are already been labeled as failure and are not told to set their sights
low. This, sadly, also becomes a self-fulfilling prophecy- of the worst kind; if you expect to
perform poorly and are of the mindset to do so, you will effectively underperform- it is as
good as guaranteed. I see no reason why we have to set children up for a fall like this so
early in life and divide them up into a two-tier education system which provides the upper
tier with a massive advantage over the lower one. I mean, our society has enough class-based
problems without manufacturing more. And that brings me to my next criticism. I believe the
education system in this country is biased towards the privileged. Let’s face it, the best
education is the one which money can buy. Fee-paying schools consistently outperform
schools in the state system, and only the wealthy can exploit the unfair advantage enjoyed by
students who attend these exclusive institutions. And if that wasn’t bad enough- as if
ordinary working class people didn’t face an uphill struggle already, if they do manage to
make it through to university, they are then expected to pay astronomical fee. In many cases,
they are simply priced out of a third- level education system or are forced to take on a
massive burden of debt to finance their studies. Meanwhile, mummy’s little Elton boy can
have his pick of universities and cost is no barrier. It is sad really how unfair the whole
system is.
Eleanor B
People often groan about the faults of our education system, but I think we just like to
complain; my gut feeling is that most people are actually only too aware of how lucky they
are to have such a high quality of formal education open to them for free all the way up to
the end of secondary school. What we take for granted- free education- is not something
students from other parts of the world necessarily enjoy. The standard of teaching in our
schools is only second-to-none. Another thing which can’t be said anywhere. I mean, in
Greece, for example, state school teachers are often so indifferent that students are forced to
attend extra study class at night- the cost of which has to be borne by their parents. Not alone
is this a waste of money, it also eats into students’ free time. The situation is similar in South
Korea- students have private lessons in the evenings to help them improve their state school
grades, and sometimes, between state school classes, private lessons and homework, there
are literally not enough hours in the day, leading to exhaustion and burnout in a worst- case
scenario, and even in the best one, a significant reduction in the amount of leisure time
available to pursue healthy activities and partake in the kind of fun and games that should
characterize youth. In Britain, we go to school from 9 a.m. until 4 p.m.; we have the evening
to enjoy being young and that is how it should be; to quote a well-known proverb, “you are
only young one”.
Michelle C
I think our education system still commands a lot of respect and I think many other countries
are still envious of the kinds of programs we have in place, but I also worry that we are not
investing enough money into schools to enable them to keep up with the latest classroom
innovations. I mean, sure, our universities are state-of-the-art, but the formative years are the
Page 12 of 17
most important of all, and as far as education is concerned, this means that it would be wise
to invest more in the facilities and resources of primary schools where young children will
reap the benefits. Technologically speaking, I would say a lot of our schools are behind the
rest of the developed world. In Japan, for example, every classroom has at least one
computer, as well as a projector screen and a number of other technology-driven interactive
tools. It is vital, in this, the information age, that we introduce kids to technology as early as
possible, and that’s why I strongly feel that there is now a greater need than ever to kit our
primary schools with the latest gadgets. Besides, the children of the information age are
becoming so accustomed to using technology in the other areas of their lives that they will be
more likely respond well to technology-based lessons than the traditional kind, which will
ultimately see them learning more effectively, engaging more genuinely and developing
more speedily.
Alan D
I have a problem with the testing mechanisms used today. I mean, if you look at the statistics
for the exams every year, there is one striking pattern; more and more people are getting As;
the nation’s results on average are getting better and better year-on-year. So, that means one
of two things: either students today are smarter than ever before, or their examinations are
watered down and do not represent a fair test. I personally believe the latter is true and I am
incensed that this is being allowed to happen. In ten or fifteen years’ time, it will have
forgotten to the stage where an A is meaningless if this continue. In order for the education
system to be taken serious, it is vital, therefore, that a complete overhaul of the examination
system takes place and that we return to a situation where examination offer a meaningful
challenge and a true test of ability. That way, when a child receives an A, his or her
achievement will feel genuine; it will have been earned and the child concerned will have the
right to feel very proud of themselves. The problem is, in this country, we have forgotten that
there is absolutely no shame in getting a B, or a C or D grade for that matter. Provided we do
our best, that is always good enough. But, in this politically correct world-gone-mad of ours,
assessors seem to think the only way forward is to give more and more of us the best grades,
devaluing the grading system completely. It is as though they don’t think we could take it if
we got anything less than an A; as though we should all somehow be perfect students. The
problem with that is that it is just not realistic, and, when you set unrealistic objectives, the
only way to achieve them is to “play” with the figures to manufacture the right result.
D. WRITING (60 points)
PART 1. Read the following extract from a professor’s lecture and use your own words to
summarize it. Your summary should be between 150 and 170 words long. (15 pts)
“Now I want to tell you about what one company found when it decided that it would turn over
some of its new projects to teams of people, and make the team responsible for planning the
projects and getting the work done. After about six months, the company took a look at how
well the teams performed. On virtually every team, some members got almost a “free ride” …
they didn’t contribute much at all, but if their team did a good job, they nevertheless benefited
from the recognition the team got. And what about group members who worked especially well
and who provided a lot of insight on problems and issues? Well…the recognition for a job well
done went to the group as a whole, no names were named. So it won’t surprise you to learn that
when the real contributors were asked how they felt about the group process, their attitude was
just the opposite of what the reading predicts. Another finding was that some projects just didn’t
move very quickly. Why? Because it took so long to reach consensus…it took many, many
meetings to build the agreement among group members about how they would move the project
along. On the other hand, there were other instances where one or two people managed to
become very influential over what their group did. Sometimes when those influencers said “That
will never work” about an idea the group was developing, the idea was quickly dropped instead
Page 13 of 17
of being further discussed. And then there was another occasion when a couple of influencers
convinced the group that a plan of theirs was “highly creative.” And even though some members
tried to warn the rest of the group that the project was moving in directions that might not work,
they were basically ignored by other group members. Can you guess the ending to this story?
When the project failed, the blame was placed on all the members of the group.”
Write your answer in the space below.
.............................................................................................................................................................................................................................................
.............................................................................................................................................................................................................................................
.............................................................................................................................................................................................................................................
.............................................................................................................................................................................................................................................
.................................................................................................................................. ...........................................................................................................
..................................................................................................................................................... ........................................................................................
........................................................................................................................................................................ .....................................................................
........................................................................................................................................................................................... ..................................................
.............................................................................................................................................................................................................. ...............................
................................................................................................................................................................................................................................. ............
.............................................................................................................................................................................................................................................
.............................................................................................................................................................................................................................................
.............................................................................................................................................................................................................................................
.............................................................................................................................................................................................................................................
.............................................................................................................................................................................................................................................
.............................................................................................................................................................................................................................................
.............................................................................................................................................................................................................................................
.............................................................................................................................................................................................................................................
............................................................................................................................................ .................................................................................................
............................................................................................................................................................... ..............................................................................
.................................................................................................................................................................................. ...........................................................
..................................................................................................................................................................................................... ........................................
........................................................................................................................................................................................................................ .....................
........................................................................................................................................................................................................................................... ..
.............................................................................................................................................................................................................................................
.............................................................................................................................................................................................................................................
.............................................................................................................................................................................................................................................
Part 2. The charts give data on the proportion of time and the places people of different
age groups in Fantasia spend and go for information on a weekly basis. Summarize the
information by selecting and reporting the main features, and make comparisons where
relevant. Write about 150 words (15pts)

Page 14 of 17
.............................................................................................................................................................................................................................................
.............................................................................................................................................................................................................................................
.............................................................................................................................................................................................................................................
.............................................................................................................................................................................................................................................
.............................................................................................................................................................................................................................................
.............................................................................................................................................................................................................................................
.............................................................................................................................................................................................................................................
............................................................................................................................................ .................................................................................................
............................................................................................................................................................... ..............................................................................
.................................................................................................................................................................................. ...........................................................
..................................................................................................................................................................................................... ........................................
........................................................................................................................................................................................................................ .....................
........................................................................................................................................................................................................................................... ..
.............................................................................................................................................................................................................................................
.............................................................................................................................................................................................................................................
.............................................................................................................................................................................................................................................
.............................................................................................................................................................................................................................................
.............................................................................................................................................................................................................................................
.............................................................................................................................................................................................................................................
.............................................................................................................................................................................................................................................
................................................................................................................................... ..........................................................................................................
...................................................................................................................................................... .......................................................................................
......................................................................................................................................................................... ....................................................................
............................................................................................................................................................................................ .................................................
............................................................................................................................................................................................................... ..............................
.............................................................................................................................................................................................................................................
.............................................................................................................................................................................................................................................
Part 3. There is an idea that young people should not learn what the world is doing, but
they should learn what they can do to the world. To what extent do you agree with the
statement? Give specific example(s) to support your answer. Write about 300 words. (30
points)
.............................................................................................................................................................................................................................................
.............................................................................................................................................................................................................................................

Page 15 of 17
.............................................................................................................................................................................................................................................
.............................................................................................................................................................................................................................................
.............................................................................................................................................................................................................................................
......................................................................................................................................................................................................................... ....................
............................................................................................................................................................................................................................................ .
.............................................................................................................................................................................................................................................
............................................................................................................................................... ..............................................................................................
.................................................................................................................................................................. ...........................................................................
..................................................................................................................................................................................... ........................................................
........................................................................................................................................................................................................ .....................................
........................................................................................................................................................................................................................... ..................
.............................................................................................................................................................................................................................................
.............................................................................................................................................................................................................................................
.............................................................................................................................................................................................................................................
.............................................................................................................................................................................................................................................
.............................................................................................................................................................................................................................................
.............................................................................................................................................................................................................................................
.............................................................................................................................................................................................................................................
.............................................................................................................................................................................................................................................
...................................................................................................................................... .......................................................................................................
......................................................................................................................................................... ....................................................................................
............................................................................................................................................................................ .................................................................
............................................................................................................................................................................................... ..............................................
.................................................................................................................................................................................................................. ...........................
..................................................................................................................................................................................................................................... ........
.............................................................................................................................................................................................................................................
.............................................................................................................................................................................................................................................
.............................................................................................................................................................................................................................................
.............................................................................................................................................................................................................................................
.............................................................................................................................................................................................................................................
.......................................................................................................................................................................................................................... ...................
.............................................................................................................................................................................................................................................
.............................................................................................................................................................................................................................................
................................................................................................................................................ .............................................................................................
................................................................................................................................................................... ..........................................................................
...................................................................................................................................................................................... .......................................................
......................................................................................................................................................................................................... ....................................
............................................................................................................................................................................................................................ .................
.............................................................................................................................................................................................................................................
.............................................................................................................................................................................................................................................
.............................................................................................................................................................................................................................................
.............................................................................................................................................................................................................................................
.............................................................................................................................................................................................................................................
.............................................................................................................................................................................................................................................
.................................................................................................................................................................................................................................... .........
.............................................................................................................................................................................................................................................
....................................................................................................................................... ......................................................................................................
.......................................................................................................................................................... ...................................................................................
............................................................................................................................................................................. ................................................................
................................................................................................................................................................................................ .............................................

Page 16 of 17
................................................................................................................................................................................................................... ..........................
...................................................................................................................................................................................................................................... .......
.............................................................................................................................................................................................................................................
.............................................................................................................................................................................................................................................
.............................................................................................................................................................................................................................................
.............................................................................................................................................................................................................................................
.............................................................................................................................................................................................................................................
........................................................................................................................................................................................................................... ..................
.............................................................................................................................................................................................................................................
.............................................................................................................................................................................................................................................
................................................................................................................................................. ............................................................................................
.................................................................................................................................................................... .........................................................................
.......................................................................................................................................................................................................................................... ...
.............................................................................................................................................................................................................................................
..............................................................................................................................................
-------------- THE END --------------
(Thí sinh không được sử dụng tài liệu. Cán bộ coi thi không giải thích gì thêm)

Page 17 of 17
TRƯỜNG THPT CHUYÊN VĨNH THI GIAO LƯU DHBB
PHÚC NĂM HỌC 2021 – 2022
---------------- MÔN THI: TIẾNG ANH 11
(Đề thi có 13 trang) Thời gian làm bài: 180 phút (Không kể giao đề)
I. LISTENING
Section 1: You will hear an interview with Steve, who talks about the love of his life,
Abby. For questions 1-5, choose the answer (A, B, C or D) which fits best according to
what you hear.
1. How did Abby feel about Steve five years ago?
A. She felt the same way as Steve did about her.
B. She was uncertain about starting any relationship.
C. She thought he was fun to be with occasionally.
D. She looked up to Steve, but didn't love him.
2. How did Steve explain Abby's change of heart initially?
A. He saw it as an aberration, brought on by boredom.
B. He thought she was having a laugh at his expense.
C. He put it down to her being lonely and unattached.
D. He decided something at work must have upset her.
3. What did Steve's work colleagues suddenly notice about him?
A. He was putting in longer hours than he had done.
B. He was showing more commitment to his work.
C. He seemed preoccupied by a personal problem.
D. He spent less time chatting with them in the office.
4. On hearing Steve's declaration,the first thing Samantha did was to
A. get some flowers for Abby.
B. tell Steve's office he was sick.
C. burst into tears at his news.
D. rush round to Abby's place.
5. At the family wedding, Steve
A. announced his plan to get married to Abby.
B. showed an American how to drive a British car.
C. was given advice by someone he didn't know well.
D. was attacked by his mother for ditching Samantha.

Section 2: You are going to hear a writer called Peter Watkins being interviewed bythe
programme presenter, Sue Manchester. He is talking about his book, which discusses
the behaviour of animals and birds in relation to the weather.
For questions 6-10 decide whether these statements are true or false.
6. Sue has little faith in the accuracy of sayings about the weather.
7. Peter says that low-flying birds suffer badly in storms.
8. According to Peter, insects have difficulty in sensing changes in the atmosphere.
9. Peter says that weather sayings used to be confined to the farming community.
10. Sue agrees with Peter about the contradictory nature of some of the sayings.

1
Section 3: You will hear a conversation. Based on what you hear, answer the questions
11-15. Write NO MORE THAN THREE WORDS AND/OR A NUMBER for each
answer.
11. What job was Sam expecting to do?
12. Which city did he fly to?
13. What document, other than his visa, did Sam show the immigration officials?
14. What did the immigration officials buy for Sam?
15. When is the “Welcome Back” party?

Section 4: You will hear a piece of news. Based on what you hear, complete the
sentences with NO MORE THAN THREE WORDS AND/OR A NUMBER from the
listening for each blank.
 The 43-year-old family-run foodstall on a quiet residential street in the suburban area
of Tokyo has been (16)_____________________ for housewives for decades.
 The price of the corn snack, for example, has increased by
(17)_____________________ for the first time for more than 40 years.
 In a society which believes in sharing social burdens, it's almost a
(18)_____________________ to raise prices while wages remain unchanged.
 Hence, (19)_____________________ had to be launched to explain for the rise.
 The inflation is allegedly necessary for Japan to have a (20)_____________________
for the economy.
 This can make a difference, leading the price to go up moderately, not
(21)_____________________.
 As a result, Japan will get out of (22)_____________________ that it has been in for
30 years.
 According to takeshi Ninami, the chief executive of Japan's
(23)_____________________ Santori, Japanese businesses face with
(24)_____________________.
 Japan has been (25)_____________________ the issue of increasing prices for
decades, but in the current situation, actions will soon be taken.

II. LEXICO-GRAMMAR
Section 1: Mark the letter A, B, C or D on your answer sheet to indicate the correct
answer to each of the following questions.
1. We are not known _______ at all; and as we grow, we feel a progress lack of individual
personality.
A. gruelingly B. severally C. expensively D. brusquely
2. The blue sundress set _______ her long blonde hair.
A. up B. off C. forth D. in
3. His poor handling of the business _______ on negligence.
A. neared B. edged C. approached D. bordered

2
4. Brain cancer requires _______ treatment such as surgery.
A. aggressive B. confrontation C. malignant D. rigorous
5. If we _______ over the details, we’ll never finish filming this episode by today.
A. niggle B. discuss C. huddle D. mob
6. Last weekend, _______ nothing to watch on television, we sang karaoke together.
A. there being B. there having C. having had D. being
7. I didn’t really want to go to the party, but I thought I’d better put _______ an appearance.
A. away B. in C. off D. on
8. The upper branches of the tallest trees produce more leaves _______ other branches.
A. than do B. than have C. than they do D. than it does
9. Given that Mary is an _______ liar, you must take what she says with a small grain of
salt.
A. incorrigible B. incurable C. irredeemable D. irremediable
10. In geometry, an ellipse may be defined as the locus of all points _______ distances from
two fixed points is constant.
A. the sum of whose B. of which the sum C. whose sum of D. which the sum of
11. _______ her job, her sons and the housework, she doesn’t have a minute for herself.
A. What with B. If it weren’t for C. Barring D. Given
12. In such a plight _______ that we had no choice but to radio for help.
A. we found ourselves B. we ourselves found
C. did we find ourselves D. did we ourselves find
13. I knew my mother would _______ a face the minute she saw my new haircut.
A. drag B. lift C. pull D. raise
14. ‘Do you think Dennis took the money?’ – ‘I wouldn’t _______ him.’
A. put it past B. think it through C. pass it over D. rub it up
15. Rather than take his time to think about the questions, the interviewee _______ out the
first answer that came into his head.
A. blundered B. blurted C. bungled D. botched
16. The luxurious office accentuated the manager’s position _______. It enhanced his power
and his sense of his own worth. And it made other people feel small.
A. on the pecking pole B. in the nibbling line
C. at the ripping post D. in the pecking order
17. I realized _______ that he was a thief.
A. sooner or later B. all a long C. at the beginning D. eventually
18. His flat looks so _______ that it is difficult to believe he just had a party last night.
A. spick and span B. by and large C. safe and sound D. sick and tired
19. He will not _________ his allegiance to his family
A. aberration B. abogate C. abject D. abjure
20. He intended to ___________ the decree issued by his predecessor.

A. abrogate B. adamant C. abject D. annex

Section 2: Supply the correct forms of the words given.

3
21. The project was subject to the usual ________ of exploratory research. (VICIOUS)

22. The cargo was ________ for safe and efficient shipping. (CONTENT)
23. She ________ questions about whether she plans to run, saying she’s focused on her
voter registration and campaign finance initiatives. (STEP)
24. ‘Villa’ was something of a ________ the place was no more than an old farmhouse.
(NAME)
25. She stood there completely ________ so I had no idea at all what she was thinking.
(EXPRESS)
26. Any actor who becomes known for one role is in danger of becoming ________.
(TYPE)
27. This school was once ________ as a military hospital during the war. (REQUIRE)
28. The rocks appear to be stationary but in the high winds that whip across this desert
landscape, they are in reality moving ________.
29. The boats surrounded the whales, drove them into nets, where they became ________
and were rendered helpless by harpoon thrusts. (MESH)
30. Perhaps ________, recent computer modeling studies predict fewer tropical cyclones if
the ocean heats up further as a result of global warming. (INTUITION)

III. READING
Section 1: You are going to read an extract from a book on human rights. Seven
paragraphs have been removed from the extract. Choose from the paragraphs A-H the
one which fits each gap (1-7). There is one extra paragraph which you do not need to
use.
VALUES FOR A GODLESS AGE
When the Berlin Wall came tumbling down in 1989 so did the plaster cast which had kept
the idea of human rights in limbo. It was now free to evolve in response to the changing
conditions of the late twentieth century.
(1)__________________________
Of course, in one sense, the quest for universal human rights standards after the Second
World War was an early attempt to communicate across national boundaries, albeit a rather
faltering endeavour, with its claims to universality challenged both in terms of authorship
and content. More recently, a loosening of the reins of the human rights dialogue has ushered
in wider debate.
(2)__________________________
Perhaps the best known of these is Amnesty International, established in 1961. Before
Amnesty, there were very few organizations like it, yet now there are thousands operating
all over the world. Whether campaigning for the protection of the environment or third-
world debt relief, any such organization is engaged in the debate about fundamental human
rights. And it is no longer just a soft sideshow.
(3)__________________________
The fact that strangers from different countries can communicate with each other through
the worldwide web is having a similar effect in dealing a blow to misinformation. During

4
one recent major human rights trial over sixty websites sprang up to cover the proceedings,
while sales of the government-controlled new paper in that country plummeted.
(4)__________________________
The effect of increased responsibility at this highest level has been to continually extend the
consideration of who is legally liable, directly or indirectly, under international human rights
law. In part, this is an acknowledgement that even individuals need to be held responsible
for flagrant breaches of others' rights, whether these are preventing protesters from
peacefully demonstrating or abusing the rights of children.
(5)__________________________
It has been noted that paradoxically, in such circumstances, it may be in the interests of
human rights organizations to seek to reinforce the legitimacy and authority of the state,
within a regulated global framework.
(6)__________________________
Part of the new trend in human rights thinking is therefore to include powerful private bodies
within its remit. The International Commission of Jurists has recently explored ways in
which international human rights standards could be directly applied to transnational
corporations.
(7)__________________________
Whatever the way ahead, the lessons of the past must be learnt. Any world view or set of
values which is presented as self-evident is ultimately doomed to failure. The case for human
rights always needs to be made and remade. In a world where globalization too often seems
like a modernized version of old-fashioned cultural imperialism, it is important to query the
claim that human rights are universally accepted.
A. The problem is that the growth of globalization makes the protection of nation states a
pointless goal in certain circumstances. Transnational corporations with multiple
subsidiaries operating in a number of countries simultaneously wield significant economic
and political power and it is often extremely difficult for the state - both home and host
governments - to exercise effective legal control over them.
B. If the proliferation of pressure groups has raised the profile of the human rights debate,
satellite television has reinforced much of the content of their campaigns. The fact that from
our armchairs we can all see live what is happening to others around the world has had an
enormous impact on the way the struggle for human rights is viewed. It would not be
remotely believable to plead ignorance nowadays, for 24-hour news coverage from the
world's hotspots reaches us all.
C. This is, after all, a uniquely propitious time, as the values and language of human rights
are becoming familiar to more and more people, who judge the merits or otherwise of
political and economic decisions increasingly in human rights terms. Arguments seem fresh
and appealing in many quarters where once they sounded weak and stale.
D. On a global scale, it is not strong states that are the problem here but weak ones, as they
fail to protect their citizens from private power - whether it is paramilitaries committing
murder and torture or transnational corporations spreading contamination and pollution.
E. One of the most significant of these is what has come to be called 'globalization', the
collapsing of national boundaries in economic, political and cultural life. From the

5
expanding role of the world's financial markets and the spread of transnational corporations
to the revolution in communications and information technology, more and more areas of
people's lives are affected by regional, international or transnational developments, whether
they are aware of this or not.
F. Not only must states not infringe rights, and enforce those rights which fall within their
direct sphere (like providing a criminal justice system or holding fair elections), but they
also have 'positive obligations' to uphold rights enshrined in human rights treaties, even
when it is private parties which have violated them.
G. The results of its investigations were published in 1 999 in a unique pamphlet on
Globalization, Human Rights and the Rule ofLaw. The issue to be faced is whether to treat
these and other corporations as 'large para-state entities to be held accountable under the
same sort of regime as states', or whether to look for different approaches to accountability
'that are promulgated by consumer groups and the corporations themselves'.
H. No longer the preserve of representatives of nation states meeting under the auspices of
the United Nations, a developing conversation is taking place on a global scale and involving
a growing cast of people - for an increasing range of pressure groups now frame their
aspirations in human rights terms.

Section 2: Fill in each numbered blank with ONE suitable word.


A new threat to our health seems to have arisen in our midst, confusion and stress
brought (1)____ by technology. All you need to do or prove this to (2)____ is to telephone
a large company; a recorded voice will (3)____ you with a bewildering list of choices, and
when you have finished answering its questions, you will probably be subjected to several
minutes of piped music before you eventually make contact with a human being. But the
stress you undergo as a result is negligible compared to the (4)____ the telegraph made on
people 150 years ago. Until (5)____, messages could only travel as fast as a messenger could
carry them. But now they could be sent great (6)____ in seconds. Before long, (7)____
cables were laid across the oceans, and thirty years later, the network reached 20,000 towns
around the world.
Information arrived so quickly, often contradicting what had previously been
transmitted, (8)____ businessmen had to work much harder to (9)____ abreast of
developments. If we find difficulty with the Internet, which is technological evolution,
(10)____ revolution, our ancestors had afar harder task in getting used to the invention in
the first place.

Section 3: Read the following passage and answer questions.


BRINGING UP CHILDREN
Where one stage of child development has been left out, or not sufficiently
experienced, the child may have to go back and capture the experience of it. A good home
makes this possible - for example, by providing the opportunity for the child to play with a
clockwork car or toy railway train up to any age if he still needs to do so. This principle, in
fact, underlies all psychological treatment of children in difficulties with their development,
and is the basic of work in child clinics.

6
The beginnings of discipline are in the nursery. Even the youngest baby is taught by
gradual stages to wait for food, to sleep and wake at regular intervals and so on. If the child
feels the world around him is a warm and friendly one, he slowly accepts its rhythm and
accustoms himself to conforming to its demands. Learning to wait for things, particularly
for food, is a very important element in upbringing, and is achieved successfully only if too
great demands are not made before the child can understand them. Every parent watches
eagerly the child’s acquisition of each new skill: the first spoken words, the first independent
steps, or the beginning of reading and writing. It is often tempting to hurry the child beyond
his natural learning rate, but this can set up dangerous feelings of failure and states of anxiety
in the child. This might happen at any stage. A baby might be forced to use a toilet too early,
a young child might be encouraged to learn to read before he knows the meaning of the
words he reads. On the other hand, though, if a child is left alone too much, or without any
learning opportunities, he loses his natural zest for life and his desire to find out new things
for himself.
Learning together is a fruitful source of relationship between children and parents.
By playing together, parents learn more about their children and children learn more from
their parents. Toys and games which both parents and children can share are an important
means of achieving this co-operation. Building-block toys, jigsaw puzzles and crosswords
are good examples.
Parents vary greatly in their degree of strictness or indulgence towards their children.
Some may be especially strict in money matters; others are severe over times of coming
home at night, punctuality for meals or personal cleanliness. In general, the
controls imposed represent the needs of the parents and the values of the community as
much as the child's own happiness and well-being.
With regard to the development of moral standards in the growing child, consistency
is very important in parental teaching. To forbid a thing one day and excuse it the next is no
foundation for morality. Also, parents should realize that “Example is better than precept”. If
they are hypocritical and do not practise what they preach, their children may grow confused
and emotionally insecure when they grow old enough to think for themselves, and realize
they have been, to some extent, deceived. A sudden awareness of a marked difference
between their parents' ethics and their morals can be a dangerous disillusion.
1. The principle underlying all treatment of developmental difficulties in children _______.
A. is in the provision of clockwork toys and trains
B. is to send them to clinics
C. is to capture them before they are sufficiently experienced
D. offers recapture of earlier experiences
2. Learning to wait for things is unsuccessful taught _______.
A. in spite of excessive demands being made
B. only if excessive demands are avoided
C. because excessive demands are not advisable
D. is achieved successfully by all children
3. The encouragement of children to achieve new skills _______.
A. should be focused on only at school B. can never be taken too far

7
C. will always assist their development D. should be balanced and moderate
4. Parental controls and discipline _______.
A. serve a dual purpose
B. are designed to promote the child’s happiness
C. reflect only the values of the community
D. should be avoided as far as possible
5. The practice of the rule “Example is better than precept” _______.
A. only works when the children grow old enough to think for themselves
B. would help avoid the necessity for ethnics and morals
C. will free a child from disillusion when he grows up
D. is too difficult for all parents to exercise
6. In the first paragraph, the author lays some emphasis on the role of the _______ in helping
the child in trouble.
A. psychiatrists B. community C. family D. nursery
7. The phrase “conforming to” in the second paragraph means _______.
A. adapting to B. accepting C. agreeing with D. following
8. The word “zest” in the second paragraph means _______.
A. appetite B. excitement C. enthusiasm D. enjoyment
9. The word “imposed” in the fourth paragraph is closest in meaning to _______.
A. excepted B. introduced C. made D. constrained
10. Hypocrisy on the part of the parents may _______.
A. result in their children’s wrong behavior
B. make their children lose faith in them
C. disqualify their teachings altogether
D. impair their children’s mind

Section 4: Read the text and do the tasks that follow.


AIR RAGE
(A) The first recorded case of an airline passenger turning seriously violent during a flight,
a phenomenon now widely known as “air rage”, happened in 1947 on a flight from Havana
to Miami. A drunk man assaulted another passenger and bit a flight attendant. However, the
man escaped punishment because it was not then clear under whose legal control a crime
committed on plane was, the country where the plane was registered or the country where
the crime was committed. In 1963, at the Tokyo convention, it was decided that the laws of
the country where the plane is registered take precedence.
(B) The frequency of air rage has expanded out of proportion to the growth of air travel.
Until recently few statistic were gathered about air rage, but those that have been indicate
that passengers are increasingly likely to cause trouble or engage in violent acts. For
example, in 1998 there were 266 air rage incidents out of approximately four million
passengers, a 400% increase from 1995. In the same period American Airlines showed a
200% rise. Air travel is predicted to rise by 5% internationally by 2010 leading to increased
airport congestion. This, coupled with the flying public’s increased aggression, means that
air rage may become a major issue in coming years.

8
(C) Aside from discomfort and disruption, air rage poses some very real dangers to flying.
The most extreme of these is when out of control passengers enter the cockpit. This has
actually happened on a number of occasions, the worst of which have resulted in the death
and injury of pilots or the intruder taking control of the plane, almost resulting in crashes. In
addition, berserk passengers sometimes attempt to open the emergency doors while in flight,
putting the whole aircraft in danger. These are extreme examples and cases of air rage more
commonly result in physical assaults on fellow passengers and crew such as throwing
objects, punching, stabbing or scalding with hot coffee.

(D) The causes of air rage are not known for certain, but it is generally thought that factors
include: passenger behavior and personality, the physical environment and changes in
society. A recent study has identified the issues that start the incidents to be as follows.
Alcohol 25%
Seating 16%
Smoking 10%
Carry on luggage 9%
Flight attendants 8%
Food 5%
(E) One of the major causes seems to be the passenger’s behavior or their personality. Fear
of flying and the feeling of powerlessness associated with flying can lead to irritable or
aggressive passengers. Also, alcohol consumed on a plane pressurized to 8000ft affects the
drinker more quickly and the effects are stronger. Many people do not take account of this
and drinking may increase any negative reaction to the flying environment they have, which,
combined with the lowering of their inhibitions, may cause air rage. Smoking withdrawal,
which some liken in severity to opiate withdrawal, is another major cause of air rage
incidents. Passengers caught smoking in the toilets occasionally assault flight attendants and
have been known to start fires. When conflicts occur in these conditions, they can escalate
into major incidents if the passenger has a violent personality or a fear of flying and because
of the enclosed nature of a plane offers no option of retreat as would be natural in a “fight
or flight” reaction.

(F) Some people feel that the physical environment of a plane can lead to air rage. Seats on
most airlines have become smaller in recent years as airlines try to increase profits. This
leads to uncomfortable and irritated passengers. Also, space for carry-on luggage is often
very small. Because up to 8% of checked in luggage is lost, misdirected or stolen, passengers
have been trying to fit larger carry-on items into these small storage areas and this can lead
to disputes that can escalate into air rage. Airlines could also be to blame by raising
passengers’ expectations too high with their marketing and advertising. Many air rage
incidents start when disappointed passengers demand to be reseated. Finally, there is some
evidence to show that low oxygen levels can raise aggression level and make people feel
more desperate. Airlines have lowered oxygen levels to save money. Now the level of
oxygen in the air that the pilots breathe is ten times higher than in cabin class.

9
(G) Another reason that has been suggested is that society is getting ruder and less patient.
The increased congestion at airports, longer queues and increased delays have only added to
this. In addition, some air rage incidents have been linked to the demanding nature of high
achieving business people, who do not like people telling them what to do and resent the
power that the cabin staff have over them. For them, a flight attendant is a waiter or waitress
who should do what the passenger wants.

(H) The strongest calls for action to control air rage have come from pilots and aircrew. The
International Transport Workers’ Federation argues that there are too many loopholes that
let people escape punishment and that the penalties are too light. They want to notify all
passengers of the penalties for air rage before taking off, rather than after the passenger
begins to cause serious problems, when it may be too late. The Civil Aviation Organization
has been organizing international cooperation and penalties have increased in recent years.
The most severe punishment so far has been a 51-month jail sentence, a fine to pay for the
jet fuel used and 200 hours community service for a man who attempted to enter the cockpit
and to open the emergency door of a domestic US flight.

(I) Various other measures are being used to control air rage. Air crews are getting training
on how to calm passengers and how to predict where incidents might result in air rage and
take action to prevent this. Other measures include, strengthening doors to stop people
entering the cockpit, training crew in the use of plastic restraints to tie down unruly
passengers and having pilots divert their planes if passengers cause problems. Banning
passengers who are guilty of air rage from flying has also been tried to a lesser extent.

Questions 1 – 8
The reading passage has nine paragraphs A – I. From the list below choose the most
suitable headings for B – I. Write the appropriate number (i – xiv) beside in boxes 1 – 8 on
your answer sheet.
NB There are more headings than paragraphs, so you do not have to use them all.

10
List of headings
i The traveler’s character.
ii Disproportionate growth.
iii Pilots and aircrew.
iv Additional action.
v Smaller seats.
vi Uncomfortable aeroplanes
vii Origins.
viii A major threat.
ix Demands for change.
x Business people.
xi The roots of the problem.
xii The pace of life.
xiii Links to the surroundings.
xiv Personal experience.
Example:
Paragraph A Answer: vii
1. Paragraph B __________
2. Paragraph C __________
3. Paragraph D __________
4. Paragraph E __________
5. Paragraph F __________
6. Paragraph G __________
7. Paragraph H __________
8. Paragraph I __________

Questions 9 – 13
Do the following statements agree with the information in the above reading sample text?
Mark them as follows:
T if the statement agrees with the information in the text.
F if the statement does not agree with the information in the text.
NG if there is no information on this in the text.
9. In the first case of air rage, the man was not punished because the plane was not
registered.
10. The statistics on air rage were collected by private monitoring groups.
11. The environment in a plane makes disagreements more likely to become serious
problems.
12. Airlines have been encouraging passengers to bring more items onboard as carry-on
luggage.
13. It has been impossible to ban passengers with histories of air-rage.

11
Section 5: You are going to read a magazine article about health and fitness. For
questions 1-10, choose from the sections (A-D). The sections may be chosen more than
once.
Which person
1. mentions being disconcerted by their lack of ability when faced with a completely new
activity?
2. was grateful for having been spurred on in their efforts?
3. suggests that prior experience of the exercise method can be advantageous?
4. suggests that they have overreached themselves during their first session?
5. is sceptical about whether a way of exercising would really appeal to them?
6. suggests that their chosen exercise programme seemed to be based on a slightly eccentric
premise?
7. rejects the idea that they are following an exercise programme to improve fitness?
8. comments on the relentless nature of the trainer?
9. contrasts the amount of pleasure to be gained from different types of exercise?
10. suggests that the outcome oftheir exercise programme was not wholly positive?
A. Cloe: New Pilates
I’ll be honest, I have never felt the natural high which scientists claim follows a bout of
intense exercise. The empirical evidence of my own body tells me that the only thing
exercise releases in my brain is loathing. So I scoffwhen the people at the gym tell me I'll be
hooked on a new type of Pilates in two sessions. My first session is an hour's one-on-one
with Daniel, my trainer; a good idea for any beginner. Although I, disappointingly, don't
actually lose any weight.over my six sessions - personally, I find it rather ups my appetite -
I can attest to its toning abilities. The classes themselves - which take a maximum of six
people - are entertaining, and as agreeable as enforced muscle fatigue can ever be. It
definitely helps ifyou'd already got to grips with some basic Pilates techniques before you
start, but, once you've got the hang of commands such as 'squeeze that imaginary grape
under your armpit; it provides a great variety of exercise.
B. Mark: Personal training
A month of sessions with a personal trainer three times a week seems like the perfect
springboard to a better future. My personal trainer,Tony, asks me what I hope to achieve. I
mutter something about losing a few pounds and toning up a bit, but the truth is I want to
get back into my tailormade suits. It's the gap between my expectations and reality that is
hardest to contend with. I know that no matter how healthy I become at the age of 36, I will
still be less fit than I was as a lazy 1 8-year-old who did no exercise at all. But if I'm honest,
I secretly believed I wouldn't actually be all that bad at this. The problem is weights. I've
never bothered with them before. I take it slowly for the first few sessions but it's hard going
and I eventually pull a muscle in my right arm. It's time for a few days off. I greet a four-
day respite with enthusiasm, but actually find myself in the gym, running faster and longer
than before and lifting weights well.
C. Ben: Sport Active
I go along to my nearest fitness centre and decide to try out the DVD of Sport Active, which
has more than 70 different exercises on it. The programme can measure and display your

12
heart rate, thanks to a monitor that straps to your forearm which sends information to the
console. I start with tennis and get an enormous kick out of hitting balls into an onscreen
net. I quickly move on to mountain biking, or, as I now call it, 'total physical punishment'.
However, even though I am an old hand at cycling, by halfway round, I have clearly lost all
ability to show off. On screen, my heart rate has rocketed up to 1 78. 'You're definitely
getting a good cardio workout here; encourages Robert, the fitness centre trainer. Could
these games damage people by suggesting the wrong positions? Robert is dismissive: 'It's
unlikely you're going to hurt yourself: I decide to carry on and after a few weeks begin to
see the benefits.
D. Tasha: Wild fitness
Wild Fitness is more than a form of exercise, Matt, my trainer, told me that it was a whole
philosophy of life: to transform yourself by learning to move and eat in the way of our
hunter-gatherer ancestors and to become strong, fast and agile. It all sounded a bit bizarre
but I was more than happy to give it a go. The first session began at Sam on a Monday
morning in Regent's Park, London, with some introductory exercises. The hardest session
came the next week when Matt told us we would sprint around the 400-metre running track
four times, with a short rest in-between - no excuses allowed. I did my best and then
discovered that the so-called rest was going to involve squat thrusts; 20 of them. Matt didn't
stop there. It was thanks to him that I did far more than I would ever have done exercising
alone and I looked thinner and was far more toned as a result, especially around my thighs
and stomach.

IV. WRITING
Section 1: Write a summary of 100-120 words on the following passage.
The reach of the media, in the present times of 24-hour channels, is to almost every nook
and corner of the world. Further, large number of people believe as correct that which
appears in media, print or electronic. It is also necessary to always bear in mind that the
judiciary is the last resort of redressal for resolution of disputes between State and the
subject, and high and low.
The confidence of the people in the institution of judiciary is necessary to be preserved at
any cost. That is its main asset. Loss of confidence in institution of judiciary would be end
of rule of law. Therefore, any act which has such tendency deserves to be firmly curbed. For
rule of law and orderly society, a free responsible press and independent judiciary are both
indispensable. Both have to be, therefore, protected.
The judgments of courts are public documents and can be commented upon, analysed and
criticized, but it has to be in dignified manner without attributing motives. Before placing
before public, whether on print or electronic media, all concerned have to see whether any
such criticism has crossed the limits as aforesaid and if it has, then resist every temptation
to make it public.
In every case, it would be no answer to plead that publication, publisher, editor or other
concerned did not know or it was done in haste. Some mechanism may have to be devised
to check the publication which has the tendency to undermine the institution of judiciary.

13
Section 2: Write a report of at least 150 words on the following graphs.
The following graphs give information about sports activity in a college last year.
Summarize the information by selecting and reporting the main features and make
comparisons where relevant.

Section 3: Write an essay of at least 150 words on the following topic.


Some people say advertising has positive economic effects. Others think it has negative
social effects because it will make people dissatisfied with who they are and what they have.
Discuss both views and give your own opinion.

_____________________THE END_____________________

14
HỘI CÁC TRƯỜNG THPT CHUYÊN KÌ THI HỌC SINH GIỎI NĂM 2022
VÙNG DUYÊN HẢI VÀ ĐỒNG BẰNG BẮC BỘ MÔN: TIẾNG ANH. LỚP 11
TRƯỜNG THPT CHUYÊN TUYÊN QUANG Thời gian: 180 phút
----------- Đề thi gồm 15 trang
ĐỀ THI ĐỀ XUẤT

SECTION I. LISTENING (50pts)


HƯỚNG DẪN PHẦN THI NGHE HIỂU
 Bài nghe gồm 4 phần; mỗi phần được nghe 2 lần, mỗi lần cách nhau 05 giây; mở đầu và kết thúc
mỗi phần nghe có tín hiệu. Thí sinh có 20 giây để đọc mỗi phần câu hỏi.
 Mở đầu và kết thúc bài nghe có tín hiệu nhạc. Thí sinh có 03 phút để hoàn chỉnh bài trước tín hiệu
nhạc kết thúc bài nghe.
 Mọi hướng dẫn cho thí sinh (bằng tiếng Anh) đã có trong bài nghe.

Part 1. For questions 1-5, listen to part of a radio interview in which two academics called, Jonh
Farrendale and Lois Granger, taking part in a discussion on the subject of attitudes to work and
choose the answer (A, D, cor D) which fits best according to what you hear. Write your answers
in the corresponding numbered boxes provided. (Expert Proficiency)

1. Lois agrees with John's point that


A. most people dread the prospect of unemployment
B. the psychological effects of unemployment can be overstated
C. some people are better equipped to deal with unemployment than others.
D. problems arise when unemployment coincides with other traumatic events.
2. Lois agrees with the listener who suggested that
A. work is only one aspect of a fulfilling life.
B. voluntary work may be more rewarding than paid work.
C. not everybody can expect a high level of job satisfaction
D. people should prepare for redundancy as they would for retirement
3. What is John's attitude towards people who see work as a means to an end?
A. He doubts their level of commitment to the job.
B. He accepts that they have made a valid choice.
C. He fears it will lead to difficulties for them later.
D. He feels they may be missing out on something important
4. When asked about so-called 'slackers' at work, John points out that
A. they accept the notion that work is a necessary evil.
B. people often jump to unfair conclusions about them.
C. their views are unacceptable in a free labour market.
D. such an attitude has become increasingly unacceptable.
5. Lois quotes the psychologist Freud in order to
A. show how intellectual ideas have shifted over time.
B. provide a contrast to the ideas of Bertrand Russell.
C. question the idea that a desire to work is a natural thing.
D. lend weight to John's ideas about increased social mobility
Your answers:
1. 2. 3. 4. 5.
Part 2: You are going to listen to a report on how European countries are dealing with the
coronavirus pandemic and decide whether these statements are TRUE (T) or FALSE (F). Write
your answer in the corresponding numbered box provided (10 pts)
https://www.youtube.com/watch?v=K30zVlSWJXw

6. Under the full lockdown in Italy, all grocery stores must be shut down.
7. There is a ban on entry for people from Austria into Italy
8. Angla Merkel warned that Coronavirus could infect up to 17% of Germany’s population.
9. Public gatherings still take place in the UK.
10. According to the professor, although the UK is taking more drastic measures than Italy, its
effectiveness is open to question.
Your answers:
6. 7. 8. 9. 10.

Part 3: You will hear a residence manager giving a short question and answer session to new
residents.Write NO MORE THAN THREE WORDS AND/OR A NUMBER for each answer.
Write your answer in the spaces provided. (10 pts)
https://ieltshelpnow.com/

11. Where will the future residence launderette be?


………………………………………………………………………..
12. What special facility does Betty offer at the Launderette?
………………………………………………………………………..
13. Which number bus does Simon advise Jo to take?
………………………………………………………………………..
14. Where can students get advice on Council Tax?
………………………………………………………………………..
15. Where does Simon say that Bill should register for his Council Tax?
………………………………………………………………………..

Part 4: For questions 16-25, listen to a talk about how pandemics are linked to climate change and
supply the blanks with the missing information. Write NO MORE THAN FOUR WORDS taken
from the recording for each answer in the space provided.

https://www.youtube.com/watch?v=Q53RnXk-2OI.

The prevailing catastrophe is considered an (16)________ with the outbreak of a nightmarish novel virus
which has a tremendous level of (17) ________. Humanity has already witnessed the contribution that
climate change has made to the rising likelihood of pandemics such as Zika, Ebola, SARS and recently,
Covid-19. By forcing wild animals to (18) ________ and move nearer to human due to loss of habitat
and inappropriate temperature, climate change has cleared the way for diseases to spread from these
animals to our community. Deforestation for (19) ________ _ in fact, is the major culprit of so serious
a situation. In addition, air pollution carries an alarming risk for (20) ________, which accounts for high
lethality level among respiratory diseases like Covid-19. The immense population density in
cosmopolitan areas worldwide also means frequent human-to-human contacts that in turn, increases the
peril of (21) ________ · Another drive behind the pandemic is civil aviation. The fact that only Chinese
passengers were (22) ________ for coronavirus initially, for instance, caused other infected travellers
to (23) ________ nationwide. To protect our planet from both the (24) ________ of climate disruption
and Covid-19, it's high time urgent action was taken. However, dangers can be expected from the
pathogens as we necessarily remove (25) ________ of life on our planet.

Your answers
16. 17. 18. 19. 20.
21. 22. 23. 24. 25.
SECTION II. GRAMMAR & VOCABULARY (30pts)
Part 1. Choose the correct answer A, B, C or D to each of the following questions and write your
answers in the corresponding numbered boxes provided. (20 pts)
26.Many people are feeling the _______ now that there is an economic recession.
A. strain B. prod C. pinch D. pain
27. The fact that his alibi didn’t check out was _______ evidence.
A. discriminating B. incriminating C. invigorating D. undulating
28. Although the company seems very successful and popular, it has ______ actual money. Everything
is built off loans and debts.
A. less or no B. little or no C. many or not D. not any or little
29. The man accused of robbery asked that he _______ on bail until the court hearing.
A. will be released B. is to released C. be released D. was releasing himself
30. The government’s environmental policy has ________ criticism.
A. led up to B. come in for C. clung on to D. gone on for
31. The city of Shanghai is a _______ of modern-day ChinA.
A. microclimate B. microcopy C. microcosm D. microscope
32. “Did you attend the economic planning lecture?”
“Yes, and unfortunately it wasn’t ________ I’d expected”
A. as nearly boring as B. nearly as boring as C. nearly boring as D. so boring as
33. It was _______ that Henry would fail his final exams because he hadn’t even opened up his books
all semester.
A. indelible B. inevitable C. inconceivable D. incomprehensible
34. Sandra must be very _______ to be able to ignore all of her colleagues’s rude remarks.
A. single-minded B. two-faced C. short-sighted D. thick-skinned
35. We've made some great improvements over the past three months, but we're still not out of the _______
A. jungle B. bush C. dark D. woods
36. Many homeless people have been ______ for years with little prospect of finding proper housing.
A. down the drain B. down and dirty C. down and out D. down in the dumps
37. The accountant ______ the company out of millions of dollars before he was caught.
A. swindled B. spun C. dwindled D. saddled
38. The political candidate always tries to ______ any difficult questions when talking to the press.
A. butter up B. fend off C. fawn over D. drive back
39. Money was short and people survived by ______and saving.
A. scrimping B. scavenging C. scouring D. scrounging
40. Jane was terribly nervous before the interview but she managed to pull herself ______ and act
confidently.
A. through B. over C. together D. off
41. Round and round ___________.
A. went the wheels of the engine B. the wheels of the engine went
C. did the wheels of the engine go D. going the wheels of the engine
42. The delight in treasure finding doesn’t always ________acquiring tremendous amounts of valuables.
A. dwell on B. poke around C. lay about D. hinge upon
43. The luxurious office accentuated the manager's position ________. It enhanced his power and his
sense of his own worth. And it made other people feel small.
A. on the pecking pole B. in the nibbling line
C. at the nipping post D. in the pecking order
44. _________ chair the meeting.
A. John was decided to B. There was decided that John should
C. It was decided that John D. John had been decided to
45. The pollution problems in the town have been ______ by mass tourism in the summer months.
A. exacerbated B. developed C. augmented D. contributed
Your answers:
26. 27. 28. 29. 30. 31. 32 33. 34. 35.
36. 37. 38. 39. 40. 41. 42. 43. 44. 45.

Part 2: Use the correct FORM of the word in capitals to fit each gap. Write your answer in the
numbered box. (10 pts)
46. A _______ celebrity featured in an advertising campaign can increase product sales. CHARISMA
47. The new computer lab is open-plan and furnished in a _______ style with basic pieces in white.
MINIMISE
48. The American landscape is being ________ by malls and fast-food restaurants. GENE
49. _______ tools, machines and even structures could redefine how humans live and work. MORPH
50. The complexity of the ecosystem will best be understood through a _______ approach. DISCIPLINE
51. The way that advertisers and the marketers play on people’s insecurities is thoroughly _______ .
MANIPULATE
52. A number of cosmetic manufacturers complained about the requirement for product comparison
tests to validate claims of ______. ALLERGY
53. The ensuring _______ caused the meeting to be rescheduled DEMONISE
54. It is an ________ fact that vaccination is not the one-stop solution to the Covid-19 pandemic.
AXIOM
55. Surely, it’s _______ for the company to claim its product has health benefits when it doesn’t FRAUD

Your answers

46. 47. 48. 49. 50.


51. 52. 53. 54. 55.

SECTION III. READING COMPREHENSION


Part 1. Read the passage below and fill each of the following numbered spaces with ONE suitable
word. Write your answers in the corresponding boxes provided. (15p)

THE BENEFITS OF SLEEPING ON THE JOB

Numerous factors dictate the optimal amount of hours an individual should sleep each night, though, for
the average working adult eight hours is the most oft-quoted figure. In recent times, getting a good
night's sleep has been steadily slipping down people's lists of priorities. So (56) ______ so, in fact, that
one in three British people now suffer from some form of sleep deprivation. But in Japan, where the
average adult gets just seven hours and five minutes of sleep (57)______night, workers don't have to
struggle through the day with heavy eyelids. Instead, they can have a short nap! The practice is called
inemuri, which literally means 'to be asleep while present', and is an acceptable feature of Japanese work
environments. In fact, (58) ______ from being embarrassed about falling asleep at work, the Japanese
are actually quite proud of nodding (59) ______ on the job. Inemuri is viewed as an indication that a
worker is working too strenuously and the practice is (60) ______ respected that some Japanese workers
even pretend to be asleep at their desks! Inemuri does have an unwritten code of practice, though. You
must sit (61)______ in your chair while you are napping, you shouldn't nap in front of a superior, and
you shouldn't nap for more than around twenty minutes a day. On the (62)______, many experts believe
the West has much to learn from Japan's enlightened attitudes towards sleeping at work - both from the
point of (63) ______ of health and productivity levels at work. For instance, recent research suggests
that American companies lose 520 hours of productivity per employee per year (64)______ failing to
manage their energy levels properly. By taking power naps, research suggests, workers can recharge
their energy levels. Nevertheless, it (65) ______ to be seen whether sleeping on the job ever becomes
culturally acceptable outside of Asia.
Your answers:

56. 57. 58. 59. 60.


61. 62. 63. 64. 65.

Part 2. Read the text below and decide which answer (A, B, C or D) best fits each gap. Write your
answers in the corresponding numbered boxes provided. (10p.)

Science plays a crucial role in identifying problems related to how natural systems function and
deteriorate, particularly when they are affected by an external factor. In turn, scientific findings shape
the policies introduced to protect such systems where necessary. Experts are frequently called upon by
politicians to provide evidence which can be used to make scientifically sound, or at least scientifically
justifiable policy decisions.

Issues arise as there are frequent disagreements between experts over the way data is gathered and
interpreted. An example of the former is the first scientific evidence of a hole in the ozone layer by the
British Antarctic Survey. (A) The findings were at first greeted by the scientific community with
scepticism, as the British Antarctic Survey was not yet an established scientific community. (B)
Moreover, it was generally believed that satellites would have picked up such ozone losses if they were
indeed occurring. (C) It was not until the methodology of NASA's Goddard Space Flight Center was
reviewed that it became apparent that data had been overlooked. (D)

With regards to the latter, controversy between scientists may arise where data analysis appears to
support one policy over another. In 1991, the World Resource Institute (WRI) published estimates of
net emissions and sinks of greenhouse gases for a number of countries, including India. The report
provoked criticisms among Indian scientists who argued that the figures had failed to take some
significant factors into account, leading to overestimated emission values. The WRI was accused of
blaming less economically developed countries for global warming; a stance which, if accepted, could
impede industrialisation and sustain, even widen, the wealth gap.

Problems regarding the scientific method are well documented and it is widely accepted by the scientific
community that, however consistent scientists are in their procedures, the results born under different
circumstances can vary markedly. A number of factors influence research, among them the organisation
of a laboratory, the influence of prevailing theories, financial constraints and the peer review process.
Consequently, scientists tend to believe they are not in a position to bear universal truths but to reveal
tendencies.

However, this is countered by two factors. Firstly, certain scientific institutions wish to maintain a
degree of status as ‘bearers of truth’. Further, policy makers uphold this understanding by requesting
scientific certainties in order to legitimise their policy decisions. According to a number of authors who
have documented this process, decision makers do not necessarily try to obtain all the information which
is or could be made available regarding an issue. Rather, they select that information which is necessary
to fulfil their goals, information termed as ‘half-knowledge’. Attempts to underplay transboundary
issues such as water provision and pollution are cases in point. Politicians clearly cannot pretend that
certain data do not exist if they are well-known in scientific communities or national borders, but some
discretion is evident, especially where there is controversy and uncertainty.

It is important to note that policies regarding scientific issues are influenced in no small part by societal
factors. These include the relative importance of certain environmental issues, the degree of trust in the
institutions conducting the research, and not least the social standing of those affected by the issue. In
other words, environmental problems are in many ways socially constructed according to the prevailing
cultural, economic and political conditions within a society. It has been suggested, for example, that
contemporary ‘post-materialist’ Western societies pay greater attention to ‘quality’ – including
environmental quality – than ‘quantity’. This theory does not necessarily assume that people of low-
income countries have no interest in environmental protection, as the example of the Chipko movement
in India clearly demonstrates but demonstrates that the way a resource is valued varies widely among
different communities.
Finally, it cannot be denied that the ‘issue of the day’ changes constantly. One issue becomes more or
less urgent than another, based on current events. Concurrently, new issues enter the political agendA.
It has been noted that it often takes a ‘policy entrepreneur’, someone who dedicates time, energy and
financial resources to a certain issue, to raise its profile. Furthermore, whether an issue is taken up by
political, environmental or media groups, depends very much on the degree to which it suits their
particular agenda, not to mention budget.

66. With reference to paragraph 1, which of the following pieces of research would NOT be relevant
to this article?
A. the effect of climate change on weather patterns in Africa
B. whether or not low-level radiation increases the risk of cancer
C. how acid rain impacts species within a lake ecosystem
D. a comparison of the species present in two areas of woodland
67. What is the purpose of the example of ozone data given in paragraph 2?
A. to show that NASA’s Goddard Space Flight Center used unreliable methods of gathering scientific data
B. to show how data gathering methods and the status of scientists may affect the way data is regarded
C. to prove that it is wrong to dismiss evidence which comes from a non-established source
D. to show how NASA and the British Antarctic Survey disagreed over the correct way to gather
ozone data.
68. Where in paragraph 2 does this sentence best fit?
This was because of the way their computers had been programmed to discard any readings which
appeared anomalous.
A. (A) B. (B) C. (C) D. (D)
69. Paragraph 3 gives an example of a dispute over __________.
A. which country was most responsible for producing greenhouse gases
B. the pollution caused by multinational companies in India.
C. how statistics were interpreted and presented.
D. erroneous data which resulted from a poorly-funded experiment.
70. In paragraph 5, ‘this’ refers to __________.
A. the scientific method and its inherent problems.
B. the belief that scientists cannot reveal universal truths.
C. the variation in scientific results under different circumstances.
D. the list of factors which influence scientific research.
71. What is meant by this sentence?
‘Further, policy makers uphold this understanding by requesting scientific certainties in order to
legitimise their policy decisions’.
A. Politicians when seeking evidence for policy-making, do not understand the fact that scientists are
unable to act as ‘bearers of truth’.
B. Politicians consider the scientific research that supports their policies as more legitimate than other research.
C. Scientific institutions encourage politicians to use them for policy-making in order to improve their status.
D. Politicians, when seeking evidence for policy-making, encourage the belief that scientists can produce
incontestable facts.
72. Which sentence best sums up the ideas in paragraph 4?
A. Scientists are aware that their work cannot present incontrovertible facts.
B. If scientists were more consistent, they could create more reliable evidence.
C. Variations in how research is conducted often affect its validity.
D. Scientists spend more time documenting problems than conducting research.
73. Why are ‘transboundary issues such as water provision and pollution’ referred to in paragraph 5?
A. to illustrate situations in which politicians pretend that certain data does not exist
B. to illustrate situations in which incorrect information is given by scientific institutions keen to
maintain their status.
C. to illustrate situations in which politicians are selectiive with regards to what data they gather
D. to illustrate situations in which policy makers request scientists to present them with scientific
certainties, even though none exist.
74. What can be inferred about the Chipko movement?
A. It was an example of how people in low-income countries have little interest in environmental protection.
B. It was an example of how different people within a community valued a resource differently.
C. It was an example of how people in a low-income community showed interest in protecting the environment.
D. It was an example of how people in a low-income community valued quantity over quality.
75. Which of the following arguments is NOT presented in paragraph 7?
A. An issue only get political or media attention if someone with a high profile is supporting it.
B. Politicians are only interested in environmental issues if it benefits them.
C. Issues don’t get public attention unless a particular person advocates it strongly.
D. Issues may be overlooked if there are other significant events happening at the time.
Your answers:

66. 67. 68. 69. 70.


71. 72. 73. 74. 75.

Part 3: Read the text and do the following tasks (13pts)

Researcher on the Tree Crown

A. The forest canopy – the term given to the aggregated crowns of trees in a forest – is thought to host
up to 40 per cent of all species, of which ten per cent could be unique to the forest roof. “We’re dealing
with the richest, least known, most threatened habitat on Earth,” says Andrew Mitchell, the executive
director of the Global Canopy Programme (GCP), a collection of groups undertaking research into this
lofty world. “The problem with our understanding of forests is that nearly all the information we have
has been gleaned from just two meters above the soil, and yet we’re dealing with trees that grow to
heights of 60 meters, or in the case of the tallest redwood 112 meters. It’s like doctors trying to treat
humans by only looking at their feet.”
B. Tropical rainforest comprises the richest of ecosystems, rivalled only by coral reel for its diversity and
complex interrelationships. And a great deal of that diversity lives up in the canopy – an estimated 70-
90 per cent of life in the rainforest exists in the trees; one in ten of all vascular plants are canopy dwellers,
and about 20-25 per cent of all invertebrates are thought to be unique to the canopy.
C. The first Briton to actually get into the canopy may have been Sir Francis Drake who, in 1573, gained his
first glimpse of the Pacific Ocean from a tall tree in Darien, Panama. However, the first serious effort to
reach and study the canopy didn’t begin until 1929. The Oxford University Expedition to British Guiana,
led by Major RWG Hingston, still ended up help of locals when it came to building an observation
platform. It was a successful expedition all the same, despite the colony’s acting governor getting stuck
high up on a winched seat during a visit. In terms of canopy access, the French have proved themselves
to be excellent innovators, taking things further with the development of ‘lighter-than-air platforms -
balloons and related equipment, to you and me. Francis Halle; from the Laboratoire de Botanique
Tropicale at Montpellier University took to a balloon in the mid-1980s in order to approach the canopy
from above. His work in French Guiana was inspired by the use in Gabon of a tethered helium balloon
by Marcel and Annette Hladick. Halle went one further by using a small purpose-built airship-a cigar-
shaped balloon with propellers to aid manoeuvrability. “We suddenly had a mobile system that could
move around the treetops; there were no other means of doing this,” says Mitchell.
D. From this, two balloon-dependent features have developed: the radeau or raft, and the luge or sledge.
The raft is a ‘floating’ platform, employed by French academics Dany Cleyet-Marrel and Laurent Pyot
and is essentially an island in the treetops. Made of kevlar mesh netting and edged with inflated neoprene
tubes, it rests on top of the canopy, allowing sampling (mostly of plants and insects) to take place at the
edges of the platform, and can stay in position for several days. The luge, on the other hand, is an inflated
hexagon similar to a traditional balloon basket but with a hole in the bottom covered with Kevlar mesh.
Such techniques aren’t without their problems, however, “balloons can cover larger areas, especially for
collection purposes, but they are extremely expensive- Jibe raft alone cost 122,000 [euro] (86,000
[pounds sterling]) in 2001], nut very effective because you can only reach the tops of the trees, and are
highly dependent on the weather,” says Dr Wilfried Morawetz, director of systematic botany at the
University of Leipzig. “Balloons can usually only be used in the early morning for two to four hours.
Last time, we could only fly three times during the whole week.” Given these factors, it comes as no
surprise that operations involving these balloons numbered just six between 1986 and 2001.
E. Smithsonian Tropical Research Institute in Panama. Smith had the idea of using a static crane to get
into the treetops. Un-tethered balloons may allow widely distributed sites to be sampled, but cranes
allow scientists to study an area of at least a hectare from soil to canopy throughout the year, year after
year. “Cranes beat any other access mode. They are cheap, reliable and fast. In two minutes I can reach
any point in our forest, which is essential for comparative measurements across species,” says Professor
Christian Korner of the University of Basel. Korner is using a static crane in a unique carbon dioxide-
enrichment experiment in Switzerland, in an attempt to discover how forests might respond to the global
increase in atmospheric carbon dioxide (see Swiss canopy-crane carbon experiment, right). For reasons
of convenience, cranes are generally situated close to cities or a research center. Leipzig University has
a crane not far from the town, the Location allowing scientists to study the effect of city pollutants on
forests. In order to increase the amount of canopy a crane can access, some have been mounted on short
rail tracks. In “1995, Dr Wilfried Morawetz was the first to use this technique, installing a crane on 150
meters of track in the Venezuelan rainforest. “In my opinion, cranes should be the core of canopy
research in the future,” he says.
F. It appears that the rest of the scientific community has now come around to Mitchell’s way of
thinking. “I think most scientists thought him mad to consider such a complex field station at first,” says
internationally respected ‘canopist’ Meg Lowman, the executive director of the Marie Selby Botanical
Gardens. “However, we’ve all come to realize that a combination of methods, a long-term approach to
ecological studies and a collaborative approach are the absolute best ways to advance canopy science.
A permanent canopy field station would allow that to happen.” With A dedicated group of canopy
scientists working together and a wide range of tools available for them to get into the treetops, we’re
now finally on our way towards a true understanding of the least-known terrestrial habitat.

Questions 76-80: The reading Passage has seven paragraphs A-F. Which paragraph contains
the following information? Write the correct letter A-F, in boxes 76-80

76 ______The Scientific significance for committing canopy study.


77 ______The first academic research attempt mentioned to getting the top canopy.
78 ______The overview idea of the forest canopy and the problem of understanding the forests.
79 ______Recognition for a long-term effect and cooperation.
80 ______An innovation accessing to treetop which proved to be an ultimate solution till now.

Questions 81-84: Complete the following summary of the paragraphs of Reading Passage
Using NO MORE THAN TWO WORDS from the Reading Passage for each answer.

Scientists keep trying new methods to access to the canopy of the treetop. Though early attempt
succeeded in building an observation platform yet the help from the (81) ______was imperative; further
innovators made by the French who built a platform with equipment by using (82) ______. Later, the
‘floating’ platform of (83) ______is serving as an island in the treetops. Then finally, there came the
next major breakthrough in Panama. Scientists applied (84) ______to access to the treetops, which are
proved to be the center of canopy research in today and n the future.
Questions 85-88: Use the information in the passage to match the category (listed A-F) with
opinions or deeds below. Write the appropriate letters A-F in boxes 85-88
NB You may use any letter more than once

A Sir Francis Drake


B Wilfried Morawetz
C Dany Cleyet-Marrel
D Francis Halle
E Christian Korner
F Alan Smith

85 ______ Scientist whose work was inspired by the method used by other researchers.
86 ______ Scientist who made a claim that a balloon could only be used in a limited frequency or time.
87 ______ Scientist who initiated a successful access mode which is cheap and stable.
88 ______Scientist who had committed canopy-crane experiment for a specific scientific project .

Your answers:

76. 77. 78. 79. 80. 81. 82.


83. 84. 85. 86. 87. 88.

Part 4: In the passage below, seven paragraphs have been removed. Read the following passage
and choose from paragraphs (A-H) the one which fits each gap. There is ONE extra paragraph
which you do not need to use. Write your answers in the corresponding numbered boxes provided.
(7 pts)

THE SOCIAL MEDIA PRODUCT PLACEMENT JUGGERNAUT

Social media is the air that celebrities breathe. No other platform can keep them so relevant in the cut-
throat business of fame. But beyond satisfying their need to connect with the public, social media is also
helping to fill bank accounts. To maintain their fabulous lifestyles, celebrities can turn to social media
where the world of Instagram and Infuencer marketing awaits. And what lucrative world it is.
89. _____________
So what do we know about specific rates for social media posts? Mid-level reality TV stars can earn
anywhere from $1,000 to $20,000 for a brand sponsored Instagram post. This can go as high as $50,000
for a TV sitcom star. That’s a lot of money, but relatively low compared to the current crop of It-girl
models who can command up to $300,000 for a single Instagram post, or an A-list actor who can receive
up to $500,000.
90. ______________
Although celebrity endorsements have been around for as long as advertising itself, their foray into
social media is new territory. Traditional print and TV advertising is replete with famous faces pushing
products we doubt they use or consume, but there is an understanding that we accept the falsehood
because we know the game, we know they ve been paid. The pseudo-ads posted as selfies on Instagram
or Twitter, however, usually contain no clear indication that money has changed hands, despite the fact
that, in many cases, it’s legally required to do so.
91. _____________
Beginning enthusiastically with ‘OMG’, the original post was liked online by hundreds of thousands of
followers. There was no doubt that she had actually used the medication, as it had been prescribed by
her doctor. But had she included the mandatory information, it would have given the very distinct, and
true; impression that it was a promotion. Ever mindful of her public image, she was aware this may not
go down well with her followers. For her sins, she was forced to remove her gushing reviews from
Instagram, Twitter and Facebook, and repost with a huge risk disclaimer.
92. _____________
Celebrities love social media because it helps them to build some semblance of authenticity and social
intimacy with fans. First they create an image of openness and honesty with their online followers and
then they exploit it in their sponsored posts. One star, for example, recently confided that her ‘secret’ to
staying trim was a certain weight loss tea. A photo of her looking slim and healthy, holding a cup of the
miraculous fat-melting intusion was there for all to see. No mention was made of frequent, gruelling
sessions with a well-paid personal trainer. A clearly sponsored post- though honest – would have
dismantled her credibility.
93. _____________
As advertising professionals will tell you, that is exactly the goal of any ad – to sell an idea or a desirable
lifestyles, and it doesn’t even have to be true or attainable. The personality vouching for the product
isn’t even required to actually use it. All that needs to happen is for consumers to want some glamour
and coolness to rub off on them. That is how advertising works. But this sneaky new form of celebrity
endorsement, with its camouflaged product promotions requires consumers to completely suspend belief
in order to place their trust in someone with an agenda
94._____________
As such, the language has to be just right, peppered with the kinds of casual terms friends use with each
other. Celebrities ‘swear by’ a new skincare product, are ‘in love with’ a new coconut water, ‘can’t live
without’ a new protein shake. The more cynical among us might question the true author of these snappy
captions – is it the paid influencer or has the brand’s marketing department come up with the text and
tailored it to the target audience? Because the audience is what it’s all about. Keeping it is important,
but first it has to be founa.
95._____________
So far, it’s been a very successful strategy, and as long as a supply of famous faces are willing to accept
cash for comments, the use of social media as a unique advertising tool won’t be going away any time
soon.
Missing Paragraphs

A. For that kind of money, celebrities are happy to plug a product hard. Juice cleanses, meal replacement
regimes, weight-loss teas, waist shapers, teeth whitening kits – they comprise a range of categories
that are perfectly suited to the highly narcissistic nature of the medium that is social media, requiring
nothing more than a love of selfies and a complete lack of shame. The questionable efficacy of the
products is no deterrent to the blatant money-grab.
B. Advertising is powerful and its manipulation runs deep when consumers are gullible to such marketing
ploys. The endorsements – casually slipped in between a candid photo with a beloved pet and a carefree
snáp with a close friend, for example – make unwitting followers feel that this product is simply a part
of the celebrity’s daily life, and that they should make it a part of theirs too. They want it, and at that
point, they’re hooked.
C. Advertisers have a major responsibility to inform celebrities, bloggers and anyone else they’re paying
to push a product that appropriate disclosures need to be made. Advertising should be identifiable as
advertising so that consumers know when they’re hearing a marketing pitch versus an independent
viewpoint.
D. Disclosure with regards to medication is one of these instances. An American celebrity got into hot
water with the U.S. Food and Drug Administration for promoting a medication without providing
information about possible side effects. The FDA, which is responsible for protecting public health
through the regulation and supervision of a range of products including dietary supplements,
prescription and over-the-counter medications, took a very dim view of this.
E. The beauty of influencer marketing for brands is that they don’t have to build their target audience from
scratch – something which is costly and time-consuming as it involves extensive market research.
Social media influencers already have a devoted fan following, so half the job has already been done.
Once the influencer has been chosen, it is just a matter of getting the followers to support the brand,
talk about it themselves to others, and buy the products.
F. Make no mistake – they are raking it in. Even those who are no longer as famous as they once were can
be offered tantaiising amounts by companies to promote products on the main personal social media
platforms – Instagram, Twitter and Facebook. The uber-famous with the largest followings are making
money hand over fist, and the amounts are staggering.
G. But what makes people susceptible to such influence? When asked, consumers said they trusted
celebrities over brands. The problem with branded marketing content is that it is biased; influencer
marketing, on the other hand, does not come across as an overt marketing attempt. Instead, it looks like
a recommendation from a trusted friend.
H. Besides failing to list the dow the fact that she was paid to make tr payment to promote a product on
their pdes railing to list the downsides of the product, the celebrity fell foul of the law by not being
explicit about that she was paid to make the post. There are rules about this sort of thing, too. Celebrities
receiving to promote a product on their platforms are required to tag the post #ad or #sponsored –
something a monetary transaction has taken place in return for their endorsement. So what’s preventing
them from adding those few letters?

Your answers:

89. 90. 91. 92.


93. 94. 95.

Part 5: You are going to read a magazine article in which five career consultants give advice
about starting a career. For questions 1 – 10, choose from the consultants (A – E). The
consultants may be chosen more than once. C1 Advanced Handbook for teachers

Starting out on your career Are you a graduate trying to plan out the best career path for yourself?
We've asked five career consultants to give some tips on how to go about it.
Consultant A
A university degree is no guarantee of a job, and job hunting in itself requires a whole set of skills. If
you find you are not getting past the first interview, ask yourself what is happening. Is it a failure to
communicate or are there some skills you lack? Once you see patterns emerging it will help you decide
whether the gaps you have identified can be filled relatively easily. If you cannot work out what the
mismatch is, get back to the selection panel with more probing questions, and find out what you need to
do to bring yourself up to the level of qualification that would make you more attractive to them: but be
careful to make this sound like a genuine request rather than a challenge or complaint.
Consultant B
Do not be too dispirited if you are turned down for a job but think about the reasons the employers give.
They often say it is because others are 'better qualified', but they use the term loosely. Those who made
the second interview might have been studying the same subject as you and be of similar ability level,
but they had something which made them a closer match to the selector's ideal. That could be experience
gained through projects or vacation work, or it might be that they were better at communicating what
they could offer. Do not take the comments at face value: think back to the interviews that generated
them and make a list of where you think the shortfall in your performance lies. With this sort of analytical
approach, you will eventually get your foot in the door.
Consultant C
Deciding how long you should stay in your first job is a tough call. Stay too long and future employers
may question your drive and ambition. Of course, it depends where you are aiming. There can be
advantages in moving sideways rather than up, if you want to gain real depth of knowledge. If you are
a graduate, spending five or six years in the same job is not too long provided that you take full advantage
of the experience. However, do not use this as an excuse for apathy. Graduates sometimes fail to take
ownership of their carcers and take the initiative. It is up to you to make the most of what's available
within a company, and to monitor your progress in case you need to move on. This applies particularly
if you are still not sure where your career path lies.
Consultant D
It is helpful to think through what kind of experience you need to get your dream job and it is not a
problem to move around to a certain extent. But in the early stages of your career, you need a definite
strategy for reaching your goal, so think about that carefully before deciding to move on from your first
job. You must cultivate patience to master any role. There is no guarantee that you will get adequate
training, and research has shown that if you do not receive proper help in a new role, it can take 18
months to master it.
Consultant E
A prospective employer does not want to that you have changed jobs every six months with no thread
running between them. You need to be able to demonstrate the quality of your experience to a future
employer, and too many moves too quickly can be a bad thing. In any company it takes three to six
months for a new employee to get up to speed with the structure and the culture of the company. From
the company's perspective, they will not receive any retum on the investment in your salary until you
have been there for 18 months. This is when they begin to get most value from you – you are still fired
up and enthusiastic. If you leave after six months, it has not been a good investment – and may make
other employers wary.

Which consultant makes the following statements?


96. Keep your final objective in mind when you are planning to change jobs.
97. It takes time to become familiar with the characteristics of a company you have joined.
98. You should demonstrate determination to improve your job prospects.
99. Make sure your approach for information is positive in tone.
100. It is not certain that you will be given very much support in your job initially.
101. Stay optimistic in spite of setbacks.
102. Promotion isn’t the only way to increase your expertise.
103. Ask for information about your shortcomings.
104. Some information you are given may not give a complete picture.
105. It will be some time before you start giving your employers their money’s worth.
Your answers:

96. 97. 98. 99. 100.


101. 102. 103. 104. 105.

SECTION IV. WRITING (60 points)


Part 1: Read the following extract and use your own words to summarize it. Your summary should be
about 140 words. You MUST NOT copy the original. (15 pts)

According to a report by Asiaweek, on-the-job injuries are no longer confined strictly to blue-collar
workers. The modern-day office has become a danger zone and computers are largely to be blamed.
Their increased use points to an increase in sick leave and doctors' visits. The new group of patients
includes writers, secretaries and data-entry clerks. Anyone who spends hours at a keyboard can be at
risk.
The most frequent complaints are wrist, hand and neck pain. These are typical signs of repetitive strain
injuries. Another related condition is called carpal tunnel syndrome caused by pressure on the median
nerve in the wrist. It results from repeated movements such as typing or using the mouse over a long
period of time. Before computers came along, typists would stop to make corrections or change paper.
These movements provided some relief. Now, typists rarely move from their computer, hitting as much
as 21,600 words an hour. In severe cases, the pain shoots up a victim's arm. Some also develop neck and
shoulder problems from holding their head in uncomfortable positions.
Computer users may also complain of eye strain, headaches, double vision and other eye problems
caused by improper use of display screens. It may be a result of staring at the screen for too long. It
could also be due to improper lighting and screen glare.
The best way to cope with such problems is to adopt healthier work habits. This means that the workers
have to hold their wrists flat when they use the keyboard. They should also tap on the keys softly and
take frequent breaks. Their feet should also be flat on the floor and their heads and backs straight. Some
may be required to change their typing technique. In severe cases, taking a rest and some anti-
inflammatory medication may be necessary.
There are also ergonomic hardware and software that blends well with a person's body or actions. Some
examples of such ergonomic hardware are tilting display screens, detachable keyboards and specially
designed keyboards that are suitable for the wrists.
Employers and managers should help employees create a comfortable working environment to reduce
the side effects of working on the computer.
Answer:
……………………………………………………………………………………………………………
……………………………………………………………………………………………………………
……………………………………………………………………………………………………………
……………………………………………………………………………………………………………
……………………………………………………………………………………………………………
……………………………………………………………………………………………………………
……………………………………………………………………………………………………………
……………………………………………………………………………………………………………
……………………………………………………………………………………………………………
……………………………………………………………………………………………………………
……………………………………………………………………………………………………………
……………………………………………………………………………………………………………
……………………………………………………………………………………………………………
……………………………………………………………………………………………………………
……………………………………………………………………………………………………………

Part 2: CHART DESCRIPTION (15 pts)


The pie charts below show the most common advantages and disadvantages of Fairmont Island,
according to a survey of visitors.
Summarise the information by selecting and reporting the main features and make comparisons where
relevant.
……………………………………………………………………………………………………………
……………………………………………………………………………………………………………
……………………………………………………………………………………………………………
……………………………………………………………………………………………………………
……………………………………………………………………………………………………………
……………………………………………………………………………………………………………
……………………………………………………………………………………………………………
……………………………………………………………………………………………………………
……………………………………………………………………………………………………………
……………………………………………………………………………………………………………
……………………………………………………………………………………………………………
……………………………………………………………………………………………………………
……………………………………………………………………………………………………………
……………………………………………………………………………………………………………
……………………………………………………………………………………………………………

Part 3: ESSAY WRITING (30 pts)


In the process of educational renovation, some people believe that unpaid community work (for
example, working for a charity, teaching sports to children) should be a compulsory part of high school
curriculums. To what extend, do you agree or disagree?
……………………………………………………………………………………………………………
……………………………………………………………………………………………………………
……………………………………………………………………………………………………………
……………………………………………………………………………………………………………
……………………………………………………………………………………………………………
……………………………………………………………………………………………………………
……………………………………………………………………………………………………………
……………………………………………………………………………………………………………
……………………………………………………………………………………………………………
……………………………………………………………………………………………………………
……………………………………………………………………………………………………………
……………………………………………………………………………………………………………
……………………………………………………………………………………………………………
……………………………………………………………………………………………………………
……………………………………………………………………………………………………………
……………………………………………………………………………………………………………
……………………………………………………………………………………………………………
……………………………………………………………………………………………………………
……………………………………………………………………………………………………………
……………………………………………………………………………………………………………
……………………………………………………………………………………………………………
……………………………………………………………………………………………………………
……………………………………………………………………………………………………………
……………………………………………………………………………………………………………
……………………………………………………………………………………………………………
……………………………………………………………………………………………………………
……………………………………………………………………………………………………………
……………………………………………………………………………………………………………
……………………………………………………………………………………………………………
……………………………………………………………………………………………………………
……………………………………………………………………………………………………………
……………………………………………………………………………………………………………
……………………………………………………………………………………………………………
……………………………………………………………………………………………………………
……………………………………………………………………………………………………………
……………………………………………………………………………………………………………
……………………………………………………………………………………………………………
……………………………………………………………………………………………………………
……………………………………………………………………………………………………………
……………………………………………………………………………………………………………
……………………………………………………………………………………………………………
……………………………………………………………………………………………………………
……………………………………………………………………………………………………………
……………………………………………………………………………………………………………
……………………………………………………………………………………………………………
……………………………………………………………………………………………………………
……………………………………………………………………………………………………………
……………………………………………………………………………………………………………
……………………………………………………………………………………………………………
……………………………………………………………………………………………………………
……………………………………………………………………………………………………………
……………………………………………………………………………………………………………
……………………………………………………………………………………………………………
……………………………………………………………………………………………………………
……………………………………………………………………………………………………………
……………………………………………………………………………………………………………
……………………………………………………………………………………………………………
……………………………………………………………………………………………………………
……………………………………………………………………………………………………………
……………………………………………………………………………………………………………
……………………………………………………………………………………………………………
……………………………………………………………………………………………………………
……………………………………………………………………………………………………………
……………………………………………………………………………………………………………
……………………………………………………………………………………………………………
……………………………………………………………………………………………………………
……………………………………………………………………………………………………………
……………………………………………………………………………………………………………
The end

Người ra đề: Đào Thị Thương (0914706060) + Chu Nhất Đại (0981694954)
SỞ GD - ĐT THÁI NGUYÊN ĐỀ XUẤT ĐỀ THI HỌC SINH GIỎI
TRƯỜNG THPT CHUYÊN KHU VỰC DUYÊN HẢI – ĐBBB NĂM 2022
------------------------- Môn: Tiếng Anh – Lớp 11

A. LISTENING (50 pts)


Part 1. You are going to listen to a talk about working in Japan. For question 1-5, listen
and decide the following statements are True (T) or False (F). (10pts)
1. Nearly one fourth of Japanese companies have their employees work more than 80 hours of
overtime per month.
2. The dramatic economic growth beginning in the 1950s propelled Japan to become the
world’s second-largest economy.
3. Inside Japan’s corporations, the culture emphasizes the success of individuals.
4. Japan has the lowest productivity amongst the G7 nations.
5. Working overtime did not ever happen in Japan.
Your answers:
1. 2. 3. 4. 5.

Part 2. You are going to listen to a podcast about minimalism. Listen and answer
the following questions with no more than three words. (10pts)
1. What do people following minimalism tend to eat?
_____________________________________________________________________
2. When did the word “minimalism” first turn up?
_____________________________________________________________________
3. Minimalist architects like Ludwig Mies van der Rohe, and Maya Lin create structures
full of what?
_____________________________________________________________________
4. Except for minimalistic music, how is minimalism?
_____________________________________________________________________
5. Who have reacted against the clutter and chaos of life and art, and created many
disparate forms of minimalism?
_____________________________________________________________________
Your answers:
1. 2. 3.

4. 5.

Part 3: Listen to the news about the colossal consequences of supervolcanoes and
choose the appropriate answer A, B, C or D to each question. (10pts)
1. Where did the strange snow fall?
A. England and the USA B. Hungary and Italy
C. Italy and England D. All over Europe
2. What was the strange phenomenon known as?
A. The year without summer B. The year of the Famine
C. The year the Apocalypse began D. The year of darkness
3. Which aspect of a supervolcanic eruption makes it the most deadly?
A. Earthquakes after the eruption B. Magma destroying everything on its way
C. Particles and gases released into the air D. The wind it causes
4. Which of these create a ‘volcanic winter’?
A. the cooling temperatures B. Ash and particles blocking out sunlight
C. Sulfur dioxide reacting in the stratosphere D. All of the above
5. Where is a volcanic Caldera nowadays?
A. Yellowstone National Park B. Mount Vesubio
C. Russia D. The Minx Dynasty
Your answers:
1. 2. 3.

4. 5.

Part 4: Listen to the second part of the lecture as you listen, complete the notes
below with no more than three words. (20pts)
In Britain shops usually open at 9:00 and close at 17:30 but in many cities, they have a
late 1. _______ once a week. They stay open 2. ______ about 20:00. British voltage is
240 V AC, 50 HZ. Many hotels will be able to supply 3. ______ for electric shavers.
Stamps can be bought at 4.______ offices. Most hotel bills include a service charge,
usually 10-12%, but in some larger 5.______, it’s 15%. People often leave 10-15% for
the waiter in the restaurants, and give 30p-50p per suitcase for porters, 10-15% of the
fare for taxis and 6.______ pounds for hairdressers and 50p to the assistant who 7.______
your hair. You must remember it is the rule to drive on the 8._______ and overtake on
the 9._______. Drivers as well as front – seat passengers must wear 10. ______ belts.
Your answers:
1. 2.
3. 4.
5. 6.
7. 8.
9. 10.

B. LEXICO-GRAMMAR (30 points)


Part I: Choose the correct answer (A, B, C, or D) to each of the following questions
and write your answers in the corresponding numbered boxes.
1. The woman was ____________ from hospital yesterday only a week after her
operation.
A. ejected B. expelled C. evicted D. discharged
2. ____________ further rioting to occur, the government would be forced to use its
emergency powers.
A. Should B. Did C. Were D. Had
3. Unfortunately, our local cinema is on the ____________ of closing down.
A. verge B. hint C. edge D. threat
4. All courses at the college are offered on a full-time basis unless ____________
indicated.
A. further B. otherwise C. below D. differently
5. Your argument ____________that Britain is still a great power, but this is no longer
the case.
A. outlines B. presupposes C. concerns D. presents
6. The construction of the new road is ____________ winning the support of local
residents.
A. thanks to B. reliant on C. dependent on D. responsible to
7. Although he didn’t actually say he wanted a radio for his birthday he did
____________ pretty strongly.
A. imply B. suggest C. mention D. hint
8. Because of cutbacks in council spending, plans for the new swimming pool had to be
____________.
A. stockpiled B. overthrown C. shelved D. disrupted
9. Don’t let that old rascal take you ____________with his clever talk – none of it is true.
A. along B. in C. about D. down
10. People living abroad are not ____________to enter for his competition.
A. enabled B. permission C. capable D. eligible
11. When attacked by his opponents, the general ________ with a strong justification for
his policy.
A. hit back B. struck up C. leapt up D. pushed forward
12. Activities in the department store were _______ by animal rights activists protesting
against the sale of fur coats.
A. disorientated B. disrupted C. deranged D. disturbed
13. In terms of protocol, the President takes ________ over all others in the country.
A. priority B. the lead C. precedence D. the head
14. I recommend that you ______ out the information for the courses online. You’ll find
everything you need there.
A. checking B. to check C. check D. for checking
15. You’ll be pleased to know that you are _____ for a scholarship if you wish to apply.
A. entitled B. eligible C. appropriate D. particular
16. It never _____ my mind that he was lying to me.
A. crossed B. racked C. took D. put
17. The children’s diet was _____ in Vitamin C, which has caused a lot of problems.
A. absent B. faulty C. deficient D. missing
18. Don’t thank me for helping in the garden. It was _____ pleasure to be working out of
doors
A. plain B. mere C. simple D. sheer
19. My sunburnt nose made me feel rather_____ for the first few days of the holiday.
A. self-effacing B. self-centred C. self-conscious D. self-evident
20. He’s a nice guy, always ready to do somebody a good_______.
A. present B. play C. turn D. pleasure
Your answers:
1 2 3 4 5 6 7 8 9 10
11 12 13 14 15 16 17 18 19 20

Part II. Provide the correct word form of the word in brackets. (10 Points)
1. Barack Obama is the first President of The United States with ___________
background. RACE
2. His family suffered from his ___________ EXPEND
3. You look rather ___________ Are you worried about something? OCCUPY
4. There’s nothing worse than the ___________ of being stuck in a traffic jam.
FRUSTRATE
5. The government’s ___________ approach has brought criticism. COMPROMISE
6. After listening to his sad story, the old woman shook her head ___________
SYMPATHY
7. Jim is one of the most ___________ members of the committee. SPEAK
8. Computers are now considered ___________in the business world. DISPENSE
9. Somebody from the audience shouted out, and the speaker was forced to stop in
___________ SENTENCE
10. They never dare to leave their only child ___________for even a moment.
ATTEND
Your answers:
1. 6.
2. 7.
3. 8.
4. 9.
5. 10.

C. READING COMPREHENSION (60 pts)

Part 1 (15 pts). Read the passage below and fill each of the following numbered
spaces with ONE suitable word. Write your answers in the corresponding boxes
provided.
SIGNS OF THE TIMES
We are familiar (0) with the saying “a picture paints a thousand words” and in the global
village the world has become, information in pictorial form is (1) _____we turn. Much
communication takes place through symbols rather than words, a case in point (2)
______ airports, where you can see the majority of the thirty-four symbols devised (3)
______ the American Institute of Graphic Arts in the 1970s. Such signs as a knife and
fork for a restaurant or a telephone for a phone booth are a boon for (4) _______ a
traveller who does not speak English or use the Latin alphabet. (5) _______ worldwide
"languages" of this kind are musical and mathematical notation, circuit diagrams, road
signs and computer icons, (6)______, again, bypass the need for words. Even a label on a
garment will carry, in symbols, washing and ironing instructions. All these (7) ______ to
be sufficient to their (8)______ restricted worlds but would it really be possible to devise
a universal symbolic system of communication independent of any spoken language,
culture-free and value-free, as dreamt of by the seventeenth-century philosopher Leibniz?
It would seem (9) ______. Chinese and Japanese pictograms and ancient Egyptian
hieroglyphics are sometimes cited as examples of such a system, yet both Japanese script
and Egyptian hieroglyphics include sound-base elements and Chinese is often
transliterated (10) ______ Romanised sound based "pin yin" script. In a word, words are
inescapable.
1. 6.
2. 7.
3. 8.
4. 9.
5. 10.

Part 2. (10 pts) Read the passage and choose the best answer to each of the
questions. Write your answer in the space provided.

The Origins of Theater


1. In seeking to describe the origins of theater, one must rely primarily on speculation,
since there is little concrete evidence on which to draw. The most widely accepted
theory, championed by anthropologists in the late nineteenth and early twentieth
centuries, envisions theater as emerging out of myth and ritual. The process perceived by
these anthropologists may be summarized briefly. During the early stages of its
development, a society becomes aware of forces that appear to influence or control its
food supply and well–being. Having little understanding of natural causes, it attributes
both desirable and undesirable occurrences to supernatural or magical forces, and it
searches for means to win the favor of these forces. Perceiving an apparent connection
between certain actions performed by the group and the result it desires, the group
repeats, refines, and formalizes those actions into fixed ceremonies, or rituals.
2. Stories (myths) may then grow up around a ritual. Frequently the myths include
representatives of hose supernatural forces that the rites celebrate or hope to influence.
Performers may wear costumes and masks to represent the mythical characters or
supernatural forces in the rituals or in accompanying celebrations. As a people becomes
more sophisticated, its conceptions of supernatural forces and causal relationships nay
change. As a result, it may abandon or modify some rites. But the myths that have grown
up around the rites may continue as part of the group’s oral tradition and may even come
to be acted out under conditions divorced from these rites. When this occurs, the first step
has been taken toward theater as an autonomous activity, and thereafter entertainment
and aesthetic values may gradually replace the former mystical and socially efficacious
concerns.
3. Although origin in ritual as long been the most popular, it is by no means the only
theory about how the theater came into being. Storytelling has been proposed as one
alternative. Under this theory, relating and listening to stories are seen as fundamental
human pleasures. Thus, the recalling of an event (a hunt, battle, or other feat) is
elaborated through the narrator’s pantomime and impersonation and eventually through
each role being assumed by a different person.
4. A closely related theory sees theater as evolving out of dances that are primarily
pantomimic, rhythmical or gymnastic, or from imitations if animal noises and sounds.
Admiration for the performer’s skill, virtuosity, and grace are seen as motivation for
elaborating the activities into fully realized theatrical performances.
5. In addition to exploring the possible antecedents of theater, scholars have also
theorized about the motives that led people to develop theater. Why did the theater
develop, and why was it valued after it ceased to fulfill the function of ritual? Most
answers fall back on the theories about the human mind and basic human needs. One, set
forth by Aristotle in the fourth century B.C., sees humans as naturally imitative – as
taking pleasure in imitating persons, things, and actions and in seeing such imitations.
Another, advanced in the twentieth century, suggests that humans have a gift for fantasy,
through which they seek to reshape reality into more satisfying forms than those
encountered in daily life. Thus, fantasy or fiction (of which drama is one form) permits
people to objectify their anxieties and fears, confront them, and fulfill their hopes in
fiction if not fact. The theater, then, is one tool whereby people define and understand
their world or escape from unpleasant realities.
6. But neither the human imitative instinct nor a penchant for fantasy by itself leads to
an autonomous theater. Therefore, additional explanations are needed. One necessary
condition seems to be a somewhat detached view of human problems. For example, one
sign of this condition is the appearance of the comic vision, since comedy requires
sufficient detachment to view some deviations from social norms as ridiculous rather
than as serious threats to the welfare of the entire group. Another condition that
contributes to the development of autonomous theater is the emergence of the aesthetic
sense. For example, some early societies ceased to consider certain rites essential to
their well-being and abandoned them; nevertheless, they retained as parts of their oral
tradition the myths that had grown up around the rites and admired them for their
artistic qualities rather than for their religious usefulness.
(Source: TOEFL articles)
1. The world “championed” in the passage is closest in meaning to
A. changed B. debated C. created D. supported
2. The world “attributes” in the passage is closest in meaning to
A. ascribes B. leaves C. limits D. contrasts
3. According to paragraph 1, theories of the origins of theater ____________
A. are mainly hypothetical
B. are well supported by factual evidence
C. have rarely been agreed upon by anthropologists
D. were expressed in the early stages of theater’s development
4. According to paragraph 1, why did some societies develop and repeat ceremonial
actions?
A. To establish a positive connection between the members of the society
B. To help society members better understand the forces controlling their food supply
C. To distinguish their beliefs from those of other societies
D. To increase the society’s prosperity
5. The word “autonomous” in the passage is closest meaning to
A. artistic B. important C. independent D. established
6. According to paragraph 2, what may cause societies to abandon certain rites?
A. Emphasizing theater as entertainment
B. Developing a new understanding of why events occur
C. Finding a more sophisticated way of representing mythical characters
D. Moving from a primarily oral tradition to a more written tradition
7. All of the following are mentioned in paragraph 5 as possible reasons that led societies
to develop theater EXCEPT:
A. Theater allows people to face what they are afraid of.
B. Theater gives an opportunity to imagine a better reality.
C. Theater is a way to enjoy imitating other people.
D. Theater provides people the opportunity to better understand the human mind.
8. Which of the following best describes the organization of paragraph 5?
A. The author presents two theories for a historical phenomenon.
B. The author argues against theories expressed earlier in the passage.
C. The author argues for replacing older theories with a new one.
D. The author points out problems with two popular theories.
9. The word “penchant” in the passage is closest in meaning to
A. compromise B. inclination C. tradition D. respect
10. Which of the sentences below best expresses the essential information in the italic
sentence in the passage? Incorrect choices change the meaning in important ways or
leave out essential information.
A. A society’s rites were more likely to be retained in the oral tradition if its myths were
admired for artistic qualities.
B. The artistic quality of a myth was sometimes an essential reason for a society to
abandon it from the oral tradition.
C. Some early societies stopped using myths in their religious practices when rites ceased
to be seen as useful for social well-being.
D. Myths sometimes survived in a society’s tradition because of their artistic qualities
even after they were no longer deemed religiously beneficial.
Your answers:
1. 2. 3. 4. 5.
6. 7. 8. 9. 10.
Part 3. (13 pts) Read the following passage and do the following tasks.
Dugongs are herbivorous mammals that spend their entire lives in the sea. Their close
relatives the manatees also venture into or live in freshwater. Together dugongs and
manatees make up the order Sirenia or sea cows, so-named because dugongs and
manatees are thought to have given rise to the myth of the mermaids or sirens of the sea.

The dugong, which is a large marine mammal which, together with the manatees, looks
rather like a cross between a rotund dolphin and a walrus. Its body, flippers and fluke
resemble those of a dolphin but it has no dorsal fin. Its head looks somewhat like that of a
walrus without the long tusks.

Dugongs, along with other Sirenians whose diet consists mainly of sea-grass; and the
distribution of dugongs very closely follows that of these marine flowering plants. As
seagrasses grow rooted in the sediment, they are limited by the availability of light.
Consequently they are found predominantly in shallow coastal waters, and so too are
dugongs. But, this is not the whole story. Dugongs do not eat all species of seagrass,
preferring seagrass of higher nitrogen and lower fibre content.

Due to their poor eyesight, dugongs often use smell to locate edible plants. They also
have a strong tactile sense, and feel their surroundings with their long sensitive bristles.
They will dig up an entire plant and then shake it to remove the sand before eating it.
They have been known to collect a pile of plants in one area before eating them. The
flexible and muscular upper lip is used to dig out the plants. When eating they ingest the
whole plant, including the roots, although when this is impossible they will feed on just
the leaves. A wide variety of seagrass has been found in dugong stomach contents, and
evidence exists they will eat algae when seagrass is scarce. Although almost completely
herbivorous, they will occasionally eat invertebrates such as jellyfish, sea squirts, and
shellfish.

A heavily grazed seagrass bed looks like a lawn mown by a drunk. Dugongs graze
apparently at random within a seagrass bed, their trails meandering in all directions
across the bottom. This is rather an inefficient means of removing seagrass that results in
numerous small tufts remaining. And this is where the dugongs derive some advantage
from their inefficiency. The species that recover most quickly from this disturbance,
spreading out vegetatively from the remaining tufts, are those that dugongs like to eat. In
addition, the new growth found in these areas tends to be exactly what hungry dugongs
like.

Dugongs are semi-nomadic, often travelling long distances in search of food, but staying
within a certain range their entire life. Large numbers often move together from one area
to another. It is thought that these movements are caused by changes in seagrass
availability. Their memory allows them to return to specific points after long travels.
Dugong movements mostly occur within a localised area of seagrass beds, and animals in
the same region show individualistic patterns of movement.

Recorded numbers of dugongs are generally believed to be lower than actual numbers,
due to a lack of accurate surveys. Despite this, the dugong population is thought to be
shrinking, with a worldwide decline of 20 per cent in the last 90 years. They have
disappeared from the waters of Hong Kong, Mauritius, and Taiwan, as well as parts of
Cambodia, Japan, the Philippines and Vietnam. Further disappearances are likely. (In the
late 1960s, herds of up to 500 dugongs were observed off the coast of East Africa and
nearby islands. However, current populations in this area are extremely small, numbering
50 and below, and it is thought likely they will become extinct. The eastern side of the
Red Sea is the home of large populations numbering in the hundreds, and similar
populations are thought to exist on the western side. In the 1980s, it was estimated there
could be as many as 4,000 dugongs in the Red Sea. The Persian Gulf has the second-
largest dugong population in the world, inhabiting most of the southern coast, and the
current population is believed to be around 7,500. Australia is home to the largest
population, stretching from Shark Bay in Western Australia to Moreton Bay in
Queensland. The population of Shark Bay is thought to be stable with over 10,000
dugongs.

G
Experience from various parts of northern Australia suggests that Extreme weather such
as cyclones and floods can destroy hundreds of square kilometres of seagrass meadows,
as well as washing dugongs ashore. The recovery of seagrass meadows and the spread of
seagrass into new areas, or areas where it has been destroyed, can take over a decade. For
example, about 900 km2 of seagrass was lost in Hervey Bay in 1992, probably because
of murky water from flooding of local rivers, and run-off turbulence from a cyclone three
weeks later. Such events can cause extensive damage to seagrass communities through
severe wave action, shifting sand and reduction in saltiness and light levels. Prior to the
1992 floods, the extensive seagrasses in Hervey Bay supported an estimated 1750
dugongs. Eight months after the floods the affected area was estimated to support only
about 70 dugongs. Most animals presumably survived by moving to neighbouring areas.
However, many died attempting to move to greener pastures, with emaciated carcasses
washing up on beaches up to 900km away.

If dugongs do not get enough to eat they may calve later and produce fewer young. Food
shortages can be caused by many factors, such as a loss of habitat, death and decline in
quality of seagrass, and a disturbance of feeding caused by human activity. Sewage,
detergents, heavy metal, hypersaline water, herbicides, and other waste products all
negatively affect seagrass meadows. Human activity such as mining, trawling, dredging,
land-reclamation, and boat propeller scarring also cause an increase in sedimentation
which smothers seagrass and prevents light from reaching it. This is the most significant
negative factor affecting seagrass. One of the dugong’s preferred species of seagrass,
Halophila ovalis, declines rapidly due to lack of light, dying completely after 30 days.

Despite being legally protected in many countries, the main causes of population decline
remain anthropogenic and include hunting, habitat degradation, and fishing-related
fatalities. Entanglement in fishing nets has caused many deaths, although there are no
precise statistics. Most issues with industrial fishing occur in deeper waters where
dugong populations are low, with local fishing being the main risk in shallower waters.
As dugongs cannot stay underwater for a very long period, they are highly prone to
deaths due to entanglement. The use of shark nets has historically caused large numbers
of deaths, and they have been eliminated in most areas and replaced with baited hooks.
Hunting has historically been a problem too, although in most areas they are no longer
hunted, with the exception of certain indigenous communities. In areas such as northern
Australia, hunting remains the greatest impact on the dugong population.

(Source: https://mini-ielts.com/1405/reading/the-dugong-sea-cow)

Questions 1-4

Summary

Complete the following summary of the paragraphs of Reading Passage, using NO


MORE THAN TWO WORDS from the Reading Passage for each answer.

Dugongs are herbivorous mammals that spend their entire lives in the sea. Yet Dugongs
are picky on their feeding Seagrass, and only chose seagrass with higher (1) _______ and
lower fibre. To compensate for their poor eyesight, they use their (2) _______ to feel
their surroundings.

It is like Dugongs are “farming” seagrass. They often leave (3) _______ randomly in all
directions across the sea bed. Dugongs prefer eating the newly grew seagrass recovering
from the tiny (4) _______ left behind by the grazing dugongs.

Your answers:

1. 2. 3. 4.

Questions 5-9

Do the following statements agree with the information given in Reading Passage?

In boxes 5-9 on your answer sheet, write

TRUE if the statement is true

FALSE if the statement is false

NOT GIVEN If the information is not given in the passage

5. The dugong will keep eating up the plant completely when they begin to feed.

6. It takes more than ten years for the re-growth of seagrass where it has been grazed by
Dugongs.
7. Even in facing food shortages, the strong individuals will not compete with the weak
small ones for food.

8. It is thought that the dugong rarely returns to the old habitats when they finished the
plant.

9. Coastal industrial fishing poses the greatest danger to dugongs which are prone to be
killed due to entanglement.

Your answers:

5. 6. 7. 8. 9.

Questions 10-13

Choose NO MORE THAN TWO WORDS AND/OR A NUMBER from the passage
for each answer.

10 What is Dugong in resemblance to yet as people can easily tell them apart from the
manatees by its tail?

11 What is the major reason Dugongs traveled long distances in herds from one place to
another?

12 What number, has estimated to be, of dugong’s population before the 1992 floods in
Hervey Bay took place?

13 What is thought to be the lethal danger which dugongs were often trapped in?

Your answers:

10. 11. 12. 13.

Part 4: (7 pts) In the passage below, seven paragraphs have been removed. For
questions 1-7, read the passage and choose from the paragraphs A-H the one which
fits each gap. There is ONE extra paragraph which you do not need to use. Write
your answers in the corresponding numbered box provided.
Mathematical Games
The majority of video games designed to provide mathematics learning fail educationally
for one of two reasons. One of these is that the designers know how to design and create
video games but know little about mathematics education (in particular, how people learn
mathematics) and in many cases don't seem to know what maths really is. The second is
that they have a reasonable sense of mathematics and have some familiarity with the
basic principles of mathematics education, but do not have sufficient education in video
game design.
1.
To create an engaging game that also supports good mathematics learning requires a
great deal more: a complete understanding of what mathematics is, how and why people
learn and do mathematics, how to get and keep them engaged in their learning and how
to represent the mathematics on the platform on which the game will be played. That too
demands much more than just superficial knowledge.
2.
Following the tradition of textbook publishing, that figure does not include any payment
to the authors who essentially create the entire pedagogic framework and content. Nor
does it take into account the money required for the fees payable to the project's
academic advisory board, without whom the project is unlikely to succeed.
3.
Though this term is specific to this context, its concept has been well known in maths
education circles for over twenty years and is recognised as the biggest obstacle to
practical mastery of middle school mathematics. To understand the precise implication of
what the term entails and appreciate how pervasive it is, it is necessary to examine the
role symbolic expressions play in mathematics.
4.
But just how essential are those symbols? Perhaps this question can best be answered
through a comparison with music. Until the invention of recording devices, symbolic
musical notation was the only way to store and distribute music, yet no one ever confuses
music with a musical score. In the same way as music is created and enjoyed within the
mind, mathematics is created and carried out in the mind. At its heart, mathematics is a
mental activity and one that throughout human history has proved to be highly beneficial
to life and society.
5.
So, why is it that many people believe mathematics itself is symbolic manipulation? And
if the answer is that it results from our classroom experiences, why is mathematics taught
that way? The answer to that second question is that mathematics is taught symbolically
because for many centuries symbolic representation has been the most effective way to
record mathematics and pass on mathematical knowledge to others.
6.
While it is true to say that people sometimes scribble down symbols when they do
everyday maths in a real-life context, for the most part, what they write down are the
facts needed to start with, perhaps the intermediate results along the way and, if they get
far enough, the final answer at the end. But the actual mathematical part is primarily a
thinking process as even when people are asked to 'show all their work', the collection of
symbolic expressions they write down is not necessarily the same as the process that goes
on in their minds when they do the maths correctly. In fact, people can become highly
skilled at doing mental mathematics and yet be hopeless at its symbolic representations.
7.
It is simply not the case that ordinary people cannot do everyday maths. Rather, they
cannot do symbolic everyday maths. From this observation, it can be concluded that the
symbol barrier is huge and pervasive. For the entire history of organised mathematics
instruction, where there has been no alternative to using static, symbolic expressions on
flat surfaces to store and distribute mathematical knowledge, that barrier has prevented
millions of people from becoming proficient in a cognitive skill set on a par in
importance with the ability to read and write.
(Source: www.researchgate.net/publication)
A. So, given the effort and expense to make a maths game, is it worth pursuing? From an
educational perspective, it certainly is. That being said, it must be acknowledged that the
vast majority of maths video games on the market essentially capitalise on just one
educationally important aspect of video games - their power to fully engage players in a
single activity for long periods of time. Only a fraction of them take advantage of another
educationally powerful feature of the medium - their ability to overcome the 'symbol
barrier'.
B. With routine mathematics, the symbolic barrier emerges. In their 1993 book Street
Mathematics and School Mathematics Terezinha Nunes, David William Carraher and
Analucia Dias Schliemann describe research conducted in the street markets of Recife,
Brazil. This and other studies have shown that when people are regularly faced with
everyday mathematics in their daily lives, they master it to an astonishing 98 per cent
accuracy. Yet when faced with the very same problems (from a mathematical
perspective) presented in the traditional symbols, their performance drops to a mere 35 to
40 per cent accuracy.
C. In both these subjects, the symbols are merely static representations on a flat surface
of dynamic mental processes. Just as the trained musician can look at a musical score and
hear the music come alive in his or her head, the trained mathematician can look at a
page of symbolic mathematics and have that mathematics come alive in the mind.
D. In other words, designing and building a good mathematics educational video game,
whether it is a massively multiplayer online game (MMO) or a single smartphone app,
requires a team of experts from several different disciplines. That means it takes a lot of
time and a substantial budget. For a simple-looking, casual game that runs on an iPad, it
can take about nine months from start to finish and cost upwards of a quarter of a million.
E. Yet tens of thousands of years of evolution have produced the most adaptive device on
the planet: the human brain. Trying to design a computer system to adapt to human
cognitive activity is like trying to build a cart that will draw a horse. It can be done, but it
will not work nearly as well as building a cart that a horse can pull.
F. To build a successful video game requires an understanding, at a deep level, of what
constitutes a game, how and why people play games, what holds their attention, and how
they interact with the different platforms on which the game will be played. That is a lot
of deep knowledge.
G. By and large, the public identifies doing maths with writing symbols, often obscure
symbols. Why do people readily make this identification? A large part of the explanation
is that much of the time they spent in the school mathematics classroom was devoted to
the development of correct symbolic manipulation skills, and symbol-filled books are the
standard way to store and distribute mathematical knowledge. So we have become used
to the fact that mathematics is presented by way of symbolic expressions.
H. Still, given the comparison with music, is it possible to break free of that historical
legacy? It would appear that it is, as long as a distinction is made between the advanced
mathematics used by scientists and engineers and the kind of maths important to ordinary
people. Advanced mathematics, on the other hand, is intrinsically symbolic, whereas
everyday maths is not and such activities as counting, proportional reasoning and
problem solving can be done mentally.
Part 5: (15 pts) The passage below consists of four sections marked A, B, C and D.
For questions 1-10, read the passage and do the task that follows. Write your
answers (A, B, C or D) in the corresponding numbered boxes provided.
Is it Art?
A. Corinne
Art is the result of an artist using her or his skill or creative imagination for a creative
purpose, to give pleasure to the viewer through its aesthetic qualities, or to get a reaction
from the audience to a wider more significant issue outside of the work of art itself. That
work of art might be a painting, a sculpture, an installation of some kind or an example
from the performing arts like dance or mime. I think we sometimes get bogged down by
the notion of 'skill'. For many in the anti modern art camp, there needs to be evidence of
the artist's craft on show before the work is taken seriously and can merit the term 'art', be
it intricate drawing skills, expert use of form or an artist's eye for colour. I'm not
suggesting that an artist need not have these credentials but hand in hand with craft is, as
I said earlier, creative imagination, the ability to see the value or beauty of something
unremarkable which would often go unnoticed by the untrained eye. Much of modern art
I think possesses this second quality which is why I often leave an exhibition of modern
art feeling that I've had the chance to reflect on something that I wouldn't normally have
given the time of day to. The art has engaged me, has had an impact, made me think
about something in a way that I wouldn't have thought about before.
B. Michael
I would certainly call myself an art enthusiast and have been for many years and in my
opinion the modern art world is full of second-rate junk which most of us, if we were
being totally honest, would agree a 4-year-old child could do. The idea that a slept-in bed
such as that 'produced' by Tracy Emin or many of the pieces by Damien Hirst and his ilk
are works of art is hard to justify as is the huge price tag that accompanies their work. I
find it particularly galling when extremely talented people out there who have spent years
honing their skills and learning the craft of drawing or painting are completely ignored.
What's more, one of the dangers of this kind of 'art' is that it serves to alienate the mass of
the population from the visual arts. The man in the street viewing one of these pieces is
left thinking the world of modern art has no value; worse still, that he lacks the
intellectual ability to understand the meaning of the piece when in fact there is little to
interpret. Thankfully, one or two great artists make it through, but I'm afraid many are
lost amongst the deluge of dross the art-world deems 'art'. For me, the first measure of the
worth of an artist must be the degree of skill exhibited in the work or at the very least a
pedigree of fine art preceeding any more abstract pieces produced by the artist such as
was the case with Picasso.
C. Robert
The idea that modern art is some kind of mass deception and that all modern artists are
talentless fraudsters just doesn't hold water. And I'm not talking here about the painters
who for centuries have made a living out of copying works of art and selling them on as
originals. I'm talking about abstract art and the idea that the great art collectors such as
the Saatchis or Rockefellas and the great museums of art around the world, would
somehow allow themselves to be duped into paying a fortune for an abstract painting or
sculpture. Are these artists really tricking these people into paying huge sums of money
for something worthless? Of course not. Though some of these works may not appear to
the layman as having any artistic merit, neither did the great impressionists or the more
abstract works of Picasso or Rothko when they were first exhibited. In the same way that
great poetry can speak to us in a way that prose never can, abstract art can engage with
the audience in more subtle and effective ways than is the case with art of a more realistic
nature. So, they may get their fingers burnt now and again but I don't think the Saatchis
will be cursing the day they spent huge sums on works of abstract art. Quite the opposite
in fact and in the process of making a canny investment they have helped further raise the
profile of some of our great modern artists.
D. Janet
Here we go again: the media are once more up in arms about the latest 'is it art' shock-
horror editorials following the latest Turner Prize shortlisting. When will they learn? For
decades art in many forms has moved away from realism and towards abstraction. Ever
since the invention and popularisation of photography, art has had to reinvent itself.
Patrons who wanted a perfect representation of themselves no longer needed to turn to
the artist. Artists started to struggle with the challenge of catching the essence of the
thing depicted rather than simply its external appearance. Abstract artists try to convey a
pure idea, not the exact replica of the subject concerned. It's true that some works of art
are so obscure that you may need to read up on the theory behind the creation, which is
usually helpfully supplied in art galleries. But this isn't always necessary. Take Guernica
by Picasso. To get a full understanding of this painting it could be argued the audience
needs to appreciate the historical context, the bombing of the Basque city during the
Spanish Civil War. It would also probably help to have a good understanding of the
techniques of abstraction that Picasso had used to create the effect. However, I think most
people viewing this masterpiece would be struck by the horror it depicts even without
this background knowledge. And I would argue it is the effect of this abstraction that
adds to the impact on us compared to a realistic portrayal of such a scene.
(Source: C2 Proficiency (CPE) Practice Tests)
Which person gives each of these opinions about modern art?
1. Defending that long-established artwork has its own cost.
2. Some people may not have an inkling of artwork.
3. Certain aesthetic qualities can be invisible until brought to our attention by the artist.
4. Picasso is an emblem of an artist who proved his craftsmanship.
5. Appreciation of the work itself is not always the artist's aim.
6. Implicit intents of artists on Arts have witnessed some transformation.
7. The works of great artists fail to make a good impression on beholders initially.
8. Abstract art is exorbitant on the whole.
9. Investing money into abstract art may be ill-advised.
10. A masterpiece can move our hearts despite its opaque context.
Your answers:
1. 2. 3. 4. 5.
6. 7. 8. 9. 10.

D. WRITING
Part 1: Read the following extract and use your own words to summarize it. Your
summary should be between 100 and 120 words. (15 points)
A genetic study involving more than 2,000 dogs, paired with 200,000 answers from dog
owners on related surveys, suggests that breed alone is a poor predictor of behavior.
Despite being one of humans' oldest animal companions, almost all modern dog breeds
were only invented about 200 years ago. Before this time — beginning more than 2,000
years ago — dogs were primarily selected for traits central to their functional roles, like
hunting, guarding, or herding. It wasn't until the 1800s that humans began selecting dogs
for their breed-defining physical and aesthetic traits.

Today, most modern dog breeds are ascribed characteristic temperaments associated with
their ancestral function. As such, the breed ancestry of individual dogs is assumed to be
predictive of temperament and behavior. This has led to, among other outcomes, a variety
of breed-specific legislation, which can include insurance restrictions or the outright ban
on owning some dog breeds.

Despite these widely held assumptions, there is a stark lack of genetic research
illustrating a link between breed and behavior. To address this, Kathleen Morrill, a
student at the Broad institute and the Chan Medical School, and colleagues used
association studies to search for common genetic variations that could predict behavioral
traits in 2,155 purebred and mixed-breed dogs.

Behavioral data was analyzed across owner-reported breeds and genetically detected
breed ancestries. The results of these tests, which included data from 78 breeds, identified
11 genetic loci strongly associated with behavior, though none of these were specific to
breed.

Among the behaviors most strongly predicted by genetics was dog biddability — how
well dogs respond to human direction. However, this, too, varied significantly among
individual dogs. Other behaviors, like aggression — a trait unfairly linked to some
breeds, like pit bulls — has more to do with the environment in which a dog is raised
than its genes.

(Adapted from https://www.aaas.org/news/dogs-breed-doesnt-determine-its-behavior)

Part 2: The table and chart below show the domestic water use and cost in 5
countries. Summarize the information by selecting and reporting the main features
and make
comparisons where relevant. You should write about 150 words. (15 points)
Part 3: Write an essay of 350 words on the following topic.
Some people think that the teenage years are the happiest times of most people’s lives.
Others think that adult life brings more happiness in spite of greater responsibilities.
Discuss both views and give your own opinion.

--THE END--

Người ra đề: Đinh Thị Thu Hường Ký tên


Số điện thoại: 0912 545 745
SỞ GD & ĐT THÁI BÌNH KỲ THI HỌC SINH GIỎI CÁC TRƯỜNG THPT CHUYÊN
TRƯỜNG THPT CHUYÊN KHU VỰC DUYÊN HẢI VÀ ĐỒNG BẰNG BẮC BỘ
LẦN THỨ XIV, NĂM 2022
ĐỀ THI ĐỀ XUẤT
Đề thi môn: TIẾNG ANH – LỚP 11
(Đề thi gồm 13 trang) Thời gian thi: 180 phút (không kể thời gian phát đề)

Điểm
Giám khảo1 Giám khảo 2 Số phách
Bằng số Bằng chữ

A: LISTENING (50 points)


HƯỚNG DẪN PHẦN THI NGHE HIỂU
 Bài nghe gồm 4 phần, mỗi phần được nghe 2 lần, mỗi lần cách nhau 30 giây, mở đầu và kết thúc mỗi phần
nghe có tín hiệu.
 Mở đầu và kết thúc bài nghe có tín hiệu nhạc. Thí sinh có 3 phút để hoàn chỉnh bài trước tín hiệu nhạc kết
thúc bài nghe.
 Mọi hướng dẫn cho thí sinh (bằng tiếng Anh) đã có trong bài nghe.

Part 1: For questions 1 – 5, listen to part of a podcast by HubSpot marketing about remote working and
decide whether the statements are True (T) or False (F). Write your answers in the corresponding
numbered boxes.
1. By 2009, there had been over 300 thousand IBM employees working remotely.
2. Localization of workers was believed to boost work productivity, agility and innovation.
3. According to a research by Harvard Business Review, many remote employees feel being discriminated by
their non-remote counterparts.
4. Over half the number of workers would like to do their part-time job at flexible hours.
5. It is advisable for managers to keep themselves posted about remote workers' situations.
Your answers
1. 2. 3. 4. 5.

Part 2: For questions 6 – 10, listen to a student called Wei Liu, talking to a restaurant manager about a
part-time job and answer the questions. Write NO MORE THAN THREE WORDS AND/OR NUMBERS
taken from the recording for each answer.
6. What kind of job is Wei Liu looking for?

___________________________________________________________________

7. What is his profession at the moment?

___________________________________________________________________

8. According to the woman a barista is someone who does what?

___________________________________________________________________

9. Apart from the name, what else is new at Tanner’s?

___________________________________________________________________

10. Why don’t you need to be an expert to bake the cakes at Tanner’s?

___________________________________________________________________

Page 1 of 14 pages
Part 3: For questions 11 – 15, listen to part of a program in which two racing drivers, Eddie Kiwitz and
Jenny Pelaw, are discussing their profession and choose the answer (A, B, C, or D) which fits best
according to what you hear. Write your answers in the corresponding numbered boxes.
11. What does Eddie say to Jenny about being the number one driver in the world?
A. She will have to adjust quickly to her new status.
B. She needs to recognise that nobody is unbeatable.
C. She must convince herself that she is worthy of the title.
D. She must keep reminding herself that she can still improve.
12. What do we learn about a leading journalist’s criticism of Jenny?
A. It led Eddie to leap to Jenny’s defence.
B. It was widely seen as unnecessarily harsh.
C. It didn’t upset Jenny as much as people thought.
D. It provided Jenny with the motivation she needed.
13. When talking about differences between their respective eras, Eddie and Jenny agree that _____
A. there is now less comradeship among drivers. B. drivers are now more prone to self-doubt.
C. racing has become substantially safer. D. nowadays drivers tend to go faster.
14. What view do Eddie and Jenny share about simulators?
A. Their influence is likely to increase as time goes on.
B. They are useful for drivers working on technique.
C. They are no substitute for natural ability.
D. Their attempt to provide fitness is ill-conceived.
15. What effect has the experience had on Cindy?
A. rather regretting some of the things he did.
B. being rather indifferent to Jenny’s enthusiasm.
C. wanting to avoid the public eye wherever possible.
D. preferring the racing world from when he was a driver.
Your answers
11. 12. 13. 14. 15.
Part 4: For questions 16 – 25, listen to a news report by National Geographic about historical revolutions
and complete the following table. Write NO MORE THAN THREE WORDS taken from the recording in
each blank.
Definition A (16) _____________________________ transition from one government to another.

 Mass frustration
 Shared motivation
Common  (17) ______________________________________ with wealth, education or power
characteristics  State crises
These characteristics help revolutions, albeit unalike, demonstrate the dynamism of human
nature and (18) ______________________________________

The Latin - Challenged traditional (19) ________________________________.


American
Revolution - Ultimately led to the battle for (20) ___________________________.

- Partly an outcome of social inequalities originating from a(n) (21)


The
French ____________________________.
Revolution
- Eventually (22) _______________________________ for good.
Examples
The - Occurred due to the (23) ______________________ of the government.
Russian
Revolution - Replaced (24) __________________________ with its communist regime.

The - Incited by contribution to (25) _______________________.


American
Revolution - One of the earliest revolutions in modern history.

Page 2 of 14 pages
B: GRAMMAR & LEXICO (30 points)
Part 1: For questions 1 – 20, choose the correct answer (A, B, C or D) to each of the following questions.
Write your answers in the corresponding numbered boxes.
1. Remember not to sneeze or cough at the wedding. ______, excuse yourself.
A. For necessary B. In due course C. If need be D. In retrospect
2. He has done things he ought not to have done and ______ undone things he ought to have done
A. leaving B. will leave C. left D. leave
3. In your place, I _______ to others for help. There’s no way of finishing the project yourself.
A. would have turned B. must have turned C. will turn D. would turn
4. ______ circling the globe faster than Jules Verne’s fictional Phileas Fogg.
A. Also known as a pioneer journalist, the exploits of Nellie Bly included
B. The exploits of Nellie Bly, a pioneer journalist, included
C. A pioneer journalist, Nellie Bly’s exploits also included
D. Included in Nellie Bly’s exploits, who was a pioneer journalist,
5. Jimmy doesn’t like my cooking, but it doesn’t bother me. _____, I’d say.
A. Everyone to his own B. Each to his own C. Each one his own D. Everyone’s his own
6. My sister _____ and then finally admitted she'd worn my shoes.
A. waxed and waned B. ranted and raved C. hemmed and hawed D. racked and ruined
7. The two groups were ______ into signing the agreement, despite their objections.
A. shanghaied B. beijinged C. wuhaned D. hongkonged
8. Congressman Saunders fired the opening ______ during a heated debate on capital punishment.
A. cannonade B. barrage C. salvo D. burst
9. Drug abuse _______ a major threat to the _______ of our society.
A. poses/fabric B. strikes/fabric C. hits/cotton D. strikes/thread
10. Large quantities of condensed milk, put up in _____ sealed tins, are sold for use in mining camps.
A. hermetically B. eccentrically C. enigmatically D. esoterically
11. Under the administration of the crooked sheriff, corruption was _______ on the police force.
A. profuse B. steadfast C. staunch D. rampant
12. He started a small delicatessen, and within weeks he was making money ________.
A. hand over fist B. hand over mouth C. neck over shoulder D. arm over leg
13. They were really excited to the project, but now they seem to have gone off the ________.
A. mill B. rails C. steam D. boil
14. She gave him a long, ________ kiss before saying goodbye.
A. transient B. persistent C. lingering D. protracted
15. The train service has been a _____ since they introduced the new schedules.
A. shambles B. rumpus C. chaos D. fracas
16. The designer refuses to gild the _____, preferring simple lines of his creations.
A. lily B. rose C. daisy D. daffodil
17. The organization had to _____ its emergency fund in response to the unexpected catastrophe.
A. set aside B. fall upon C. dip into D. take up
18. Before a judge takes a decision, he has to take all the evidence into ______.
A. consideration B. speculation C. deliberation D. exemplification
19. The country is an economic ______ with chronic unemployment and pervasive crime.
A. lost cause B. basket case C. false dawn D. dark horse
20. The job creation scheme is still in its ______.
A. childhood B. babyhood C. initiation D. infancy
Your answers
1. 2. 3. 4. 5. 6. 7. 8. 9. 10.
11. 12. 13. 14. 15. 16. 17. 18. 19. 20.
Part 2: For questions 21 – 30, write the correct form of each bracketed word in each sentence in the
corresponding numbered spaces provided in the column on the right.
Your answers
21. He represents not the continuing power of symbolic _____ but its recession into
21. _____________
the dead past. (INVEST)
22. The cost of living keeps rising, especially for _____ expenditure such as rent and
22. _____________
mortgage payments, healthcare and childcare. (DISCRETION)
23. Our expansion plans will have to be ______ in the current economic crisis.
23. _____________
(BURN)

Page 3 of 14 pages
24. The judge dismissed his DUI charge but convicted him for improper lane
24. _____________
changing and refusing the _____ test. (BREATHE)
25. ______ is the quickest method of removing potassium from the bloodstream.
25. _____________
(FILTER)
26. Within the system, he’s a pitiful nothing, unable to speak without a _____, an
26. _____________
affirmative-action phony doomed to inevitable defeat. (PROMPT)
27. Many voters feel that the governor has not acted in a(n) ______ manner for
27. _____________
someone who serves as the state's chief executive. (FIT)
28. Rob suspected this was perhaps the first real _____ he had had in his life,
28. _____________
something that probably his own father had never done. (DRESS)
29. It is the recognition of identity _____ that has allowed these women to construct a
29. _____________
new identity brick by brick. (CLOSE)
30. These recordings were often of very poor technical quality, many having been
30. _____________
made in _____ studios behind record shops. (MAKE)

C: READING (60 points)


Part 1: For questions 1 – 10, read the text below and fill in each of the following numbered blanks with
ONE suitable word. Write your answers in the corresponding numbered boxes.
In the immediate post-war years, the city of Birmingham scheduled some 50,000 small working class
cottages as slums (1) ______ for demolition. Today that process is nearly complete. Yet it is clear that, quite
apart from any question of race, an environmental problem (2) ______. The expectation built into the planning
policies of 1945 was that in the foreseeable future the city would be a better place to live in. But now that slum
clearance has run its (3) ______, there seems to be universal agreement that the total environment where the
slums (4) ______ stood is more depressing than ever.
For the past ten years the slum clearance areas have looked like bomb sites. The buildings and places
which survive (5) ______ so on islands in a sea of rubble and ash. When the slums were there they supported
an organic community life and each building, each activity, fitted in as part of the (6) ______. But now that they
have been destroyed, nothing meaningful appears to remain, or (7) ______ those activities which do go on do
not seem to have any meaningful relation to the place. They happen there because it is an empty stage which
no one is using anymore.
Typical of the inner-city in this sense is the Birmingham City Football Ground. Standing in unsplendid (8)
______ on what is now wasteland on the edge of Small Heath, it brings into the area a stage army on twenty or
so Saturdays a year who come and cheer and then go away again with little concern any more for the place
where they have done their cheering. Even they, however, have revolted recently. ‘The ground’ says the leader
of the revolt, ‘is a slum’, thus putting his (9) ______ on the fact that the demolition of houses creates rather than
(10) ______ problems of the inner-city.
Your answers
1. 2. 3. 4. 5.

6. 7. 8. 9. 10.
Part 3: For questions 11 – 20, read the passage below and choose the correct answer (A, B, C or D) to
each of the following questions. Write your answers in the corresponding numbered boxes.
WRITING FICTION
Because I am a novelist myself, I am always faintly fussed by the idea of creative writing courses. I
completely accept that you can teach the craft, that you can give instruction on how to structure a book, how to
vary space and tension, how to write dialogue. But what you can’t teach, it seems to me is the right kind of
interpretation of what has been observed. It worries me to think of all those earnest pupils who have diligently
mastered the mechanics, wondering with varying degrees of misery and rag why the finished recipe just hasn’t
somehow worked.
The great writer Samuel Coleridge explained it. He said that there are two kinds of imagination, the
primary and the secondary. We all, he said, possess the primary imagination, we all have the capacity to
perceive, to notice. But what only poets (loosely translated as all truly creative people, I suppose) have - the
secondary imagination is the capacity to select, and then translate and illuminate everything that has been
observed so that it seems to the audience something entirely new, something entirely true, something exciting,
wonderful and terrible.

Page 4 of 14 pages
There is, after all, nothing new to say about the human condition. There is nothing to say that
Shakespeare or Sophocles hasn’t already, inimitably, brilliantly, said. Codes of product, fashions in morality and
ethics, all may come and go. But what the human heart has desired - and feared – down the ages goes on being
very much the same. The novelist’s task is to follow the well-trodden, time-worn path of human hopes and
terrors. Never forget: betrayal may be as old as time, it may happen every nanosecond of every minute that’s
ever been, but the first time it happens to you feels like the first time in the history of the world. A cliché is a
cliché only if it is comfortably taking place in someone else’s life.
This empathy is vital in the writing of fiction. Coleridge’s view of the poet as prophet to the hungry
hordes is, in truth, a bit grand for me. I admire it, but I am not, personally, quite up to it. I am happier seeing the
novelist, sleeves rolled up, in the thick of it alongside the reader, bleeding when pricked, in just the same way
that the reader does. The only capacity I would claim is that I have an instinct to select, from everything I have
noticed in half a century’s beady-eyed people-watching, the telling detail, the apt phrase. I seem to be good at
the rhythms of dialogue. I seem to know how not to overwrite. But that is it really. Except that the older I get, the
more prepared I am to surrender and trust to the power of the unconscious mind. Maybe this is a modest form of
the secondary imagination, maybe not. Whatever it is, it produces a level and intensity of communication that
causes people to buy my books and write to me about them in numbers that I still can’t get over.
What I do believe, fervently, is that we are all in this boat together – writer, reader, critic. I have a tattered
little quotation that lies on my desk and becomes more valuable to me as time goes on. It comes from the
autobiography of the celebrated nineteenth-century writer Anthony Trollope. He said many remarkable things in
this book, but my own personal favourite is on the subject of the novelist’s central preoccupation. Trollope is not
so much concerned with the landscape of the grand passions as with something else, something less glamorous
perhaps, but just as intense and certainly more universal: ‘My task’, he wrote, ‘is to chronicle those little daily
lacerations upon the spirit.’
I feel a thrill of recognition every time I read that, or even think about it. That is what the writer’s life is all
about for me. The point of it is to emphasise that we are none of us immune to longing, or disappointment (much
under-rated, in my view, as a source for distress), or frustration, or idiotic hope, or bad behaviour. What fiction
does, in this difficult world, is to reassure us that we are not alone, nor we are (most of us) lost causes. There is
a theory that suffering strengthens and elevates us in a way that joy can never somehow do. I’m not so sure
about that. Isn’t it just that we have, on the whole, so much more suffering than joy that we have resolved, out of
our great surviving instinct, to insist that something worthwhile must be made of it? And isn’t fiction a handrail,
of a kind, which we can all grasp while we blunder about in the dark? Isn’t fiction written by people for people
about people? And is there a subject more fascinating or more important?
11. What view does the novelist express about creative writing courses?
A. A few good books emerge from them.
B. It would be inappropriate for her to teach on them.
C. Students are frustrated by the poor teaching on them.
D. Some aspects of writing skills can be successfully taught on them.
12. The novelist implies that a writer’s most valuable asset is ______.
A. an instinct for the unusual
B. a gift for meticulous observation
C. the ability to put a fresh interpretation on the everyday world
D. the ability to highlight sensational aspects of our existence
13. What is stated about writers in the third paragraph?
A. They should not exploit their readers’ fears.
B. They should revisit well-established themes.
C. They should be prepared to exaggerate their personal experience.
D. They should not try to keep pace with changes in literary tastes.
14. The phrase ‘the well-trodden, time-worn path’ refers to themes of writing that are ______.
A. familiar and long-standing B. extraordinary and profound
C. up-to-date and catchy D. simple and soulful
15. The word ‘prophet’ refers to writer as a(n) ______ person.
A. conservative B. receptive C. impartial D. emotional
16. The novelist states that one of her own strengths as a writer lies in ______.
A. her depiction of character B. her construction of plot
C. her command of language D. her knowledge of psychology

Page 5 of 14 pages
17. Why does novelist admire Anthony Trollope?
A. He portrays the fact that everyone suffers in some way.
B. He realises that all writers need a strong sense of place.
C. He understands that everyone craves deep emotion.
D. He is aware that all writers have a particular obsession.
18. The word ‘lacerations’ refers to ______ events.
A. exhilarating B. epoch-making C. pathetic D. trivial
19. The novelist describes fiction as ‘a handrail, of a kind’ because it ______.
A. reflects the negative aspects of emotion B. enables us to deal with failure
C. helps us make sense of complex events D. offers reassurance in an uncertain world
20. Which theme recurs in this text?
A. The need for novelists to avoid complex philosophical questions
B. The need for novelists to develop their writing techniques
C. The need for novelists to give an accurate reflection of the spirit of the time
D. The need for novelists to identify closely with readers’ preoccupations
Your answers
11. 12. 13. 14. 15.

16. 17. 18. 19. 20.


Part 3: Read the following passage and answer the questions 21 – 33.
ANCIENT CHINESE CHARIOTS
A. The Shang Dynasty or Yin Dynasty, according to traditional historiography, ruled in the Yellow River valley in
the second millennium. Archaeological work at the Ruins of Yin (near modern-day Anyang), which has been
identified as the last Shang capital, uncovered eleven major Yin royal tombs and the foundations of palaces
and ritual sites, containing weapons of war and remains from both animal and human sacrifices.
B. The Tomb of Fu Hao is an archaeological site at Yinxu, the ruins of the ancient Shang Dynasty capital Yin,
within the modem city of Anyang in Henan Province, China. Discovered in 1976,it was identified as the final
resting place of the queen and military general Fu Hao. The artifacts unearthed within the grave included
jade objects, bone objects, bronze objects etc. These grave goods are confirmed by the oracle texts, which
constitute almost all of the first hand written record we possess of the Shang Dynasty. Below the corpse was
a small pit holding the remains of six sacrificial dogs and along the edge lay the skeletons of human slaves,
evidence of human sacrifice.
C. The Terracotta Army was discovered on 29 March 1974 to the east of Xian in Shaanxi. The terracotta
soldiers were accidentally discovered when a group of local farmers was digging a well during a drought
around 1.6 km (1 mile) east of the Qin Emperors tomb around at Mount Li (Lishan), a region riddled with
underground springs and watercourses. Experts currently place the entire number of soldiers at 8,000 —
with 130 chariots (130 cm long), 530 horses and 150 cavalry horses helping to ward of any dangers in the
afterlife. In contrast, the burial of Tutank Hamun yielded six complete but dismantled chariots of unparalleled
richness and sophistication. Each was designed for two people (90 cm long) and had its axle sawn through
to enable it to be brought along the narrow corridor into the tomb.
D. Excavation of ancient Chinese chariots has confirmed the descriptions of them in the earliest texts. Wheels
were constructed from a variety of woods: elm provided the hub, rose-wood the spokes and oak the felloes.
The hub was drilled through to form an empty space into which the tampering axle was fitted,the whole
being covered with leather to retain lubricating oil. Though the number of spokes varied, a wheel by the
fourth century BC usually had eighteen to thirty-two of them. Records show how elaborate was the testing of
each completed wheel: flotation and weighing were regarded as the best measures of balance, but even the
empty spaces in the assembly were checked with millet grains. One outstanding constructional asset of the
ancient Chinese wheel was dishing. Dishing refers to the dish-like shape of an advanced wooden wheel,
which looks rather like a flat cone. On occasion they chose to strengthen a dished wheel with a pair of struts
running from rim to rim on each of the hub. As these extra supports were inserted separately into the felloes,
they would have added even greater strength to the wheel. Leather wrapped up the edge of the wheel aimed
to retain bronze.
E. Within a millennium, however, Chinese chariot-makers had developed a vehicle with shafts, the precursor of
the true carriage or cart. This design did not make its appearance in Europe until the end of the Roman
Empire. Because the shafts curved upwards, and the harness pressed against a horse’s shoulders, not his
neck, the shaft chariot was incredibly efficient. The halberd was also part of chariot standard weaponry. This
halberd usually measured well over 3 metres in length, which meant that a chariot warrior wielding it

Page 6 of 14 pages
sideways could strike down the charioteer in a passing chariot. The speed of chariot which was tested on the
sand was quite fast. At speed these passes were very dangerous for the crews of both chariots.
F. The advantages offered by the new chariots were not entirely missed. They could see how there were
literally the warring states, whose conflicts lasted down the Qin unification of China. Qin Shi Huang was
buried in the most opulent tomb complex ever constructed in China, a sprawling, city-size collection of
underground caverns containing everything the emperor would need for the afterlife. Even a collection of
terracotta armies called Terra- Cotta Warriors was buried in it. The ancient Chinese, along with many
cultures including ancient Egyptians, believed that items and even people buried with a person could be
taken with him to the afterlife.
For questions 21 – 24, decide whether the following statements are True (T), False (F) or Not Given (NG).
Write your answers in the corresponding numbered spaces provided in the column on the right.
Your answers
When discovered, the written records of the grave goods proved to be accurate. 21. _________
Human skeletons in Anyang tomb were identified as soldiers who were killed in the war. 22. _________
The Terracotta Army was discovered by people lived who lived nearby, by chance. 23. _________
The size of the King Tutankhamen’s tomb is bigger than that of in Qin Emperors’ tomb. 24. _________
For questions 25 – 30, complete the following note using ONE WORD ONLY taken from the passage for
each blank. Write your answers in the corresponding numbered spaces.

 The hub is made of wood from the tree of (25) _______


 The room through the hub was to put tempering axle in which is wrapped up
by leather aiming to retain (26) _______
 The number of spokes varied from 18 to (27) _______
 The shape of wheel resembles a (28) _______
 Two 9 was used to strengthen the wheel (29) _______
 Leather wrapped up the edge of the wheel aimed to remain (30) _______
For questions 31 – 33, write your answers to the questions below, using NO MORE THAN THREE
WORDS AND/OR A NUMBER from the passgae for each blank.
31. What body part of horse was released the pressure from to the shoulder?
_________________________________________________________
32. What kind road surface did the researchers measure the speed of the chariot?
_________________________________________________________
33. What part of his afterlife palace was the Emperor Qin Shi Huang buried in?
_________________________________________________________
Part 4: In the passage below, seven paragraphs have been removed. For questions 34 – 40, read the
passage and choose from paragraphs (A-H) the one which best fits each gap. There is ONE EXTRA
PARAGRAPH you do not need to use. Write your answers in the corresponding numbered boxes.
The 110 young men, women, and children who boarded the Clotilda in May 1860 came from Bantè, Dahomey,
Kebbi, Atakora, and other regions of Benin and Nigeria. Among them were people from the Yoruba, Isha, Dendi,
Nupe, and Fon ethnic groups. Their parents had named them Kossola, Kupollee, Abile, Abache, Gumpa.
34.
One man, Kupollee, had a small hoop in each ear, which meant he had been initiated in an ile-orisa-house of the
god - into the religion of the Yoruba. Ossa Keeby came from Kebbi in Nigeria, a kingdom renowned for its
professional fishermen. Like 19-year-old Kossola (later known as Cudjo Lewis), several were victims of a raid by
the slave-trading kingdom of Dahomey. Kossola said he came from modest means, but his grandfather was an
officer of a Bantè king. At 14 he trained as a soldier and later began initiation into the Yoruba oro, the male
secret society. A young girl, Kêhounco (Lottie Dennison), was kidnapped, as were many others. Their forced
journeys ended in a slave pen in Ouidah.
35.
In the morning the dejected group waded neck-deep across a lagoon to reach the beach, where canoes
transported them over the dangerous, sometimes deadly, surf to the Clotilda. What happened next haunted
them forever. They were forced to remove their clothes. The Africans’ total nakedness was a rule of the slave
trade, officially - although quite ineffectively - to maintain cleanliness. The last Clotilda survivors still bristled
years later at the humiliation of being called naked savages by Americans who believed nudity was “African.”
36.

Page 7 of 14 pages
Slave ships were places of unspeakable misery. Solidarity was vital, and those who suffered together forged
lifelong relationships that sometimes spanned generations - if they were not separated again. On the Clotilda,
over a month and a half, such a community was born.
37.
Short of workers for their developing plantations, slaveholders in the Deep South had for years bought people
from the upper South at prices they found outrageous. With the international slave trade illegal, some turned to
smuggling. In Alabama, despite Foster and Meaher’s precautions, the “secret” arrival was all over town and in
the press within a day or two. Meanwhile, the young Africans had disembarked into the desolate, mosquito-
infested canebrakes of Dabney’s Clarke County plantation. Moved from one place to another to avoid detection,
they were fed meat and cornmeal that made them sick. They welcomed the rags, pieces of cornsacks, and skins
they were given in lieu of clothes. When federal authorities sent a crew led by a U.S. marshal to find them, the
Africans had already been moved to Burns’s plantation. They “almost grieved themselves to death,” they
confided half a century later.
38.
Timothy Meaher was arrested, released on bail, tried, and cleared of all charges. Federal cases against Burns
Meaher and Dabney were dismissed because “said negroes” were never found. Foster was fined $1,000 for
failing to pay the duties on his “imports.” Timothy Meaher awarded himself 16 males and 16 females; Burns took
20 of the captives, including Kêhounco; and James Meaher took Kossola and seven of his companions. Foster
received 16 individuals, among them Abile (Celia Lewis). Each person bought for $100 in Ouidah was now worth
$1,000, and once acclimated could be sold for $2,000, or $60,000 in today’s dollars.
39.
When Meaher’s cook, Polly, slapped one of the young girls, she screamed like a “wild cat in the darkness,” Hart
said. Her shipmates came running from the fields with rakes, spades, and sticks in hand. Polly darted up the
stairs to Mary Meaher’s room. They followed her and banged on the door. Polly quit. One day Burns’s overseer
tried to whip a young woman. They all jumped on him, grabbed the lash, and beat him up. He never tried to
brutalize them again. One of the Africans, Sakarago, argued with a white man and was unconcerned by the high
price he could pay for his audacity. But it appears that where the shipmates were isolated, just two or three to a
plantation, they were poorly treated. Redoshi (Sallie Smith) told civil rights activist Amelia Boyton Robinson that
“the slave masters and overseers beat us for every little thing when we didn’t understand American talk.”
40.
For five years the shipmates labored in the cotton, rice, and sugarcane fields. In Mobile several men worked on
the river ships, firing the furnaces with tons of timber, loading and unloading bales of cotton. During the Civil
War, forced to build the city’s fortifications, they lived in abject conditions. At last, on April 12, 1865, freedom
came when the Union Army entered the city. The Africans celebrated to the beat of a drum.
THE MISSING PARAGRAPHS
A. On July 8 the shipmates glimpsed land in the distance. They heard a noise they likened to a swarm of bees.
It was the sound of a tugboat towing the Clotilda up Mobile Bay. They were transferred to a steamboat
owned by Timothy Meaher’s brother Burns and taken upriver to John Dabney’s plantation while Foster took
his ship to Twelve Mile Island. There was no hiding the squalid remnants of a slaving voyage, and Foster
risked the death penalty if caught. He lit loose wood or perhaps lantern oil, and the ship he had built five
years earlier went up in flames.
B. Amid the sheer horror and misery, the captives found support and solidarity, until foreign slavers irreparably
tore their newfound community apart. According to newspaper interviews and oral histories given by the
survivors over the years, when Clotilda captain Foster entered the grounds, people were ordered to form
circles of 10. After inspecting their skin, teeth, hands, feet, legs, and arms, he selected 125 individuals. In the
evening they were told they would leave the next day. Many spent the night crying. They had no idea what
loomed ahead and did not want to be separated from their loved ones.
C. The next phase of the shipmates’ tribulation was their entry into the savage plantation world inhabited by
black and white strangers. Up to then they had been Yoruba, Dendi, Nupe, or Fon, with different languages
and cultures. At that moment they became Africans. Identifying with a continent was as alien to them as it
was to Europeans. But they embraced their new identity with pride, regardless of others’ contempt. Noah
Hart, enslaved on Timothy Meaher’s plantation, recalled that they looked fierce, yet they never threatened
the African Americans on the plantation or quarreled among themselves. Acting as a group, they “wouldn’t
stand a lick” from whites or blacks. Several times they engaged in collective acts of resistance, unafraid of
the consequences.
D. Before the transfer was over, Foster saw steamers approaching. Afraid he would be caught, he sailed away,
leaving 15 people on the beach. For the first 13 days at sea, every captive remained confined in the hold.
Decades later, in 1906, when Abache (Clara Turner) talked of the filth, the darkness, the heat, the chains,
and the thirst to a writer from Harper’s magazine, “her eyes were burning, her soul inexpressibly agitated at
the memory.” Despair, agony, and horror were compounded for powerless parents unable to alleviate their
children’s fears and suffering. One woman, later known as Gracie, had four daughters on board; the
youngest, Matilda, was about two years old. The lack of water was torture, and the meals - molasses and

Page 8 of 14 pages
mush - did not help. The sugary foods only intensified their thirst. “One swallow” twice a day was all they got,
and it tasted like vinegar. The rain they caught in their mouths and hands was a fleeting relief. There was
sickness, and two people died.
E. If their hometown was a nurturing haven, the African homelands were the idyllic places their mothers and
fathers dreamed of. “They say it was good there,” recalled Eva Allen Jones, Kupollee’s daughter. “I seen
them sit down and shed tears. I see my father and Uncle Cudjo weep and shed tears talking about going
home.”
F. The Africans largely kept to themselves and maintained practices they had grown up with. The people from
Atakora, in present-day Benin, buried their dead in deep graves, the corpses wrapped in bark. The Yoruba
plunged their new-borns into a creek, looking for signs of vitality. One Fon couple tattooed their son’s chest
with the image of a snake biting its tail, a sacred symbol of the kingdom of Dahomey.
G. Timothy Meaher, eager to quickly settle his affairs, organized a sale. As their new family was separated once
again, the shipmates cried and sang a farewell song, wishing one another “no danger on the road.” While
about 80 were taken to Mobile, the Mercury newspaper of July 23, 1860, reported, “some negroes who
never learned to talk English, went up the Railroad the other day ... There were twenty-five of them,
apparently all of the pure, unadulterated African stock.” As the group was walking, a circus passed by, and
when the Africans heard an elephant, they screamed, “Ile, ile, ajanaku, ajanaku,” (“home,” “elephant,” in
Yoruba and Fon). They spent the rest of their lives scattered across the Black Belt of Alabama. Gracie was
sold along with two of her daughters, but agonisingly, she never knew what happened to her other two.
H. Some were long-distance traders, likely carrying salt, copper, and fabric. They may have produced iron.
Others may have woven cloth, harvested yams, or made palm oil. Some women were married and had
children; they likely worked as farmers or market traders.
Your answers
34. 35. 36. 37. 38. 39. 40.
Part 5: You are going to read an article about Prairie Fever. For questions 41 – 50, choose the best
answer from sections (A–E). Some of the choices may be required more than once. Write your answers
in the corresponding numbered spaces in the right column.
PRAIRIE FEVER
A. How the British aristocracy was drawn to the frontier lands of 19th-century America is perhaps the most
bizarre episode in the country's epic immigration story, and is revealed in a remarkable new book, Prairie
Fever, by veteran BBC documentary maker Peter Pagnamenta. Lured by romantic tales of the American
outdoors by writers such as James Fennimore Cooper, and the real-life gun-slinging escapades of Wild Bill
Hickock, these eccentric newcomers wanted the U.S. on their own terms. In settlements with reassuringly
British names, such as Runnymede and Victoria, the British aristocracy set about ensuring that there was
one corner of America that was forever England.
B. The pioneers started arriving in the 1830s. Some were sportsmen drawn by the promise of unlimited buffalo
to hunt, others true adventurers. They were led by Scotsman Sir William Stewart, a Waterloo veteran who
spent seven years trekking through the Rockies, rubbing shoulders with mountain men, and fending off
marauding bears and Indians. His companion, Charles Murray, son of the Earl of Dunmore, lived for a spell
with the Pawnee Indians. The Old Etonian had to swallow his pride when his hosts ate his dog, but he
impressed with rock-throwing contests in which he used skills honed in the Highland Games. Sadly, few of
the lords that followed were nearly so adaptable. They often treated the locals and their customs with utter
contempt. Sir George Gore — a classic example of the breed -- went on a $100,000, three-year hunting
expedition beginning in 1854 in Missouri. American officials later accused him of slaughtering 6,000 buffalo,
single-handedly endangering the Plains Indians Food supply. Later, the English settlers wound up the
Americans even more because of their air of superiority.
C. By the 1870s, however, their American hosts had more to complain about than aristocratic rudeness – the
British wanted to settle permanently. The British ruling classes had realised that the American West wasn't
just a good place to hunt and carouse, but also the perfect dumping ground for younger sons with few
prospects at home, America, desperate for new settlers to farm prairie states like Kansas and Iowa,
welcomed them with open arms. Back in Britain, the Press followed the settlers closely. ‘It was hot but
everyone looked happy ... how much more sensible and useful lives they live there than they would live here
at home!’ the Times reported. Yet more astute observers noted that the British settlers never grasped the
American work ethic. For them, running their farms came a poor second to hunting and enjoying themselves.
D. The prairie states were already dotted with ‘colonies’, each made up exclusively of workers from one part of
America or one group of immigrants such as Danes or Russians. In 1873, an enterprising Scottish
gentleman farmer named George Grant had a brainwave - a colony in western Kansas populated entirely by
the British upper classes, by stipulating that they had to have at least £2,000 in funds and would each get no
less than a square mile of land, he kept out the rabble. Victoria, as Grant patriotically called his settlement,
was talked back home as a ‘Second Eden’, but the new arrivals —many of whom had never farmed in their
lives- soon discovered it was a hard place to play the country gentleman. No rain would fall for months and
the temperature could soar to 105F in the shade. Worst of all, nobody had mentioned the dense clouds of

Page 9 of 14 pages
grasshoppers that would suddenly arrive and eat everything. Despite their neighbours’ derision at these
remittance men’ (so named because they relied on allowances from their parents), the two hundred or so
colonists gamely battled on.
E. In general, the colonists’ dreams came to nothing, and many headed home. But there was one event that
definitively ended the British aristocracy’s love affair with the West. Encouraged by the vast sums to be
made from cattle ranching, some wealthy British investors bought huge tracts of land. One investor alone
amassed 1.75 million acres and 100,000 cattle. Enough was enough. Tolerant when the British were
buffoonish adventurers, Americans felt threatened once they became too rich. U.S. politicians stoked anti -
foreigner resentment, aided by widows out of their homes and rumours that some were so snobby they
referred to their cowboys as ‘cow-servants’. Congress passed the Alien Land Act limiting foreign companies
to buying no more than 5,000 acres in future.
In which section is each of the following mentioned? Your answers

 the opinion that the settlers never got their priorities right 41. ________

 the fact the settlers wanted nothing less than a home away from home 42. ________

 the wish to maintain exclusivity in the British colonies 43. ________

 the inability of the settlers to become truly independent of Britain 44. ________

 the view that the English were naive in their expectations of the USA 45. ________

 a difference in the locals’ and settlers’ cultural taboos 46. ________

 the view that the British settlers were victims of their own success 47. ________

 the fact that America offered a solution to a problem 48. ________

 a newspaper showing lack of perception 49. ________

 the disregard of the settlers for the locals’ way of life 50. ________

D: WRITING (60 points)


Part 1: Read the following extract and summarise it with your own words. Your summary should be
between 100 and 120 words long. YOU MUST NOT COPY THE ORIGINAL.
A computer chip is a tiny piece of material (usually made of silicon), that contains a complex electronic
circuit. These chips are essential in modern computers and a variety of other electronic devices. The circuit on a
computer chip, sometimes called an integrated circuit, is made up of electronic components built into the chip.
Most chips are no larger than a fingernail.
Two Americans - Jack Kilby, an engineer, and Robert Noyce, a physicist, who worked independently -
patented the first computer chips in 1959. During the 1960s, scientists developed chips for guided missiles and
satellites. Engineers soon began to build smaller and faster computers by using chips in place of conventional
circuits. The first microprocessors were produced in 1971 for use in desktop calculators.
There are two main kinds of computer chips. The first is called a microprocessor, which carries out the
instructions that make up computer programs, and the other type is called a memory chip, which holds computer
programs and other data. Memory chips are used primarily in computers. Microprocessors are used in
computers and hundreds of other products. A microprocessor serves as the 'heart' of every personal computer.
Larger computers have more than one such chip. Other products controlled by microprocessors include video
games, digital watches, microwave ovens and some telephones.
The body of most chips is made of silicon. This material is used because it is a semiconductor. In its pure
form, silicon does not conduct electricity at room temperature. But if certain impurities are added to silicon, it can
carry an electric current. Manufacturers 'dope' silicon chips with such impurities as boron and phosphorus. The
doped regions form the chip's electronic components, which control the electric signals carried on the chip. The
type and arrangement of the impurities determine how each component controls signals. Most components
serve as switches called transistors. Others serve mainly as capacitors, which store an electric charge; diodes,
which prevent current from flowing in one direction but not the other, and resistors, which control voltage.
The manufacturing of a computer chip begins with a wafer of doped silicon. The wafer measures from
2.5 to 20 centimeters in diameter. A photographic process reduces a large master design for the integrated
circuit to microscopic size. Technicians use these microscopic designs, called masks, as stencils to make
hundreds of chips on one wafer. After the wafer has been processed, it is divided into individual chips. Some
chips contain millions of components. Certain parts of these components measure less than 1 micrometer (0.001

Page 10 of 14 pages
millimeter) across. Manufacturers create thin lines of metal - usually aluminium - on the chip to connect these
tiny components.

Part 2: This graph ranks active male tennis players by the number of singles Grand Slam titles they have
won so far. Write a report of about 150 words describing the information in the graph. Select and report
the main features and make comparisons where relevant.

Page 11 of 14 pages
Part 3: Write an essay of about 350 words to express your opinion on the following topic:
In any situation, progress requires discussion among people who have contrasting points of view.
Write a response in which you discuss the extent to which you agree or disagree with the statement and explain
your reasoning for the position you take. In developing and supporting your position, you should consider ways
in which the statement might or might not hold true and explain how these considerations shape your position.

Page 12 of 14 pages
Page 13 of 14 pages
THE END

Page 14 of 14 pages
TRƯỜNG THPT KỲ THI HSG CÁC TRƯỜNG THPT CHUYÊN KHU VỰC
CHUYÊN SƠN LA DUYÊN HẢI VÀ ĐỒNG BẰNG BẮC BỘ
NĂM HỌC 2021 - 2022
ĐỀ ĐỀ XUẤT Môn thi: TIẾNG ANH
Ngày thi: 14/07/2022
(Đề thi có 18 trang) Thời gian làm bài: 180 phút không kể thời gian phát đề

I. LISTENING (50 points)


PART 1: For questions 1-5, you will hear an explorer called Richard Livingstone talking about a
trip he made in the rainforest of South America. Listen and decide whether the statements are
True (T) or False (F). Write your answers in the corresponding numbered boxes provided.
1. They went all the way by boat.
2. Richard said that during the walk, they were always both cold and wet.
3. In a deserted camp, they found some soup made from unusual meat and vegetables.
4. After the meal, they began to feel worried about what they have done.
5. Before leaving the camp, they left the sum of 50 dollars to thank the host.
Your answers:

1. 2. 3. 4. 5.
PART 2: For questions 6-10, listen to a piece of news and answer each question with NO MORE
THAN THREE WORDS AND/OR A NUMBER. Write your answers in the corresponding
numbered spaces provided.
6. How are temperatures compared to pre-industrial levels in the second scenario?
________________________________________________________________
7. What is one way in which 10 feedback loops keep the earth cool?
________________________________________________________________
8. What would happen if all the world’s permafrost were melt?
________________________________________________________________
9. What is Amazon rainforest being quickly converted into?
________________________________________________________________
10. What has none of the feedback systems done?
________________________________________________________________
Your answers:

6. 7. 8. 9. 10.

Part 3. For questions 11-15, listen to a discussion in which two people, Derek Grant and Lucy
Wadham, talk about the current state of mass tourism and choose the answer (A, B, C or D)
which fits best according to what you hear. Write your answers in the corresponding numbered
boxes provided.

11. From the figures quoted by Lucy, it can be concluded that ______.
A. most of the revenue generated from safari packages goes to the airline.
B. mass tourism inevitably leads to poverty.
C. hotels in developing countries do not have high standards.
D. there aren't sufficient local products to cater for tourists.

Page 1 of 18
12. What does Derek say about governments whose countries are tourist destinations? A. They are
constantly trying to increase their profit margins.
B. They ignore the negative impacts of mass tourism.
C. They have very limited bargaining power.
D They are unfamiliar with aspects of modern business methods.

13. Derek's example of excessive water consumption caused by mass tourism shows how _____
A. ineffective the laws are in developing countries.
B. local food production can be adversely affected by it.
C. difficult it is to find sources of pure water.
D. valuable a resource water has become in recent years.

14. Lucy says that when a resort loses its appeal, _____
A. the locals are left to fend for themselves.
B. money has to be spent on a new infrastructure.
C. the cost of package holidays falls considerably.
D. it becomes vulnerable to natural disasters.

15. According to Derek, tourism ______


A. does more harm than good in the long run.
B. is too big an industry to be tampered with.
C. is largely responsible for global inequality.
D. promotes cultural sensitivity most of the time.
Your answers:

11. 12. 13. 14. 15.

Part 4. For questions 16-25, listen to a talk about pandemic diseases and supply the blanks with
the missing information. Write NO MORE THAN FOUR WORDS taken from the recording for
each answer in the space provided.
TOP 5 DEADLIEST PANDEMIC DISEASES

1. Smallpox
• first emerged around 400BC
• caused 16.______________________ all over the body
• the only disease declared to be 17.______________________
2. Bubonic Plague
• also known as the 18.______________________
• caused by a bacterium spread by 19.______________________
• 20.______________________, called buboes, occurred in the body
3. The Spanish Flu
• began and ended in a 21.______________________
• filled the lungs of patients with 22.______________________
4. Malaria
• categorized as a blood disease
• caused by 23.______________________

Page 2 of 18
• Anopheles mosquitoes would 24.______________________ infected blood and pass it on to the
next person they bite
5. HIV/AIDS
• often sexually transmitted
• HIV 25.______________________ the immune system
Your answers:
16. 17. 18. 19. 20.

21. 22. 23. 24. 25.

II. LEXICAL AND GRAMMAR (20 points)


Part 1. Choose the correct answer A, B, C, or D to each of the following questions. Write your
answers in the corresponding numbered boxes provided.
1. Stephen really lost his ________ when his dental disappointment was cancelled yet again.
A. head B. voice C. calm D. rag
2. It only took me a few minutes to get the printer up and ________ after taking it out of
the box.
A. walking B. proceeding C. running D. going
3. It was a hot summer day and ice cream salesmen were doing a ________ trade.
A. busy B. lucrative C. bustling D. roaring
4. Bob is so short-tempered; he should try to ________ his anger.
A. curb B. control C. temper D. stunt
5. If you ________ your demands, they may be accepted bu others.
A. control B. temper C. stunt D. curb
6. As far as I can ________, she wasn't there on that occasion.
A. memorise B. reminisce C. recollect D. remind
7. I'm sure it won't rain, but I'll take an umbrella (just) to be on the ________ side.
A. sunny B. straight C. safe D. secure
8. I was in ________ at that comedy show. I could barely breathe it was so funny.
A. stitches B. pleats C. shreds D. tears
9. She tried to ________ Tom’s importance to the company in order to gain a promotion for herself.
A. diminish B. swindle C. reduce D. shrink
10. I hate the way Tony ________ around looking so self – important.
A. struts B. scampers C. slithers D. slinks
11. Most frequently, the earthquake lasts 30 to 60 seconds, so usually there is no time to avert the
mortal ________ once the shaking starts.
A. upkeep B. upturn C. upshot D. upswing
12. They live under a constant pressure of being ________ and subsequently replaced by someone
who is younger, faster and more accomplished.
A. outcast B. outshone C. outstayed D. outgrown
13. My brother loves watching horror movies but I find them too ________ myself.
A. gruesome B. untimely C.ghostly D. sterile
14. There was a lot of _______ as throngs of people tried to see the famous actor walking through
the mall

Page 3 of 18
A. stamina B. discipline C. counsel D. commotion
15. Luckily my wallet was handed in to the police with all its contents _______.
A. preserved B. unscathed C. contained D. intact
16. Serena is still _______ ignorant of the fact that she is about to be made redundant.
A. blissfully B. decorously C. jubilantly D. ecstatically
17. No decision has been taken about the building of the new airport. The authorities are still
_______.
A. beating about the bush B. comparing apples and oranges
C. sitting on the fence D. holding all the aces
18. Oil spills will _______ even the healthiest of marine ecosystem.
A. play havoc on B. break ground with
C. pay the consequences for D. take their toll on
19. Don’t look so worried! You should take the boss’s remarks with a ________of salt.
A. teaspoon B. pinch C. grain D. dose
20. Rachel has an amazing ________ of jokes that she uses to good effect at parties.
A. body B. repertoire C. variation D. store

Your answers:
1. 2. 3. 4. 5.

6. 7. 8. 9. 10.

11. 12. 13. 14. 15.

16. 17. 18. 19. 20.

Part 2. For questions 21-30, write the correct form of each bracketed word in each sentence in
the corresponding numbered boxes provided.

21. A portion of the proceeds will be ________ for providing school fees for poor children for the
coming academic year. (mark)

22. The administration ropes in all educational institutions, government offices, public sector
________ and universities for the purpose of mobilising funds. (take)

23. Contraception is less ________ or affordable in South America. (cure)

24. While learning has changed for students in this new century, we are ________ by the boundless
opportunity presented in our lifetime. (bold)

25. It does not become an economic ________ but it does become a ‘newly industrialized country’,
like Malaysia, Taiwan and South Korea. (power)

26. We believe that the most effective enforcement tool is self-policing and ________ (strain).

27. It is possible to humanely raise and slaughter a variety of food animals, including ________
poultry and beef cattle. (range)

Page 4 of 18
28. Most of this feature includes behind-the-scenes video ________ of the crew working and
goofing off. (foot)

29. Together they forged a(n) ________ intellectual climate that has profoundly shaped my career.
(vigor)

30. We will investigate the tradeoffs among data ________, data hiding capacity, and probabilities
of extraction errors in different applications. (perceive)

Your answers:

21. 22. 23. 24. 25.

26. 27. 28. 29. 30.

III. READING (60 points)

Part 1. For questions 1-7, you are going to read a magazine article. Seven paragraphs have been
removed from the extract. Choose from paragraphs A-H the one which fits each gap. There is
one extra paragraph you do not need to use. (7 points)
The Inuit
The way of life of aboriginal peoples the world over has been in decline for decades now, if not
centuries. Slowly but surely, all of it, from its spiritual underpinnings to its actual geographical
homeland, is being whittled away by the developed world. Even now that it is very nearly too late,
the demise of these cultures is seen as just one of many problems needing our generous attention.
Once again, we in positions of power have cast ourselves in the role of teacher with plenty to pass
on to our needy pupils when, in reality, we are the ones who have much to learn.
1
Their spiritual views, for example, provide the basis for all other activities, lending them in turn a
coherence and meaning that ensure that nothing is taken for granted. They believe that everything
possesses its own spirit – not only people and animals, but also inanimate objects and phenomena
such as the wind. These “inua”, as they are called, have independent existences of their own, and
those that are hosted by particularly strong animals or men can take on a physical presence
whenever they wish.
2
This belief, in turn, influenced other areas, such as art. Weapons like knives and harpoons were
intricately crafted, as this was believed to reflect the hunter’s esteem for the “inua”. Materials
selected for weapons were chosen because they were familiar and comforting to the prey; so, sea
mammals were commonly hunted with weapons made from walrus tusks, showing a concern for the
hunted which bordered on sympathy. Given that a single animal could provide food, oil, clothing,
and even boats – often made out of skins – it is easy to see how it warranted the proffered
reverence.
3
Rather than revealing the folly of peoples we consider more primitive than us, such practices and
tales show a deep awareness of and respect for the true relationship between people and their
environment. As also revealed in Inuit carvings of two-faced creatures, one face human and the
other animal, the relationship is one of mutual interdependence; nature preys on us as we prey on it,
and both factors in the equation need the other.

Page 5 of 18
4
In the mid-nineteenth century, European whalers began to actually live in the Arctic, where they felt
they could better control the whaling industry. Whales were hunted for oil and fuel, as well as for
whalebone, which among other things, was used for making women’s corsets. Inuit men and
women were hired to work on the whalers’ bases and also on the ships, and slowly abandoned their
traditional way of life.
5
Nowadays, our role in the erosion of Inuit tradition has changed, but it continues undebated, if in a
more modern way. To help solve the problems rampant in Inuit society, such as poverty and
unemployment, governments have encouraged the promotion of tourism in Inuit lands, unlikely
though this may seem. Dog-sledding adventures and whaling expeditions are now advertised online.
You, too, can experience life in the frozen north, learn the seventeen words for snow and live in an igloo.
6
Our influence has altered Inuit art, as well. Thanks to collectors’ relentless appetite for ivory, a ban
on the hunting of animals for their tusks has been necessary to preserve these creatures from
extinction, thus depriving the Inuit of materials for their weapons and crafts. Soapstone has taken its
place, as its softness makes it easy to carve. As such, it is especially suitable for mass production,
and today, soapstone carvings are being churned out at breakneck speed, often not even by Inuit
carvers, to meet the demand from collectors.
7
Certainly, it is too late to turn back the clock, but is it too late to learn from the past? The
industrialized nations are often depressingly slow learners, despite all their laudable technological
achievements. But our own irreverent age would undoubtedly benefit from the meaning and
guidance afforded by the traditional Inuit view of nature and the planet.
Missing paragraphs:
A. The practice of engaging the help of Shamans for hunting purposes testifies to the respect and
fear with which the natural world was viewed. A shaman was thought to have a special spirit,
one which was stronger and in closer contact with others. He was often called upon the
intervene in the hunt and persuade the prey to give itself to the hunters. Inuit legends also
illustrate feelings of awe for the natural world. Sedna, for example, was a drowning girl whose
severed fingers are transformed in the water into whales and seals.
B. Wood was scarce, so Inuit art was generally carved out of ivory, caribou antlers or local stone,
which had to be mined during the warmer months, sometimes at great distances from the
hunting base. The carvings had simple shapes and smooth lines which were reminiscent of the
flowing snowy landscape.
C. Needless to say, these spirits were respected by the Inuit, because they affected their daily
lives in so many ways. The sea, for example, could be bountiful, or it could withhold its gifts.
Animals could be hunted or they could evade the hunters. A whale’s spirit, if offended, could
direct its host well away from the whalers, or, having been shown the proper respect, it could
allow itself to be caught.
D. This attitude is something which could inform our own worldview and form the basis of a
more rational approach to our planet. Unfortunately, we appear to have too much confidence
in our supposed superiority to listen. Feelings like this are nothing new; indeed, we have been
contributing to the decline of the Inuit for quite some time.
E. There are those who will argue that these efforts have benefited the ailing Inuit culture, and
perhaps they are right to some extent. Tourism brings in money, helps raise awareness of Inuit
culture and the problems surrounding its survival, and selling art, however cheapened and
removed from tradition, does the same. We cannot change history, the argument goes, and at

Page 6 of 18
least these are ways to help revive and maintain those few crafts and skills which survive
among the Inuit.
F. Soon, fashions changed and fur was in demand, so the Inuit became trappers. They had
previously hunted big game, which required the combined efforts of the extended family unit,
but trapping foxes was a solitary pursuit, and involvement in this field further eroded their
traditional way of life. By the time the fur trade collapsed in the mid-twentieth century, the
Inuit lifestyle and economy had changed so drastically that it was impossible to revert to the
old way of life.
G. This is particularly true in the case of the native people of the northern polar regions of the
world, who could instruct us mightily if we let them. The Inuit, whose name means ‘living
people’, are believed to have migrated westward from Greenland about a thousand years ago,
merging with or possibly assimilating other, older peoples. Many aspects of their culture
reveal a noble and cohesive world view which shows a deep respect for all living creatures as
well as the natural environment, and acknowledges their interconnectedness.
H. Fortunately, the prohibitive cost and the arduous trek to the far north, which involves several
flights in single-engine aircraft and several hours on a snowmobile, have deterred all but the
most determined travellers from such northern fantasy trips. But it is only a matter of time
before insatiable adventure-seekers look to these regions as the last frontier to visit and
impress their friends with.
Your answers:
1. 2. 3. 4. 5. 6. 7.
Part 2. There are four passages marked A, B, C, and D. For questions 8-17, read the passages
and do the task that follows. Write your answers in the corresponding numbered boxes provided.
Dorothy Who?
The only British woman scientist to win the Nobel prize should be a household name in her own
country, says Georgina Ferry, but she is little known.
A. For the past four years, I have been subjecting friends and acquaintances to the Dorothy
Hodgkin test. It's very simple: when asked what I am working on, I tell them I am writing the
first biography of Dorothy Hodgkin. If their eyes light up, and they say things like 'Surely
there's one already!' they have passed.
Why should people in Britain know about Dorothy Hodgkin? The fact that she is the only
British woman scientist to have won a Nobel prize ought to be enough. Anyone who held the
same distinction in literature would be a household name. But Hodgkin, who died in 1994, was
a remarkable individual by any standards, as many-faceted as the crystals she studied. Her life
reflects some of the greatest upheavals of the 20th century: among them, the advancement of
women's education and the globalisation of science.
When I began my research, I set out to read some scientific biographies. One of Hodgkin's
friends recommended a new biography of Linus Pauling. Pauling was a close friend and
contemporary of Hodgkin, worked in the same branch of science and shared a commitment to
campaigning against nuclear weapons. I hurried to the main bookshop in the university town
where I live, only to discover that not a single biography of Pauling was on the shelves. I now
realise I was naive to be surprised that Pauling was not deemed sufficiently interesting to
British readers, even though he was the most influential chemist of the 20th century and a
winner of Nobel prizes for both chemistry and peace.
B. Even scientists themselves have doubted the value of the scientific biography. 'The lives of
scientists, considered as Lives, almost always make dull reading', wrote the late Peter
Medawar, another Nobel laureate, who laid most of the scientific groundwork that now makes
organ transplants possible.
If scientists propagate this negative view, it is hardly surprising if publishers and booksellers

Page 7 of 18
share it. Treating scientists differently from everybody else as biographical subjects is one of
the outstanding symptoms of the 'two cultures' mentality, the belief that there is an
unbridgeable divide of understanding between the arts and sciences, still prevalent in the
literary world. Few but the towering giants of science make it into the biography sections of
bookshops.
Of course it is nonsense to say scientists, as a group, lead less interesting lives than artists and
writers, or actors, or politicians. For some, the fastidiousness involved in maintaining scientific
credibility extends to any kind of media appearance. A leading geneticist once told me he was
happy to be interviewed about his work, but did not want to be quoted directly or
photographed, because he did not want to be perceived as ‘self-promoting’.
C. The avoidance of the personal conveys a false impression of the enterprise of science that
discourages young people from joining in, and fosters more public suspicion than it dispels.
Fortunately, gaps are appearing in the smokescreen. Contemporary scientists now regularly
appear in the public eye in contexts other than the straightforward scientific interview. For
instance, Professor Richard Dawkins presents prizes to winners of a TV quiz, and geneticist
Steve Jones advertises cars on television. No doubt these activities have raised eyebrows in
laboratories but they have done more to make scientists recognisable as people than any
number of academic papers.
The publishing world is also undergoing a transformation. Scientific biographies and
autobiographies, if they appeared at all, used to be rather scholarly but dull and overreverent.
The life which the scientist in question led outside work marriage, children, things most people
regard as fairly central to their existence - was often dismissed in a couple of paragraphs. That
changed with Richard Feynman's Surely You're Joking, Mr Feynman?, the hilarious and
affecting memoir of a man who also happened to be one of the century's greatest theoretical
physicists. More recently, even the greatest names in science, such as Isaac Newton, Charles
Darwin, Albert Einstein and Marie Curie have been allowed to appear with all their flaws
clearly visible. To the reader, it does not matter that Einstein's relationship with his family is
'irrelevant' to his General Theory of Relativity. The question of how creative genius copes with
emotional ups and downs, trivial practicalities, the social demands of ordinary life, is absorbing
in its own right.
D. Dorothy Hodgkin was devoted to her scientific work. Her most important successes were
solving the structure of penicillin and vitamin B12, which won her the Nobel prize for
chemistry in 1964, and of insulin, which her group solved in 1969. In each case she pushed the
technique into realms of complexity others deemed unreachable at the time.
But she also had three children to whom she was devoted and was married to a frequently
absent husband with a career as a historian. Her personal life is not strictly relevant to her work
as a scientist, but surely we can all learn from her capacity to unite the disparate threads of her
life into a coherent whole. There is much in her life of universal interest, but it would be
disloyal of me to imply that this does not include the science itself. Scientific inquiry was the
passion of Hodgkin's life, as it has to be for any successful scientist. How to communicate the
nature of this passion is the hardest task for the scientific biographer. Most readers are not
equipped with enough fundamental scientific concepts to grasp more complex ideas without a
lot of explanation. Understanding scientific ideas is not really any more difficult than reading
Shakespeare or learning a foreign language it just takes application. It is sad to think that
educated people, who would be embarrassed if they failed to recognise the name of some
distinguished literary or artistic figure, continue to live in happy ignorance of the rich heritage
represented by scientists such as Dorothy Hodgkin.

Which section mentions the following? Your answers:


 the continuing general scarcity of biographies of scientists 8. ________

Page 8 of 18
 certain parallels between the lives of two people 9. ________
 an attitude which is common to scientists and people working in the book trade 10. ________
 the lack of trust people sometimes have in scientists 11. ________
 someone whose scientific research went much further than others had 12. ________
believed possible
 biographies which include the less positive aspects of a scientist's life 13. ________
 the lessons to be taken from someone else's life 14. ________
 growing public interest in the everyday lives of brilliant people 15. ________
 the greatest difficulty in writing the biography of a scientist 16. ________
 someone who was modest about the interest of their own life to others 17. ________

Part 3. Read the following passage and choose the best answer (A, B, C or D) according to the
text. Write your answers (A, B, C or D) in the corresponding numbered boxes.
Language diversity has always been part of the national demographic landscape of the United
States. At the time of the first census in 1790, about 25% of the population spoke languages other
than English (Lepore, 2002). Thus, there was a diverse pool of native speakers of other languages at
the time of the founding of the republic. Today, nationwide, school districts have reported more
than 400 languages spoken by language-minority students classified as limited English proficient
(LEP) students (Kindler, 2002). Between 1991 and 2002, total K-12 student enrollment rose only
12%, whereas LEP student enrollment increased 95% during this same time period (National
Clearinghouse for English Language Acquisition, 2002b). This rapid increase and changing
demographics has intensified the long debate over the best way to educate language-minority
students.
Historically, many groups attempted to maintain their native languages even as they learned
English, and for a time, some were able to do so with relatively little resistance until a wave of
xenophobia swept the country during World War 1 (Kloss, 1977/1998). Other groups, Africans, and
Native Americans encountered repressive politics much earlier. During the 1960s, a more tolerant
policy climate emerged. However, for the past two decades there has been a steady undertow of
resistance to bilingualism and bilingual education. This article provides historical background and
analyzes contemporary trends in language-minority education within the context of the recent
national push for accountability, which typically takes the form of high-stakes testing.
The origins of persistent themes regarding the popular antagonisms toward bilingual education
and the prescribed panaceas of "English immersion" and high-stakes testing in English need to be
scrutinized. As background to the contemporary context, we briefly discuss the history of language
politics in the United States and the ideological underpinnings of the dominant monolingual English
ideology. We analyze the recent attacks on bilingual education for what this attack represents for
educational policy within a multilingual society such as the United States. We emphasize
multilingualism because most discussions of language policy are framed as if monolingualism were
part of our heritage from which we are now drifting. Framing the language policy issues in this way
masks both the historical and contemporary reality and positions non-English language diversity as
an abnormality that must be cured. Contrary to the steady flow of disinformation, we begin with the
premise that even as English has historically been the dominant language in the United States since
the colonial era, language diversity has always been a fact of life. Thus, efforts to deny that reality
represent a "malady of mind" (Blaut, 1993) that has resulted in either restrictionist or repressive
language policies for minorities.
As more states ponder imposing restrictions on languages of instruction other than English-as
California, Arizona, and Massachusetts have recently done-it is useful to highlight several questions
related to the history of language politics and language planning in the United States. Educational
language planning is frequently portrayed as an attempt to solve the language problems of the
Page 9 of 18
minority. Nevertheless, the historical record indicates that schools have generally failed to meet the
needs of language-minority students (Deschenes, Cuban, & Tyack, 2001) and that the endeavor to
plan language behavior by forcing a rapid shift to English has often been a source of language
problems that has resulted in the denial of language rights and hindered linguistic access to
educational, social, economic, and political benefits even as the promoters of English immersion
claim the opposite.
The dominance of English was established under the British during the colonial period, not by
official decree but through language status achievement, that is, through "the legitimization of a
government's decisions regarding acceptable language for those who are to carry out the political,
economic, and social affairs of the political process" (Heath, 1976, p.51). English achieved
dominance as a result of the political and socioeconomic trade between England and colonial
administrators, colonists, and traders. Other languages coexisted with English in the colonies with
notable exceptions. Enslaved Africans were prohibited from using their native tongues for fear that
it would facilitate resistance or rebellion. From the 1740s forward, southern colonies simultaneously
institutionalized "compulsory ignorance" laws that prohibited those enslaved from acquiring
English literacy for similar reasons. These restrictive slave codes were carried forward as the former
southern colonies became states of the newly United States and remained in force until the end of
the Civil War in 1865 (Weinberg, 1977/1995). Thus, the very first formal language policies were
restrictive with the explicit purpose of promoting social control.
18. What is the primary purpose of including the statistic from the 1790 census in the introductory
paragraph?
A. To explain how colonizing the US eradicated language diversity.
B. To show concrete evidence that language diversity in the US is not a new phenomenon.
C. To note that before that time, there was no measure of language diversity in the US.
D. To demonstrate that census data can be inaccurate.
19. The article compares two sets of statistics from the years 1991-2002, increases in K-12
enrollment and increases in LEP students, to highlight______.
A. that the two numbers, while often cited in research, are insignificant
B. that while many people with school-age children immigrated to the US during this time, an
equal amount left the country as well
C. that language diversity had no impact on US student enrollment during this time
D. that while the total amount of students enrolled in US schools may have grown slowly, the
amount of those students who were LEP increased dramatically
20. According to the second paragraph, many groups maintained their native languages without
resistance into the 20th century EXCEPT__________.
A. Native Americans and African Americans B. Irish Americans and African Americans
C. Mexican Americans and Native Americas D. Native Americans and Dutch Americans
21. Why is the word "undertow" emphasized in the second paragraph?
A. To explain how certain groups continued to carry their native languages with them despite the
opposition from those against language diversity.
B. To show the secretive and sneaky nature of those opposed to language diversity.
C. To call attention to the ebb and flow of language resistance during the 20th century,
experiencing periods of both rest and extremism.
D. To explain that, while many groups tried to maintain their native languages, many gave in to
social and political pressure to use only English.
22. What is the best way to describe the function of the third paragraph in this excerpt?
A. The paragraph provides its primary thesis as well an outline of the article's main points.
B. The paragraph is an unnecessary and irrelevant inclusion.
C. The paragraph serves to reveal the conclusions of the article before detailing the data.
D. The paragraph firmly establishes the article's stance against language diversity.
23. What is the best summary of why the phrase "multilingualism" is emphasized in the third
paragraph?

Page 10 of 18
A. Language repression stems from the US's unwillingness to recognize the languages of its
foreign allies.
B. Because language is constantly changing and often goes through multiple phases over time.
C. The authors firmly believe that speaking more than one language gives students a substantial
benefit in higher education.
D. Language policy discussions often assumes that the US has a monolinguistic history, which is
untrue and poses language diversity as threatening.
24. Phrases such as "prescribed panaceas" and "malady of the mind" are used in the third
paragraph to______.
A. defend the point that the US must standardize its language education or there will be severe
results
B. point out that language is as much a physical process as an intellectual one
C. illustrate how certain opponents of language diversity equate multilingual education with a
kind of national disease
D. demonstrate how the stress of learning multiple languages can make students ill
25. According to the fourth paragraph, all of the following are potential negatives of rapid English
immersion EXCEPT__________.
A. it can lead to a denial of language rights for particular groups
B. students become more familiar with conversational expressions and dialect
C. it can prevent access to certain benefits that are always available to fluent speakers
D. it can promote feelings of alienation among groups that are already in a minority status
26. The best alternate definition of "language status achievement" is __________.
A. when enough scholarly work has been produced in a language, it is officially recognized
B. those who are in power socially and economically determine the status of a language
C. languages fall into a hierarchy depending upon the numbers of populations that speak them
D. the position of a language in which no others may coexist with it
27. From the context of the final paragraph, what does "compulsory ignorance" mean?
A. Populations at the time were required only to obtain a certain low level of education.
B. Slave populations were compelled to only speak in their native languages and not learn
English.
C. That slaves were forcibly prevented from developing their native language skills out of fear
that they would gain power.
D. Slave owners would not punish slaves who did not wish to learn and speak only English.
Your answer:
18. 19. 20. 21. 22.
23. 24. 25. 26. 27.
Part 4. For questions 28 - 37, fill each of the following numbered blanks with ONE suitable word
and write your answers in the correspondent numbered boxes provided. (10 pts)
WHERE HAVE ALL THE GOOD CARTOONS GONE?
Childhood will never be the same again. Remember Saturday mornings spent lounging on
the sofa, hour (28)________ hour, watching your favourite cartoons? (29)________ there have been
a better reward for the long school week that had had to be endured? Bugs Bunny, Donald Duck,
Mickey Mouse brought virtually live into (30)________ living rooms. Back then, they were in
black and white, and back then, they were meant to amuse, to entertain.
It seems this has changed – and definitely (31)________ the worse. Now when you turn on
the television on a Saturday or Sunday morning, you do (32)________ at your own risk! Be
prepared to confront violence in all its animated glory: exploding bombs, falling buildings, blazing
weapons, and bad guy after bad guy. I don’t see (33)________ is funny about this warped vision of
our times and our society. Nor do I see what’s worth watching on these programmes with
(34)________ gruesome caricatures of good and evil. Who is responsible for children’s
programming these days?

Page 11 of 18
It cannot be good for today’s youth to be exposed (35)________ this type of entertainment.
(36)________ best, they are missing out on the humour, sensitivity and moral lessons that were to
be had from the cartoons of old. At worst, their childish brains are (37)________ filled with scenes
of non-stop violence and ideas that are morally corrupt. Childhood should be a time of innocence,
short-lived as it may be in these turbulent times in which we live. Perhaps we should bear this in
mind the next time we see our child glued to the TV on a Saturday morning.
Your answer:
Your answers:
28. 29. 30. 31. 32.
33. 34. 35. 36. 37.

Part 5. Read the text and do the tasks that follow.


The Farmers! Parade of history

A
History of Fanner trading company: In 1909 Robert Laidlaw establishes mail-order company
Laidlaw Leeds in Fort Street, Auckland. Then, Branch expansion: purchase of Green and Colebrook
chain store; further provincial stores in Auckland and Waikato to follow. Opening of first furniture
and boot factory. In 1920, Company now has 29 branches; Whangarei store purchased. Doors open
at Hobson Street for direct selling to public. The firm establishes London and New York buying
offices. With permission from the Harbour Board, the large FARMERS electric sign on the
Wyndham Street frontage is erected.
B
In 1935, if the merchandise has changed, the language of the catalogues hasn’t Robert Laidlaw, the
Scottish immigrant who established die century-old business, might have been scripting a modern-
day television commercial when he told his earliest customers: Satisfaction, or your money back. “It
was the first money back guarantee ever offered in New Zealand by any firm,” says Ian Hunter,
business historian. “And his mission statement was, potentially, only the second one ever found in
the world.” Laidlaw’s stated aims were simple to build the greatest business in New Zealand, to
simplify every transaction, to eliminate all delays, to only sell goods it would pay the customer to
buy.
C
This year, the company that began as a mail-order business and now employs 3500 staff across 58
stores turns 100. Its centenary will be celebrated with the release of a book and major community
fundraising projects, to be announced next week. Hunter, who is writing the centenary history, says
“coming to a Fanners store once a week was a part of the New Zealand way of life”. By 1960, one
in every 10 people had an account with die company. It was the place where teenage girls shopped
for their first bra, where newlyweds purchased their first dinner sets, where first pay cheques were
used to pay off hire purchase furniture, where Santa paraded every Christmas.
D
Gary Blumenthal’s mother shopped there, and so does he. The fondest memory for the Rotorua
resident? “We were on holiday in Auckland… I decided that upon the lookout tower on top of the
Farmers building would be a unique place to fit the ring on my new fiancee’s finger.” The
lovebirds, who had to wait for “an annoying youth” to leave the tower before they could enjoy their
engagement kiss, celebrate their 50th wedding anniversary in June.

Page 12 of 18
E
Farmers, says Hunter, has always had a heart. This, from a 1993 North & South interview with a
former board chairman, Rawdon Busfield: “One day I was in the Hobson Street shop and I saw a
woman with two small children. They were clean and tidily dressed, but poor, you could tell. That
week we had a special on a big bar of chocolate for one shilling. I heard the woman say to her boy,
‘no, your penny won’t buy that’. He wasn’t wearing shoes. So I went up to the boy said,’ Son, have
you got your penny? ‘He handed it to me. It was hot he’d had it in his hand for hours. I took the
penny and gave him the chocolate.”
F
Farmers was once the home of genteel tearooms, children’s playgrounds and an annual sale of
celebration for birthday of Hector the Parrot (the store mascot died, aged 131, in the 1970s his
stuffed remains still occupy pride of place at the company’s head office). You could buy houses
from Farmers. Its saddle factory supplied the armed forces, and its upright grand overstrung pianos
offered “the acme of value” according to those early catalogues hand-drawn by Robert Laidlaw
himself. Walk through a Farmers store today and get hit by bright lights and big brands. Its Albany
branch houses 16 international cosmetics companies. It buys from approximately 500 suppliers, and
about 30% of those are locally owned.
G
“Eight, 10 years ago,” says current chief executive Rod McDermott, “lots of brands wouldn’t
partner with us. The stores were quite distressed. We were first price point focused, we weren’t
fashion focused. “Remove the rose-tinted nostalgia, and Farmers is, quite simply, a business, doing
business in hard times. Dancing with the Stars presenter Candy Lane launches a clothing line? “We
put a trial on, and we thought it was really lovely, but the uptake wasn’t what we thought it would
be. It’s got to be what the customer wants,” says McDermott.
H
He acknowledges retailers suffer in a recession: “We’re celebrating 100 years because we can and
because we should.” Farmers almost didn’t pull through one economic crisis. By the mid 1980s, it
had stores across the country. It had acquired the South Island’s Calder Mackay chain of stores and
bought out Haywrights. Then, with sales topping $375 million, it was taken over by Chase
Corporation. Lincoln Laidlaw, now aged 88, and the son of the company’s founder, remembers the
dark days following the stock market crash and the collapse of Chase. “I think, once, Farmers was
like a big family and all of the people who worked for it felt they were building something which
would ultimately be to their benefit and to the benefit of New Zealand… then the business was
being divided up and so that kind of family situation was dispelled and it hasn’t been recovered.”
For a turbulent few years, the stores were controlled, first by a consortium of Australian banks and
later Deka, the Maori Development Corporation and Foodland Associated Ltd. In 2003, it went
back to “family” ownership, with the purchase by the James Pascoe Group, owned by David and
Anne Norman the latter being the great-granddaughter of James Pascoe, whose first business
interest was jewellery.
I
“Sheer power of the brand,” says McDermott, “pulled Farmers through and now we’re becoming
the brand it used to be again.” Farmers was the company that, during World War n, topped up the
wages of any staff member disadvantaged by overseas service. Robert Laidlaw a committed
Christian who came to his faith at a 1902 evangelistic service in Dunedin concluded his original
mission statement with the words, “all at it, always at it, wins success”. Next week, 58 Farmers
stores across the country will announce the local charities they will raise funds for in their centenary
celebration everything from guide dog services to hospices to volunteer fire brigades will benefit.

Page 13 of 18
Every dollar raised by the community will be matched by the company. “It’s like the rebirth of an
icon,” says McDermott.
Questions 38-42
The reading Passage has seven paragraphs A-I
Which paragraph contains the following information?
Write the correct letter A-I, in boxes provided.
38. Generosity offered in an occasion.
39. Innovation of offer made by the head of company.
40. Fashion was not its strong point.
41. A romantic event on the roof of farmers.
42. Farmers were sold to a private owned company.
Your answer
38. 39. 40. 41. 42.
Questions 43-47
Complete the following summary of the paragraphs of Reading Passage
Using NO MORE THAN TWO WORDS from the Reading Passage for each answer.

6. Farmers was first founded as a 43 ______in Auckland by Mr Laidlaw.


7. Farmers developed fast and bought one 44 ______then.
8. During oversea expansion, Farmers set up 45 ______in cities such as London.
9. Farmers held a 46 ______once a year for the well-known parrot.
10. In the opinion of Lincoln Laidlaw, Farmers is like a 47 ______for employees, not just for
themselves but for the whole country.
Your answer
43. 44. 45. 46. 47.

Questions 48-50
Use the information in the passage to match the people (listed A-C) with opinions or deeds below.
Write the appropriate letters A-C in boxes provided.
NB You may use any letter more than once.
A Lincoln Laidlaw
B Rod McDermott
C Ian Hunter
48. Product became worse as wrong aspect focused.
49. An unprecedented statement made by Farmers in New Zealand.

Page 14 of 18
50. Character of the company was changed.
Your answer
48. 49. 50.

IV. WRITING (60 points)


Part 1. Read the following extract and use your own words to summarize it. Your summary
should be between 100 and 120 words.
In his nearly 30 years studying vaccines, Paul Goepfert, M.D., director of the Alabama
Vaccine Research Clinic at the University of Alabama at Birmingham, has never seen any vaccine
as effective as the three COVID vaccines — from Pfizer, Moderna, and Johnson & Johnson —
currently available in the United States. “A 90 percent decrease in risk of infections, and 94 percent
effectiveness against hospitalization for the Pfizer and Moderna vaccines is fantastic,” he said. But
what makes vaccine experts such as Goepfert confident that COVID vaccines are safe in the long term?
There are several reasons, actually. Vaccines, given in one- or two-shot doses, are very
different from medicines that people take every day, potentially for years. And decades of vaccine
history — plus data from more than a billion people who have received COVID vaccines starting
last December — provide powerful proof that there is little chance that any new dangers will
emerge from COVID vaccines. Goepfert says we already know enough to be confident the COVID
vaccines are safe, starting with the way vaccines work and continuing through strong evidence from
vaccine history and the even stronger evidence from the responses of people who have received
COVID-19 vaccines worldwide over the past six months. “Many people worry that these vaccines
were ‘rushed’ into use and still do not have full FDA approval — they are currently being
distributed under Emergency Use Authorizations,” Goepfert said. “But because we have had so
many people vaccinated, we actually have far more safety data than we have had for any other
vaccine, and these COVID vaccines have an incredible safety track record. There should be
confidence in that.”
https://www.uab.edu/news/health/item
...............................................................................................................................................................
...............................................................................................................................................................
...............................................................................................................................................................
...............................................................................................................................................................
...............................................................................................................................................................
...............................................................................................................................................................
...............................................................................................................................................................
...............................................................................................................................................................
...............................................................................................................................................................
...............................................................................................................................................................
...............................................................................................................................................................
...............................................................................................................................................................
................................................................................................................................................................
Part 2. Describing a graph
The graph below shows the gold medals team Great Britain has won in 4 sports during 6 Olympics.
Summarize the information by selecting and reporting the main features, and make comparisons
where relevant. You should write about 150 words.
Page 15 of 18
...............................................................................................................................................................
...............................................................................................................................................................
...............................................................................................................................................................
...............................................................................................................................................................
...............................................................................................................................................................
...............................................................................................................................................................
...............................................................................................................................................................
...............................................................................................................................................................
...............................................................................................................................................................
...............................................................................................................................................................
...............................................................................................................................................................
...............................................................................................................................................................
...............................................................................................................................................................
...............................................................................................................................................................
...............................................................................................................................................................
...............................................................................................................................................................
...............................................................................................................................................................
...............................................................................................................................................................
...............................................................................................................................................................
...............................................................................................................................................................
Part 3. Write an essay of 350 words on the following topic:
History should be a core subject in high school like Maths, Literature, and English.
Do you agree or disagree with the statement? Discuss this statement and give your opinion.
...............................................................................................................................................................

Page 16 of 18
...............................................................................................................................................................
...............................................................................................................................................................
...............................................................................................................................................................
...............................................................................................................................................................
...............................................................................................................................................................
...............................................................................................................................................................
...............................................................................................................................................................
...............................................................................................................................................................
...............................................................................................................................................................
...............................................................................................................................................................
...............................................................................................................................................................
...............................................................................................................................................................
...............................................................................................................................................................
...............................................................................................................................................................
...............................................................................................................................................................
...............................................................................................................................................................
...............................................................................................................................................................
...............................................................................................................................................................
...............................................................................................................................................................
...............................................................................................................................................................
...............................................................................................................................................................
...............................................................................................................................................................
...............................................................................................................................................................
...............................................................................................................................................................
...............................................................................................................................................................
...............................................................................................................................................................
...............................................................................................................................................................
...............................................................................................................................................................
...............................................................................................................................................................
...............................................................................................................................................................
...............................................................................................................................................................
...............................................................................................................................................................
...............................................................................................................................................................
...............................................................................................................................................................
...............................................................................................................................................................
...............................................................................................................................................................

Page 17 of 18
...............................................................................................................................................................
...............................................................................................................................................................
...............................................................................................................................................................
...............................................................................................................................................................
...............................................................................................................................................................
...............................................................................................................................................................
...............................................................................................................................................................
...............................................................................................................................................................
...............................................................................................................................................................
...............................................................................................................................................................
...............................................................................................................................................................
...............................................................................................................................................................
...............................................................................................................................................................
...............................................................................................................................................................
...............................................................................................................................................................
...............................................................................................................................................................
...............................................................................................................................................................
...............................................................................................................................................................
...............................................................................................................................................................
...............................................................................................................................................................
...............................................................................................................................................................
...............................................................................................................................................................
...............................................................................................................................................................
...............................................................................................................................................................

(You may write overleaf if you need more space)


- THE END -

Page 18 of 18
SỞ GD&ĐT HẢI DƯƠNG ĐỀ THI CHỌN HỌC SINH GIỎI VÙNG
TRƯỜNG THPT CHUYÊN DUYÊN HẢI - ĐỒNG BẰNG BẮC BỘ
NGUYỄN TRÃI NĂM HỌC 2021-2022
Môn: Tiếng Anh – Lớp 11
(Đề thi đề xuất) (Thời gian: 180 phút – không kể thời gian giao đề)

SECTION A. LISTENING (50 points)

PART 1: You will hear two nutritionists, Fay Wells and George Fisher, discussing
methods od food production. For questions 1-5, choose the answer (A, B, C or D) which
fits best according to what you hear.
1. Looking at reports on the subject of GM foods, Fay feels ____________.
A. pleased to read that the problem of food shortages is being addressed
B. surprised that the fears of the public are not allayed by them
C. frustrated by contradictory conclusions
D. critical of the scientists' methodology
2. What does George suggest about organic foods?
A. Consumers remain surprisingly poorly informed about them.
B. People need to check out the claims made about them.
C. They need to be made more attractive to meat-eaters.
D. They may become more widely affordable in future.
3. What is George's opinion of 'vertical farming'?
A. It could provide a realistic alternative to existing methods.
B. It's a highly impractical scheme dreamt up by architects.
C. It's unlikely to go much beyond the experimental stage.
D. It has the potential to reduce consumption of energy.
4. George and Fay agree that the use of nanotechnology in food production will
____________.
A. reduce the need for dietary supplements
B. simplify the process of food-labelling
C. complicate things for the consumer
D. introduce potential health risks
5. In Fay's view, returning to self-sufficiency is only an option for people who
____________.
A. have no need to get a return on their investment
B. are willing to accept a high level of regulation
C. reject the values of a consumer society
D. already have sufficient set-up funds
Your answers:
1. 2. 3. 4. 5.

PART 2: Read the statements, listen and decide whether they are true (T) or false (F).
1. Emily interprets the statistical information she quotes as reflecting the particular appeal of
travelling exhibitions.
2. Scott points out that an impressive museum building can distract attention from the
exhibits.
3. When asked about tour groups, Emily suggests that people should feel prejudiced against
them.
4. How most people had few expectations before arriving surprised Scott when he was doing
research into why people visited a museum.
5. Emily and Scott agree that virtual museums can’t replicate the real-life experience.
Your answers:
1. 2. 3. 4. 5.

PART 3. Answer the questions (no more than 3 words).


21. What did Klara originally want to do her project on?
22. Where can Klara read about housing prices every day?
23. What did Klara’s friend suggest including information on?
24. What does Klara need to use in the middle part?
25. How long does Klara have to finish the project?
Your answers:
1. 2. 3. 4. 5.

PART 4. For questions 1 – 10, listen to a piece of news about Australia lifts one of last
COVID-19 Public Health Mandates and complete the summary using NO MORE
THAN THREE WORDS OR NUMBER for each gap. Write your answers in the
corresponding boxes provided.
Face masks are one of the most 1._________________ of the pandemic in Australia,
where some of the world’s toughest 2. _________________ were imposed like
3._________________ and vaccination orders for key workers.
A number of the restrictions have been lifted in the states of New South Wales and
Queensland, along with the 4. _________________ on Friday and in Western Australia on
Saturday. However, they haven’t lifted the 5. _________________ in South Australia or
Victoria.
Catherine Bennett, the chair in 6. _________________ at Deakin University, wanted
the mask mandates to end. She said wherever people are, whether it is at local shops, your
workplace or the airport, they have to be conscious of their exposure.
Adrian Esterman, the chair in 7. ________________ and epidemiology at the
University of South Australia, thought that forcing people to wear masks on flights in
Australia was a bad move and territory governments are lying to everyone that life can go
8._________________.
Some statistics: Australia has had 9. _________________ coronavirus infections
with 9,000 deaths. 95% of the population over 16 years of age have received two doses of
a COVID-19 vaccine. About 70% have had a 10._________________ .
Your answers:
1. 2. 3. 4. 5.
6. 7. 8. 9. 10.

SECTION B. LEXICO- GRAMMAR (30 points)


Part 1. Choose the best option A, B, C, or D to complete the following sentences and write
your answers in the corresponding numbered boxes. (20 points)
1. The opposition party won the election on a _____ of economic reform.
A. lectern B. podium C. platform D. dogma
2. I’m a ______ at this game-I only learned to play it a few weeks ago.
A. novelty B. novice C. learner D. newcomer
3. The accountant _____ the company out of millions of dollars before he was caught.
A. swindled B. spun C. dwindled D. saddled
4. The judge’s sentence is _____ and can not be changed.
A. incongruous B. invariable C. irrevocable D. irreconcilable
5. It usually takes me 40 minutes to get into town, but today ______ because there was a lot
of traffic.
A. Took twice that B. I took twice more
C. it took twice that D. it had taken twice than that
6. _____ we will lose clients due to the current financial climate, the company is still
expected to reach its target for the quarter.
A. Even though it appears likely that
B. Though it likely appears that
C. In spite of the likelihood
D. Nevertheless likely it appears that
7. Customers are tempted to break _____ with so many alluring products available online.
A. the ice B. the mold C. the cycle D. the bank
8. the smell of freshly baked bread ____ fond memories of her childhood days.
A. evicted B. evoked C. evolved D. evaded
9. One of the defendants _____ and was on the run until his arrest.
A. jumped bail B. made bail C. stood bail D. posted bail
10. The business started off small, but now has become a large media and entertainment
_______.
A. Metropolis B. conglomerate C. coalition D. alliance
11. I spent the whole night in curlers ____ get my new hairdo wet in the rain the net day.
A. only to B. to only C. only as to D. for only to
12. the college is planning to launch an online learning program, but the date of
implementation is uncertain.
A. since then B. by then C. beyond that D. after a while
13. “What did the thief look like?” “He was ____ his chin.”
A. young with a scar in
B. quite youngly with a scar on
C. quite young with a scar at
D. quite young with a scar on
14. The political candidate always tries to ____ any difficult questions when talking to the
press.
A. butter up B. fend off C. fawn over D. drive back
15. Violation of the school’s code of conduct could result in a weeklong ____ for students.
A. expulsion B. discharge C. eviction D. suspension
16. Sandra had _____ to snakes and spiders.
A. a conversion B. a distortion C. an aversion D. an aggression
17. A great diplomat and a firm political leader is able to speak _____ about government
legislation.
A. over the hump B. out of breath
C. out of fire D. off the cuff
18. Although some banking institutions allow their customers to ____ payment, it is not
advised.
A. defer B. deter C. deflect D. diverge
19. After an official investigation the defendant was _____ and set free.
A. validated B. authenticated C. exonerated D. rehabilitated
20. That country’s diplomatic _______ was the largest group at the conference.
A. contingent B. battalion C. franchise D. chapter

Your answers:
1. 2. 3. 4. 5.
6. 7. 8. 9. 10.
11. 12. 13. 14. 15.
16. 17. 18. 19. 20.

Part 2. Write the correct form of the words given in the brackets.( 10 points)
1. The resultant disruptions in trade and agriculture, and the _______ of the countryside, left
long-term scars. (POPULATE)
2. Put otherwise, in a context in which there are unequal power relations, a _______
homogenisation seems likely, if not inevitable. (DELETE)
3. The intense magenta color indicates intense positive staining in this _______ aggregate.
(CELL)
4. Today the press is free and _______ by censorship, and the private electronic media
flourish. (CUMBERSOME)
5. Coffee beans and tea leaves are _______ with this solvent. (CAFFEINE)
6. This is a procedure which should become standard in future work by _______
organizations. (GOVERN)
7. When it was over, she began life _______ in France. (NEW)
8. All these former offenses were swiftly ______. ( CRIME)
9. I stand not upon the _______ words of a challenge. (DEFINE)
10. Economists and some psychologists _______ employ such tasks in their studies.
(EXCLUDE)

Your answers:
1. 2. 3. 4. 5.
6. 7. 8. 9. 10.

C. READING (60 points)


Part 1: In the passage below, seven paragraphs have been removed. Read the passage
and choose from the paragraphs A-H the one which fits each gap. There is ONE extra
paragraph which you do not need to use. ( 7 points)
Is There A Limit To Our Intelligence?
Increasing IQ scores suggests that future generations will make us seem like dimwits by
Tom Govern
Almost thirty years ago James R. Flynn, a researcher at the University of Otago in New
Zealand, discovered a phenomenon that social scientists still struggle to explain: IQ scores
have been increasing steadily since the beginning of the 20th century. Nearly 30 years of
follow-up studies have confirmed the statistical reality of the global uptick, now known as
the Flynn effect. And scores are still climbing.
1.
The Flynn effect means that children will, on average, score just under 10 points higher on
IQ tests than their parents did. By the end of this century our descendants will have nearly
a 30-point advantage over us if the Flynn effect continues. But can it continue or is there
some natural limit to the Flynn effect and to human intelligence?
2.
Most of the IQ gains come from just two subtests devoted to abstract reasoning. One deals
with “similarities” and poses questions such as “How are an apple and an orange alike?” A
low-scoring answer would be “They’re both edible.” A higher-scoring response would be
“They’re both fruit,” an answer that transcends simple physical qualities. The other subtest
consists of a series of geometric patterns that are related in some abstract way, and the test
taker must correctly identify the relation among the patterns.
3.
“If you don’t classify abstractions, if you’re not used to using logic, you can’t really
master the modern world,” Flynn says. “Alexander Luria, a Soviet psychologist, did some
wonderful interviews with peasants in rural Russia in the 1920s. He would say to them,
‘Where there is always snow, bears are always white. There is always snow at the North
Pole. What colour are the bears there?’ They would say they had never seen anything but
brown bears. They didn’t think of a hypothetical question as meaningful.”
4.
A naive interpretation of the Flynn effect quickly leads to some strange conclusions.
Extrapolating the effect back in time, for example, would suggest that the average person
in Great Britain in 1900 would have had an IQ of around 70 by 1990 standards. “That
would mean that the average Brit was borderline mentally retarded and wouldn’t have
been able to follow the rules of cricket,” says David Hambrick, a cognitive psychologist at
Michigan State University. “And of course, that’s absurd.”
5.
So, what will the future bring? Will IQ scores keep going up? One thing we can be sure of
is that the world around us will continue to change, largely because of our own actions.
6.
Therefore, our minds and culture are locked in a similar feedback loop. We are creating a
world where information takes forms and moves with speeds unimaginable just a few
decades ago. Every gain in technology demands minds capable of accommodating the
change, and the changed mind reshapes the world even more. The Flynn effect is unlikely
to end during this century, presaging a future world where you and I would be considered
woefully premodern and literal.
7.
Perhaps we should not be so surprised by the existence of something like the Flynn effect.
Its absence would be more startling; it would mean we were no longer responding to the
world we are creating. If we are lucky, perhaps we will keep building a world that will
make us smarter and smarter—one where our descendants will contemplate our simplicity.
A The villagers were not stupid. Their world just required different skills. “I think the
most fascinating aspect of this isn’t that we do so much better on IQ tests,” Flynn says.
“It’s the new light it sheds on what I call the history of the mind in the 20th century.”

B Of course, our minds are changing in ways other than those which can be measured by
IQ tests. “People are getting faster.” Hambrick says. “Previously, it had been thought that
200 milliseconds is about the fastest that people can respond. But if you ask people who
have done this sort of research, they’re having to discard more trials. We text, we play
video games, we do a lot more things that require really fast responses.

C Almost as soon as researchers recognized the Flynn effect, they saw that the ascending
IQ scores were the result almost entirely of improved performances on specific parts of
the most widely used intelligence tests. It would seem more natural to expect
improvements in crystallized intelligence—the kind of knowledge picked up in school.
This is not happening, though. The scores in the sections that measure skills in arithmetic
and vocabulary levels have remained largely constant over time.

D A paradox of the Flynn effect is that these tools were designed to be completely
nonverbal and culture-free measurements of what psychologists call fluid intelligence—an
innate capacity to solve unfamiliar problems. Yet the Flynn effect clearly shows that
something in the environment is having a marked influence on the supposedly culture-free
components of intelligence in populations worldwide. Detailed studies of generational
differences in performance on intelligence tests suspect that our enhanced ability to think
abstractly may be linked to a new flexibility in the way we perceive objects in the world.

E Flynn likes to use a technological analogy to describe the long-term interaction between
mind and culture. “The speeds of automobiles in 1900 were absurdly slow because the
roads were so lousy,” he says. “You would have shaken yourself to pieces.” But roads and
cars co-evolved. When roads improved, cars did, too, and improved roads prompted
engineers to design even faster cars.

F “To my amazement, in the 21st century the increase is still continuing,” says Flynn,
whose most recent book on the subject—Are We Getting Smarter?— was published in
2012. “The latest data show the gains in America holding at the old rate of three-tenths of
a point a year.”

G Consequently, we may not be more intelligent than our forebears, but there is no doubt
our minds have changed. Flynn believes the change began with the industrial revolution,
which engendered mass education, smaller families, and a society in which technical and
managerial jobs replaced agricultural ones. Education, in turn, became the driver for still
more innovation and social change, setting up an ongoing positive feedback loop between
our minds and a technology-based culture that does not seem likely to end any time soon.

H Formal education, though, cannot entirely explain what is going on. Some researchers
had assumed that most of the IQ increases seen over the 20th century might have been
driven by gains at the left end of the intelligence bell curve among those with the lowest
scores, an outcome that would likely be a consequence of better educational opportunities.
However, a close examination of 20 years of data revealed that the scores of the top 5 per
cent of students were going up in perfect lockstep with the Flynn effect.

Your answers:
1. 2. 3. 4. 5. 6. 7.

Part 2. Read the following text and fill in the blank with ONE suitable word. Write your
answers in corresponding numbered boxes. (15 points)
Writing is a medium of (1) ______ that most people use daily, and perhaps even take for
granted. Yet, without a shadow of a (2) ______, it has been one of the key elements in the
development of society. Writing systems and the invention of books have meant that
knowledge can be passed (3) ______ reality through the generations.
Much evidence suggests that (4) ______ was in the Middle East that systems of writing
were initially developed, and these were born out of practical necessity. As individuals
grouped together in cities, this (5) ______ rise to more frequent trading of goods, but
keeping a running order of these goods was an (6) ______ battle, especially since they were
often communally stored.
(7) ______ the face of these difficulties, a better method of controlling and accounting (8)
______ stock was developed, and this was the very first system of writing. It initially took
the form of pictures drawn in clay tablets to represent a particular commodity, with lines
corresponding to the number of items a person had. In time, these drawings gave (9) ______
to symbols, which were more efficient for the writer, and then to more detailed forms of
written record. This is when writing evolved to more than just lists of nouns, and started to
(10) ______ the shape of the fully formed sentences we see today.
Your answers:
1. 2. 3. 4. 5.
6. 7. 8. 9. 10.

Part 3. Read the following passage and choose the best answer to each of the following
questions. Write your answers in corresponding numbered boxes. (10 points)
While he spoke my very conscience and reason turned traitors against me, and charged me
with crime in resisting him. They spoke almost as loud as Feeling: and that clamored wildly.
“Oh, comply!” it said. “Think of his misery; think of his danger—look at his state when left
alone; remember his headlong nature; consider the recklessness following on despair—
soothe him; save him; love him; tell him you love him and will be his. Who in the world
cares for you or who will be injured by what you do?” L6
Still indomitable was the reply—“I care for myself. The more solitary, the more friendless,
the more unsustained I am, the more I will respect myself. I will keep the law given by God;
sanctioned by man. I will hold to the principles received by me when I was sane, and not
mad—as I am now. Laws and principles are not for the times when there is no temptation:
they are for such moments as this, when body and soul rise in mutiny against their rigor;
stringent are they; inviolate they shall be. If at my individual convenience I might break
them, what would be their worth? They have a worth—so I have always believed; and if I
cannot believe it now, it is because I am insane—quite insane: with my veins running fire,
and my heart beating faster than I can count its throbs. Preconceived opinions, foregone
determinations, are all I have at this hour to stand by: there I plant my foot.” L16
I did. Mr. Rochester, reading my countenance, saw I had done so. His fury was wrought to
the highest: he must yield to it for a moment, whatever followed; he crossed the floor and
seized my arm and grasped my waist. He seemed to devour me with his flaming glance:
physically, I felt, at the moment, powerless as stubble exposed to the draught and glow of a
furnace: mentally, I still possessed my soul, and with it the certainty of ultimate safety. The
soul, fortunately, has an interpreter—often an unconscious, but still a truthful interpreter—
in the eye. My eye rose to his; and while I looked in his fierce face I gave an involuntary
sigh; his gripe was painful, and my over-taxed strength almost exhausted. L24
“Never,” said he, as he ground his teeth, “never was anything at once so frail and so
indomitable. A mere reed she feels in my hand!” And he shook me with the force of his
hold. “I could bend her with my finger and thumb: and what good would it do if I bent, if I
uptore, if I crushed her? Consider that eye: consider the resolute, wild, free thing looking
out of it, defying me, with more than courage—with a stern triumph. Whatever I do with its
cage, I cannot get at it—the savage, beautiful creature! If I tear, if I rend the slight prison,
my outrage will only let the captive loose. Conqueror I might be of the house; but the inmate
would escape to heaven before I could call myself possessor of its clay dwelling-place. And
it is you, spirit—with will and energy, and virtue and purity— that I want: not alone your
brittle frame. Of yourself you could come with soft flight and nestle against my heart, if you
would: seized against your will, you will elude the grasp like an essence—you will vanish
ere I inhale your fragrance. Oh! Come, Jane, come!” L36
As he said this, he released me from his clutch, and only looked at me. The look was far
worse to resist than the frantic strain: only an idiot, however, would have succumbed now.
I had dared and baffled his fury; I must elude his sorrow: I retired to the door.
“You are going, Jane?”
“I am going, sir.”
“You are leaving me?”
“Yes.”
“You will not come? You will not be my comforter, my rescuer? My deep love, my wild
woe, my frantic prayer, are all nothing to you?”
What unutterable pathos was in his voice! How hard it was to reiterate firmly, “I am going.”

1.Jane’s attitude toward Mr. Rochester is best characterized as


A. sympathetic. B. uncaring. C. despising. D. reckless.
2.Based on the information in the passage, it can be inferred that Jane refuses Rochester’s
advances because
A. she does not love him as much as he loves her.
B. it would violate her personal ideals.
C. he thinks that she is weak and frail.
D. she wishes to cause him injury.
3. Which choice provides the best evidence for the answer to the previous question?
A. Lines 1-2 (“While . . . him”)
B. Lines 8-10 (“I will . . . now”)
C. Lines 21-23 (“The soul . . . eye”)
D. Lines 29-30 (“Whatever . . . creature”)
4. In context, the phrase “I am insane—quite insane” in line 14 refers chiefly to
A. a severe mental illness that Jane suffers from.
B. a mental state brought on by God’s law.
C. a feeling that currently urges Jane to reject Rochester.
D. a reduction of judgment due to emotion.
5.As used in line 17, “wrought” most nearly means
A. hammered. B. made. C. excited. D. wrung.
6. The fourth paragraph (lines 25-36) provides a contrast between
A. Jane’s body and her will.
B. Rochester’s love and anger toward Jane.
C. a bird and its cage.
D. Jane’s purity and impurity
7. The inmate Rochester mentions in line 31 refers to
A. a criminal locked away in jail.
B. Rochester trapped in his emotions.
C. Jane stuck in the traditions of her time.
D. the possible behavior of Jane’s spirit.
8. Which choice provides the best evidence for the answer to the previous question?
A. Lines 23-24 (“My eye . . . exhausted”)
B. Lines 27-28 (“I could . . . her”)
C. Lines 32-34 (“And it . . . frame”)
D. Lines 37-38 (“The look . . . now”)
9. As used in line 38, “worse” most nearly means
A. less desirable. B. more difficult.
C. of lower quality. D. unskillful.
10. Based on the information in the final paragraph, it can be reasonably inferred that Jane
values
A. her emotions over her reason.
B. freedom over social convention.
C. her principles over her feelings.
D. true love above all else.
Your answers:
1. 2. 3. 4. 5.
6. 7. 8. 9. 10.

Part 4: Read the passage and do the tasks below.


Sleepy Students Perform Worse
A. Staying up an hour or two past bedtime makes it far harder for kids to learn, say
scientists who deprived youngsters of sleep and tested whether their teachers could tell the
difference. They could. If parents want their children to thrive academically, “Getting
them to sleep on time is as important as getting them to school on time," said psychologist
Gahan Fallone, who conducted the research at Brown Medical School.
B. The study, unveiled Thursday at an American Medical Association (AMA) science
writers meeting, was conducted on healthy children who had no evidence of sleep- or
learning-related disorders. Difficulty paying attention was among the problems the sleepy
youngsters faced - raising the question of whether sleep deprivation could prove even
worse for people with attention deficit hyperactivity disorder, or ADHD. Fallone now is
studying that question, and suspects that sleep problems “could hit children with ADHD
as a double whammy”.
C. Sleep experts have long warned that Americans of all ages do not get enough shuteye.
Sleep is important for health, bringing a range of benefits that, as Shakespeare put it,
“knits up the ravelled sleave of care”. Not getting enough is linked to a host of problems,
from car crashes as drivers doze off to crippled memory and inhibited creativity. Exactly
how much sleep correlates with school performance is hard to prove. So, Brown
researchers set out to test whether teachers could detect problems with attention and
learning when children stayed up late - even if the teachers had no idea how much sleep
their students actually got.
D. They recruited seventy-four 6- to 12-year-olds from Rhode Island and southern
Massachusetts for the three-week study. For one week, the youngsters went to bed and
woke up at their usual times. They already were fairly good sleepers, getting nine to 9.5
hours of sleep a night. Another week, they were assigned to spend no fewer than ten hours
in bed a night. The other week, they were kept up later than usual: First -and second-
graders were in bed no more than eight hours and the older children no more than 6.5
hours. In addition to parents’ reports, the youngsters wore motiondetecting wrist monitors
to ensure compliance.
E. Teachers were not told how much the children slept or which week they stayed up late,
but rated the students on a variety of performance measures each week. The teachers
reported significantly more academic problems during the week of sleep deprivation, the
study, which will be published in the journal Sleep in December, concluded. Students who
got eight hours of sleep or less a night were more forgetful, had the most trouble learning
new lessons, and had the most problems paying attention, reported Fallone, now at the
Forest Institute of Professional Psychology.
F. Sleep has long been a concern of educators. Potter-Burns Elementary School sends
notes to parents reminding them to make sure students get enough sleep prior to the
school’s yearly achievement testing. Another school considers it important enough to
include in the school’s monthly newsletters. Definitely, there is an impact on students’
performance if they come to school tired. However, the findings may change physician
practice, said Dr. Regina Benjamin, a family physician in Bayou La Batre, who reviewed
the data at the Thursday’s AMA meeting. “I don't ask about sleep” when evaluating
academically struggling students, she noted. “I’m going to start.”
G. So how much sleep do kids need? Recommended amounts range from about ten to
eleven hours a night for young elementary students to 8.5 hours for teens. Fallone insists
that his own second-grader get ten hours a night, even when it meant dropping soccer -
season that practice did not start until 7:30 — too late for her to fit in dinner and time to
wind down before she needed to be snoozing. “It’s tough,” he acknowledged, but “parents
must believe in the importance of sleep."
Questions 1-4
The text has 7 paragraphs (A - G).
Which paragraph contains each of the following pieces of information?
1 Traffic accidents are sometimes caused by lack of sleep.
2 The number of children included in the study
3 How two schools are trying to deal with the problem
4 How the effect of having less sleep was measured
Questions 5-8
Complete the following sentences using NO MORE THAN TWO WORDS from the
text for each gap.
5.Fallone is now studying the sleep patterns of children with _______________
6.The researchers used _______________that show movement to check that children went
to bed at the right time.
7.Students with less sleep had problems with memory, remembering new material, and
_______________
8.Fallone admitted that it was _______________ for children to get enough sleep.
Questions 9-13
Do the following statements agree with the information given in Reading Passage . In
boxes 9 - 13 on your answer sheet, write
TRUE if the statement agrees with the information
FALSE if the statement contradicts the information
NOT GIVEN If there is no information on this
9. The results of the study were first distributed to principals of American schools,
10. Some of the children in the study had previously shown signs of sleeping problems.
11.The study could influence how doctors deal with children’s health problems.
12.Fallone does not let his daughter play soccer.
13.Staying up later is acceptable if the child is doing
Your answers:
1. 2. 3. 4. 5.
6. 7. 8. 9. 10.
11. 12. 13.

Part 5. You are going to read extracts from an article about snowflakes. For questions 1-
10, choose from the sections (A-E). The extracts may be chosen more than once.
In which extract ...
... is a point of contention amongst scientists over the 1……………………………
effects of something highlighted?
... does the writer give an insight into their personal 2……………………………
outlook on life?
... is the difficulty in proving something likened to 3…………………………….
searching for an everyday object?
... does the writer examine the different ways likeness can 4……………………………
be interpreted?
... does the writer hint at the inconveniences snowflakes 5……………………………
can cause in everyday life?
... is the composition of young snow crystals differentiated 6……………………………
in some detail?
... are the range of possible forms flakes can take defined 7……………………………
as almost never-ending?
... does the writer first explain that two developed 8…………………………..
snowflakes can rarely be the same?
... does the writer suggest the closer something is 9…………………….…..
inspected, the less likely an outcome is?
... does the writer suggest that simplification can have a 10…………………………
positive impact on the world?

A
It is often claimed that no two snowflakes are alike, but what exactly is the veracity of this
statement?
Well, although you wouldn’t think it to glance at them, snow crystals are rather intricate.
For that reason, the answer is by no means clear-cut. For instance, scientists remain unsure
as to how temperature and humidity affect growth. Indeed, moving somewhat tangentially
for a moment, nor are they yet certain of the wider climactic effect flakes have. For
example, they know that clouds of snow crystals reflect sunlight during the day, producing
a cooling affect; although at night they sort of blanket the planet, absorbing the heat it
gives off, doing the reverse. So whether such clouds contribute to global warming or not is
up for debate on account of these competing effects.

B
As for snow crystals themselves, they undergo various stages of formation before they
become fully developed snowflakes. In the developmental stages, they are more simple
structures, then they later branch out and become complex. To start with, they resemble
fairly plain and uniform six-sided prisms that are hard to distinguish from one another.
Such underdeveloped crystals do often fall to the ground prematurely as precipitation. In
this case, the probability of close likeness amongst different ones is quite high in relative
terms. So, hypothetically, it’s quite possible to find two more or less the same, but,
in practice, this would be like looking for a needle in a haystack – two, actually, so good
luck trying to prove it.

C
However, snowfall is typically comprised of crystals at a more advanced stage of
development – true snowflakes, if you will – and here the odds change considerably with
the likelihood of very close resemblance dramatically reduced. This is because the ways in
which fully developed crystals can arrange themselves are almost infinite. Once crystals
have branched out to form large flakes, then, the chances of finding identical twins are,
therefore, extremely remote.

D
Another problem with this question is how you define ‘alike’. After all, to the naked eye,
most flakes look more or less indistinguishable, irrespective of size or shape. Indeed, even
under a microscope, more simple crystal formations are strikingly similar to one another,
though the unique characteristics of fully formed snowflakes will be revealed. However,
an understanding of the science of physics confirms the extreme rarity of identical twins
even amongst superficially similar flakes. In other words, at a molecular level, likeness is
a near impossibility, so the more closely we examine a flake and the more strictly we
define the notion of likeness, the less probable it becomes to ever identify two crystals
which are truly alike.

E
It is, in a way, somewhat reassuring, though, that something as seemingly simple as a
snowflake which is in actuality incredibly complex, can still be uniformly beautiful in
another purer, more innocent sense. For, once the flakes have made landfall and begun to
amass, snow is, to a degree, just snow, and it takes on that kind of magical, fairy-tale
quality that only it can evoke in so many people, but particularly the young, who have less
need to worry about the logistical implications of it amassing in ever greater quantities,
and, indeed, who usually welcome the closure of facilities, particularly academic ones,
that is normally commensurate with such accumulations. For it is the way of the universe
as a whole, is it not? Order springs from chaos, beauty is born from the most unlikely,
disordered and chance set of circumstances. Indeed, as a self-proclaimed glass-half-full
person, I like to think that we, human beings, are not all that dissimilar to snowflakes,
actually. After all, each one of us is, on some level, utterly unique, and yet, remove all the
complexities of life and the over-analysis, and, on another, we are all precisely the same;
hopeful, flawed, loving, caring, jealous and imperfect; perfectly so. The sooner we
understand that, the better for both our species and the wider world we inhabit, snow-
covered or otherwise.
Your answers:
1. 2. 3. 4. 5.
6. 7. 8. 9. 10.

IV. WRITING (60 points)


Part 1. Read the following extract and use your own words to summarise it. Your
summary should be between 100 and 120 words long.
Across cultures, wisdom has been considered one of the most revered human qualities.
Although the truly wise may seem few and far between, empirical research examining
wisdom suggests that it isn’t an exceptional trait possessed by a small handful of bearded
philosophers after all – in fact, the latest studies suggest that most of us have the ability to
make wise decisions, given the right context.
‘It appears that experiential, situational, and cultural factors are even more powerful
in shaping wisdom than previously imagined,’ says Associate Professor Igor Grossmann of
the University of Waterloo in Ontario, Canada. ‘Recent empirical findings from cognitive,
developmental, social, and personality psychology cumulatively suggest that people’s
ability to reason wisely varies dramatically across experiential and situational contexts.
Understanding the role of such contextual factors offers unique insights into understanding
wisdom in daily life, as well as how it can be enhanced and taught.’
Coming up with a definition of wisdom is challenging, but Grossmann and his
colleagues have identified four key characteristics as part of a framework of wise reasoning.
One is intellectual humility or recognition of the limits of our own knowledge, and another
is appreciation of perspectives wider than the issue at hand. Sensitivity to the possibility of
change in social relations is also key, along with compromise or integration of different
attitudes and beliefs.
Grossmann and his colleagues have also found that one of the most reliable ways to
support wisdom in our own day-to-day decisions is to look at scenarios from a third-party
perspective, as though giving advice to a friend. Research suggests that when adopting a
first-person view point, we focus on ‘the focal features of the environment’ and when we
adopt a third-person, ‘observer’ viewpoint we reason more broadly and focus more on
interpersonal and moral ideals such as justice and impartiality. Looking at problems from
this more expansive viewpoint appears to foster cognitive processes related to wise
decisions.

Part 2: The graph below provides information on the average cost of three kinds of
cereals in England and Wales over an eight-month period in 2014, while the table
shows the quantities of cereals sold during the same period.
Summarise the information by selecting and reporting the main features, and make
comparisons where relevant.
Write at least 150 words
AVERAGE PRICE PER TONNE IN ENGLAND AND WALES
AVERAGE QUANTITIES OF CEREALS SOLD (TONNES) IN ENGLAND AND
WALES

January February March April May June July August


WHEAT 76,800 85,500 163,500 56,300 30,800 50,700 66,300 131,700
BARLEY 21,500 18,900 32,400 15,700 11,300 11,500 38,100 45,100
OATS 2350 1820 3580 1700 960 600 945 2310
……………..……………..……………..……………..……………..……………..……
……………..……………..……………..……………..……………..……………..……
……………..……………..……………..……………..……………..……………..……
……………..……………..……………..……………..……………..……………..……
……………..……………..……………..……………..……………..……………..……
……………..……………..……………..……………..……………..……………..……
……………..……………..……………..……………..……………..……………..……
……………..……………..……………..……………..……………..……………..……
……………..……………..……………..……………..……………..……………..……
……………..……………..……………..……………..……………..……………..……
……………..……………..……………..……………..……………..……………..……
……………..……………..……………..……………..……………..……………..……
……………..……………..……………..……………..……………..……………..……
……………..……………..……………..……………..……………..……………..……
……………..……………..……………..……………..……………..……………..……
……………..……………..……………..……………..……………..……………..……
……………..……………..……………..……………..……………..……………..……
……………..……………..……………..……………..……………..……………..……
……………..……………..……………..……………..……………..……………..……
……………..……………..……………..……………..……………..……………..……
……………..……………..……………..……………..……………..……………..……
……………..……………..……………..……………..……………..……………..……
……………..……………..……………..……………..……………..……………..……

Part 3: Write an essay of about 350 words to express your opinion on the following
issue (30 points)
“In modern society, some people argue that schools become unnecessary as children
can study at home via the Internet. Do you agree or disagree?”
Give reasons for your answer and include any relevant examples from your own
knowledge or experience. Write at least 350 words.
……………..……………..……………..……………..……………..……………..……
……………..……………..……………..……………..……………..……………..……
……………..……………..……………..……………..……………..……………..……
……………..……………..……………..……………..……………..……………..……
……………..……………..……………..……………..……………..……………..……
……………..……………..……………..……………..……………..……………..……
……………..……………..……………..……………..……………..……………..……
……………..……………..……………..……………..……………..……………..……
……………..……………..……………..……………..……………..……………..……
……………..……………..……………..……………..……………..……………..……
……………..……………..……………..……………..……………..……………..……
……………..……………..……………..……………..……………..……………..……
……………..……………..……………..……………..……………..……………..……
……………..……………..……………..……………..……………..……………..……
……………..……………..……………..……………..……………..……………..……
……………..……………..……………..……………..……………..……………..……
……………..……………..……………..……………..……………..……………..……
……………..……………..……………..……………..……………..……………..……
……………..……………..……………..……………..……………..……………..……
……………..……………..……………..……………..……………..……………..……
……………..……………..……………..……………..……………..……………..……
……………..……………..……………..……………..……………..……………..……
……………..……………..……………..……………..……………..……………..……

THE END
Người ra đề

Nguyễn Thị Bích Vân (0904114546)


ĐỀ THI ĐỀ XUẤT
MÔN TIẾNG ANH 11
TRƯỜNG THPT CHUYÊN NGUYỄN TẤT THÀNH – YÊN BÁI

LISTENING (5 points)
Part 1. Listen to a talk about the Ant IPO delay and decide whether these statements are True (T),
False (F) or Not Given (NG).
1. Shanghai authorities disrupted Ant’s IPO at short notice on the grounds of its fragile business plans.
2. Ant Group’s IPO would have been unprecedented in the fintech industry.
3. Interference from China is the main reason accounting for the questionable status of Hong Kong as
a major financial centre.
4. Winston Ma forecasts that the fintech industry will not cease to enjoy a period of phenomenal growth
in the wilderness.
5. Recent regulations from the Chinese government will shift the role of Ant Group as a fintech
company.
Your answers
1. 2. 3. 4. 5.

Part 2. Listen to a talk about Neptune and answer the questions. Write NO MORE THAN THREE
WORDS taken from the recording for each answer in the corresponding numbered boxes provided.
1. What is the distance between Neptune and the Sun?
______________________________________________________________________
2. What is Neptune’s core made up of besides water ice?
______________________________________________________________________
3. What gives Neptune blue color?
______________________________________________________________________
4. What can strong winds recorded on Neptune do?
______________________________________________________________________
5. What is the name of the spacecraft that has visited Neptune?
______________________________________________________________________

Part 3. Listen to part of an interview with an artist about the subject of art and choose the answer (A,
B, C or D) which fits best according to what you hear. Write your answers in the corresponding
numbered boxes provided.
1. How does Brendan feel about his work?
A. He prefers negative criticism to no reaction at all.
B. His intention is to provoke critics with his art.
C. He believes his art can only be viewed subjectively.
D. His definition of art is at odds with general opinion.
2. According to Brendan, standing on a chair
A. is an example of what art should be.
B. would be art if it was intended to be.
C. is an example of mediocre art.
D. would not be considered art by most people.
3. Brendan says that some successful artists
A. are not very good at drawing.
B. have not been able to create original art.
C. lack the ability to express ideas.
D. use excessively old-fashioned techniques.
4. Brendan doesn't think that
A. throwing paint at a canvas would constitute art.
B. it is difficult to come up with original ideas.
1
C. critics are open-minded enough.
D. modern art has nowhere left to go.
5. The invention of the camera
A. allowed more people to indulge their passion for taking portraits.
B. enabled photographers to imitate life more closely.
C. meant that artists were no longer restricted to copying from life.
D. changed the way society viewed the role of the artist.
Your answers
1. 2. 3. 4. 5.

Part 4. Listen to a talk about sleep and supply the blanks with the missing information. Write NO
MORE THAN THREE WORDS taken from the recording for each answer in the space provided.
SLEEP AND MENTAL HEALTH
1. The culprit behind our ______________________ relationship to sleep originates from our ancient
understanding of the subject.
2. Parents of small children have a disposition to be ______________________ in routine negotiations.
3. Every reversal becomes a drama, every disappointment turns into a catastrophe and every excitement
shifts into ______________________.
4. An innovatively ______________________ approach to bedtimes when growing up can be
considered an expression of independence and individuality.
5. There are various ways of expressing our perceptions about lives, ranging from positive narratives to
appalling tales of complete ignorance and ______________________.
6. When exhaustion sets in, we tend to think in a ______________________ way.
7. When we lie in bed, we think that we bear a resemblance to a ______________________, for instance,
a rabbit or a squirrel.
8. Given the harsh grown-up life, we need to be free to ______________________ this.
9. It is inferred from curled squirrel position that mental problems cannot be universally handled by
______________________.
10. It is not until we have treated ourselves with a ______________________ or a long night’s sleep
that we understand the reasons to live.

LEXICO - GRAMMAR (3 points)


Part 1. Choose the best option to complete the following sentences.
1. Sandra astounded all the spectators by winning the match ______ down.
A. heads B. hands C. hearts D. feet
2. When his manager went on a business trip, Mark stepped into the ______ and chaired the meeting.
A. hole B. breach C. pool D. crack
3. It is public knowledge that new magazines often use free gifts or other _____ to get people to buy
them.
A. gimmicks B. snares C. plots D. scams
4. We should never have quarreled like that. Let’s bury the ______and forget all about it.
A. axe B. argument C. hatchet D. subject
5. It is often difficult for a householder to ______ squatters and regain possession of his or her
property.
A. eliminate B. withdraw C. evict D. vacate
6. I'm not a serious investor, but I like to ______ in the stock market.
A. splash B. splatter C. paddle D. dabble
7. The teacher said 'Well done' and patted me on the head. I can't stand people who treat me so
______.
A. pompously B. maternally C. snobbishly D. patronizingly
8. The investigation was instigated ______the Prime Minister.
A. on the part of B. consequence of C. subsequent to D. at the behest of
2
9. Teachers have the authority to discipline pupils by ______ of their position as teachers.
A. view B. virtue C. means D. way
10. The consultant called in by the firm brought a ______ of experience to bear on the problem.
A. wealth B. realm C. bank D. hoard
11. The thick fog ______ out any possibility of our plane taking off before morning.
A. ruled B. struck C. stamped D. crossed
12. The new curriculum has been designed to ______ students learning by combining theory with hands-
on practice.
A. endow B. optimize C. sharpen D. estimate
13. When I was younger, I wanted to be an air pilot but I soon went ____ the idea when I realised I hated
flying.
A. out B. off C. up D. with
14. People can make themselves walk on nails or through fire. It’s a question of mind over ______.
A. body B. material C. matter D. facts
15. We had a ______ sale to sell off all the stuff we found in the attic
A. parking B. garage C. station D. garden
16. Because of the dominance of retail chain-stores, most shopping centers show the same bland
_______ and no imagination.
A. similarity B. likeness C. equality D. uniformity
17. She wears the most _______ color combinations you could ever imagine.
A. hiding B. hideout C. hidebound D. hideous
18. It would help _______ me, if you could go to the Post Office for me.
A. totally B. absolutely C. enormously D. largely
19. The plastic surgery must have cost the ________, but there’s no denying she looks younger.
A. world B. planet C. universe D. earth
20. A few of the older campers were sent home after a week as they were ________.
A. lenient B. erratic C. unruly D. indulgent
Your answers:
1. 2. 3. 4. 5.
6. 7. 8. 9. 10.
11. 12. 13. 14. 15.
16. 17. 18. 19. 20.

Part 2. Give the correct form of the words in brackets.


The upper layers of Earth's oceans have cooled (1. SIGNIFY) _______ over the past two years,
even though the planet as a whole is warming up. While this may just be part of the natural (2.VARY)
______ of oceans, climatologists are still confounded by the massive unaccountable loss of heat.
Scientists have been (3. INCREASE)______ concerned by rising sea temperatures over the last 50 years
but these new (4.FIND)______ tell a different story.
Generally speaking, the (5.ABSORB)______ of heat by the oceans reduces atmospheric
warming. Now (6. MEASURE) ______ taken by the National Oceanic and Atmospheric Administration
have put a wrinkle in the trend. The researchers used data from 3000 floating buoys which monitor the
oceans (7. WORLD) ______. They found that the oceans dropped in temperature by an
(8.BELIEVE)______ 0.02 degrees centigrade between 2011 and 2013. Now, that may not seem like
much, but trying to account for the missing energy is proving to be enormously (9. PROBLEM) ______.
It is possible that (10. VOCANO) ______ eruptions are one main cause of the phenomenon, but no firm
answers have yet been provided.
Your answers:
1. 2. 3. 4. 5.
6. 7. 8. 9. 10.

3
READING (6 points)
Part 1. Fill in each blank with ONE word
Anyone who has (1) ______ been to a yoga or meditation class will know the enormous benefits
of something as simple and natural as breathing. Inhale slowly and steadily, and you can relax your
entire body. Stop and focus on the flow of (2) ______ breath you take in and out, and you can quieten
and focus your mind. In (3)______ positive breathing will help you feel calmer, bring down your blood
pressure and increase your mental alertness and energy levels.
Yet, breathing is so instinctive that most of the time we're hardly even aware of it. Of course,
that's (4)______ why it's easy to develop bad habits and why many of us don't do it as well as we
(5)______. But with a little practice, (6)_____ it right can bring instant health benefits. These include
feeling more relaxed and being more mentally alert. You may also find that there's an improvement in
(7)______ physical symptoms, such as bloating and stomach pains, dizziness, headaches, pins and
needles and low energy.
Learning to breathe correctly can dramatically improve your wellbeing and (8) ______ of life.
Most people think that poor breathing means that you don't get enough oxygen, but it's (9)______ the
carbon dioxide you're missing out (10)______
Your answers:
1. 2. 3. 4. 5.
6. 7. 8. 9. 10.

Part 2. Read passage and choose the answer A, B, C or D that fits best according to the text.
It has become conventional wisdom that spending too much time playing video games has a detrimental
effect on children’s studies and their social development. However, some educationalists are now
questioning this theory and are using video games as effective educational tools thus bridging the gap
between recreational and educational activities.

Due to the sophisticated nature of today’s games, teachers are able to justify the inclusion of video and
online games for many pedagogical reasons. There may, for example, be sociological, psychological,
and ethical implications built into the gameplay. Harvey Edwards, who teaches IT classes in London,
was one such educator who decided to use video games in his lessons. To do this, he chose Minecraft,
an online game in which players create and develop imaginary worlds. He was somewhat uneasy about
attempting such an unconventional approach, not because of some students’ unfamiliarity with the game
but rather due to them not being able to make sense of what he was trying to do with it. He worried that
it might interfere with his learners’ focus, but he couldn’t have been more surprised by the results.
Minecraft is an example of a ‘sandbox game’, in which gamers roam around and change a virtual world
at will. Instead of having to pass through numbered levels to reach certain places, there’s full access
from start to finish. The original version can be adapted to control which characters and content are left
in. Each student can then be allocated tasks – such as house-building, locating items or problem-solving
– which they must complete within the game. Elements of more general skills can be subtly incorporated
into the lessons, such as online politeness and safety, teamwork and resolving differences. Edwards feels
that presenting such lessons in the context of a game students probably already know and enjoy enables
him to connect with them at greater depth, and in more motivational ways.
Bolstered by his success, Edwards introduced his approach to another school nearby. He recalls that the
first couple of sessions didn’t live up to his expectations. Those who had played Minecraft before were
keen for others to adopt their own style of play. Unsurprisingly, this assortment of styles and opinions
as to how the game should proceed were far from harmonious. However, the sessions rapidly
transformed into something more cohesive, with the learners driving the change. With minimal teacher
input, they set about choosing leaders and established several teams, each with its own clearly-defined
role. These teams, now party to clear common goals, willingly cooperated to ensure that their newborn
world flourished, even when faced with the toughest of challenges.
4
‘Human’ inhabitants in a Minecraft ‘society’ are very primitive and wander around the imaginary world,
waiting for guidance from players. [A] This dynamic bears a resemblance to traditional education, an
observation highlighted by Martina Williams, one of the leaders of the group. [B] ‘Through the game,
we were no longer passive learners in the classroom, being told what and how to learn, but active
participants in our own society. [C] The leaders, meanwhile, had a vision for their virtual world as a
whole, encouraging everyone to play their part in achieving the group’s goals. [D] Through creating
their own characters and using these to build their own ‘world’, students will have gained some
experiential understanding of societal structure and how communities work.
But not everyone is convinced by video games’ potential academic value. While many progressive
commentators cite extensive evidence to maintain that video games encourage collaboration and build
problem-solving skills, more traditional factions continue to insist they are a distraction that do not merit
inclusion in any curriculum. Even less evangelical cynics, who may grudgingly acknowledge games
have some educational benefit, assert that this is only the case in the hands of creative educators.
However, the accusation most often levelled at video games is that they detract from the social aspect
of the classroom, particularly taking part in discussions. Dr Helen Conway, an educational researcher,
argues that video games can be used to promote social activities. ‘Students become animated talking
about the game and how to improve their gameplaying and problem-solving skills,’ she says. ‘I find it
strange, this image that many people have,’ Conway says. ‘Children are often totally detached from
their peers when undertaking more traditional activities, like reading books, but we never suggest that
books are harmful because they’re a solitary experience.
1. The first time Edwards used a game in his classes, he was
A. convinced that learners would realise why he wanted them to play it.
B. convinced that learners would see the reasons for playing it.
C. anxious that he had chosen the wrong one for learners to play.
D. sure that his reasons for getting learners to play it were valid.
2. The writer suggests that Minecraft is a good choice of educational game because
A. any number of learners can use it simultaneously.
B. teachers can remove any inappropriate material.
C. gamers can create educative tasks whilst playing it.
D. players can develop their skills in a step-by-step way.
3. Which of the following words in the fourth paragraph is used to convey a feeling of approval?
A. keen B. harmonious C. driving D. newborn
4. In the fifth paragraph, the writer draws a comparison between a Minecraft ‘society’ and
A. relationships within the group as they played.
B. the way in which countries organise themselves.
C. typical students in a school environment.
D. how leadership operates in different situations.
5. In the sixth paragraph, the writer feels that critics of video games in education
A. are unwilling to admit that using them in class has benefits.
B. make accurate observations about teachers who use them.
C. use flawed research to support their objections to using them.
D. acknowledge the drawbacks of more traditional teaching methods.
6. The words ‘this image’ in the sixth paragraph refer to
A. people who criticise gaming in education.
B. students discussing a game in a group.
C. a group of students reading individually.
D. a solitary player absorbed in a game.
5
7. Where does this sentence belong to in the fifth paragraph?
Each group member had ideas as to how their function should develop.
A. [A] B. [B] C. [C] D. [D]
8. The word ‘subtly’ in the third paragraph is closest in meaning to __________.
A. intricately B. ingeniously C. ingenuously D. haphazardly
9. The word ‘grudgingly’ in the sixth paragraph is closest in meaning to __________.
A. gleefully B. vivaciously C. genially D. reluctantly
10. Which of the following best describes the author's attitude towards the application of gaming into
education?
A. supportive B. neutral C. cynical D. satirical
Your answers:
1. 2. 3. 4. 5.
6. 7. 8. 9. 10.

Part 3. Read the following passage and do the tasks that follow.
The history of aspirin is a product of a rollercoaster ride through time, of accidental discoveries, intuitive
reasoning and intense corporate rivalry.
A. In the opening pages of Aspirin: The Remarkable Story of a Wonder
Drug, Diarmuid Jeffreys describes this little white pill as ‘one of the most amazing creations in
medical history, a drug so astonishingly versatile that it can relieve headache, ease your aching
limbs, lower your temperature and treat some of the deadliest human diseases’.
B. Its properties have been known for thousands of years. Ancient Egyptian physicians used
extracts from the willow tree as an analgesic, or pain killer. Centuries later the Greek physician
Hippocrates recommended the bark of the willow tree as a remedy for the pains of childbirth and
as a fever reducer. But it wasn't until the eighteenth and nineteenth centuries that salicylates the
chemical found in the willow tree became the subject of serious scientific investigation. The race
was on to identify the active ingredient and to replicate it synthetically. At the end of the
nineteenth century a German company, Friedrich Bayer & Co. succeeded in creating a relatively
safe and very effective chemical compound, acetylsalicylic acid, which was renamed aspirin.
C. The late nineteenth century was a fertile period for experimentation, partly because of the hunger
among scientists to answer some of the great scientific questions, but also because those
questions were within their means to answer. One scientist in a laboratory with some chemicals
and a test tube could make significant breakthroughs whereas today, in order to map the human
genome for instance, one needs ‘an army of researchers, a bank of computers and millions and
millions of dollars’.
D. But an understanding of the nature of science and scientific inquiry is not enough on its own to
explain how society innovates. In the nineteenth century, scientific advance was closely linked
to the industrial revolution. This was a period when people frequently had the means, motive
and determination to take an idea and turn it into reality. In the case of aspirin that happened
piecemeal - a series of minor, often unrelated advances, fertilised by the century’s broader
economic, medical and scientific developments, that led to one big final breakthrough.
E. The link between big money and pharmaceutical innovation is also a significant one. Aspirin’s
continued shelf life was ensured because for the first 70 years of its life, huge amounts of money
were put into promoting it as an ordinary everyday analgesic. In the 1970s other analgesics, such
as ibuprofen and paracetamol, were entering the market, and the pharmaceutical companies then
focused on publicising these new drugs. But just at the same time, discoveries were made

6
regarding the beneficial role of aspirin in preventing heart attacks, strokes and other afflictions.
Had it not been for these findings, this pharmaceutical marvel may well have disappeared.
F. So the relationship between big money and drugs is an odd one. Commercial markets are
necessary for developing new products and ensuring that they remain around long enough for
scientists to carry out research on them. But the commercial markets are just as likely to kill off'
certain products when something more attractive comes along. In the case of aspirin, a potential
‘wonder drug* was around for over 70 years without anybody investigating the way in which it
achieved its effects, because they were making more than enough money out of it as it was. If
ibuprofen or paracetamol had entered the market just a decade earlier, aspirin might then not be
here today. It would be just another forgotten drug that people hadn't bothered to explore.
G. None of the recent discoveries of aspirin's benefits were made by the big pharmaceutical
companies; they were made by scientists working in the public sector. 'The reason for that is
very simple and straightforward,' Jeffreys says in his book. 'Drug companies will only pursue
research that is going to deliver financial benefits. There's no profit in aspirin any more. It is
incredibly inexpensive with tiny profit margins and it has no patent any more, so anyone can
produce it.' In fact, there's almost a disincentive for drug companies to further boost the drug, he
argues, as it could possibly put them out of business by stopping them from selling their more
expensive brands.
H. So what is the solution to a lack of commercial interest in further exploring the therapeutic
benefits of aspirin? More public money going into clinical trials, says Jeffreys. ‘If I were the
Department of Health. I would say “this is a very inexpensive drug. There may be a lot of other
things we could do with it." We should put a lot more money into trying to find out.'
I. Jeffreys' book which not only tells the tale of a 'wonder drug' but also explores the nature of
innovation and the role of big business, public money and regulation reminds us why such
research is so important.
Questions 1-6
The Reading Passage has nine paragraphs A-I.
Choose the correct heading for each paragraph (A-F) from the list of headings below.
Write the correct number (i-x) in boxes 1-6.
List of headings:
i. The most powerful analgesic
ii. The profit potential of aspirin
iii. Saved from oblivion by drug companies
iv. Recognition of an important medicinal property
v. A double-edged sword
vi. An unstructured pattern of development
vii. Major pharmaceutical companies
viii. A wonder drug
ix. Roots of the scientific advancements in the 19th century
x. The discovery of new medical applications

7
1. Paragraph A
2. Paragraph B
3. Paragraph C
4. Paragraph D
5. Paragraph E
6. Paragraph F

Questions 7-13
Do the following statements agree with the views of the writer?
YES if the statement agrees with the views of the writer

NO if the statement contradicts the views of the writer

NOT GIVEN if it is impossible to say what the writer thinks about this
7. The discovery of aspirin’s full medicinal properties was a very unusual one.
8. The 19th century saw significant changes in the way in which scientific experiments were carried out.
9. For nineteenth-century scientists, small-scale research was far from enough to make important
discoveries.
10. The creation of a market for aspirin as a painkiller was achieved through commercial advertising
campaigns.
11. In the 1970s sales of new analgesic drugs overtook sales of aspirin.
12. Between 1900 and 1970, there was little research into aspirin because commercial companies
thought it had been adequately exploited.
13. Jeffrey suggests that there should be state support for further research into the possible applications
of aspirin.
Your answers:
1. 2. 3. 4. 5.
6. 7. 8. 9. 10.
11. 12. 13.

Part 4. You are going to read a magazine article. Seven paragraphs have been removed from the
article. Choose from the paragraphs A - H the one which fits each gap (24 - 30). There is one extra
paragraph which you do not need to use.
Mobile Misgivings
It's getting hard to be anonymous. To do anything, you have to prove who you are. Want to buy
something or draw some cash? That's a wodge of credit cards to lug around, and a plethora of four-digit
PINs to remember. Even before stepping out of the front door, you've got to find your driving licence
or rail pass, perhaps even your passport.

1.

Inside every digital mobile phone is a SIM card. SIM stands for Subscriber Information Module, and
the chip embedded in the SIM card is what makes the mobile yours. For now, the SIM just identifies
you to the phone system, and maybe holds details of your favourite phone numbers. In future it could

8
identify you to everyone who needs to know who you are and would enable you to carry out transactions
which require a form of identification.

2.

The Finnish government is looking at using SIM's in place of a national identity card - and eventually a
passport. Under this plan, the SIM wilt become a person's legal proof of identity. And there's no reason
why it couldn't unlock your health records, social security details and other personal information. One
click and a hospital would know exactly who it's dealing with.

3.

People can loge or mislay their phones, and they are a tempting target for thieves, who can easily dispose
of them on the black market. That's bad enough when there's only a large phone bill at stake. When your
phone becomes the key to your identity, secrets and cash, you'll want so make sure it stays safely locked
up, even if only the gadget itself falls into the wrong Lands. "Having something that contains all this
information would be extremely rash," says Roger Needham, managing director of Microsoft's British
research laboratory in Cambridge. "People will simply find it unacceptable."

4.

The beauty of this system is that the identifier would act as one half of what's called a public key
encryption system. The identifier, kept safe inside the phone, acts as a key, known to no one else. To
read a message locked with this private key requires a second, public key, which can be freely
distributed.

5.

An increasing number of countries are passing laws to give private keys the same legal force as
signatures. This has unleashed a flood of encryption systems, and the problem now is to get governments
and companies to agree on a standard. "It needs to be simple, secure and transparent," says Mica
Nierninen, head of mobile commerce company More Magic Software, in Helsinki. "We have the maths
to show that it is secure. The only problem now is making it global."

6.

The private key is protected by a PIN, and the card will shut itself off if wrong numbers are keyed in
three times. To switch it back on, the owner must take it to a police station with another form of ID. If
a card is stolen, the police will cancel it permanently. Either way, information on the card stays safe.

7.

Pearson thinks consumers, too, will learn to trust a chip with their identity, not least because it will make
life so much easier. A private key will do away with hard-to-remember log-in codes and passwords for
websites, as well as all those credit cards and PINS. "People already give up their privacy quite happily
just to get access to a website," he says. "As long as they get something out of it."

A. But for these dreams to become reality, there'll have to be a revolution in public attitudes. People will
have to let go of their apprehensions about e-commerce and learn to trust their mobiles; "Cultivating
that trust is a very difficult thing to do and takes it lot of time," says Ian Pearson, resident futurologist
at British Telecom.
B. Your credit history could be accessed by your bank manager with your agreement, which would
negate the need for you to visit the bank. The manager could communicate with you through your mobile

9
phone and either give you advice over the phone or invite you to the bank for a face-to-face consultation.
This has already been piloted in Britain and has received a positive reception.
C. The solution, according to experts in the field, is to share precious information on secure servers
accessible via a WAP connection on the web. The SIM would only store a personal identifier - a long
string of digits that would unlock the servers and give access to the information they hold. To use the
identifier, the phone's owner would have to punch in a PIN.
D. "Even in its embryonic form, FINEID gives people a secure way to access sensitive information,"
says Vatka. "And when you get it in a mobile phone you're not even tied to a terminal," he says. Many
believe that identity theft will be inevitable no matter how careful safeguards are. But since it is already
taking place and this system is more secure, businesses will probably be keen to adopt it.
E. In a few years, this plastic and paper baggage could be history. A single chip hidden in your cellphone
will be all you need - a little treasure that holds your complete identity. But beware! Lose your phone,
and your identity and money go with it. The big question is whether people will be willing to trust so
much to a sliver of silicon.
F. You might use this set-up to send a request to a bank using its public key to see the details of your
account, which it would decrypt using its private key. The bank would then send you the requested
information encrypted with your public key, which only your private key could decrypt. Thus both
messages would be secure.
G. To pay for a meal, say, you will use the phone to transfer money through the phone network to a
restaurant's computer. There will be no payment slip to sign because your SIM will do it for you.
Likewise, when you board a plane you won't have to wait in line for a boarding pass and seat number.
H. "The Finnish government has taken the initiative with a national standard that companies can use
free of charge," says Vesa Vatka of the Finnish Population Register Centre in Helsinki. "At the moment
this system - called FINEID - uses a smart card and a card reader attached to a computer, but the plan is
to integrate to a SIM," says Vatka.
Your answers:
1. 2. 3. 4. 5.
6. 7.

Part 5. The passage below consists of four paragraphs marked A-D. Read the passage and do the
task that follows. Write your answers (A-D) in the corresponding numbered boxes provided.
Studying the law
A. Ordinary people regularly encounter law in a variety of circumstances. Freely-negotiated commercial
contracts may bind them to act in particular ways. By becoming members of a sports club or a trade
union they agree to comply with a set of rules. Sometimes these forms of law will use the courts to
enforce their arrangements. In other cases privately-instituted adjudication bodies are established, a third
party being appointed to decide whether an agreement or rule has been broken or not. These kinds of
arrangements may seem very different from the normal idea of law, especially if law is thought of mainly
in terms of the criminal law. However, it is possible to see law simply as a way of regulating our actions,
of deciding what can be done and what cannot be done. Most laws are not about something spectacular
but, rather, about the details of ordinary life. Every time a purchase is made, a contract is made. Both
parties make promises about what they will do; one to hand over the goods, one to pay the price. In this
and other ways, everybody is involved in law every day of their lives.
B. Legal rules can be divided up in many different ways. The rules show differences in purpose, in
origin and form, in the consequences when they are breached, and in matters of procedure, remedies

10
and enforcement. One of the most fundamental divisions in law is the division between criminal and
civil law. Newcomers to the study of law tend to assume that criminal law occupies the bulk of a lawyer's
caseload and of a law student's studies. This is an interesting by-product of the portrayal of the legal
system by the media. Criminal law weighs very lightly in terms of volume when measured against non-
criminal (that is, civil) law. There are more rules of civil law than there are of criminal law; more court
cases involve breach of the civil law than that of the criminal law.
C. The term 'national law' is used to mean the internal legal rules of a particular country, in contrast to
international law which deals with the external relationships of a state with other states. There is no
world government or legislature issuing and enforcing laws to which all nations are subject. The
international legal order has no single governing body and operates by agreement between states. This
means that the creation, interpretation and enforcement of international law lie primarily in the hands of
states themselves. Its scope and effectiveness depend on the sense of mutual benefit and obligation
involved in adhering to the rules. Disputes about the scope and interpretation of international law are
rarely resolved by the use of international courts or binding arbitration procedures of an international
organisation. This is because submission to an international court or similar process is entirely voluntary
and few states are likely to agree to this if there is a serious risk of losing their case or where important
political or national interests are at stake.
D. One source of detailed information about the legal system is statistical analyses. Information about
the number of cases handled by a court shows in specific terms what a court's workload is. Changes in
these from year to year may indicate some effects of changes in the law and practice. Statistical tests
can establish that there is a relationship, a correlation, between different things. For example, the length
of a sentence for theft may correlate with the value of the items stolen or the experience of the judge
who heard the case. This means that the sentence will be longer if, for example, more items are stolen
or the judge is more experienced. A correlation can provide evidence for a theory. Such confirmation is
important; without it we have little to establish the impact the law has, being forced to rely on individual
instances of its application and having to assume that these have general truth. Empirical study of the
operation of law may reveal areas of improvement. It can also confirm that, measured by particular
standards, the courts are working well.

In which extract are the following mentioned?


1. the relative frequency of certain types of legal cases
2. input by those who are not directly involved in a dispute
3. how common transactions assume certain guarantees
4. ascertaining the effectiveness of a legal system
5. determining the consequences of altering the legal system
6. the influence of popular depictions of the law
7. a reluctance to submit to formal legal processes
8. how a decentralised legal system depends on a feeling of reciprocity
9. the lack of drama in the way the law operates
10. the absence of a certain type of legal institution
Your answers:
1. 2. 3. 4. 5.
6. 7. 8. 9. 10.

11
WRITING (6 pts)
Part 1. Read the following text and use your own words to summarize it between 120 and 140
words long. You MUST NOT copy the original.
Throughout history, various people have demonstrated a high degree of confidence in the ability of
certain animals to predict the weather. It may seem surprising today in view of the complex equipment
now involved in weather forecasting to understand that in certain cases, the behavior of animals does
indeed provide an indication of inclement weather. Sensitivity of certain animals to falling air pressure
or to low-frequency sound waves that humans cannot hear, which are indicators of approaching storms,
causes behaviors in animals that certain societies have come to recognize as predictors of storms.
A number of animals are remarkably sensitive to variations in air pressure, and some of these animals
show consistent, noticeable, and predictable behaviors as air pressure drops before a storm hits. Where
the air pressure drops before a storm, some animals move closer to the ground to equalize the pressure
in their ears: some birds such as swallows tend to stay on the ground or roost in trees instead of soaring
in the skies when a storm is imminent because of the decreasing air pressure. Other animals make more
noise than usual as air pressure drops: an unusual amount of quacking by ducks and a high volume of
croaking by frogs are both indicators that are believed to occur because of the high degree of sensitivity
of ducks and frogs to the change in pressure. Finally, still other animals become more active before
storms as a reaction to the falling air pressure: dolphins and porpoises seem to be taking part in a frenzied
sort of play, and bees and ants become more active prior to storms, most likely because of their
sensitivity to lower pressure.
There is good reason to believe that the fact that these animal behaviors seem to occur regularly prior
to storms may have a scientific basis and that the animals demonstrating these behaviors may actually
be good short-range weather forecasters. However, their ability to predict long-range weather patterns
is rather suspect. Certain proverbs, for example, are based on what is most likely the idea that squirrels
are good indicators of long-range weather patterns. One proverb indicates that, if a squirrel seems busier
than usual in gathering nuts, then a long and cold winter is on its way; however, this behavior in squirrels
is more likely due to a large supply of nuts available for gathering, which occurs because of earlier good
weather, and is not an indicator of cold weather to come. Another proverb about squirrels indicates that
if a squirrel grows a long and bushy tail in the fall, then a particularly harsh winter is on its way; in this
case too, the squirrel develops a long and bushy tail because of earlier good weather and not as a warning
of bad weather to come.
…….…………………………………………………………………………………………………...
…….…………………………………………………………………………………………………...
…….…………………………………………………………………………………………………...
…….…………………………………………………………………………………………………...
…….…………………………………………………………………………………………………...
…….…………………………………………………………………………………………………...
…….…………………………………………………………………………………………………...
…….…………………………………………………………………………………………………...
…….…………………………………………………………………………………………………...
…….…………………………………………………………………………………………………...
…….…………………………………………………………………………………………………...
…….…………………………………………………………………………………………………...
…….…………………………………………………………………………………………………...
…….…………………………………………………………………………………………………...

12
Part 2. The chart below shows the percentage of adults of different age in the UK who used the
Internet everyday from 2003-2006. Summarize the information by selecting and reporting the
main features and make comparisons where relevant.

…….…………………………………………………………………………………………………...
…….…………………………………………………………………………………………………...
…….…………………………………………………………………………………………………...
…….…………………………………………………………………………………………………...
…….…………………………………………………………………………………………………...
…….…………………………………………………………………………………………………...
…….…………………………………………………………………………………………………...
…….…………………………………………………………………………………………………...
…….…………………………………………………………………………………………………...
…….…………………………………………………………………………………………………...

Part 3. Write an essay on the following topic


Society is based on rules and laws. It could not function if individuals were free to do whatever they
wanted. To what extent do you agree or disagree?
…….…………………………………………………………………………………………………...
…….…………………………………………………………………………………………………...
…….…………………………………………………………………………………………………...
…….…………………………………………………………………………………………………...
…….…………………………………………………………………………………………………...
…….…………………………………………………………………………………………………...
…….…………………………………………………………………………………………………...

13
TRƯỜNG ĐẠI HỌC NGOẠI NGỮ KỲ THI HỌC SINH GIỎI CÁC TRƯỜNG THPT CHUYÊN
TRƯỜNG THPT CHUYÊN NGOẠI NGỮ KHU VỰC DUYÊN HẢI VÀ ĐỒNG BẰNG BẮC BỘ
LẦN THỨ XIII, NĂM 2022

ĐỀ THI MÔN: TIẾNG ANH


ĐỀ ĐỀ NGHỊ
LỚP 11
(Đề thi gồm 24 trang)
Thời gian: 180 phút (Không kể thời gian giao đề)
( Thí sinh làm bài trực tiếp vào đề)

A. LISTENING (50 points)


SECTION 1: (10pts)
Part 1: You will hear part of a radio phone-in programme about consumer
competitions that appear in magazines or are run by shops, in which advice is given to
people who regularly enter them. For questions 1-5, choose the answer (A, B, C or D)
which fits best according to what you hear.
1. Diana has phoned because she ______
A. feels that she is the victim of an injustice.
B. is reluctant to consult a lawyer yet.
C. fears she misunderstood an agreement she made.
D. wants to avoid falling out with her best friend.
2. Kathy tells Diana that ______
A. her problem is a rather unusual one.
B. she should have been more careful when dealing with her friend.
C. it is unfortunate that her friend has the attitude that she has.
D. she would regret taking legal action.
3. What does Kathy tell Ron about using different names when entering competitions?
A. People who do so are regularly caught out.
B. It may affect the quality of a competitor's entries.

Page 1 of 24
C. There are rarely occasions when it might be justified.
D. It is unusual for competitors to decide to do so.
4. What has led Stan to phone in?
A. an inadequate response to a complaint he has made
B. a feeling of confusion as to the rules of a competition
C. a belief that he has been sent inaccurate information
D. a desire for more openness about the results of competitions
5. What does Kathy tell Stan about the competition he entered?
A. Some of the phrasing of the instructions is ambiguous.
B. The rules allow for results that may appear unfair.
C. A deliberate attempt has been made to mislead competitors.
D. It is the sort of competition that it is best not to enter.

SECTION 2: (10pts)
Listen to the recording and then decide whether the following statements are True (T)
or False (F)
1. Paul decided to get work experience in South America because he wanted to learn about
Latin American life.
2. Paul originally intended to get involved in an agriculture project.
3. In the village community, he learnt how important it was to respect family life.
4. Paul thinks his project manager was very supportive of him.
5. Paul was surprised to be given an extension to his contract.

SECTION 3:
Listen to the recording and complete the following notes. Write NO MORE THAN
THREE WORDS AND/OR NUMBER

Page 2 of 24
Study Skills Tutorial – Caroline Benning
Dissertation topic: (1) ………………….
Strengths:  (2) ………………………
 computer modelling
Weaknesses:  lack of background information
 poor notetaking skills
Possible strategy Benefits Problems
peer group discussion increase dissertations tend to
(3) ……………………… contain the same ideas

use the (4) provides structured limited places


……………………… programme
consult study skills books are a good source of can be too general
reference
Recommendations:  use a card index
 read all notes (5) …………………….
Next tutorial date: 25 January

SECTION 4: (20 pts)


Listen to the recording and them complete the summary below.
Write NO MORE THAN THREE WORDS AND/OR A NUMBER for each answer.

Saving for the future


According to George Bernard Shaw, men are supposed to understand (1)
……………………., economics and finance. However, women are more
prepared to (2) ……………………. about them. While women tend to save for
(3) …………………….and a house, men tend to save for (4)
……………………. and for retirement.

Page 3 of 24
Research indicates that many women only think about their financial future when
a (5) ……………………. occurs. This is the worst time to make decisions. It is
best for women to start thinking about pensions when they are in their (6)
………………… A good way for women to develop their (7) …………………..
in dealing with financial affairs would be to attend classes in (8)
………………….. . When investing in stocks and shares, it is suggested that
women should put a high proportion of their savings in (9)
………………………. In such ways , women can have a comfortable ,
independent (10) ………………………. .
B. LEXICO AND GRAMMAR (30 pts)
Part 1: Choose the word/ phrase that best completes each of the following sentences.
Write your answer in the corresponding numbered boxes. (10 pts)
1. I thought the bill for just two people was simply ____________.
A. exhaustive B. exorbitant C. execrable D. exalted
2. It was a ___________ lie and nobody at all believed it.
A. blatant B. salient C. conspicuous D. pronounced
3. The rates of extreme poverty have halved in recent decades. But 1.2 million people still
live ____________.
A. on the breadline B. on the tenterhooks
C. on the house D. on the cards
4. The Prime Minister’s declaration that taxes will be raised has put the public
____________ an uproar.
A. at B. in C. to D. with
5. I personally never ____________ the idea that to be attractive you have to be thin.
A. dream up B. hold back C. buy into D. fall out
6. Gina finally ____________ and admitted that she had broken the vase.
A. threw in the towel B. made a clean breast of it
C. read her riot act D. let her hair down

Page 4 of 24
7. It must be true. I heard it straight from the ____________ mouth.
A. dog’s B. cat’s C. camel’s D. horse’s
8. Obama expressed regret as a US drone strike has ____________ killed innocent hostages.
A. incongruously B. vehemently C. inadvertently D. graciously
9. Two weeks ____________, we will celebrate our coronation
A. hence B. yet C. hitherto D. albeit
10. Mr. Smith is held in high ____________ for his dedication to the school.
A. respect B. esteem C. honor D. homage
11. The argument drove a ____________ between party members.
A. wedge B. linchpin C. needle D. sword
12. It is hard to know whether to believe such ____________ of doom, possibly because what
they are saying seems too terrible to be true.
A. champions B. warriors C. prophets D. giants
13. Patrick is too ____________ a gambler to resist placing a bet on the final game.
A. instant B. spontaneous C. compulsive D. continuous
14. I was at___________ to make it clear that I wasn’t blaming either of them.
A. efforts B. trouble C. pains D. endeavors
15. Tom’s normally very efficient but he’s been making a lot of mistakes ___________.
A. of late B. for now C. in a while D. shortly
16. So ____________ that they actually finished three weeks early.
A. were the builders fast B. fast the builders were
C. the builders were fast D. fast were the builders
17. Most of Annie Jump Cannon’s career as an astronomer involved the observation,
classification, and _____.
A. she analyzed stars B. the stars’ analysis
C. stars were analyzed D. analysis of stars
18. Designed by Frederic Auguste Batholde, _____ .
A. the United Sates was given the Statue of Liberty by the people of France
B. the people of France gave the Statue of Liberty to the United States
Page 5 of 24
C. the Statue of Liberty was given to the United States by the people of France
D. the French people presented the United States with a gift, the Statue of Liberty
19. The early railroads were _______ the existing arteries of transportation: roads, turnpikes,
canals, and other waterways.
A. those short lines connected B. short lines that connected
C. connected by short lines D. short connecting lines
20. The genus Equus became extinct in North America during the glacial period in the
unconscious and it was not reintroduced until ______ by Spaniards.
A. brought there B. was brought there
C. bringing it there D. it brought there
Your answers:
1. 2. 3. 4. 5.
6. 7. 8. 9. 10.

Part 2: Write the correct form of each bracketed word in each of the following sentences.
(10 pts)
1. _______________ pain is the reason for around 5% of all emergency department
visits. (ABDOMENT)
2. I was late because I _______________ how much time I would need. (ESTIMATE)
3. Since his bad habits were never broken when he was a child, they are now
_______________. (CORRECT)
4. Are those _______________ they put in food really necessary. (ADD)
5. The reason for Bruce Lee’s dealth was sleeping pill _______________. (DOSE)
6. If you can respond calmly and _______________, then I think that would be the best
way forward. (AGGRESSIVE)
7. Bill Gate is totally millionaire _______________ he started his own business with no
financial help at all. (SELF)

Page 6 of 24
8. The comedy Bringing Up Baby presents practically non-stop dialogue delivered at
_______________ speed. (NECK)
9. The new regulations will be _______________ for small businesses. (BURDEN)
10. You can ask a _______________ for advice on what kind of food you should eat to
keep you healthy. (DIET)
C. READING (60pts)
Part 1: For questions 1 – 7, choose which of the paragraph A – G into the numbered gaps
in the following magazine article. There is one extra paragraph, which does not fit in any
of the gaps.
CHEWING GUM CULTURE
It's fashionable, classless and Americans chew 12 million sticks of it a day.
Discover how an ancient custom became a big business
Chewing gum contains fewer than 10 calories per stick, but it is classified as a food and must
therefore conform to the standards of the American Food and Drug Administration. Today's
gum is largely synthetic with added pine resins end softeners which help to hold the flavour
and improve the texture.
1.
American colonists followed the example of the Amero-Indians of New England and chewed
the resin that formed on spruce trees when the bark was cut. Lumps of spruce for chewing
were sold in the eastern United States in the early 1800s Making it the first commercial
chewing gum in the country.
Modern chewing gum has its origins in the late 1860s with the discovery of chicle, milky
substance obtained from the sapodilla tree of the Central American rainforest.
2.
Yet repeated attempts to cultivate sapodilla commercially have failed. As the chewing gum
market has grown, synthetic alternatives have had to be developed.
3.

Page 7 of 24
Most alarming is the unpleasant little chicle fly that likes to lodge its egg in the tapper’s ears
and nose. Braving these hazards, barefooted and with only a rope and an axe, an experienced
chiclero will shin a mature tree in minutes to cut a path in the bark for the white sap to flow
down to a bag below.
4.
Yet, punishing though this working environment is, the remaining chicleros fear for their
livelihood. Not so long ago, the United States alone imported 7,000 tonnes of chicle a year
from Central America. Last year just 2000 tonnes were tapped in the whole of Mexcio’s
Yucatan peninsula. As chewing gum sales have soared, so the manufacturers have turned to
synthetics to reduce costs and meet demands.
5.
Plaque acid, which forms when we eat, causes this. Our saliva, which neutralizes the acid and
supplies minerals such as calcium, phosphate and fluoride, is the body’s natural defence.
Gum manufacturers say 20 times of chewing can increase your salivary flow.
6.
In addition, one hundred and thirty-seven square kilometers of America is devoted entirely to
producing the mint that is used in the two most popular chewing gums in the world

A. Gum made from this resulted in the smoother, more satisfying and more elastic chew,
and soon a whole industry was born based on this product.
B. Meanwhile, the world's gum producers are finding indigenous ways of marketing their
products. In addition to all the claims made for gum - it helps you relax, peps you up
and eases tension ( soldiers during both world wars were regularly supplied with gum)
– gum’s greatest claim is that it reduces tooth decay.
C. Research continues on new textures and flavors. Glycerine and other vegetable oil
products are now used to blend the gum base. most new flavors are artificial artificial -
but some flavors still need natural assistance.

Page 8 of 24
D. This was not always the case, though. The ancient Greeks chewed a gum-like resin
obtained from the bark of the mastic tree, a shrub found mainly in Greece and Turkey.
Grecian women, especially, favored mastic gum to clean the teeth and sweeten their
breath.
E. Each chiclero must carry the liquid on his back to a forest camp, where it is boiled until
sticky and made into bricks. Life at the camp is no picnic either, with a monotonous
and often deficient maize-based diet washed down by the local alcohol distilled from
sugar cane.
F. The chicleros grease their hands and arms to prevent the sticky gum sticking to them.
The gum is then packed into a wooden mould, pressed down firmly, initially and dated
ready for collection and export.
G. Today the few remaining chicle gatherers, chicleros, eke out a meagre and dangerous
living, trekking for miles to tap scattered sapodilla in near – 100% humidity.
Conditions are appalling: highly poisonous snakes lurk ready to pounce and insects
abound.
Your answer
1. 2. 3. 4.
5. 6. 7.

Part 2: For questions 1-10, read the text below and think of the word which best fits each
space. Use only one word in each space. There is an example at the beginning (0). (10 pts)
TRAFFIC JAMS ARE NOTHING NEW
In the age before the motor car, what was travelling in London like? Photographs
taken 100 years ago showing packed streets indicate that it was much the (1) ________ as it
is now. commuters choose the car to get to work properly travel at an (2) _________ speed of
17 kmp from their homes in the suburbs to offices in the center. (3) ________ is virtually is
the same speed that they (4) ________ have travelled at in a horse and carriage a century ago.

Page 9 of 24
As towns and cities grow, (5) ________ does traffic, whether in the form of the horse and
carriage (6) ________ the modern motor car. It would seen that, wherever (7) ________ are
people who need to go somewhere, they would (8) ________ be carried than walk or pedal.
The photographs show that, in terms of congestion and speed , traffic in London hasn't
changed over the past 100 years. London has had traffic jams ever since it became a huge
city. It is only the vehicles that have changed.
However, although London had traffic congestion long before the care came along, the age of
the horse produced little unpleasant apart from the congestion. Today, exhaust fumes create
dangerous smogs that cause breathing problems (9) ________ a great many people. (10)
_________problems could be reduced if many of us avoided jams by using bicycles or taking
a brisk walk to school or work
Your answer
1. 2. 3. 4. 5.
6. 7. 8. 9. 10.
Part 3: Read the following passage and choose the best answer (A, B, C or D) according
to the text. Write your answers (A, B, C or D) in the corresponding numbered boxes (15pts).
THE RIVERS OF THE AMAZON
There are three different types of rivers associated with the Amazon rainforest. Of
course, these types exist elsewhere in the world, yet, in the Amazon, they are most distinct.
They are classified as whitewater, clearwater, and blackwater rivers, and each displays
elements that make it ecologically distinct from the others. Clearly, one of the main reasons
they have been named as such is due to the qualities of their appearance, but there are also
fundamental differences in everything from their water consistency to abundance or lack of
organisms within each type of river. While whitewater and blackwater rivers occupy areas on
the floor of the forest at minimal elevations, clearwater rivers tend to be present at higher
elevations in mountainous regions. Still, each contributes in its own way to one of the most
diverse ecosystems on the planet, the Amazon.

Page 10 of 24
The clearwater rivers of the Amazon region, also called bluewater rivers, are noted for
their crystal—clear water and are typically found flowing through the rocks of the highlands
and at upper elevations. One of the main reasons why the waters are so clear is that the rocks
they flow through and over are extremely old, making erosion by the river waters difficult.
This lack of erosion lends to a lack of sediment in the water and its more transparent nature.
Clearwater rivers are also much swifter than whitewater or blackwater rivers due to their
abrupt changes in gradients. Overall, clearwater rivers have a very high mineral content, which
allows plant life as well as algae to proliferate. Because of the abundance of algae, many other
types of species of fish are able to thrive as well.
A second major type of river found in the Amazon is the whitewater river. The
formidable Amazon River itself is in this class. Ironically, whitewater rivers are usually light
chocolate-colored and have extremely low visibility because they contain heavy loads of
sediment. Two of the main causes of their high sediment content are the natural erosion of
the river basin itself as well as the deforestation of the rainforest. As deforestation continues,
soil is no longer anchored by the roots of vegetation and trees and is instead washed into the
river during the heavy rains that frequent the rainforest. Still, whitewater rivers are inherently
rich in nutrients and therefore support numerous types of aquatic organisms. In contrast to
clearwater rivers, whitewater rivers such as the Amazon tend to flow much more slowly
because of the low gradients involved. For example, the Amazon itself experiences only a
107—meter change in elevation from Peru to its mouth, a distance that covers over about
3,219 kilometers.
The final major type of river in the Amazon is known as a blackwater river, of which
the Rio Negro in Brazil is a prime example and also the largest in the world. Blackwater rivers
are the most common type in the Amazon. They exhibit a deep, dark brown color due to the
decomposition of leaves and vegetation in the waters. Most vegetation contains the chemical
tannin, which is released into blackwater rivers as it begins to decompose. Blackwater rivers
have excellent water clarity due to their highly acidic nature, which also benefits the river in
a couple of other ways. A high acid content is an excellent trait for a river to have because it
keeps the river clean and, in many ways, more sanitary than others. The acid kills parasites
Page 11 of 24
and bacteria which can threaten fish populations, as well as insect larvae such as that of
mosquitoes, reducing the spread of dangerous diseases such as malaria.
While the Amazon is a highly diverse ecosystem, one of the reasons this is so is due to
the different types of river systems within it. The Amazon is the main vein, and most
blackwater and clearwater rivers are estuaries branches of it until they finally link at certain
points along the Amazon’s main route. For example, the Amazon and the Rio Negro finally
meet at Manaus, Brazil, and, once they do, their distinct ecosystems are combined to form an
even more complex river system of life and water quality until it eventually empties into the
ocean.
1. In paragraph 1, the author's description of the major types rivers mentions which of the
following?
A. They are classified by what is able to exist in them.
B. They are determined by their length and breadth.
C. They are unique only to the area of the Amazon.
D. They are all located at altitudes at or beneath sea level.
2. The word "transparent" in the 2nd passage is closest in meaning to _____.
A. veneered B. refreshing C. glassy D. beneficial
3. Which of the following can be inferred from paragraph 2 about clearwater rivers?
A. Their water is not really as clear as their name implies.
B. They flow at a slower pace than blackwater rivers.
C. There is not much algae present in clearwater rivers.
D. They do not exist on more level areas in the Amazon.
4. According to paragraph 2, which of the following is NOT true of clearwater rivers?
A. They eventually connect with the Amazon River itself.
B. They have very high levels of nutrients and minerals.
C. They have enough sediment for algae to multiply.
D. They often occur where ancient stones or boulders are Present.
5. The author discusses "natural erosion" in paragraph 3 in order to _____.

Page 12 of 24
A. indicate the types of threats the Amazon region faces
B. relate why whitewater rivers are not as clear as some others
C. show how deforestation contributes little to whitewater rivers
D. suggest that sediment is not the main cause of the color of river
6. According to paragraph 3, deforestation is a negative factor in the s because _____.
A. too many trees are cut down, never to grow back
B. valuable soil is washed away into the rivers
C. other species of animals will not have a natural habitat
D. rivers become more erosive without strong boundaries
7. The word "inherently" in the passage is closest in meaning to _____.
A. partially B. subsequently C. naturally D. externally
8. According to paragraph 4, blackwater rivers are named so because _____.
A. they lack the necessary nutrients needed to produce life
B. they are in direct contrast to the two other types of river
C. they are colored by the decomposition of vegetation
D. they are the most common type of river in the Amazon
9. The word “it” in the passage refers to _____.
A. vegetation B. chemical C. tannin D. rivers
10. The word “vein” in the passage is closest in meaning to _____.
A. clasp B. diversion C. appendage D. thread
Your answers:
1. 2. 3. 4. 5. 6. 7. 8. 9. 10.

Part 4 : Read the following passage and do the tasks that follows. Write your answer in the
space provided. (15 pts)

Page 13 of 24
WHAT’S THE PURPOSE OF GAINING KNOWLEDGE?
A. ‘I would found an institution where any person can find instruction in any subject’. That
was the founders motto for Cornell University, and it seems an apt characterization of the
different university, also in the USA, where I currently teach philosophy. A student can
prepare for a career in resort management, engineering, interior design, accounting, music,
law enforcement, you name it. But what would the founders of these two institutions have
thought of a course called “Arson for Profit”? I kid you not: we have it on the books. Any
undergraduates who have met the academic requirements can sign up for the course in our
program in ‘fire science’.

B. Naturally, the course is intended for prospective arson investigators, who can learn all the
tricks of the trade for detecting whether a fire was deliberately set, discovering who did it,
and establishing a chain of evidence for effective prosecution in a court of law. But
wouldn’t this also be the perfect course for prospective arsonists to sign up for? My point
is not to criticize academic programs in fire science: they are highly welcome as part of
the increasing professionalization of this and many other occupations. However, it’s not
unknown for a firefighter to torch a building. This example suggests how dishonest and
illegal behavior, with the help of higher education, can creep into every aspect of public
and business life.

C. I realized this anew when I was invited to speak before a class in marketing, which is
another of our degree programs. The regular instructor is a colleague who appreciates the
kind of ethical perspective I can bring as a philosopher. There are endless ways I could
have approached this assignment, but I took my cue from the title of the course:
‘Principles of Marketing’. It made me think to ask the students, ‘Is marketing principled?’
After all, a subject matter can have principles in the sense of being codified, having rules,
as with football or chess, without being principled in the sense of being ethical. Many of
the students immediately assumed that the answer to my question about marketing

Page 14 of 24
principles was obvious: no. Just look at the ways in which everything under the sun has
been marketed; obviously it need not be done in a principled (=ethical) fashion.

D. Is that obvious? I made the suggestion, which may sound downright crazy in light of the
evidence, that perhaps marketing is by definition principled. My inspiration for this
judgement is the philosopher Immanuel Kant, who argued that any body of knowledge
consists of an end (or purpose) and a means.

E. Let us apply both the terms ‘means’ and ‘end’ to marketing. The students have signed up
for a course in order to learn how to market effectively. But to what end? There seem to
be two main attitudes toward that question. One is that the answer is obvious: the purpose
of marketing is to sell things and to make money. The other attitude is that the purpose of
marketing is irrelevant: Each person comes to the program and course with his or her own
plans, and these need not even concern the acquisition of marketing expertise as such. My
proposal, which I believe would also be Kant’s, is that neither of these attitudes captures
the significance of the end to the means for marketing. A field of knowledge or a
professional endeavor is defined by both the means and the end; hence both deserve
scrutiny. Students need to study both how to achieve X, and also what X is.

F. It is at this point that ‘Arson for Profit’ becomes supremely relevant. That course is
presumably all about means: how to detect and prosecute criminal activity. It is therefore
assumed that the end is good in an ethical sense. When I ask fire science students to
articulate the end, or purpose, of their field, they eventually generalize to something like,
‘The safety and welfare of society,’ which seems right. As we have seen, someone could
use the very same knowledge of means to achieve a much less noble end, such as personal
profit via destructive, dangerous, reckless activity. But we would not call that firefighting.
We have a separate word for it: arson. Similarly, if you employed the ‘principles of
marketing’ in an unprincipled way, you would not be doing marketing. We have another

Page 15 of 24
term for it: fraud. Kant gives the example of a doctor and a poisoner, who use the identical
knowledge to achieve their divergent ends. We would say that one is practicing medicine,
the other, murder.

Choose the correct heading for paragraphs B-F from the list of headings below. Write
the correct number, i-viii, next to Questions 1-5.
List of Headings
i. Courses that require a high level of commitment
ii. A course title with two meanings
iii. The equal importance of two key issues
iv. Applying a theory in an unexpected context
v. The financial benefits of studying
vi. A surprising course title
vii. Different names for different outcomes
viii. The possibility of attracting the wrong kind of student
Example Answer
Paragraph A iii
1. Paragraph B
2. Paragraph C
3. Paragraph D
4. Paragraph E
5. Paragraph F
Your answers:
1. 2. 3. 4. 5. 6.

Questions 6-9: Complete the summary below. Choose NO MORE THAN TWO WORDS
from the passage for each answer. Write your answer in the space provided.

Page 16 of 24
THE ‘ARSON FOR PROFIT’ COURSE
This is a university course intended for students who are undergraduates and who are studying
(6)________________. The expectation is that they will become (7)________________
specializing in arson. The course will help them to detect cases of arson and find
(8)________________ of criminal intent, leading to successful (9)________________ in the
courts.
Your answers:
6. 7. 8. 9.
Questions 10-13: Do the following statements agree with the views of the writer in the
reading passage
YES if the statement agrees with the views of the writer
NO if the statement contradicts the views of the writer
NOT GIVEN if it is impossible to say what the writer thinks about this

10. It is difficult to attract students onto courses that do not focus on a career.
11. The “Aron for Profit” course would be useful for people intending to set fire to buildings.
12. Fire science courses are too academic to help people to be good at the job of firefighting.
13. The writer’s fire science students provided a detailed definition of the purpose of their studies.
Your answers:
10. 11. 12. 13.

Part 5: Read the text below and answer questions (10pts).


THE BRAIN GAIN
With New Zealand becoming renowned as a great place to live, it was the first-choice
destination for a new generation of talented migrants looking for a better life. Sharon
Stephenson talks to four of them.

Page 17 of 24
A. Nicky Meiring, Architect
Listen to Nicky Meiring talk about South Africa and it soon becomes evident that she's
mourning for a country she once called home. 'The current economic situation has made
South Africa quite a hard place to live in,' she says, 'but I do miss it.' Nicky first arrived in
Auckland in 1994 and got a job in an architectural practice in Auckland where she soon
settled in. She says 'New Zealand often feels like utopia. I just love the tranquillity and the
fact you can lead a safe and ordinary life.' She lives and works from a renovated factory
where her mantelpiece is littered with awards for the design of her summer house on Great
Barrier Island. 'Although the design of buildings is fairly universal, houses here are generally
constructed of timber as opposed to brick and when it comes to the engineering of buildings,
I have to take great heed of earthquakes which isn't an issue in South Africa,' she says. 'But
the very fact that my training and points of reference are different means I have something to
offer. And I'm so glad I have the opportunity to leave my stamp on my new country."

B. Jenny Orr, Art Director


American Jenny Orr's southern accent seems more at home in the movies than in New
Zealand's capital, Wellington. 'I'm from Alabama, but no, we didn't run around barefoot and
my father didn't play the banjo!' she jokes, in anticipation of my preconceptions. Having
worked in corporate design for ten years in the USA, she was after a change and thought of
relocating to New Zealand. It didn't take long for her to land a job with an Auckland design
firm, where she was able to gain experience in an unfamiliar but challenging area of design -
packaging -and before long, she was headhunted to a direct marketing agency which recently
transferred her to Wellington. While she admits she could have the same salary and level of
responsibility at home, 'it would probably have been harder to break into this kind of field.
I'm not saying I couldn't have done it, but it may have taken longer in the US because of the
sheer number of people paying their dues ahead of me.' Ask Jenny how she's contributing to
this country's 'brain gain' and she laughs. 'I don't see myself as being more talented or
intelligent but opposing views are what make strategies, concepts and designs better and I
hope that's what I bring.'
Page 18 of 24
C. Sarah Hodgett, Creative Planner
What happens when all your dreams come true? Just ask Sarah Hodgett. Sarah says that she
had always dreamed of a career in advertising. 'But I was from the wrong class and went to
the wrong university. In the UK, if you're working class you grow up not expecting
greatness in your life. You resign yourself to working at the local factory and knowing your
place.' New Zealand, on the other hand, allowed her to break free of those shackles. 'It's a
land of opportunity. I quickly learned that if you want to do something here, you just go for
it, which is an attitude I admire beyond belief.' Within a month of arriving, she'd landed a
job in customer servicing with an advertising agency. Then, when an opening in research
came up, she jumped at the chance. 'My job is to conduct research with New Zealanders,' she
explains. 'So I get to meet people from across the social spectrum which is incredibly
rewarding.' Being a foreigner certainly works in her favour, says Sarah. 'Because a lot of my
research is quite personal, respondents tend to see me as' impartial and open-minded and are
therefore more willing to share their lives with me.' She certainly sees New Zealand in a
good light. 'I wish New Zealanders could see their country as I do. That's why it saddens me
that they don't think they're good enough on the global stage.'

D. Lucy Kramer, School Director


Born in Sydney, Australia, Lucy Kramer left for London when she was 23 to further her career
as a stockbroker. 'London certainly lived up to my expectations and I had a very exciting, very
hectic lifestyle,' Lucy explains. But after four years she felt burnt out and was becoming
increasingly disillusioned with her job. 'People at work were far too competitive for my liking,'
she says. It was at this time she made two life-changing decisions. 'I signed up for a teacher-
training course and shortly after that met my partner, Graeme. He asked me to come back to
New Zealand with him and I didn't hesitate.' It wasn't long before she found work in a large
Auckland school and, since then, she has rapidly worked her way up to a management position.
'It's fair to say I'm not earning what I used to but my New Zealand colleagues are much more
easy- going. A good atmosphere more than makes up for the drop in salary. Another thing that
Page 19 of 24
impresses me is that you can leave your stuff on a seat in a cafe and it'll still be there half an
hour later. People are pretty trustworthy here. Sometimes it bothers me that we're so remote -
you can feel a bit cut off from what's going on in the rest of the world, but on the whole, I'd
say it's one of the best moves I ever made.'
For questions 1–10, identify which section A–D each of the following is mentioned. Write
ONE letter A–D in the space provided. Each letter may be used more than once.

According to the text, which woman ….. Your


answers
1. mentions the way in which she was disadvantaged in the country she left?
2. mentions a negative point about a job she has had?
3. explains an advantage of choosing to pursue her career in New Zealand?
4. mentions an aspect of living in New Zealand that she can find frustrating?
5. appreciates the approach to achieving goals in New Zealand?
6. expresses a sense of regret about leaving her country?
7. appreciates the honesty she feels exists in New Zealand?
8. states the fact she is happy to make an impression?
9. denies conforming to a certain stereotype?
10. appreciates New Zealand for its sense of calm and normality?

D. WRITING (60 pts)


Part 1: Summarize the following passage in no more than 120 words.
There are so many lessons one can learn about life from a dog. Imagine this scenario: it is
raining heavily outside and you need to leave for someone's house. The dog is up and eager,
to go with you. You tell it to stay home. As you leave, you see it squeezing out through the
gap in the doorway. You scold it and order it back home. Then at every turn you make, you
suddenly see it following you sheepishly at a distance. It follows at the risk of being

Page 20 of 24
reprimanded for the sore reason of being somewhere nearby. How else can we experience so
selfless an instance of love and faithfulness? We can learn a lifelong lesson from this sincere
warm display of perpetual companionship.

Observe the eating habits of your dog. It does not eat, except when hungry. It does not drink,
unless it is thirsty. It does not gorge itself. It stops eating when it has had enough.

A dog also sets a perfect example of adaptability. If it is moved to a strange place, it is able to
adapt itself to that place and to its thousand peculiarities without a murmur of complaint. It is
able to learn and adapt to a new family's ways and customs. It is quick and ready to please.
Man, being accustomed to comfort and wealth will be lost if suddenly stripped of all he is
accustomed to.

A dog also teaches us a thing or two about, unselfish love. When a dog knows death is
approaching, it tries, with its last vestige of strength, to crawl away elsewhere to die, in order
to burden its owners no more.

A dog does things with all vigor. However, when there is nothing to do, it lies down and rests.
It does not waste its strength and energy needlessly. Many working people are burning the
candles at both ends. Many suffer nervous breakdowns due to stress. Perhaps, they should
learn to rest like a dog does.

A dog above all is truly man's best friend.


………………………………………………………………………………………………………………………………………………………………

………………………………………………………………………………………………………………………………………………………………

………………………………………………………………………………………………………………………………………………………………

………………………………………………………………………………………………………………………………………………………………

………………………………………………………………………………………………………………………………………………………………

………………………………………………………………………………………………………………………………………………………………

………………………………………………………………………………………………………………………………………………………………

………………………………………………………………………………………………………………………………………………………………

………………………………………………………………………………………………………………………………………………………………

Page 21 of 24
………………………………………………………………………………………………………………………………………………………………

………………………………………………………………………………………………………………………………………………………………

………………………………………………………………………………………………………………………………………………………………

………………………………………………………………………………………………………………………………………………………………

………………………………………………………………………………………………………………………………………………………………

………………………………………………………………………………………………………………………………………………………………

………………………………………………………………………………………………………………………………………………………………

Part 2: Chart description.(15pts)


You should spend about 20 minutes on this task.
The graph below gives information from a 2008 report about consumption of energy in
the USA since 1980 with projections until 2030.
Summarize the information by selecting and reporting the main features, and make
comparisons where relevant.
Write at least 150 words.

Page 22 of 24
………………………………………………………………………………………………………………………………………………………………

………………………………………………………………………………………………………………………………………………………………

………………………………………………………………………………………………………………………………………………………………

………………………………………………………………………………………………………………………………………………………………

………………………………………………………………………………………………………………………………………………………………

………………………………………………………………………………………………………………………………………………………………

………………………………………………………………………………………………………………………………………………………………

………………………………………………………………………………………………………………………………………………………………

………………………………………………………………………………………………………………………………………………………………

………………………………………………………………………………………………………………………………………………………………

………………………………………………………………………………………………………………………………………………………………

………………………………………………………………………………………………………………………………………………………………

………………………………………………………………………………………………………………………………………………………………

………………………………………………………………………………………………………………………………………………………………

………………………………………………………………………………………………………………………………………………………………

………………………………………………………………………………………………………………………………………………………………

………………………………………………………………………………………………………………………………………………………………

………………………………………………………………………………………………………………………………………………………………

………………………………………………………………………………………………………………………………………………………………

………………………………………………………………………………………………………………………………………………………………

………………………………………………………………………………………………………………………………………………………………

…………………………………………………………………………………………………………………………………………………………

Part 3: Write an essay of about 350 words to express your opinion on the following
issue (30 pts)

Too much emphasis is placed on testing these days. The need to prepare for
tests and examinations is a restriction on teachers and also exerts unnecessary
pressure on young learners.

To what extent do you agree or disagree with this opinion?


Give reasons for your answer and include any relevant examples from your own
knowledge or experience.
Page 23 of 24
………………………………………………………………………………………………………………………………………………………………

………………………………………………………………………………………………………………………………………………………………

………………………………………………………………………………………………………………………………………………………………

………………………………………………………………………………………………………………………………………………………………

………………………………………………………………………………………………………………………………………………………………

………………………………………………………………………………………………………………………………………………………………

………………………………………………………………………………………………………………………………………………………………

………………………………………………………………………………………………………………………………………………………………

………………………………………………………………………………………………………………………………………………………………

………………………………………………………………………………………………………………………………………………………………

………………………………………………………………………………………………………………………………………………………………

………………………………………………………………………………………………………………………………………………………………

………………………………………………………………………………………………………………………………………………………………

………………………………………………………………………………………………………………………………………………………………

………………………………………………………………………………………………………………………………………………………………

………………………………………………………………………………………………………………………………………………………………

………………………………………………………………………………………………………………………………………………………………

………………………………………………………………………………………………………………………………………………………………

………………………………………………………………………………………………………………………………………………………………

………………………………………………………………………………………………………………………………………………………………

………………………………………………………………………………………………………………………………………………………………

………………………………………………………………………………………………………………………………………………………………

………………………………………………………………………………………………………………………………………………………………

………………………………………………………………………………………………………………………………………………………………

………………………………………………………………………………………………………………………………………………………………

………………………………………………………………………………………………………………………………………………………………

………………………………………………………………………………………………………………………………………………………………

………………………………………………………………………………………………………………………………………………………………

………………………………………………………………………………………………………………………………………………………………

………………………………………………………………………………………………………………………………………………………………

………………………………………………………………………………………………………………………………………………………………

………………………………………………………………………………………………………………………………………………………………

-The end-
Page 24 of 24
SỞ GD&ĐT THANH HOÁ KỲ THI HỌC SINH GIỎI CÁC TRƯỜNG THPT CHUYÊN
TRƯỜNG THPT CHUYÊN KHU VỰC DUYÊN HẢI VÀ ĐỒNG BẰNG BẮC BỘ
LAM SƠN NĂM 2022

ĐỀ ĐỀ XUẤT ĐỀ THI MÔN: TIẾNG ANH – KHỐI 11


Thời gian: 180 phút (Không kể thời gian giao đề)
(Đề thi gồm 15 trang)

LISTENING (50 points)


Part 1: Listen to a dialogue between two students and answer the questions with NO MORE THAN
THREE WORDS.
1. What important aspect of history do they need to learn?
_________________________________________________
2. What do we need to know about our ancestors?
_________________________________________________
3. What does the study of anthropology include?
_________________________________________________
4. What aspect of geography is focused on in the course?
_________________________________________________
5. How are social studies often criticized for?
_________________________________________________

Part 2: Listen to a talk about malware and decide if the statements are true or false.
6. Malwares mostly aim to steal secret data from digital devices.
7. All emails carry a certain type of malware.
8. Viruses will enter computers and spread when users receive an executable file.
9. Trojans not only steal confidential data but also enable cybercriminal to enter your computer.
10. The spread of both viruses and worms depends on infected host file.
Your answer:
6. 7. 8. 9. 10.

Part 3: You will hear part of a discussion programme where Florence, a marketing expert, and Mark, a
retail analyst, discuss impulse buying. For questions 11-15, choose the answer (A, B, C or D) which fits
best according to what you hear.

11. What does Mark imply when talking about the items made people purchase through impulse
shopping?
A. Many expensive items are often purchased in this way.
B. Impulse shopping can result in consumers buying unnecessary items.
C. Most impulse buys take place in clothes shops.
D. People very often purchase items that they cannot afford.
12. What point do both Mark and Florence make about the retail industry?
A. It actively encourages impulse-buying behaviours.
B. It maximizes its profits by offering promotions on expensive items.
C. It has conducted extensive research into influencing people's way of thinking.
D. It often presents products of lesser quality as a good deal.
13. What do Mark and Florence agree has made impulse buying easier?
A. more disposable income
1
B. the availability of cheaper products
C. a wider variety of payment methods
D. advances in technology
14. When describing the relationship between stress and shopping, Florence says that
A. shopping may help to briefly reduce stress levels.
B. all impulse buys are done when the consumer is stressed.
C. the act of shopping can be stressful in itself.
D. consumers shop online to avoid stress.
15. What advice does Mark have for anyone wanting to curb their spending habits?
A. avoid the shops altogether.
B. be aware that emotions guide purchasing decision.
C. delay making a purchase.
D. set a monthly budget for one’s spending.
Your answer:
11. 12. 13. 14. 15.

Part 4: You will listen to a talk about nuclear waste. For questions 16-25, fill in the gap with NO MORE
THAN THREE WORDS.
In Onkalo, Finland, the world’s first 16.__________________ for high-level nuclear waste is under
construction.
One drawback of developing nuclear energy is spent fuel rods of 17.___________________, which Finland is
planning to dispose permanently.
The rods will be kept in 18.__________________ buried nearly half a kilometer underground for at least
100,000 years until the 19__________________ decay to acceptable levels.

There are a series of barriers to keep the rods safe underground.


First barrier: The rock
- have few 20._____________________
- be impervious to water
Second barrier: Bentonite
- be water absorbing
- plug 21._____________________
- block microbes
- speed up 22._____________________
Third barrier: Copper
- won’t corrode due to being 23._________________

The plan in Finland was better than some other countries thanks to
- 24.________________ in the sighting process
- promise of jobs from the company
- promise of a new senior center
- Finns’ trust in state institutions
Around 2120, after having entombed 6500 tons of waste in Onkalo, all access tunnels will be backfilled and
sealed up, service structure will be 25.______________, no sign of the repository will remain.
Your answer:
16. 17. 18. 19. 20.
21. 22. 23. 24. 25.

LEXICO-GRAMMAR (30 points)


Part 1: Choose the correct answer to complete the sentence
2
1. ________ my dad, I would never have started playing tennis in the first place.
A. Were it for B. Were it to be for C. Had it not been for D. Were it not for
2. We _________ as well try to sell this old sofa before throwing it out.
A. could B. should C. might D. can
3. I’d rather you ________ anything for tonight, but I suppose it’s too late to change things now.
A. not arrange B. don’t arrange C. didn’t arrange D. hadn’t arranged
4. I feel ________ to inform the committee that a number of members are very unhappy with.
A. my duty B. this my duty C. it my duty D. that my duty
5. ________ I’d like to help you out , I’m afraid I just haven’t got any spare money at the moment.
A. Even B. Despite C. Much as D. Try as
6. The company’s announced it’s ________ over 1,000 workers.
A. lay off B. set off D. switch off D. kill off
7. You’ll regret it if you don’t ________ your important data.
A. do up B. back up C. key in D. test out
8. I didn’t ________ to be a millionaire- I just wanted to run a successful business.
A. crop up B. wind down C. set out D. tide over
9. My fingers are tired. I’ve been ________ at this keyboard for hours.
A. slaving away B. breaking away C. plodding away D. hammering away
10. So ________ what you are saying is that I am being made redundant.
A. in effect B. in good faith C. in that D. in between
11. We have it ________ that they’re thinking of closing the local office.
A. on good authority B. authority on C. in authority D. with authority
12. Since it was a minor offence, Derek was sent to ________ prison.
A. an open B. an easy C. a soft D. a weak
13. Don’t worry- the documents are safely _______ lock and key at my place.
A. in B. on C. under D. within
14. Ian’s really under his boss’s ________ and isn’t allowed to make any decisions himself.
A. finger B. hand C. thumb D. eye
15. Our neighbour is a bit strange, but ________ and let live, I always say.
A. take B. come C. do D. live
16. This is the third time he’s been caught- they are really going to throw the ________ at him this time.
A. book B. rod C. row D. pen
17. I’ve spent the whole morning vacuuming the inside of the car. It’s now clean as a ________.
A. dust B. cucumber C. whistle D. lab
18. You have to go through a number of ________ stages before you become an expert.
A. medium B. average C. moderate D. intermediate
19. The couple were finally ________ by the landlord after not paying rent for six month.
A. demolished B. rejected C.evicted D. evacuated
20. He’s extremely ________; he turns up every day at nine o’clock on the dot.
A. punctual B. temporary C. long-standing D. subsequent
Your answer:
1. 2. 3. 4. 5. 6. 7. 8. 9. 10.
11. 12. 13. 14. 15. 16. 17. 18. 19. 20.

Part 2: Provide the correct form of words in brakets.


21. The Mekong Delta region was at the (front) of climate change adaptation in Viet Nam.

22. The degradation had been caused by a range of factors including lack of sediment, and the land use in the
coastal zone pressing the dike (sea).

23. The inadequate water management and overexploitation of groundwater resulted in land (subside).

3
24. Also known as IT security, (secure) measures are designed to combat threats against networked systems and
applications.

25. The international supporters have committed to supporting Viet Nam in strengthening the coastal zones of
the Mekong Delta and the (resile) of its inhabitants.

26. The environmental ministry is writing more detailed instructions under this law to (actual) and monitor
waste classification.

27. Waste classification in each family is undoubtedly needed and a (do).

28. KOICA is trying to solve the environmental problems through cooperation with various partners including
academia, (start) and private businesses.

29. The amendment to the law against domestic violence has been discussed at the group stage, and deputies
have been (brain) acts that can be included.

30. No matter how heavy or light it is, domestic violence leaves (repair) mental scars.

Your answer:

21. 22. 23. 24. 25.


26. 27. 28. 29. 30.

READING (60 points)


Part 1: Fill in the gap with ONE suitable word. (15pts)
What do Einstein, Leonardo da Vinci, Julius Caesar, and Napoleon have in common? They were all left-
handed! Beginning early in history, southpaws were feared, neglected, and punished because use of the left
hand was considered 1.________ . In fact, the Latin word for “left” is “sinister.” Due to
this 2._________ against the left, left-handed people were forced to switch hands. This change caused many
problems for lefties in school, which resulted in frustration, bad behavior, and high drop-out rates. Maybe that is
why lefties were 3.________ as unintelligent, an idea that could not be further from the truth.
Researchers studying the brain have found differences in the left side and right side of the brain. Researchers
believe that people who use more of the left side of their brain tend to be more 4.________. They also tend to
use language better and solve problems faster. However, one study also showed that left-brain people tend to
have 5.________ memories.
So, which side of the brain do lefties use? Actually, both. Right-handed people use the left side of the brain
more, whereas left-handed people use both sides of the brain almost 6.________. The part connecting the two
halves of the brain is usually larger in left-handed people. Therefore, information can pass more 7.________
from one side of the brain to the other. The left hemisphere of the brain controls speech, language,
writing, logic, mathematics, and science. The right hemisphere controls music, art, creativity, perception, and
emotion. Since lefties use both sides, they are often both creative and scientific.
Approximately ten percent of the world is left-handed, and the 8.________ of left-handed males to left-handed
females is two to one. Thankfully, parents and teachers no longer 9.________ left-handedness as a problem to
be cured. It may actually contribute to a child’s excellence! If allowed to learn and develop in their own way,
lefties will 10.________ in school. Perhaps one of these lefties might turn out to be the next Einstein or da
Vinci.
(Reading challenge 2)
Your answer:

1. 2. 3. 4. 5.
6. 7. 8. 9. 10.
4
Part 2: Read the passage and answer the questions followed. (13pts)

The Phantom Hand

This illusion is extraordinarily complelling the first time you encounter it.

There is a very striking illusion in which you can feel a rubber hand being touched as if it were your
own. To find out for yourself, ask a friend to sit across from you at a small table. Set up a vertical partition on
the table, rest your right hand behind it where you cannot see it, and place a plastic right hand in view. Ask your
assistant to repeatedly tap and stroke your concealed right hand in a random sequence. Tap, tap, tap, stroke, tap,
stroke, stroke. At the same time, while you watch, they must also tap and stroke the visible plastic dummy at
exactly the same time in the same way. If your friend continues the procedure for about twenty or thirty
seconds, something quite strange will happen: you will have an uncanny feeling that you are actually being
stroke on the fake hand. The sensations you feel will seem to emerge directly form the plastic.

Why does this happen? Matthew Botinick and Jonathan Cohen, at the University of Pittsburgh and
Carnegie Mellion University, who reported the so-called rubber hand illusion in 1998, have suggested that the
similarity in appearance fools the brain into mistaking the false hand for your real hand. They believe this
illusion is strong enough to overcome the discrepancy between the position of your real hand that you can feel
and the site of the plastic hand you can see.

But that is not the whole story. William Hirstein and Kathleen Carrie Armel of the University of
California discovered a further twist: the object your helper touches does not even need to resemble your hand.
The same effect is produced if they tap and stroke the table. Try the same experiment, but this time gets your
acquaintance to rub and tap the surface in front of you while making matching movements on your real,
concealed hand. You will eventually start feeling touch sensations emerge from the wood surface.

This illusion is extraordinarily compelling the first time you encounter it. But how can scientists be
certain that the subject really believes that they are feeling sensations through the table? Kathleen Carrie Armel
again and Vilayanur S Ramachandran learned that, once the illusion has developed, if you “threaten” the table
by aiming a blow at it, the person winces and even starts sweating. This reaction was demonstrated objectively
by measuring a sudden decrease in electrical skin resistance caused by perspiration. It is as if the table becomes
incorporated into a person’s own body image so that it is linked to emotional centres in the brain; the subject
perceives a threat to the table as a threat to themselves.

This may all sound like a magic trick, but it does have practical applications. In fact, the experiments
were inspired by work with patients who had phantom limbs. After a person loses an arm from injury, they may
continue to sense its presence vividly. Often, the phantom seems to be frozen in a painfully awkward position.
To overcome this, a patient was asked to imagine putting their phantom arm behind a mirror. By then putting
their intact arm on the reflective side, they created the visual illusion of having restored the missing arm. If the
patient now moved the intact arm, its reflection- and thus the phantom- was seen to move. Remarkably, it was
felt to move as well, sometimes relieving the painful cramp.

Beyond a practical application, these illusions also demonstrate some important principles underlying
perception. Firstly, perception is based largely on marching up sensory inputs. As you feel your hand being
tapped and stroked and see the table or dummy hand being touched in the same way, your brain asks itself,

5
“What is the likelihood that what I see and what I feel could be identical simply by chance? Nil. Therefore, the
other person must be touching me.” Secondly, this mechanism seems to be based on automatic processes that
our intellect cannot override. The brain makes these judgments about the senses automatically; they do not
involve conscious thought. Even a lifetime of experience that an inanimate object is not part of your body is
abandoned in light of the perception that it is.

All of us go through life making certain assumptions about our existence. “My name has always been
Joe” someone might think. “ I was born in San Diego” and so on. All such beliefs can be called into question at
one time or another for various reasons. One premise that seems to be beyond question is that you are anchored
in your body. Yet given a few seconds of the right kind of stimulation, even this obvious fact is temporarily
forsaken, as a table or a plastic hand seem to become a part of you.

Question 11-14: The text reports the findings of three teams of researchers. Match statements 11-14 with
the correct team A, B or C.

A. Botvinick and Cohen B. Hirsteiri and Armel C. Armel and Ramachandran

11. The illusion does not depend on the “phantom” looking like a real hand.

12. The brain can disregard spatial information.

13. If the fake hand is threatened, the subject will show signs of fear.

14. A hand-shaped object is required for the illusion.

Questions 15-17: Choose the correct letter A, B, C or D in answer to these questions.

15. How do researchers explain the fact that subjects respond physically when someone threatens to hit the table
in front of them?

A. The table becomes an integral part of the image subjects have of themselves.

B. It is a reflex action triggered by the movement of the other person’s hand.

C. An electrical connection is established between the subject and the table.

D. Over time, the subject comes to believe that the table is one of these possessions.

16. What does the phantom hand experiment show us about the nature of human perception?

A. It is based on conscious thought processes.

B. It is primarily an unconscious process.

C. It is closely related to intellectual ability.

D. It relies only on sensory information.

17. Which of these statements best summarizes the wider implications of the experiments described in the text?

A. The experiments are valuable in treating patients who have lost limbs.

B. The experiments cast doubt on a fundamental human assumption.


6
C. The experiments show humans to be less intelligent than was once thought.

D. Human beings arrive at the truth by analysing the evidence of their senses.

Questions 18-23: Complete the summary below. Choose ONE WORD FROM THE TEXT AND/OR A
NUMBER for each answer.

It is a recognized phenomenon that patients who have been injured and lost 18._______ sometimes continue to
have feelings, like pain or 19.________, in these parts of their body. In order to assist patients like this, doctors
can use a 20.________ placed vertically on a flat surface. The patient imagines that he is putting his phantom
arm behind the mirror and his 21. ________ arm in front. When the patient moves the latter, the 22.________
also moves, giving the patient the illusion that his non-existent arm is moving. In some cases, this illusory
movement may succeed in 23.________ the patient’s discomfort.

Your answer:

11. 12. 13. 14. 15.


16. 17. 18. 19. 20.
21. 22. 23.

Part 3: You are going to read an extract from an autobiography. Seven paragraphs have been removed
from the extract. Choose from the paragraphs A-H the one which fits each gap 24-30. There is one extra
paragraph wich you do not need to use. (7pts)

The Hammond Organ

It’s September 1995 and I’m on my way home to Austin, Texas from Bankok. Breaking the journey in Los
Angeles, I spot an ad for an organ in the classifieds. It’s a 1954 Hammond B2. I can’t resist this little gem, so I
buy it-sight unseen- and arrange to have it collected, crated and trucked to Texas.

24
Ever since I heard Green Onions by Booker T and the MG’s on the radio, the sound of a Hammond organ has
moved me. Although ta the time I didn’t know exactly what Booker T. was playing, I knew I wanted to make
that noise. I didn’t even know how to play an organ, but the way it swirled and swam and it your ears off, I
knew somehow I had to have one. So I did my research in the music shops, and found out that the coolest-
sounding organs were all Hammonds, bu that the L100, while it still had that special sound, was lighter and
cheaper than the other models. Not that any of them were cheap, which didn’t mch matter, because I had no
money.

25
But when I called them up, they were very helpful. There was no drawback. The only thing I could not do was
move it, once they’d set it up. That wasn’t going to be a problem. The problem would be explaining the arrival
of this beautiful monster to Mum and Dad. But I wasn’t thinking that far ahead. I wasn’t really thinking at all,
apart from wondering-when could it be delivered? “Tomorrow”, “Okey”. And that was it. The next morning at
about 10am there was a knock at the door and two men in white coats were standing on the doorstep. After I’d
signed papers and promised not to move it, we pushed the dining table and chairs back against the wall.

26

7
It was all polished and shiny and made our dinning room suite look quite tatty. They showed me how to start it
up and we shooked hands. It couldn’t have been simpler. “ See you in two weeks then.” “Yes, Okey, bye”
slamd. “Aarrgh!” I screamed and ran upstairs to get the record player from the bedroom, set it up on top of the
bookcase, plonked Green Onions on the turntable and cranked it up! Yes, yes, yes, nothing could stop me now. I
had lost my mind and I’d never find it again.

27
The next thing to master was the Leslie cabinet. This was where the sound came out. The Leslie is a combined
amplifier and speaker cabinet, but it has two speakers which point up and down. The sound travels through
revolving rotors, which throws the music out in waves. It’s what makes the sound of every Hammond bit and
swim in your ears. You can regulate the speed it rotates and it’s very powerful.

28
When Dad came whistling his way up the path after work, I went to the door to head him off. “Hello Dad”. “
What’s up?” “ Nothing much . Well, I’ve got something to tell you’. “yes” “Er, Dad, you’ll never guess what
I’ve got” “ What have you got?” “ A Hammond organ”.

29
He was down the hall and peering round the door suspiciously before I could stop him. “Blimey” he said.
“Well, I am blowed. Where’s the dining room table gone?” he was in the doorway, trying to squeeze past the
monster organ and the Leslie. “It’s great, isn’t it?” “ Well, it is big… how are we going to eat with this thing in
here, and why didn’t you ask me or your mum?” “Sorry, but it’ll only be here for a couple of weeks, listen to
this,” I played the first part of Green Onions on it. “Not bad, eh?” “I dunno” he was thinking. “ Here, don’t day
a word, let me break it to your mum.”

30
I bought it on the “never, never”. Dad co-signed the hire purchase forms for me because I was under age.

------------------------------

The missing paragraphs

A. This meant that there was now enough room. Very carefully, they wheeled in a brand new Hammond organ
and matching bench with the Playing Guide and connecting cables tucked inside the lid, and a band new Leslie
147 speaker cabinet, which filled up the entire room. My face must have been a picture. This was great!

B. I found all that out by fiddling around with it for hours that day until I got some results. Basically, I just
taught myself. The wonderful thing about the Hammond is it sounds good without too much effort. It’s not like
the bagpipes or the violin, where even after a lot of work it can still sound bad.

C. However, I never had any ambition as a kid to play the piano, let alone the organ. It was all my mum’s fault.
She’d had a dream of playing the piano since she was a kid, but growing up in the little town of Montrath in the
centre of Ireland, as one of 11 kids, there was hardly money for shoes let alone piano lessons. And as she hadn’t
been able to afford them when she was young, I was going to get them whether I wanted them or not.

D. “What’s a Hammond organ?” “It’s free. I’ve got it for two weeks, then they’ll come and take it away and no
charge whatsoever”. “Where is it then?” “ It’s in the back room, it’s fantastic and it’s not costing a penny.”

8
E. Then, thumbing through the back pages of the Melody Maker, I noticed an ad for Boosey and Hawkes, in
Regent Street, who were offering to let me: “Try a Hammond Organ in your own home on two weeks’ free
approval”. “Yeah, right,” I thought. “Pull the other one” I tried to figure out what the catch could be, because I
couldn’t believe they’d let me get my sweaty hands on a genuine Hammond without money changing hands or
at least making a promise to buy.

F. Somehow I knew that meant it was going to be all right. The men in white coats came to take it away two
weeks later and my new mahogany Hammond organ and matching Leslie cabinet arrived the following week.

G. Sometimes, a smell can trigger a memory so strong and true it unravels years in an instant, like the smell of
oil paint, which takes me straight back to my art school days. So, as they unbolt the container, even before I get
to see how beautiful the instrument is, the combination of furniture polish and Hammond oil wafts up my nose
and I get a flashback to 1964, when I caught that odd mixture for the first time.

H. Now I had to figure out how to play the beast and get the same as that. Carefully listening to sustained notes
on the record, I pushed and pulled the drawer bars in and out until I got the same sound. Then if I played the
part right, the sound would change- just like the record.

Your answer:

24. 25. 26. 27. 28. 29. 30.

Part 4: Read the passage and answer the questions (15 pts)

THE TRANSPORTATION REVOLUTION


By the close of the eighteenth century, the outlines of a world economy were clearly visible. Centered in
Western Europe, it included Russia, India, the East Indies, the Middle East, northern and western Africa, and
the Americas. Trade had increased greatly and shipping had grown in volume and speed, connecting the
markets of the world more closely than ever before. The world market, however, was confined to the coasts
and along rivers, and its effects were rarely felt a hundred miles inland. The expansion of economic
activity into the interior, and its spread throughout China, Japan, Oceania, and Africa, was a major development
of the nineteenth century. It was largely accomplished through a revolution in transportation, particularly the
development of the steamship, canals, and railroads.
Since the fifteenth century, the wooden sailing ship had been the main instrument of European economic
and political expansion. Sailing ships constantly grew in carrying capacity and speed with improvements in
design, and they were built of easily available materials. The age of sailing ships reached its zenith in the
middle of the nineteenth century, the era of the great ocean-plying clippers that carried the majority of
international trade.
Before 1850, the bulk of internal trade was carried by water. In Western Europe, there had been several
attempts to supplement the excellent river network with canals. However, it was the demands of the Industrial
Revolution, particularly the need to transport huge quantities of coal, that stimulated large scale canal building
in the years 1760, 1850, first in Britain and then in Western Europe and the United States. The introduction of
steamboats gave an additional impetus to river navigation and canal construction. The steamship rose in stature
in the 1870s, when technical progress reduced the amount of coal the steam engine consumed. Technical
innovation, along with the completion of the Suez Canal in 1869, enabled the steamship to surpass the sailing
ship as the chief instrument of international trade.
Methods of land transport continued to be slow, uncertain, and expensive until the boom in railroad
construction at mid century. In 1840 there were 5.500 miles of rail track throughout the world; just twenty years
later, there were 66,000 miles. Of these, 50 percent were in North America and 47 percent were in Europe. The
rail lines built during that period served populated areas where considerable economic activity already existed,
9
yet a global ideology of railroads gradually emerged: the belief that railroads could populate and bring wealth to
undeveloped regions.
In Britain and the United States, private companies built hundreds of uncoordinated rail projects, but in
continental Europe railroad construction became a concern of the state, which provided overall control and a
large share of capital. Until 1914, the building of railroads remained the most important reason for the export of
capital as well as the main method of developing new territories. British capital financed the majority of the
railroads built in India, Canada, and Latin America. The U.S. transcontinental railroad played a key role in
populating and developing huge tracts of land in North America, as did the Trans-Siberian Railway in Asia.
In the course of the nineteenth century, around 9 million square miles of land were settled in North and
South America and Oceania. This was made possible by the decline in transportation costs, which greatly
extended the area from which bulky products such as grains and minerals could be marketed. The introduction
of refrigeration on railcars and steamers in the 1870s opened huge markets for meat, dairy products, and fruit
in North America and Europe. The 1870s also saw the adoption of steel rails, electric signals, compressed-air
brakes, and other inventions that made railroads a leading source of technical innovation in the nineteenth
century,
In the world context, the rise of the railroad was inseparable from that of the steamship. The economic
and geographic consequences of these two innovations complemented one another. Both had the effect of
increasing the size of markets as well as the amount of economic activity worldwide.
31. Which sentence below best expresses the essential information in the highlighted sentence in paragraph 1?
Incorrect choices change the meaning in important ways or leave out essential information.
A. International trade had to take place near oceans and rivers and did not extend to interior regions.
B. After several centuries of slow growth, the world economy was no longer confined by geography.
C. The effects of economic activity were felt everywhere, but especially along coasts and rivers.
D. World markets expanded rapidly, affecting people who lived hundreds of miles from the coast.
32. The word zenith in paragraph 2 is closest in meaning to
A. final goal
B. slow period
C. natural limit
D. high point
33. What factor led to an increase in canal building?
A. Competition among the world powers
B. The need to move large quantities of coal
C. Improvements in the design of sailing ships
D. An increase in the export of capital
34. The word impetus in paragraph 3 is closest in meaning to
A. push
B. style
C. shock
D. cost
35. Which of the following is given as a reason for the rise of the steamship over the sailing ship?
A.Wood for the construction of sailing ships became scarce.
B. The steamship could travel at faster speeds than the sailing ship.
C. Steamships were better than sailing ships at navigating canals.
D.Technical advances made the steamship require less coal.
36. According to the passage, what was a major result of railroad building in the nineteenth century?
A. The majority of wealth became concentrated in a few powerful nations.
B. Competition increased between private and state-owned companies.
C. There was an increase in the demand for an educated workforce.
D. Large parts of the world became populated and economically developed.
37. Why does the author mention refrigeration in paragraph 6?
A. To show how the steam engine contributed to refrigeration.
B. To illustrate the standard of living of North America and Europe.
10
C. To give an example of an innovation that expanded markets.
D. To argue that refrigeration was the most important invention of the time.
38. All of the following were effects of the transportation revolution EXCEPT
A. the spread of trade to new regions
B. innovations in technology
C. population decline in major cities
D. an increase in the size of world markets
39. It can be inferred from the passage that the author most likely believes which of the following about the
transportation revolution of the nineteenth century?
A. There will never again be so many developments in transportation in a single century.
B. Improvements in transportation had a direct impact on world economics.
C. The transportation revolution was part of a worldwide political revolution.
D. Technical innovations of that time led to similar inventions in the next century.
40. Look at the four places A, B, C and D, which indicate where the following sentence could be added to the
passage. Where would the sentence best fit?
Railroad construction required enormous amounts of capital and was financed in diverse ways.
(A) In Britain and the United States, private companies built hundreds of uncoordinated rail projects, but in
continental Europe railroad construction became a concern of the state, which provided overall control and a
large share of capital. (B) Until 1914, the building of railroads remained the most important reason for the
export of capital as well as the main method of developing new territories. (C) British capital financed the
majority of the railroads built in India, Canada, and Latin America. (D) The US. transcontinental railroad played
a key role in populating and developing huge tracts of land in North America, as did the Trans-Siberian Railway
in Asia.
Your answer:

31. 32. 33. 34. 35. 36. 37. 38. 39. 40.

Part 4: You are going to read an extract from an article about the Greek philosopher Socrates. For
questions 41-50, choose from the sections (A-D). The sections may be chosen more than once.

In which section are the following mentioned?

41. relationships between people in Socrates’ time

42. the continuing importance of Socrates’ beliefs

43. the writer’s theory concerning what happened to Socrates

44. why little is known about Socrates as a man

45. How the witer set about getting information relevant to Socrates

46. the difference between common perceptions of Socrates and what he was really like

47. an aim that Socrate was critical of

48. the realization that finding out about Socrates was a difficult task

49. how well known Socrates was during his time

50. an issue that Socrates considered in great detail

11
Seeking Socrates
It may be more than 2,400 years since his death, but the Greek philosopher can still teach us a thing or two
about leading “the good life”. Bettany Hughes digs deeper.

A. Sharing breakfast with an award-winning author in an Edinburgh hotel a few years back, the conversation
came round to what I was writing next. “A book on Socrates” I mumbled through my muesli. “Socrates!” he
exclaimed. “What a brilliant doughnut subject. Really rich and succulent with a great hole in the middle where
the central character should be” I felt my smile fade because, of course, he was right. Socrates, the Greek
philosopher, might be one of the most famous thinkers of all time, but, as far as we know, he wrote not a single
word down. Born in Athens in 469B.C, condemned to death by a democratic Athenian court in 399B.C,
Socrates philosophized freely for close on half a century. Then he was found guilty of corrupting the young and
of disrespecting the city’s traditional gods. His punishment? Lethal hemlock poison in a small prison cell. We
don’t have Socrates’ personal archive; and we don’t even know where he was buried. So, for many, he has
come to seem aloof and nebulous- a daunting intellectual figure-always just out of reach.

B. But that is a crying shame. Put simply, we think the way we do because Socrates thought the way he did. His
famous aphorism, “the unexamined life is not worth living”, is a central tenet for modern times. His
philosophies-24 centuries old- are also remarkably relevant today. Socrates was acutely aware of the dangers of
excess and overindulgence. He berated his peers for a selfish pursuit of material gain. He questioned the value
of going to fight under an ideological the value of going to fight under an ideological banner of “democracy”.
What is the point of city walls, warships and glittering statues, he asked, if we are not happy? The pursuit of
happiness is one of the political pillars of the West. We care entering what has been describes as “an age of
empathy”. So Socrates’forensic, practical investigation of how to lead “the good life” is more illuminating,
more necessary than ever.

C. Rather than being some kind of remote, tunic-clad beardy who wandered around classical columns, Socrates
was a man of the streets. The philosopher tore through Athens like a tornado, drinking, partying, sweating in the
gym as hard as, if not harder than the next man. For him, philosophy was essential to human life. His mission:
to find the best way to live on earth. As Cicero, the Roman author, perceptively put it: “Socrates brought
philosophy down from the skies”. And so to try to put him back on to the streets he loved and where his
philosophy belonged, I have spent 10 years investigating the eastern Mediterranean landscape to find clues of
his life and the “Golden Age of Athens”. Using the latest archaeology, newly discovered historical sources, and
the accounts of his key followers, Plato and Xenophone, I have endeavoured to create a Socrates-shaped space,
in the glittering city of 500BC Athens- ready for the philosopher to inhabit.

D. The street jargon used to describe the Athens of Socrates’ day gives us a sense of its character. His
hometown was known as “sleek”, “oily”, “violet-crowned”, “busybody” Athens. Lead curse tablets left in
drains, scribbled down by those in the world’s first true democracy, show that however progressive fifth-
century Athenians were, their radical political experiment- allowing the demos (the people) to have kratos
(power)- did not do away with personal rivalries and grudges. Far from it. In fact, in the city where every full
citizen was a potent politician, backbiting and cliquery came to take on epic proportions. By the time of his
death Socrates was caught up in this crossfire.

E. His life story is a reminder that the word “democracy” is not a magic wand. It does not automatically
vaporize all ills. This was Socrates’ beef, too- a society can only be good not because of the powerful words it
bandies around, but thanks to the moral backbone of each and every individual within it. But Athenians became
12
greedy, they overreached themselves, and lived to see their city walls torn down by their Spartan enemies, and
their radical democracy democratically voted out of existence. The city state needed someone to blame. High-
profile, maddening, eccentric, freethinking, free-speaking Socrates was a good target. Socrates seems to me to
be democracy’s scapegoat. He was condemned because, in fragile times, anxious political masses want
certainties-not the eternal questions that Socrates asked of the world around him.

Your answer:

41. 42. 43. 44. 45. 46. 47. 48. 49. 50.

WRITING (60 points)

Part 1: Summarize the following passage in within about 100-120 words. (15 pts)

Nanotechnology is a new field of applied science. It is an effort to create very tiny machines on a nano scale. A
nano is a unit of measurement which stands for ten to the negative power of nine. It is used to describe very
small things.

One example of nanotechnology in modern use is the making of polymers. These are based on molecular
structure. Another is the design of computer chip layouts. These are based on surface science.

At the nano-size level, the properties of many materials change. For example, copper changes from opaque to
clear. Solid gold becomes liquid at room temperature. Insulators like silicon become conductors. All of these
activities open up many potential risks.

Due to their altered states, nano particles become more mobile. They are also more likely to react with other
things. There are four ways for nano particles to enter the human body. They can be inhaled, swallowed,
absorbed through the skin, or injected. Once these particles are in the body, they are highly mobile.

In fact, the way these particles react inside living things is still not fully understood. But scientists guess that
these tiny objects could easily overload defensive cells. This would weaken a body's defenses against diseases.
Humans could easily lose control of particles this size. This would lead to mass epidemics that would cause
widespread disease and death.

Another concern about nanotechnology is of the environmental risks. One report details the possible disaster of
the Earth being covered in a gray, sticky substance. This terrible event is attributed to the unrestrained self-
replication of microscopic robots. These robots are called nanobots and are able to control themselves.

Therefore, scientists need to collect much more data before they are allowed to create and release nanobots.
They should be highly regulated by laws that only allow licensed scientists to do safe experiments.

Part 2: The bar chart below gives information about the percentage of the population living in urban
areas in different parts of the world. (15pts)

Summaries the information by selecting and reporting the main features and make comparisons where relevant.

13
100
90
80
70
60
50 1950
40
2007
30
20 2050
10
0
Africa Asia Europe Latin North Oceania
America America
Changes in percentage of population in urban areas

Part 3: Essay writing (30pts)

In the past, shopping was a routine domestic task. Many people nowadays regarded it as a hobby.

To what extent do you think this is a positive trend? Give reasons for your answer and include any relevant
examples from your own knowledge or experience. Write about 350 words.

THE END

14
HỘI CÁC TRƯỜNG THPT CHUYÊN ĐỀ THI CHỌN HỌC SINH GIỎI KHU VỰC
KHU VỰC DH VÀ ĐB BẮC BỘ DUYÊN HẢI VÀ ĐB BẮC BỘ NĂM 2022
MÔN THI: ANH - LỚP: 11
ĐỀ THI ĐỀ XUẤT (Thời gian làm bài 180 phút không kể thời gian giao đề)
THPT Chu Văn An – Hà Nội

I. LISTENING (50 points)


PART 1. (10 points)
You will hear Alice Brown interviewing Professor Robert Atkins about health scares.
For questions 1-5, choose the answer (A, B, C, D) which fits best according to what
you hear.

1. How does Professor Atkins feel about the frequency of health scares in the media?
A. irritated that the media print nonsense
B. reconciled to health scares being a necessary evil
C. resigned to the media misunderstanding science
D. worried that the health scares might be real
2. What is Alice’s attitude to the threat of the lethal diseases?
A. She worries that new ones will occur.
B. She is doubtful that they can be contained.
C. She is concerned that they are spread more easily today.
C. She believes they pose less of danger today.
3. Professor Atkins believes that the concerns people have today arise from _______.
A. a lack of spiritual belief
B. being misled by scientists and doctors
C. bewilderment when their assumptions are challenged
D. worry about how diseases are communicated
4. What do Alice and Professor Atkins agree about when it comes to health scares?
A. some businesses have a vested interest in promoting them
B. some manufacturers see them as the best form of publicity
C. some doctors are at fault for not criticizing them sufficiently
D. some researchers are looking for publicity
5. What worries Professor Atkins about health scares?
A. They could lead to people taking too many pills.

1
B. They are more damaging than real diseases.
C. They might make people disregard potential risks.
D. They are difficult to disprove.
Your answers
1. 2. 3. 4. 5.
PART 2. (10 points)

Listen to the conversation between Sasha and Jim, then complete the True/False exercise
below.

1. Sasha is currently unemployed.


2. Jim understands now why Sasha had problems in her previous apartment.
3. The problem with the television is that Sasha has sometimes forgotten to turn the TV
off.
4. Sasha didn’t know that Jim used the lights in the apartment as an anti-crime measure.
5. Sasha never realized the different cleaner was causing a problem in the shower.
Your answers

1. 2. 3. 4. 5.
PART 3. (10 points)
Listen to a talk about how to prevent a food crisis and answer the questions. Write
NO MORE THAN FOUR WORDS taken from the recording for each answer in the
corresponding numbered boxes provided.
1. What stimulates Africa’s economic boom?
2. What will become heavily dependent on AI in Europe?
3. What have fossil fuels been considered in the Middle East?
4. What will substitute for oil disputes in the future?
5. What will come to the throne by 2039?
Your answers

1. 2. 3. 4. 5.
PART 4. (20 points)
Listen to a radio discussion about the pros and cons of opening a new fast food
restaurant in a small town called Manley and complete the following sentences. Write

2
NO MORE THAN THREE WORDS taken from the recording for each answer in the
corresponding numbered boxes provided.

- The proposed opening of the new Burger World has provoked (1) _______ in the local
community.
- Mr. Brownlea thinks it will harm the young people and the (2) _______.
- He would prefer a new restaurant offering (3) _______.
- He admits that Burger World prices are (4) _______
- He believes that fast food chains are causing the disappearance of (5) _______ around the
world.
- Mrs. Masters argues that preventing the new outlet’s opening in Manley will not (6)
_______ of changing food habit worldwide.
- Mr. Brownlea is worried that the new fast food will increase the problem of (7) _______.
- This will make older residents feel (8) _______.
- Mrs. Masters feels that it will be beneficial for young people to use Burger World as a (9)
_______.
- She believes the new restaurant will boost (10) _______ of the town.
Your answers
1. 2. 3. 4. 5.
6 7 8 9 10
II. GRAMMAR AND LEXICOLOGY (30 points)
1. Choose the best answer. (20 points)
1. The athlete was _______ in defeat and praised his opponent's skill, for which he received
a lot of praise.
A. abrasive B. valiant C. chivalrous D. magnanimous
2. I wish you’d stop _________ with that watch. It needs to be repaired by a professional
A. dabbling B. striving C. tinkering D. preserving
3. His driving is so bad that I always turn _______ when I am sitting with him in the car.
A. around the bend B. in a fog
C. off his trolley D. green around the gills
4. He _____ denied the accusations, saying they were totally false
A. excessively B. superficially C. strenuously D. unstintingly
5. The runner was far ahead for most of the race, but at the end she won only _______.
A. larger than life B. on the dot C. by a whisker D. a notch above

3
6. She’s raising two kids, holding down a full-time job, and trying to maintain some
_______ of a personal life.
A. semblance B. vestige C. inkling D. portent
7. Negotiations went _______, but we did manage to reach an agreement on the contract by
the deadline.
A. down to the short strokes B. down for the count
C. down to the ground D. down to the wire
8. Given the competitiveness of the National Contest, I have to knock it out of the _______
or I won’t be able to win any prizes.
A. park B. nail C. roof D. ring
9. She was extremely elegant and obviously very _________
A. entitled B. upheld C. upstanding D. well-heeled
10. I know it’s hard at first when you start a new job, but you’ll soon ______.
A. show your teeth B. find your feet C. follow your nose D. try your hand
11. The preparations ____________ by the time the guests ____________.
A. bad been finished/ arrived B. have finished I arrived
C. had finished/ were arriving D. have been finished/ were arrived
12. ____________, playing music is an effective way for them to open their heart to the
outside world.
A. Being visually impaired people B. Such were their visual impairments
C. Having been visually impaired D. For those with visual impairments
13. One day, ____________, she announced that she was leaving.
A. in the red B. in the pink C. over the moon D. out of the blue
14. John was out of his ______________ in the advanced class, so he moved to the
intermediate class.
A. class B. depth C. league D. head
15. One of the most important things you have to remember when you join this game is that
you should ____________.
A. go to your head B. keep your wits about you
C. keep your head in the clouds D. gather your wits
16. Sailing is not ____________ an art.

4
A. more science than B. so much a science as C. as scientific as
D. worth science for
17. ____________ you cut down your carbohydrate intake, you ___ weight by now.
A. Had I would have lost B. Were/ would have lost C. If/ will Jose
D. Did/ would
18. It's very important that we ____________ as soon as there’s any change in the
patient’s condition.
A. be notified B. being notified C. are notified · D. were notified
19. There are ____________ words in English having more than one meaning. Pay close
attention to this fact.
A. a large many B. quite many C. a great many D. quite a lot
20. ____________ motivate learning is well documented.
A. That is computers B. Computers that C. That computers D. It is those
computers
Your answers
1. 2. 3. 4. 5.
6. 7. 8. 9. 10.
11. 12. 13. 14. 15.
16. 17. 18. 19. 20.

2. Complete these sentences, using the suitable form of the given words in brackets.
Write your answers in the corresponding numbered boxes provided. (10 points)
1. Unfortunately the game was cancelled because of a __________ pitch. (WATER)
2. People in coastal areas live mainly on the _________________, which allows them to
earn a great deal of money from the sea products. (CULTURE)
3. She made a _________________ return to the stage after several years working in
television. (TRIUMPH)
4. Antiseptics and __________ are widely used in hospital to kill the bacteria. (INFECT)
5. Thanks to the _________________ policy, more and more forests have been formed on
locations that used to be treeless. (FOREST)

5
6. Both are _______, extravagant in character, highly lyrical and immediately establish the
soloist as a romantic protagonist. (CLAIM)
7. We might have to employ someone temporarily as a _______ measure until we can hire
someone permanently. (STOP)
8. Upholders of the scientific faith shudder at the implications of having to mix it with such
_______ subjective and impure elements. (REDEEM)
9. _______ should summarize the main facts about their performance based on the staff
evaluation criteria of the company. (APPRAISE)
10. It encouraged experimenters to propose ________ or novel approaches to problems.
(BEAT)
Your answers
1. 2. 3. 4. 5.
6. 7. 8. 9. 10.

III. READING (60 points)


Part 1: In the passage below, seven paragraphs have been removed. Choose from
paragraphs (A-H) the one which fits each gap (1-7). There is one extra paragraph you
do not need to use. Write your answers in the corresponding numbered boxes
provided. (7 points)
The white and silver buildings of the VLT or Very Large Telescope at the ESO (European
Organisation for Astronomical research in the Southern Hemisphere) stand tall and
imposing on top of a mountain shining brightly under the desert sun. At night they come
alive, the outer walls open up and silently slide through 360 degrees, allowing an
uninterrupted view of the Southern Hemisphere sky. Inside, a giant eye looks deep into the
stars and beyond, looking for life, mysteries and making sense out of the darkness.
1.
Getting to the place is a journey of discovery in itself. After leaving the Pacific Ocean, you
head south and hook up with the Panamericana Highway and into the desert, the driest on
Earth. It's a two-hour drive to the observatory, but it feels longer as the harsh light, the
rocky, dusty desert, the complete absence of any form of life, except for the giant trucks
plying their trade along the highway, is unsettling.
6
2.
The reason, we are told, is simple. Astronomers need a clear view of the sky at night.
Optimal conditions are to be found in deserts; there are fewer of the negative factors like
light pollution that can make the four telescopes which form VLT work less effectively.
Here no lights are allowed after dark, all windows are screened, and even the main
residence where 108 people sleep, leaks no more than 40W when the lights are turned on.
3.
These monsters and their smaller auxiliaries bring to mind the set of Star Wars. But at
sunset, they take on another look. This could be Stonehenge, another magic circle where
our forefathers tried to make sense of the stars. Soon after arriving, we are taken to see the
inside of one of the telescopes. They have all been named in the indigenous Mapuche
language following a competition among Chilean schoolchildren.
4.
Once these have been completed, the telescope is handed over to the team who will operate
one or more of the telescopes from a control room. All night long these giants will be
moving and pointing to the sky helping the scientists unravel new problems. Fourteen
countries contribute around 160 million euros to their joint astronomical cause, and Paranal
is allocated 20% of that figure. Standing in the shadow of the VLT, one wonders what all
this taxpayers' money buys.
5.
They all answered with the usual 'finding out where we came from, where we are going, are
we alone in the universe...?' And, as the ESO has no commercial use and is a not-for-profit
organization, it's easy to imagine these scientists indulging in their research and being cut
off from reality.
6.
One of Paranal's great achievements was the discovery of a planet outside our solar system.
It is huge: five times bigger than Jupiter, and the work being done now is aimed at
understanding the physical and chemical composition of this, and other, giant Earth-like
planets. Truly a quest for life in outer space.
7.

7
'We needed even sharper images to settle the issue of whether any other configuration is
possible and we counted on the ESO VLT to provide those,’ says Reinhard Genzel, director
at the Max-Planck Institute for Extraterrestrial Physics. 'Now the era of observational
physics has truly begun.'

Missing paragraphs
A But talking further revealed a simple truth: that having pretty much discovered all there
is to know about our world on the Earth, astronomy looks at the vast Terra Incognita
which surrounds us. These scientists see themselves very much as a mixture of
Renaissance men and women: all questing for further knowledge.
B Every evening an engineer is assigned to one of these telescopes and his or her job is to
get it ready so that it can then be taken over at night by a three-person team. The
engineer runs through a series of tests in preparation for the work which will be done
later that night.
C The central unit inside weighs 450 tonnes and houses the main 8.5 m mirror. A second,
smaller mirror is made from beryllium, a rare metal. The external walls can all slide
open to allow the telescope to point in any direction as it rotates soundlessly on its base.
D As we drive further into the desert, the road starts to rise gradually, with hills and steep
valleys all around us. The environment is harsh in the extreme and it's hard to imagine
that a community of European scientists have chosen this place to establish a world-
leading laboratory.
E It is a question that many in Paranal find a little difficult to answer. Maybe because
scientists, due to the nature of their research and also, maybe, their mindsets, tend to
focus on very specific areas of competence and therefore are not required to have a
broader 'strategic' view.
F The other big consideration in the desert is the absence of cloud cover and, higher up,
the lack of atmospheric dust and all the other interference caused by humans or nature
which contributes to partially hiding the secrets of the universe. 'Twinkle, twinkle little
star' is just what astronomers do not want to hear, as this means there is debris between
the eye, the telescopes and the stars.
G We are in Paranal, in Chile's Atacama Desert, where at 2,600 m above sea level, Europe

8
has its most advanced astronomical observatory. It's a leading site, a joint undertaking
by fourteen European countries focused on developing the most advanced scientific
tools for observing the universe and enhancing the knowledge base for industry,
H education and culture.
Astronomers have also used the data from VLT for another purpose - to attempt to find
out how old the universe is. It seems that the oldest star is 13.2 billion years old, which
means the universe must be even older. They also use VLT to look into galaxies beyond
ours, and where they continue to find evidence of supermassive black holes, where all
kinds of violent activity occur.
Your answers
1. 2. 3. 4. 5. 6. 7.

Part 2. Complete the following passage by filling each blank with ONE suitable word.
Write your answers in the corresponding numbered boxes provided. (15 points)
Since the World Health Organization (WHO) declared the COVID-19 outbreak a
pandemic back in March 2020, the virus has claimed more than 2.5 million lives globally
with upwards of 113 million cases being (1) by laboratory tests (March 2021).
The pandemic has impacted almost every corner of life, causing global economies to
stall, changing the way we work and interact (2) our loved ones, and stretching healthcare
(3) to the limit. Governments around the world have been forced to implement harsh
restrictions (4) human activity to curb the spread of the virus.
COVID-19 vaccination is now offering a way to transition out of this phase of the
pandemic. Without them, many scientists believe that natural herd (5) would not have been
sufficient to restore society to its normal (6) quo and that it would have resulted in extreme
fatality. This is something that has been echoed by many health (7) including the WHO. In
a scenario (8) access to vaccines, strict behavioral measures may have had to remain for the
foreseeable future.
Fortunately, the beginning of 2021 saw numerous vaccines given emergency (9) and begin
their roll out in countries across the world. As of March 2021, just shy of 300 million
vaccine doses had been administered worldwide. The figures give (10) of a return to

9
‘normal’. However, global COVID-19 vaccination faces several challenges which may
impact its success.
Your answers
1. 2. 3. 4. 5.
6. 7. 8. 9. 10.

Part 3. Read the passage below and answers the questions (10 points)
The presence or absence of water has a direct bearing on the possibility of life on other
planets. In the nineteenth century, it was commonly accepted that life, perhaps even
intelligent life, was widespread in the solar system, and Mars was an obvious target in the
search for life. New photographic technology offered a way for astronomers to learn more
about the red planet. In 1888, Italian astronomer Giovanni Schiaparelli produced images
that showed a network of long, thin, dark lines crossing the surface of Mars. He called these
features canali in Italian, which became “canals” or “channels” in English. The strange
appearance of the canals suggested to some scientists that they had been formed artificially
rather than naturally. The mystery deepened when Schiaparelli observed that many of the
canals in the photographs were actually double.
Other photographic images of Mars revealed its seasonally changing polar ice caps
and features that appeared to be ancient islands located in what was now a dry streambed.
When the islands were first discovered, some scientists speculated that a thick water-laden
atmosphere capable of generating heavy rains and had once existed on Mars. However,
others remained unconvinced of the presence of water. Then, in 1963, a team of
astronomers obtained a good photographic plate of the near-infrared spectrum of Mars. The
photograph showed that, faintly but definitely, water vapor lines could be seen. This
photograph established that there really was water on Mars, though the amount was very
small. Today, the presence of water vapor in the Martian atmosphere is generally accepted,
as is the belief that the atmosphere was once much denser than it is now, with a much
greater abundance of water vapor.
The surface of Mars is dry today, but it does contain significant amounts of ice and
signs that liquid water once flowed over the planet. All of the locations where evidence of
water has been found are ancient, probably formed every early in Martian history. Data

10
transmitted from spacecraft on Mars in 2004 have revealed that water was once common
across a vast region of the planet, possibly as shallow lakes or seas that dried out and then
filled up again. There are signs that the wind blew debris around during dry stages. These
seas and lakes extended across hundreds of thousands of square miles, creating habitable
conditions during long stretches of time billions of years ago.
Evidence of water includes the presence of various minerals known as evaporates,
deposits left behind when liquid water turns to vapor. Small areas of mineral deposits have
been found in Valles Marineris, a huge hole on Mars that is larger than the Grand Canyon
on Earth. The minerals there contain water, so they had to be formed in the presence of
water. Geologic research has also turned up clay and gypsum deposits that were formed by
water in the soil. Rocks that clearly formed in water extend throughout 300 meters of
layered materials in several locations across the Martian plains. The layers were built up
over time, which means water was present, at least temporarily, for extended periods on
ancient Mars.
Besides the ice packs at Mars’s poles, astronomers have discovered a frozen sea near
its equator. This frozen sea is the size of the North Sea on Earth and appears similar to the
ice packs on Antarctica. Scientists have also detected evidence of lava flows 20 million
years ago as well as signs that some volcanoes may still be active. Several recently formed
volcanic cones near Mars’s North Pole indicate that the planet’s core may interact with the
surface, meaning there was both warmth and moisture in the recent past – circumstances
that might have supported life.
Liquid water is the key ingredient for life as we know it. Of all the other planets in the
solar system, Mars is most like Earth. In 20111 a team of researchers used computer
modeling to compare data on temperature and pressure conditions on Earth with those on
Mars to estimate how much of Mars could support Earth- like organisms. Their results
showed that three percent of Mars could sustain life, although most of these regions are
underground. Below the planet’s surface, conditions are right for water to exist as a liquid.
Additional evidence of water on the planet’s surface came in 2012, when a robot landed on
Mars and transmitted hig-resolution images showing a streambed with coarse gravel that
had likely been deposited by flowing water. All of this evidence of water does not

11
necessarily mean life ever emerged there; however, it does suggest that Mars meets all the
requirements that are needed for life to exist.

1. The word ‘target’ in paragraph 1 is closest in meaning to ______.


A. watery planet B. symbol of strength C. missing link D. object of
interest
2. According to the passage, what has been a major focus of research about Mars?
A. How Mars compares to other planets in the solar system
B. Who built the network of canals on the surface of Mars
C. Whether signs of water indicate that life has existed on Mars
D. How soon astronauts from Earth will be able to go to Mars
3. Astronomers of the 19th and 20th centuries studied Mars mainly through ______.
A. ancient writings B. photographic images
C. Martian soil samples D. data sent by spacecraft
4. It can be inferred from paragraph 1 that Schiaparelli’s observation of canals on Mars
led to ______.
A. direct proof that life has existed on Mars
B. the rejection of Schiaparelli’s ideas by other scientists
C. the search for canals on other planets in the solar system
D. new questions about intelligent life on Mars
5. What discovery led some scientists to think that the Martian atmosphere had
produced heavy rains in the past?
A. A network of canals on the surface
B. Ancient islands in a dry streambed
C. Water vapor lines on a photographic plate
D. Volcanic cones near the planet’s North Pole
6. Which sentence below best expresses the essential information in the highlighted
sentence in paragraph 2?
A. Most scientists believe there is water vapor in the Martian atmosphere, which is
now less dense than it was in the past.

12
B. The amount of water vapor in the Martian atmosphere has changed many times in
the past, and scientists generally accept this.
C. The atmosphere of Mars used to contain only water vapor, but now scientists
know that several other gases are also present.
D. Scientists used to believe that Mars had no atmosphere, but now most think it has
a very dense atmosphere of water vapor.
7. Layers of rock in several places on the Martian plains are evidence that ______.
A. Mars was formed at the same time as Earth
B. both wind and water erosion occurred there
C. water was present there for a long time
D. liquid water is no longer present on Mars
8. All of the following indicate the presence of water on Mars EXCEPT ______.
A. images of polar ice caps B. a 1963 photograph
C. clay and gypsum deposits D. evidence of lava flows
9. Write the correct letter [A], [B], [C] or [D] that indicates where the following
sentence could be added to the passage?
Astronomers already knew that Mars had some kind of atmosphere because of the
occasional presence of bright features that looked like clouds.
The presence or absence of water has a direct bearing on the possibility of life on
other planets. [A] In the nineteenth century, it was commonly accepted that life, perhaps
even intelligent life, was widespread in the solar system, and Mars was an obvious target
in the search for life. [B] New photographic technology offered a way for astronomers
to learn more about the red planet. In 1888, Italian astronomer Giovanni Schiaparelli
produced images that showed a network of long, thin, dark lines crossing the surface of
Mars. [C] He called these features canali in Italian, which became “canals” or
“channels” in English. The strange appearance of the canals suggested to some scientists
that they had been formed artificially rather than naturally. [D] The mystery deepened
when Schiaparelli observed that many of the canals in the photographs were actually
double.

13
10. An introductory sentence for a brief summary of the passage is provided below.
Complete the summary by selecting the THREE answer choices that express the
most important ideas in the passage.
Scientists have long searched for evidence of water on Mars
-
-
-

A. Early photographs of Mars showed what appeared to be canals, polar ice caps,
and ancient islands.
B. Dry streambed, lakes, and seas suggest that Mars does not have enough water to
support life.
C. There is water vapor in the Martian atmosphere and evidence that liquid water
once flowed on the surface.
D. Mars contains many types of evaporates, minerals left behind when liquid water
becomes vapor.
E. Geologic research, photography and computer modeling provide evidence of
water – and possibly life – on Mars.
F. Because Mars is so similar to Earth, scientists believe that humans will be able to
live on Mars in the future.

Your answers:
1. 2. 3. 4. 5.
6. 7. 8. 9. 10.

Part 4. Read the passage and do the tasks below (13 points)
Section A
• The human community can be regarded as a system, holistic in nature, seeking survival.
Throughout the ages, observers of human behaviour have repeatedly identified four
major patterns or configurations of behaviour. Such holistic sorting of behaviour
patterns has been recorded for at least twenty-five centuries.

14
• In 450 B.C., Hippocrates described four such dispositions he called temperaments - a
choleric temperament with an ease of emotional arousal and sensitivity, a phlegmatic
temperament with cool detachment and impassivity, a melancholic temperament with a
very serious, dour, and downcast nature, and a sanguine temperament full of
impulsivity, excitability, and quick reactivity. During the Middle Ages, Philippus
Paracelsus described four natures whose behaviours were said to be influenced by four
kinds of spirits: nymphs, sylphs, gnomes, and salamanders.
• Most twentieth-century psychologists abandoned holistic observation of human
behaviour for a microscopic examination of parts, fragments, traits, and so on. To them,
all human beings were basically alike, and individual differences were due to chance or
conditioning.
• Two German psychologists, Ernst Kretschmer and Eduard Spranger, were among the
few to continue to view individuals holistically in terms of patterns. Inspired by their
work, a modern psychologist, David Keirsey, noted common themes in the various
observations and the consistent tendency of human behavious to sort itself into four
similar patterns. Linda Berens continues to expand our understanding of the four
temperaments through the unique contributions, including the core needs, values,
talents, and behaviours of the four temperament patterns – as illustrated by The
Temperament Targets.
• These four major patterns are referred to as temperaments. They describe the ways
human personality interacts with the environment to satisfy its needs. Each of the four
types of humours corresponded to a different personality type.
Section B
• The Sanguine temperament personality is fairly extroverted. People of a sanguine
temperament tend to enjoy social gatherings and making new friends. They tend to be
creative and often day dream. However, some alone time is crucial for those of this
temperament. Sanguine can also mean very sensitive, compassionate and thoughtful.
Sanguine personalities generally struggle with the following tasks all the way through.
They are chronically late, and tend to be forgetful and sometimes a little sarcastic.
Often, when pursuing a new hobby, interest is lost quickly when it ceases to be
engaging or fun. A sanguine person is happy, friendly, warm, eager and has an ability to

15
sympathize with others. He has lots of friends and everyone thinks he is a fun company.
But he is often very self-centred and he lacks self-control. He has a tendency to
exaggerate and he is emotionally unstable.
Section C
• A person who is choleric is a doer. They have a lot of ambition, energy and passion, and
try to instill it in others. They can dominate people of other temperaments, especially
phlegmatic types. Many great charismatic military and political figures were cholerics.
They have a strong will. They are independent, practical, efficient and productive.
When they get something in their mind, nothing can stop them. Unfortunately, choleric
people have to spend a lot of their time alone, as their social side is so poorly-
developed. They scare people away with their coldness and their angry and cruel words.
They are insensitive and overly dominating.
Section D
• A person who is a thoughtful ponder has a melancholic disposition. Often very kind and
considerate, melancholies can be highly creative – as in poetry and art – and can
become occupied with the tragedy and cruelty in the world. A melancholic is also often
a perfectionist. They are often self-reliant and independent. A melancholic person is
often extremely talented. He can analyse things very deeply. He has an eye for beauty.
He is disciplined and diligent. But he has a whole load of dark sides. He is often
pessimistic and is depressed easily. He is too critical of himself and of others. He is
revengeful and easily offended.
Section E
• Phlegmatics tend to be self-content and kind. They can be very accepting and
affectionate. They may be very receptive and shy and often prefer stability to
uncertainty and change. They are very consistent, relaxed, rational, curious, and
observant, making them good administrators and diplomats. Unlike the Sanguine
personality, they may be more dependable.
• It’s easiest to get along with a phlegmatic person. He is calm and easygoing. His
carefree attitude and good sense of humour attract people. He is stable and reliable and
very diplomatic. But he is not perfect, either. He is often irritatingly slow and unable to
make up his mind. He is selfish and stingy. Sometimes he is totally indifferent to

16
everything around him. All he’s interested in is his daydreams.
Section F
• The concept of temperament can generally be defined as a behavioural or emotional
trait that differs across individuals, appears early in life, is relatively stable over the life-
span, and is, at least to some degree, influenced by biology. One common defining
characteristic of temperament is that it appears quite early in development and is
relatively stable over the life-span. It is for this reason that the study of temperament has
often focused on infancy and early-childhood. The assumption has been that
temperament is not an acquired characteristic; one doesn’t learn one’s temperament,
rather one is simply born with a given temperament profile. Further, these “in-born”
traits persist throughout the life span, though they may change form. For example, an
infant may have a withdrawal oriented temperament, but lacking the physical capacity
to move independently may cry when presented with novel situations or toys or people,
etc. The infant’s crying then elicits a caregiver to either remove the infant from the
stimulus or the stimulus from the infant. In either case it effectively increases the
distance between the infant and that which makes him or her uncomfortable. As the
child ages through early childhood, middle childhood, and adolescence and so on, he or
she no longer needs to rely exclusively on others to control their environment. They can
themselves retreat from situations that make them uncomfortable and/or they may
actively seek out environments that are of low stimulus intensity. In adolescence they
may begin to use depressant drugs such as alcohol, narcotics, or nicotine to effectively
withdraw from situations that make them uncomfortable, but for which there is strong
social pressure to pursue (parties, school functions, etc.) These behaviours are quite
different in form, but yield the same functional effect.
Questions 1-6: The reading passage has six sections, A-F. Choose the correct heading
for each section from the list of headings below. Write the correct number, i-ix, in the
corresponding numbered boxes provided.
List of Headings
I Cautious and caring people
Ii Connection between characteristics and body
Iii In-depth thinking and intelligent people

17
Iv Changing behaviours
V Active and optimistic people
Vi Theories from ancient philosophers
Vii Four personalities on the basis of body fluid
viii Demanding and unsympathetic people
Ix The in-born and permanent temperament

1. Section A
2. Section B
3. Section C
4. Section D
5. Section E
6. Section F
Questions 7-13: Do the following statements agree with the information given in the
reading passage? In the corresponding numbered boxes provided, write:
TRUE if the statement reflects the claims of the writer
FALSE if the statement contradicts the claims of the writer
NOT GIVEN if it is impossible to say what the writer thinks about this
7. The four temperaments can be traced back reliably to philosophy, notably in the work of
Hippocrates.
8. To all twentieth-century psychologists, the personal characteristics came from different
situations.
9. People of a sanguine temperament are imaginative and creative, and are often the ones
who enthusiastically promote new ideas on the job.
10. If someone has a strong will, he or she must be a great charismatic military or political
person.
11. Most melancholies can be poets or artists because they are highly creative.
12. Phlegmatic people are usually indifferent to everything.
13. Temperament is an “in-born” characteristic which is stable throughout one’s life.
Your answers
1. 2. 3. 4. 5.

18
6. 7. 8. 9. 10.
11 12 13

Part 5. (15 points)


You are going to read an extract from an article about paintings. For questions 1–10,
choose from the sections (A–E). The sections may be chosen more than once. Write
your answers in the corresponding numbered boxes provided.
A. Luisa Sutton
A Bar at the Folies-Bergère, by Edouard Manet
Manet was inviting some kind of response in the way in which he presented women in his
work and he succeeded in bridging the gap between classical traditions and painting
modern life. Above all, I have tremendous respect for the fact that he was a breakthrough
artist: a champion of realist modernism who was censured for breaking the mould. Through
the medium of painting, Manet constantly reassessed the prevailing attitudes of the world
he was living in. Today we are used to multiple perspective - seeing the same image from
different angles. This was not so in Manet’s time and in this painting we see him crossing
boundaries as he switches reality by employing a mirror to reflect his subjects.
B. Paul Harris
Henry VII, 29 October 1505, by unknown artist
Visually, this is a stunning portrait; Henry moves towards the viewer from the parapet
wearing the red robes of Lancaster, his hands on the ledge. It is immediately exciting and
emotive. Henry VII was on the lookout for a new bride and this was painted to be sent to
the court of Maximilian, much as we would send a photo today. So the provenance is clear.
Portraits of other English monarchs, Richard III in particular, are, in comparison, stiff and
remote. Henry VII’s portrait speaks in a very particular way. His eyes look at one. He is
Renaissance man but, at the same time one sees a shrewd, wise and wily man who,
throughout his reign, managed to amass the fortune of the Tudor dynasty.
C. Tom Newman
James VI and I, 1618, by Paul Van Somer
I used to work for an art handling company in New York, and I came to realise how
wonderful paintings are as entities. Old paintings last for so long because of the materials

19
used – the oil is so robust, it expands or contracts depending on the heat. They can be rolled
up and taken around the world, they’ll never die. This portrait, in particular, made a huge
impression on me. Works of art often lose their power as soon as they’re placed in a
museum. This painting is where it belongs – in a palace. Subject to who you speak to,
James is either a buffoon or a tactical genius, but in this work he looks so stately. The
painting was clearly commissioned to convey regality – and it worked on me, 400 years
later.
D. Paula Smith
Mr and Mrs Andrews, by Gainsborough
I chose this painting as it has personal relevance for me. I grew up in my grandmother’s
house in London. She was an excellent copyist of Gainsborough. We had copies of all of
his paintings, except for this one, which my grandmother didn’t approve of. I’ve always
found it incredibly beautiful though. The two figures in this wonderful painting have very
enigmatic expressions. What are they up to? What are they thinking? And then what are we
to make of the landscape? It’s an agricultural scene, in the middle of the day, but there are
no agricultural workers anywhere to be seen. Where on earth is everybody? What a strange
atmosphere the place has, a long ago era that will never be recaptured.
E. Lynn D’Anton
An Old Woman Cooking Eggs, 1618, by Velàzquez
What is most striking about this painting is surely its veracity. One gets the feeling that one
is looking into a room in which there are no obstacles to understanding. Nothing comes
between the subject and the observer. The artist here is the perfect observer. When I saw it
a few years ago in the National Gallery of Scotland, set alongside many other works from
Velazquez’s youth, there was no doubt in my mind that it was a masterpiece. I think that it
is easy for many people to empathise with this painting in one way or another.
In which section are the following mentioned?
1. the inscrutable nature of the subjects
2. the artist’s ability to give an insight into temperament
3. the integrity of the image portrayed
4. the view that the artist was an innovator
5. delight in a painting’s ability to endure

20
6. the background to a painting being well documented
7. the view that a painting’s impact depends on its surroundings
8. a painting which gives an image of a lost world
9. admiration for an artist who dared to challenge conventional ideas
10.conflicting opinions about the subject of a painting
Your answers
1. 2. 3. 4. 5.
6. 7. 8. 9. 10.

IV. WRITING (60 points)


Part 1. (15 points)
Read the following articles and summarize the measures recommended in both
articles in order to improve health and performance. You should write a paragraph of
between 80 and 100 words.
Article 1: Discover the healing power of positive thinking
We all know that strong emotions have powerful physical effect. Feeling nervous
before an important interview can send you rushing to the bathroom, while a sudden attack
of anxiety can send your heart racing and leave you feeling faint and dizzy. But new
research has revealed the incredible healing power of the brain and how learning to relax
and think positively can have dramatic health benefits. And there is now overwhelming
evidence that your mental and emotional state can also have a direct impact on your body’s
ability to fight disease and cope with pain.
Bob Lewin, Professor of Rehabilitation at York University, took a group of heart
patients through an eight-week angina management, relaxation techniques, goal-setting,
yoga and exercise. The results were staggering. Fifty per cent of the patients who had been
on waiting list for bypass surgery were taken off by their cardiologists who decided that
they no longer needed it.
So how do you make it work for yourself? Well, it’s far more complex than just
learning to look on the bright side. The key variable in patients getting well is the extent to
which they feel in control of their own emotions. Reorganizing your life and learning self-
help techniques can help put you back in control of these.

21
Article 2: How to stay cool even when you’re quaking
We all carry round a baggage of attitudes and beliefs that colours our response to new
situations. If you’re lucky, these will be “can-do” messages, but many of us are
programmed for failure. Perhaps every time you stepped out of the door when you were
little, your parents cried “Be careful!”, as if doom and disaster lurked at every turn, or
friends say, “I wouldn’t attempt that if I were you!”, if you hear negative statements often
enough, you learn to expect the worst. The immediate reaction to a new or daunting
situation is “I can’t handle it”.
“Most people’s confidence is a level or two below their competency.” Says clinical
psychologist Averil Leimon, director of a company which helps personnel transform their
behaviour. “People need to understand that they really are better than they believe.”
Everybody feels fearful in unfamiliar situations that doesn’t mean we should avoid
them. Taking risks, even tiny ones like picking up a telephone to make a complaint, is a
necessary part of accepting adult responsibility. The best strategy you can adopt is to
understand why you feel so fearful and learn how to deal with it, then, when you succeed in
a difficult situation, you’ll feel more confident about approaching it next time around.
The people you admire for their apparent confidence and ability to cope with any
situation are probably feeling just as daunted as you would be, but they don’t let it stand in
their way.
Your summary:
.............................................................................................................................................
.............................................................................................................................................
.............................................................................................................................................

Part 2. (15 points)


The line graph below shows changes in the amount of coffee exported from three
countries between 2002 and 2012. Summarize the information by selecting and
reporting the main features and make comparisons where relevant.

22
.............................................................................................................................................
.............................................................................................................................................
.............................................................................................................................................

Part 3. (30 points)


In many countries today insufficient respect is shown to older people. What do you
think may be the reasons for this? What problems might this cause in society?
Give reasons for your answer and include any relevant examples from your own
knowledge or experience.
Write no more than 350 words.

.............................................................................................................................................
.............................................................................................................................................
.............................................................................................................................................

THE END
Người ra đề

Phương Nhân (0936868540)

23
HỘI CÁC TRƯƠNG THPT CHUYÊN KÌ THI HỌC SINH GIỎI NĂM 2022
VÙNG DUYÊN HẢI VÀ ĐÔNG BẮC MÔN: TIẾNG ANH LỚP 11
BẮC BỘ Thời gian: 180 phút
TRƯỜNG THPT CHUYÊN HÙNG Đề thi gồm 18 trang
VƯƠNG
--------------------
ĐỀ THI ĐỀ XUẤT

SECTION I. LISTENING (50pts)

HƯỚNG DẪN PHẦN THI NGHE HIỂU

 Bài nghe gồm 4 phần; mỗi phần được nghe 2 lần, mỗi lần cách nhau 05 giây; mở đầu
và kết thúc mỗi phần nghe có tín hiệu. Thí sinh có 20 giây để đọc mỗi phần câu hỏi.

 Mở đầu và kết thúc bài nghe có tín hiệu nhạc. Thí sinh có 03 phút để hoàn chỉnh bài
trước tín hiệu nhạc kết thúc bài nghe.

 Mọi hướng dẫn cho thí sinh (bằng tiếng Anh) đã có trong bài nghe.

Part 1: For questions 1-5, listen to a talk about private education and decide whether
these statements are True (T), False (F), or Not Given (NG). Write your answers in the
corresponding numbered boxes provided. (10pts)
1. Enrollment in private primary schools has registered greater increase compared with

that in private secondary schools over the last 15 years.


2. The demand for private schools stems from profound socio-economic changes.

3. Private education plays an important role in solving illiteracy in several massive states
in Pakistan.

4. Inclusivity is one noticeable factor that many private schools lack.


5. High levels of tuition fee in the private sector are understandable considering the high
quality of education that private schools offer.

1. 2. 3. 4. 5.

Part 2: For questions 6-10, listen to a talk about Neptune and answer the questions.
Write NO MORE THAN FOUR WORDS taken from the recording for each answer in
the corresponding numbered boxes provided. (10pts)
6. What is the distance between Neptune and the Sun?
7. What is Neptune’s core made up of besides water ice?
8. What gives Neptune blue color?
9. What can strong winds recorded on Neptune do?
10. What is the name of the spacecraft that has visited Neptune?

6. 7. 8. 9. 10.

Part 3: For questions 11-15, listen to a radio interview in which two academics called
John Farrendale and Lois Granger, taking part in a discussion on the subject of
attitudes to work and choose the answer (A, B, C or D) which fits best according to what
you hear. Write your answers in the corresponding numbered boxes provided. (10pts)
11. Lois agrees with John's point that
A most people dread the prospect of unemployment.

B the psychological effects of unemployment can be overstated.


C some people are better equipped to deal with unemployment than others.

D problems arise when unemployment coincides with other traumatic events.


12. Lois agrees with the listener who suggested that

A work is only one aspect of a fulfilling life.


B voluntary work may be more rewarding than paid work.

C not everybody can expect a high level of job satisfaction.


D people should prepare for redundancy as they would for retirement.

13. What is John's attitude towards people who see work as a 'means to an end'?
A He doubts their level of commitment to the job.
B He accepts that they have made a valid choice.

C He fears it will lead to difficulties for them later.


D He feels they may be missing out on something important.

14. When asked about so-called 'slackers' at work, John points out that
A they accept the notion that work is a necessary evil.

B people often jump to unfair conclusions about them.


C their views are unacceptable in a free labour market.

D such an attitude has become increasingly unacceptable.


15. Lois quotes the psychologist Freud in order to

A show how intellectual ideas have shifted over time.


B provide a contrast to the ideas of Bertrand Russell.
C question the idea that a desire to work is a natural thing.
D lend weight to John's ideas about increased social mobility

11. 12. 13. 14. 15.

Part 4: For questions 16-25, listen to a recording about a medical robot named Grace
and complete the summary below using words taken from the recording. Write NO
MORE THAN THREE WORDS in each blank. (20pts)
- Both Grace and her sister – the well-known (16) __________ Sophia – was created by a
company called Hanson Robotics.
- Grace is only capable of determining your (17) __________ and also your temperature with
a(n) (18) __________.
- In terms of mental health treatment, Grace can socially stimulate patients’ mood, entertain
them or do (19) __________.

- According to David Hanson Grace’s (20) ___________, which gives her the ability to
socially interact by making natural engagement easier, was designed with the goal of
preventing (21) __________ from being overwhelmed.
- Grace is the result of a(n) (22) __________ between Hanson Robotics and another company
called Singularity Studio.

- Once Grace is mass produced, production costs – currently (23) ___________ the prices of
luxury cars – will begin to drop.
- In July or August, the beta version of Grace will be produced on the (24) _________ before
testing and deployment, all of which is part of the (25) ___________.

1. 2. 3. 4. 5.
6. 7. 8. 9. 10.

SECTION II. GRAMMAR & VOCABULARY (30pts)

Part 1: Choose the correct answer A, B, C or D to each of the following questions and
write your answers in the corresponding numbered boxes provided. (20 pts)
1. Rescuers cast a _____ to the drowning man and hauled him out of the sea.
A. lifeline B. lifeboat C. lifeguard D. lifesaver
2. Although citizen-centred schemes involve residents in debates, full political _____ is kept
by local councillors and MPs.
A. autonomy B. autarchy C. autocracy D. authority
3. He answered the teacher’s question so quickly that it seemed as though he had _____ the
answer out of the air.
A. grabbed B. snatched C. seized D. plucked
4. This curtain material _____ easily.
A. hangs itself B. makes itself C. creases D. bend
5. There is no need to get so _____ about being turned down. There are other advertising
agencies out there, you know.
A. destitute B. descendant C. despondent D. despicable
6. He likes nothing better than to spend his Sunday mornings _____ in the gardens.
A. pottering about B. hanging around C. whiling away D. winding down
7. Having lost her home, Lucy got _____ a gang of people who hang around causing trouble.
A. in with B. up to C. on with D. by on
8. Julie felt unfairly _____ when she spoke out against a company proposal and the entire staff
team turned against her.
A. prosecuted B. persecuted C. oppressed D. suppressed
9. It is impossible to miss the _____ of the Generation X in America.
A. manifestation B. propriety C. depreciation D. coalescence
10. Apart from one or two _____ of brilliance from Owen, England put on a rather poor
performance.
A. spells B. flashes C. storms D. spells
11. The way people store their emotions if more corporeal than _____.
A. telegenic B. asymmetric C. psychogenic D. telepathic
12. His fight to _____ four black men of the rape of a seventeen-year-old white girl two years
ago partially inspired the group to protest.
A. exculpate B. exonerate C. bereave D. misappropriate
13. After the hurricane, all that was left of our house was a pile of _____.
A. rabble B. rubble C. ramble D. rumble
14. Having sacked three employees, the boss was obliged to provide each with _____ pay.
A. retirement B. unemployment C. dismissal D. severance
15. Don’t get yourself _____ up over such a trivial matter
A. done B. worried C. whipped D. worked
16. I _____ with the performances but I got the flu the day before.
A. Was to have helped B. helped C. was to help D. had helped
17. She_____ fainted when she heard her father died.
A. Rather than B. nothing but C. all but D. near
18. Three candidates will be short-listed for the post but we do not know _____.
A. whom B. those C. which D. what ones
19. Stars differ differently from planets _____ they are self-luminous whereas planets shine
by reflected lights.
A. From which B. when C. and D. in that
20. Owning and living in a freestanding house is still a goal of young adults, _____ earlier
generations.
A. as did B. as it was of C. like that of D. so have

1. 2. 3. 4. 5. 6. 7. 8. 9. 10.
11. 12. 13. 14. 15. 16. 17. 18. 19. 20.

Part 2: Use the correct FORM of the word in capitals to fit each gap. Write your answer
in the numbered box. (10 pts)
1. A couple of victories would improve the team's _____ enormously. (MORAL)

2. What the _____ army lacked was not discipline, but numbers, and a coherent strategy.
(SUFFRAGE)

3. “People” is a _____ word. (SYLLABLE)


4. Questions were asked at the eye clinic but these are said to have brought merely a brisk
and _____ response. (OFFICIAL)

5. His _____ gains are all safely stashed away in a Swiss bank. (GET)
6. If a screen does not contain everything one wants, further lexicographic information can be
obtained by clicking on a _____. (LINK)

7. _____ is the transplantation of living cells, tissues or organs from one species to another.
(TRANSPLANTATION)

8. For major grain crops such as wheat, rice, jowar, and bajra, the _____ prices functioned as
the minimum support prices. (CURE)
9. The documented differences between men and women in scientific career paths do not
match what would be expected in a true _____ (MERIT)

10. In Paris, proud _____ never went out of fashion. (INTELLECT)

1. 2. 3. 4. 5.
6. 7. 8. 9. 10.

SECTION III. READING COMPREHENSION (60pts)


Part 1: Read the passage below and fill each of the following numbered spaces with
ONE suitable word. Write your answers in the corresponding boxes provided. (15pts)

Striking Back at Lightning With Lasers


Seldom is the weather more dramatic than when thunderstorms strike. Their electrical fury
inflicts death or serious injury on around 500 people each year in the United States (1) _____.
As the clouds roll in, a leisurely round of golf can become a terrifying dice with death - out in
the (2) _____, a lone golfer may be a lightning bolt’s most inviting target. And (3) _____ is
damage to property too. Lightning damage costs American power companies more than $100
million a year.
(4) _____ researchers in the United States and Japan are planning to hit back. Already in
laboratory trials they have tested strategies for neutralising the (5) _____ of thunderstorms,
and this winter they will brave real storms, equipped with an armoury of lasers that they will
be pointing towards the heavens to discharge thunderclouds before lightning can (6) _____.
The idea of forcing storm clouds to discharge their lightning on command is not new. In the
early 1960s, researchers tried firing rockets trailing wires into thunderclouds to (7) _____ up
an easy discharge path for the huge electric charges that these clouds generate. (8) _____
technique survives to this day at a test site in Florida run by the University of Florida, with
support from the Electrical Power Research Institute (EPRI), based in California. EPRI,
which is funded by power companies, is looking at (9) _____ to protect the United States’
power grid from lightning strikes. “We can cause the lightning to strike where we want it to
using rockets,” says Ralph Bernstein, manager of lightning projects at EPRI. The rocket site
is providing precise measurements of lightning voltages and allowing engineers to check how
(10) _____ equipment bears up.

1. 2. 3. 4. 5.
6. 7. 8. 9. 10.

Part 2: Read the following passage and choose the best answer (A, B, C or D) according
to the text. Write your answers (A, B, C or D) in the corresponding numbered boxes.
(10pts)
Undercover journalism
Journalism is too small or too distant a word to cover it. It is theatre; there are no second
takes. It is drama – it is improvisation, infiltration and psychological warfare. It can be
destructive in itself before any print has seen the light of day. It is exhilarating, dangerous and
stressful. It is the greatest job. It is my job.

I am an undercover reporter. For the past year or so, I have been a football hooligan, a care
worker, a bodyguard and a fashion photographer. It is a strange life and difficult one. In the
course of a day, I have assumed four different personalities, worn four different wardrobes
and spoken four different street dialects, and left a little of me behind in each of those worlds.
More important than this, though, are the experiences and emotions I’ve taken away with me.
It’s hard to put a label on them. They have seeped in and floated out of my psyche, but
somewhere in the backyard of my mind the footprints of this strange work are left behind.
I have as yet no real notion as to what, if any, long-term impact they will have. For the
moment, I relish the shooting gallery of challenges that this madness has offered me. In the
midst of all these acting roles and journalistic expeditions, I have endeavoured not to sacrifice
too much of my real self. I have not gone native and I am still sane. At least for the moment.

In the course of any one investigation, you reveal yourself in conversation and etiquette,
mannerism and delivery – of thousands of gesticulations and millions of words – and cover
yourself with the embroidery of many different disguises. If one stitch is loose or one word
misplaced, then everything could crash, and perhaps violently so.
Certainly, as a covert operator, the journalistic safe line is a difficult one to call. Every word
you utter is precious, every phrase, insinuation and gesture has to be measured and
considered in legal and ethical terms. Even the cadence of your voice has to be set to
appropriate rhythms according to the assumed role, the landscape and the terrain of your
undercover patch.
The golden rule is this: as an undercover reporter you must never be the catalyst for events
that would not otherwise have occurred, had you not been there. The strict guidelines within
broadcasting organisations about covert filming mean that, every time I go into the field, a
BBC committee or compliance officer has to grant permission first. It’s a strange but
necessary experience for someone like me, who operates on instinct and intuition, but it’s a
marriage that works well.

The undercover reporter is a strange breed. There is no blueprint that exists. It is your own
journalistic ethos and within those parameters you try to tread a safe line, both in terms of
your journalism and personal safe-keeping. And of course, there’s a high price you pay for
this kind work, home is now a BBC safe house. The only visitors to my bunker are work
colleagues. It’s not a pleasant lifestyle, but I have taken on all the stories in the full
knowledge of the risks involved.

Though I embarked upon my journey with enthusiasm and determination, the climate in
which we undertake this journalistic and documentary mission is an increasingly hostile one.
It is one in which covert filming has come under scrutiny because of concerns about fakery
and deception and the featuring of hoax witnesses. Issues concerning privacy, the use of
covert filming techniques across the media – from current affairs to the tabloid newspapers –
and the way journalists work with these tools have been rigorously appraised. I personally
welcome this scrutiny.
Hi-tech surveillance equipment allows me to tell the story as it unfolds, surrounded by its
own props, revealing its own scars and naked sinews, and delivered in its own dialect. There
is no distortion and only one editorial prism – mine. While the sophisticated technology
allows a visual and aural presentation of events, mentally I rely on the traditional method of
jotting things down to rationalise my thoughts and gain a coherent picture of all that I was
involved in. This is my delivery system – how I narrate.
Inevitably the spotlight has shone on me but those who have worked on either paper trail
investigation in newspapers or in television will know that it will fade. I am happy to return
to the career of a desk journalist because I recognise that the tools we have used are tools of
last resort. I’ll be returning to the more usual journalistic methods: telephone and computer
notebook rather than secret cameras and hidden microphones. But the aim will be the same:
to shed light into the darker corners of society where the vulnerable are most at risk.
1. Which of the following does the writer NOT suggest about his job in general?

A. Journalism is not truly a word to represent it.


B. Those involved hail from various occupations.
C. It entails people to act in different roles.

D. It has a miscellaneous collection of characteristics.


2. As implied by the author, what distinguishes undercover journalism from regular kinds?

A. the range of subjects it touches on


B. the effects of its destructive power
C. the degree of spontaneity in it
D. the harm that it can cause

3. What does the writer suggest about his attitude towards his job?
A. He has a recollection of most emotions triggered by it.

B. He attaches much of his own personality to the roles he assumes.


C. His interest in the job has been retained.

D. He abominates the ordeals involved in it.


4. What does the writer imply about undercover investigators?

A. They are required to be circumspect so as not to conceal their identities.


B. They have to be cautious in order not to cling to a preplanned set of actions.

C. They should try not to be factors causing changes in events.


D. They need to be observant to the reactions of other people while conducting tasks.
5. The writer suggests that undercover investigators:

A. have to live with the consequences of exposing themselves.


B. resent sticking to rules laid down by their employers.

C. tend to be a similar kind of person.


D. operate according to a similar code of conduct.

6. As indicated by the writer, tribulations arise within the profession because


A. the application of cutting-edge technologies is conducive to attempts to falsify
information.

B. covert filming has been put into question following worries about the effects of
documentaries.
C. deceptive testimonies have emerged to degrade the quality of covert filming.

D. how journalists make use of their equipment has come under scrutiny.
7. Regarding the harsher working environment in his profession, it can be implied that the
author:

A. harbours feelings of repulsion at it.


B. displays embrace of it.

C. finds it rather disconcerting.


D. expresses insouciance towards it.
8. What can be inferred about the method used by the author while working as a reporter?
A. He embellishes the stories with details not clearly reflecting what happened

B. He allows technology to cater for every stage of the process.


C. He uses writing as a way of brainstorming ideas and approaching what he would like to
include.

D. He lets the events speak for themselves with the aid of modern technology.
9. What does the passage suggest about the author’s intentions for the future?

A. He will adopt the more traditional work as a journalist.


B. He will make attempts to return to normalcy after all events.

C. He will have recourse to the tools used when there are no alternatives.
D. He will go to some lengths to direct the limelight away from him.

10. The writer sees the primary aim of journalism as:


A. combating the corruption within the society.

B. enlightening people about the disadvantage of the weak.


C. highlighting causes of present-day issues.
D. unraveling the mystery of criminal cases.

1. 2. 3. 4. 5.
6. 7. 8. 9. 10.

Part 3: Read the following passage and choose the best answer (A, B, C or D) according
to the text. Write your answers (A, B, C or D) in the corresponding numbered boxes. (13
pts)

Company innovation

A. In a scruffy office in midtown Manhattan, a team of 30 artificial-intelligence programmers


is trying to simulate the brains of an eminent sexologist, a well-known dietician, a celebrity
fitness trainer and several other experts. Umagic Systems is a young firm, setting up websites
that will allow clients to consult the virtual versions of these personalities. Subscribers will
feed in details about themselves and their goals; Umagic’s software will come up with the
advice that the star expert would give. Although few people have lost money betting on the
neuroses of the American consumer, Umagic’s prospects are hard to gauge (in ten years’
time, consulting a computer about your sex life might seem natural, or it might seem absurd).
But the company and others like it are beginning to spook large American firms, because they
see such half-barmy “innovative” ideas as the key to their own future success.
B. Innovation has become the buzz-word of American management. Firms have found that
most of the things that can be outsourced or re-engineered have been (worryingly, by their
competitors as well). The stars of American business tend today to be innovators such as
Dell, Amazon and Wal-Mart, which have produced ideas or products that changed their
industries

C. A new book by two consultants from Arthur D. Little records that, over the past 15 years,
the top 20% of firms in an annual innovation poll by Fortune magazine have achieved double
the shareholder returns of their peers. Much of today’s merger boom is driven by a desperate
search for new ideas. So is the fortune now spent on licensing and buying others’ intellectual
property. According to the Pasadena-based Patent & Licence Exchange, trading in intangible
assets in the United States has risen from $15 billion in 1990 to $100 billion in 1998, with an
increasing proportion of the rewards going to small firms and individuals.

D. And therein lies the terror for big companies: that innovation seems to work best outside
them. Several big established “ideas factories”, including 3M, Procter & Gamble and
Rubbermaid, have had dry spells recently. Gillette spent ten years and $1 billion developing
its new Mach 3 razor; it took a British supermarket only a year or so to produce a reasonable
imitation. “In the management of creativity, size is your enemy,” argues Peter Chemin, who
runs the Fox TV and film empire for News Corporation. One person managing 20 movies is
never going to be as involved as one doing five movies. He has thus tried to break down the
studio into smaller units—even at the risk of incurring higher costs.

E. It is easier for ideas to thrive outside big firms these days. In the past, if a clever scientist
had an idea he wanted to commercialise, he would take it first to a big company. Now, with
plenty of cheap venture capital, he is more likely to set up on his own. Umagic has already
raised $5m and is about to raise $25m more. Even in capital-intensive businesses such as
pharmaceuticals, entrepreneurs can conduct early-stage research, selling out to the big firms
when they reach expensive, risky clinical trials. Around a third of drug firms’ total revenue
now comes from licensed-in technology.

F. Some giants, including General Electric and Cisco, have been remarkably successful at
snapping up and integrating scores of small companies. But many others worry about the
prices they have to pay and the difficulty in hanging on to the talent that dreamt up the idea.
Everybody would like to develop more ideas in-house. Procter & Gamble is now shifting its
entire business focus from countries to products; one aim is to get innovations accepted
across the company. Elsewhere, the search for innovation has led to a craze for
“intrapreneurship”—devolving power and setting up internal ideas-factories and tracking
stocks so that talented staff will not leave.

G. Some people think that such restructuring is not enough. In a new book Clayton
Christensen argues that many things which established firms do well, such as looking after
their current customers, can hinder the sort of innovative behaviour needed to deal with
disruptive technologies. Hence the fashion for cannibalisation—setting up businesses that
will actually fight your existing ones. Bank One, for instance, has established Wingspan, an
Internet bank that competes with its real branches (see article). Jack Welch’s Internet
initiative at General Electric is called “Destroyyourbusiness.com”.

H. Nobody could doubt that innovation matters. But need large firms be quite so pessimistic?
A recent survey of the top 50 innovations in America, by Industry Week, a journal, suggested
that ideas are as likely to come from big firms as from small ones. Another skeptical note is
sounded by Amar Bhidé, a colleague of Mr Christensen’s at the Harvard Business School and
the author of another book on entrepreneurship. Rather than having to reinvent themselves,
big companies, he believes, should concentrate on projects with high costs and low
uncertainty, leaving those with low costs and high uncertainty to small entrepreneurs. As
ideas mature and the risks and rewards become more quantifiable, big companies can adopt
them.

I. At Kimberly-Clark, Mr Sanders had to discredit the view that jobs working on new
products were for “those who couldn’t hack it in the real business.” He has tried to change the
culture not just by preaching fuzzy concepts but also by introducing hard incentives, such as
increasing the rewards for those who come up with successful new ideas and, particularly, not
punishing those whose experiments fail. The genesis of one of the firm’s current hits,
Depend, a more dignified incontinence garment, lay in a previous miss, Kotex Personals, a
form of disposable underwear for menstruating women.

J. Will all this creative destruction, cannibalisation and culture tweaking make big firms
more creative? David Post, the founder of Umagic, is sceptical: “The only successful
intrapreneurs are ones who leave and become entrepreneurs.” He also recalls with glee the
looks of total incomprehension when he tried to hawk his “virtual experts” idea three years
ago to the idea labs of firms such as IBM though, as he cheerfully adds, “of course, they
could have been right.” Innovation unlike, apparently, sex, parenting and fitness is one area
where a computer cannot tell you what to do.

Questions 1 – 6. Which section contains the following information? Write the correct
number on the given boxes.
i The unpredictability of the public’s viewpoints about a certain topic in the future
ii A list of certain institutions that are having fewer business activities

iii A type of firms that are resorted to compulsive consumption for new ideas
iv The insatiable thirst for outstanding innovations being an impetus to big impacts on the
market.

v Some moguls which expressed financial concerns when investing in the acquisition of
smaller companies.
vi The reason why American business trends are highlighting the importance of initiatives

vii A company that is able to going through economic falters itself.


viii Small firms that can make certain impacts on bigger ones when the former possesses
more potential ideas.

ix Example of three famous American companies’ innovation


x A type of firms that are regarded fly-by-night when investing in entrepreneurship.

1. Section A 4. Section D

2. Section B 5. Section E

3. Section C 6. Section F

1. 2. 3. 4. 5. 6.

Questions 7-13. In boxes 7-13, write: Y (Yes), N (No), NG (Not Given)


Yes if the statement agrees with the claims of the writer
No if the statement contradicts the claims of the writer
Not Given if it is impossible to say what the writer thinks about this

7. Peter infers his unwillingness to invest more in restructuring his organization in return for
better creativity management.

8. Some small organizations have a craving for ideas that are regarded as an admixture of
“innovative” and “strange”.
9. Umagic is head and shoulders above other competitors in such a new field.

10. A new trend that has already superseded “entrepreneurship” in one area may directly
impact living organizations.
11. Big giants prioritize innovations with low certainty on the understanding that big risks are
parallel to big profits.

12. It takes many years for Mr Sanders to successfully ditch preconceived ideas in his
organization.
13. The author expressed a positive attitude towards the development of innovations at the
end of the passage.

7. 8. 9. 10. 11. 12. 13.

Part 4: In the passage below, seven paragraphs have been removed. For questions 1-7,
read the passage and choose from paragraphs A-H the one which fits each gap. There is
ONE extra paragraph which you do not need to use. Write your answers in the
corresponding numbered boxes provided. (7pts)
How satisfying to pull a chain again. Something went out of British plumbing with the arrival
of the integrated cistern, but everything that went out of British plumbing with the Victorians
has been reinstated in the stateliest form in the bathroom of our suite at the Pool House Hotel.
If only for that achievement alone, it deserves its AA accolade, awarded last Thursday, of
Scottish Hotel of the Year.

1.
In Wester Ross, the old parish of Gairloch - a glorious body of country clasped between the
long sea arms of Loch Torridon and Loch Broom - has all the classic components of the West
Highland landscape. It has the mighty Torridon range, the oldest rock in Britain; the moor-
and-mountain wilderness of Letterewe and the island-studded mirror of Loch Maree - an
inland loch more beautiful than any other, including Loch Lomond. It also has a lonely coast,
sandy bays, leafy glens, Hebridean vistas and numerous whitewashed villages.
2.

The Pool House building on the loch's foreshore, where the River Ewe enters the lake after a
short but vigorous journey from Loch Maree, doesn't look like a traditional Highland lodge.
But scrape away the white paint and roughcast and you will expose pink Torridonian
sandstone - the preferred building blocks of local lairds for three centuries.
3.

Osgood Mackenzie, who caused thin, acid layers of peat on a windswept headland to bloom
with the trees, shrubs and flowers of the temperate world from Chile to Tasmania, lived for a
time in Pool House while he worked obsessively on his horticultural masterpiece. Meanwhile,
his English wife whiled away the hours by carving a chain of Tudor roses in the banisters of
the central staircase.

4.
When the present owners, the Harrison family, made the decision to replace their 13
bedrooms with four themed suites it was Liz Miles who became the creative force. Liz
tracked down the extravagant wallpapers - putto friezes, Michelangelo ceilings, celestial
maps - and sourced most of the antique fittings and furniture.
5.

With some reluctance my husband and I jump ship - forcing ourselves out of the sumptuous
fantasy of Campania, with its 130-year-old cast iron and brass four-poster, to confront the
reality of the weather. As enthusiasts for the elemental challenge of the West Highland
seaboard, we have a busy programme: a rugged walk, a wildlife cruise and, as the softest
option locally available, a visit to Inverewe Garden.
6.

The headland couldn't be more exposed, but the squalls of rain beating in from the Atlantic
sail over our heads en route to the mountains. We flush grouse and snipe from the heather on
our three-hour walk and glimpse red deer. By the time we reach the great sea stacks of Stack
Dubh and Stac Buidhe, there are shafts of sunshine striking the wings of gannets, fulmars and
shags.
7.
We don't. But we do see grey seals, harbour porpoises, great skuas and - quite a spot for a
trainee birdwatcher like me - a huddle of rare, black-throated divers. Warblers and other
songbirds attend our visit to Inverewe Garden, now owned and maintained by the National
Trust for Scotland and not, perhaps, at their best on the cusp between summer and autumn.
But they are still remarkable.
Missing paragraph:

A. The Mackenzie country was dominant in this part of country. Pool House's golden age was
Victorian, when the Highlands became a sporting playground for the gentry. There was
salmon to pull from the River Ewe, deer to stalk and grouse to pot, but for a time the lodge
was home to one of the less predatory Mackenzies: a man who liked to let things grow rather
than cut them down.

B. There was only one willow tree on the promontory where Osgood Mackenzie began his
project in 1862. Now there is a prodigious stand of Scots pine and other native woodland,
planted to supply the windbreak for his exotic trees and shrubs. The contrast is beautiful.
C. Some visitors say they would be willing to pay merely to tour the rooms. Many are
especially fascinated with Diadem, which is modelled on the style of a first-class cabin on the
Titanic. Margaret Harrison's grandfather was a cousin of Captain Smith, the liner's master;
but Peter Harrison, who takes a keen interest in military history, has named all the suites after
warships as a tribute to Pool House's function during the Second World War, when it was the
Navy's headquarters for co-ordinating the North Atlantic and Murmansk convoys.

D. Not all the cottages in these old crofting townships are second homes or self-catering
units. The scattered "capital" of the parish, Gairloch, is something of a boom community,
with energetic young locals raising new houses on scenic building plots. At nearby Poolewe,
which has the botanical curiosity and tourist honeypot of Inverewe Garden, an old shooting
lodge is turning back the clock to find a future.
E. We usually do our own route finding but we want to investigate Rua Reidh Lighthouse,
where Fran Cree and Chris Barrett run residential walking and activity holidays on one of the
most remote headlands of the mainland. Just getting there is an adventure; and the airy nature
of the clifftop paths, with their views to the Outer Hebrides and dizzy drops into empty
beaches, makes us glad of the expert presence of Chris, who is a member of the local
mountain rescue team. "We get called out about a dozen times a year," she tells us.
F. Seldom has washing been such a treat. As I wallow beneath the cascading canopy of the
Shanks Independent Spray Bath (built in Glasgow in 1875 on a scale comparable to the boiler
of a Clyde steamship), I feel a certain kinship with the grey seal idling in the water outside.
From the bathroom window I can see the glassy surface of Loch Ewe - and much of its
wildlife.
G. Our tally of wildlife soars on the sturdy Starquest, which skipper Ian Birks steers to the
wide mouth of Loch Gairloch and the first tugging of the Atlantic. It's the whale-watching
season - the Minch is part of the minke's larder - and it's part of Ian's purpose to monitor their
movements for the Sea Watch Foundation. "But I never advertise these trips as whale-
watching cruises, otherwise people expect to see whales."

H. The carvings are still there, one of the few remnants of the 19th century to survive. How
can this be? From the outside Pool House may look like a made-over inn with 1960s add-ons;
step inside and you enter into the rich, decorative and occasionally camp interior of a
Victorian country house. Yet almost all its finest features, from huge, wood-panelled
bathrooms to marble and polished steel fireplaces, have been retrieved from architectural
salvage yards and put in place over the past three years.

1. 2. 3. 4. 5. 6. 7.

Part 5: You are going to read an article containing reviews of computer games. Choose
from the reviews (A-E). The reviews may be chosen more than once. Write your
answers in the corresponding numbered boxes provided. (15pts)
A. The retreat was a hasty as it was disorganized, and the fall of Galdor was completed. Had
they held out until the bitter end matching their foes blow for blow with conviction and
strength, they could at least have held their heads high, but Ruan knew all semblance of hope
had evaporated with the battle pressure, and that was what worried him the most, for Oldark
had yet to unleash his full wrath upon the people of Appleton. Though overcome with
frustration, Ruan knew in his heart of hearts that he couldn’t hold Redhorn and the others to
account for their wimpishness; after all, in much the same wat as Rome was not built in a
day, nor were farmers reshaped into warriors overnight. Weary, tired and near broken, Ruan
new the task ahead of him was gargantuan, yet, he told himself, again and again, that he must
succeed.
B. At least their numbers were little depleted, he thought. Such had been the haste to flee that
his men had largely escaped unscathed. Perhaps a handful, he estimated, had fallen, and
perhaps six dozen more had been wounded. Then, momentarily, he almost lost all his
composure as the realization hit him hard like a knife delivered heart-bound with deadly
accuracy. Orlach was not amongst the motley assemblage regrouping around him. What fate
had befallen him? His dearest brother – not kin, it’s true, but their bond to succeed bereft of
the courage Orlach imbued in him? He was, he thought, leader in but name only, for it was
Orlach’s fearlessness that had always driven him forth, sustained him and helped him keep
faith despite the faintest of hopes. Without Orlach, all was lost.

C. Ruan collected himself, eyeing its hapless followers and knowing he must deliver a
rousing message of hope with convincing, albeit false conviction. He held him arm aloft and
a hush descended over the gathering crowd. He gestured towards the woods, where his own
precious spouse and the other brave women of Appleton stood ready to defend their children
to the last; safe as yet, but for how long? He knew they would never abandon their post and
their responsibilities so pitifully as he and his men just had. There would beat no hasty retreat
if it came to it, though he hoped it never would, for that would surely be the end, and he and
his brethren would have utterly failed. So he told his weary listeners they were farmhands no
more and he gestured again towards the hidden dwelling in the woods.

D. Meanwhile, the plumes of smoke coming from what had once been their beloved village
were already evident on the distant horizon. Oldark had razed it to the ground. But Appleton
was alive, he told them, in their hearts and souls. And it would be resurrected. Oldark could
not destroy Appleton: not while a single Appletonian heart was yet beating. Indeed, so far, he
had not even come close. He had but peeled away the surface layter. Houses could be rebuilt.
Yes, the first layer of skin was gone, and it felt raw and bitter now, Ruan ceded. But there
were two more layers, were there not? The body of Appleton had not even been pierced the
once as yet. He stared at his men and thundered his words. They were the skin and bones; the
veins and the arteries; the muscle and sinew, he told them. Their brave spouses, the child
protectorate, the vital organs, the beating heart of Appleton. And the soul, what else could it
be? The younglings. It ran and played and skipped and screamed and smiled everyday; it
brought them love and happiness and completion. What else could it be indeed?
E. Suddenly, this hapless band of farmers realized that, really, the battle had barely begun.
What was lost was nothing, and all was yet to fight for. No less cowed than before, from
somewhere deep inside rumbled the warrior soul and their bellies fired with a primeval sense
of purpose. They would stand and fight yet, and do so with every last sinew of strength they
could muster from their bones, and Oldark would be dethroned. His reign of terror would and
as surely as the harshest winter must eventually retreat and give way to spring, and Appleton
would blossom again. And there would be running and playing and skipping and screaming
and smiling once more. Ruan felt Orlach’s strength within him as though he were still there
and his words no longer carried false conviction. He and his brethren were as men possessed,
and they would prevail.

In which extract…

does Ruan almost panic and lose control? 1. .............


is one of Ruan’s close relatives mentioned? 2. .............
is the nature of a defeat described? 3. .............
does the loss of the close friend leave Ruan feeling dejected? 4. .............
does Ruan compare the different genders of his people unfavourably? 5. .............
are children attributed a sort of spiritual significance? 6. .............
do we learn the fate of Appleton village? 7. .............
does the memory of someone give Ruan more resolve? 8. .............
does Ruan liken his people’s suffering to a wound? 9. .............
does Ruan find some relief in the fact that defeat was not heavy in 10. .............
casualities?

SECTION IV. WRITING (60pts)


Part 1: Read the following extract and use your own words to summarize it. Your
summary should be about 140 words. You MUST NOT copy the original. (15pts)
The Nature of Genius

There has always been an interest in geniuses and prodigies. The word ‘genius’, from the
Latin gens (=family) and the term ‘genius’, meaning ‘begetter’, comes from the early Roman
cult of a divinity as the head of the family. In its earliest form, genius was concerned with the
ability of the head of the family, the paterfamilias, to perpetuate himself. Gradually, genius
came to represent a person's characteristics and thence an individual's highest attributes
derived from his ‘genius’ or guiding spirit. Today, people still look to stars or genes,
astrology or genetics, in the hope of finding the source of exceptional abilities or personal
characteristics.

The concept of genius and of gifts has become part of our folk culture, and attitudes are
ambivalent towards them. We envy the gifted and mistrust them. In the mythology of
giftedness, it is popularly believed that if people are talented in one area, they must be
defective in another, that intellectuals are impractical, that prodigies burn too brightly too
soon and burn out, that gifted people are eccentric, that they are physical weaklings, that
there's a thin line between genius and madness, that genius runs in families, that the gifted are
so clever they don't need special help, that giftedness is the same as having a high IQ, that
some races are more intelligent or musical or mathematical than others, that genius goes
unrecognised and unrewarded, that adversity makes men wise or that people with gifts have a
responsibility to use them. Language has been enriched with such terms as ‘highbrow’,
‘egghead’, ‘blue-stocking’, ‘wiseacre’, ‘now-all’, ‘boffin’ and, for many, ‘intellectual’ is a
term of denigration.

The nineteenth century saw considerable interest in the nature of genius, and produced not a
few studies of famous prodigies. Perhaps for us today, two of the most significant aspects of
most of these studies of genius are the frequency with which early encouragement and
teaching by parents and tutors had beneficial effects on the intellectual, artistic or musical
development of the children but caused great difficulties of adjustment later in their lives, and
the frequency with which abilities went unrecognised by teachers and schools. However, the
difficulty with the evidence produced by these studies, fascinating as they are in collecting
together anecdotes and apparent similarities and exceptions, is that they are not what we
would today call norm-referenced. In other words, when, for instance, information is collated
about early illnesses, methods of upbringing, schooling, etc., we must also take into account
information from other historical sources about how common or exceptional these were at the
time. For instance, infant mortality was high and life expectancy much shorter than today,
home tutoring was common in the families of the nobility and wealthy, bullying and corporal
punishment were common at the best independent schools and, for the most part, the cases
studied were members of the privileged classes. It was only with the growth of paediatrics
and psychology in the twentieth century that studies could be carried out on a more objective,
if still not always very scientific, basis.

Part 2: CHART DESCRIPTION (15pts)


The bar chart below shows the percentage of government spending on roads and
transport in 4 countries in the years 1990, 1995, 2000, 2005.

Summarise the information by selecting and reporting the main features, and make
comparisons where relevant.

Percentage of government spending on road and transport (1990 -


2005)
30

25

20
Percentage

1990
15 1995
2000
10
2005
5

0
Italia Portugal UK USA

Part 3: ESSAY WRITING (30pts)


Some people believe that no one should do the same job for all their working life. Others
argue that doing the same job brings advantages for individuals, companies, and
society. Discuss both views and give your opinion.
Give reasons for your answer and include any relevant examples from your own
knowledge and experience. Write at least 250 words.

Người ra đề: Trần Thị Ánh Nguyệt - 0913310100


HỘI CÁC TRƯƠNG THPT CHUYÊN KÌ THI HỌC SINH GIỎI NĂM 2022
VÙNG DUYÊN HẢI VÀ ĐÔNG BẮC MÔN: TIẾNG ANH LỚP 11
BẮC BỘ Thời gian: 180 phút
TRƯỜNG THPT CHUYÊN HÙNG Đề thi gồm 18 trang
VƯƠNG
--------------------
ĐỀ THI ĐỀ XUẤT

SECTION I. LISTENING (50pts)

HƯỚNG DẪN PHẦN THI NGHE HIỂU

 Bài nghe gồm 4 phần; mỗi phần được nghe 2 lần, mỗi lần cách nhau 05 giây; mở đầu
và kết thúc mỗi phần nghe có tín hiệu. Thí sinh có 20 giây để đọc mỗi phần câu hỏi.

 Mở đầu và kết thúc bài nghe có tín hiệu nhạc. Thí sinh có 03 phút để hoàn chỉnh bài
trước tín hiệu nhạc kết thúc bài nghe.

 Mọi hướng dẫn cho thí sinh (bằng tiếng Anh) đã có trong bài nghe.

Part 1: For questions 1-5, listen to a talk about private education and decide whether
these statements are True (T), False (F), or Not Given (NG). Write your answers in the
corresponding numbered boxes provided. (10pts)
1. Enrollment in private primary schools has registered greater increase compared with

that in private secondary schools over the last 15 years.


2. The demand for private schools stems from profound socio-economic changes.

3. Private education plays an important role in solving illiteracy in several massive states
in Pakistan.

4. Inclusivity is one noticeable factor that many private schools lack.


5. High levels of tuition fee in the private sector are understandable considering the high
quality of education that private schools offer.

1. 2. 3. 4. 5.

Part 2: For questions 6-10, listen to a talk about Neptune and answer the questions.
Write NO MORE THAN FOUR WORDS taken from the recording for each answer in
the corresponding numbered boxes provided. (10pts)
6. What is the distance between Neptune and the Sun?
7. What is Neptune’s core made up of besides water ice?
8. What gives Neptune blue color?
9. What can strong winds recorded on Neptune do?
10. What is the name of the spacecraft that has visited Neptune?

6. 7. 8. 9. 10.

Part 3: For questions 11-15, listen to a radio interview in which two academics called
John Farrendale and Lois Granger, taking part in a discussion on the subject of
attitudes to work and choose the answer (A, B, C or D) which fits best according to what
you hear. Write your answers in the corresponding numbered boxes provided. (10pts)
11. Lois agrees with John's point that
A most people dread the prospect of unemployment.

B the psychological effects of unemployment can be overstated.


C some people are better equipped to deal with unemployment than others.

D problems arise when unemployment coincides with other traumatic events.


12. Lois agrees with the listener who suggested that

A work is only one aspect of a fulfilling life.


B voluntary work may be more rewarding than paid work.

C not everybody can expect a high level of job satisfaction.


D people should prepare for redundancy as they would for retirement.

13. What is John's attitude towards people who see work as a 'means to an end'?
A He doubts their level of commitment to the job.
B He accepts that they have made a valid choice.

C He fears it will lead to difficulties for them later.


D He feels they may be missing out on something important.

14. When asked about so-called 'slackers' at work, John points out that
A they accept the notion that work is a necessary evil.

B people often jump to unfair conclusions about them.


C their views are unacceptable in a free labour market.

D such an attitude has become increasingly unacceptable.


15. Lois quotes the psychologist Freud in order to

A show how intellectual ideas have shifted over time.


B provide a contrast to the ideas of Bertrand Russell.
C question the idea that a desire to work is a natural thing.
D lend weight to John's ideas about increased social mobility

11. 12. 13. 14. 15.

Part 4: For questions 16-25, listen to a recording about a medical robot named Grace
and complete the summary below using words taken from the recording. Write NO
MORE THAN THREE WORDS in each blank. (20pts)
- Both Grace and her sister – the well-known (16) __________ Sophia – was created by a
company called Hanson Robotics.
- Grace is only capable of determining your (17) __________ and also your temperature with
a(n) (18) __________.
- In terms of mental health treatment, Grace can socially stimulate patients’ mood, entertain
them or do (19) __________.

- According to David Hanson Grace’s (20) ___________, which gives her the ability to
socially interact by making natural engagement easier, was designed with the goal of
preventing (21) __________ from being overwhelmed.
- Grace is the result of a(n) (22) __________ between Hanson Robotics and another company
called Singularity Studio.

- Once Grace is mass produced, production costs – currently (23) ___________ the prices of
luxury cars – will begin to drop.
- In July or August, the beta version of Grace will be produced on the (24) _________ before
testing and deployment, all of which is part of the (25) ___________.

1. 2. 3. 4. 5.
6. 7. 8. 9. 10.

SECTION II. GRAMMAR & VOCABULARY (30pts)

Part 1: Choose the correct answer A, B, C or D to each of the following questions and
write your answers in the corresponding numbered boxes provided. (20 pts)
1. Rescuers cast a _____ to the drowning man and hauled him out of the sea.
A. lifeline B. lifeboat C. lifeguard D. lifesaver
2. Although citizen-centred schemes involve residents in debates, full political _____ is kept
by local councillors and MPs.
A. autonomy B. autarchy C. autocracy D. authority
3. He answered the teacher’s question so quickly that it seemed as though he had _____ the
answer out of the air.
A. grabbed B. snatched C. seized D. plucked
4. This curtain material _____ easily.
A. hangs itself B. makes itself C. creases D. bend
5. There is no need to get so _____ about being turned down. There are other advertising
agencies out there, you know.
A. destitute B. descendant C. despondent D. despicable
6. He likes nothing better than to spend his Sunday mornings _____ in the gardens.
A. pottering about B. hanging around C. whiling away D. winding down
7. Having lost her home, Lucy got _____ a gang of people who hang around causing trouble.
A. in with B. up to C. on with D. by on
8. Julie felt unfairly _____ when she spoke out against a company proposal and the entire staff
team turned against her.
A. prosecuted B. persecuted C. oppressed D. suppressed
9. It is impossible to miss the _____ of the Generation X in America.
A. manifestation B. propriety C. depreciation D. coalescence
10. Apart from one or two _____ of brilliance from Owen, England put on a rather poor
performance.
A. spells B. flashes C. storms D. spells
11. The way people store their emotions if more corporeal than _____.
A. telegenic B. asymmetric C. psychogenic D. telepathic
12. His fight to _____ four black men of the rape of a seventeen-year-old white girl two years
ago partially inspired the group to protest.
A. exculpate B. exonerate C. bereave D. misappropriate
13. After the hurricane, all that was left of our house was a pile of _____.
A. rabble B. rubble C. ramble D. rumble
14. Having sacked three employees, the boss was obliged to provide each with _____ pay.
A. retirement B. unemployment C. dismissal D. severance
15. Don’t get yourself _____ up over such a trivial matter
A. done B. worried C. whipped D. worked
16. I _____ with the performances but I got the flu the day before.
A. Was to have helped B. helped C. was to help D. had helped
17. She_____ fainted when she heard her father died.
A. Rather than B. nothing but C. all but D. near
18. Three candidates will be short-listed for the post but we do not know _____.
A. whom B. those C. which D. what ones
19. Stars differ differently from planets _____ they are self-luminous whereas planets shine
by reflected lights.
A. From which B. when C. and D. in that
20. Owning and living in a freestanding house is still a goal of young adults, _____ earlier
generations.
A. as did B. as it was of C. like that of D. so have

1. 2. 3. 4. 5. 6. 7. 8. 9. 10.
11. 12. 13. 14. 15. 16. 17. 18. 19. 20.

Part 2: Use the correct FORM of the word in capitals to fit each gap. Write your answer
in the numbered box. (10 pts)
1. A couple of victories would improve the team's _____ enormously. (MORAL)

2. What the _____ army lacked was not discipline, but numbers, and a coherent strategy.
(SUFFRAGE)

3. “People” is a _____ word. (SYLLABLE)


4. Questions were asked at the eye clinic but these are said to have brought merely a brisk
and _____ response. (OFFICIAL)

5. His _____ gains are all safely stashed away in a Swiss bank. (GET)
6. If a screen does not contain everything one wants, further lexicographic information can be
obtained by clicking on a _____. (LINK)

7. _____ is the transplantation of living cells, tissues or organs from one species to another.
(TRANSPLANTATION)

8. For major grain crops such as wheat, rice, jowar, and bajra, the _____ prices functioned as
the minimum support prices. (CURE)
9. The documented differences between men and women in scientific career paths do not
match what would be expected in a true _____ (MERIT)

10. In Paris, proud _____ never went out of fashion. (INTELLECT)

1. 2. 3. 4. 5.
6. 7. 8. 9. 10.

SECTION III. READING COMPREHENSION (60pts)


Part 1: Read the passage below and fill each of the following numbered spaces with
ONE suitable word. Write your answers in the corresponding boxes provided. (15pts)

Striking Back at Lightning With Lasers


Seldom is the weather more dramatic than when thunderstorms strike. Their electrical fury
inflicts death or serious injury on around 500 people each year in the United States (1) _____.
As the clouds roll in, a leisurely round of golf can become a terrifying dice with death - out in
the (2) _____, a lone golfer may be a lightning bolt’s most inviting target. And (3) _____ is
damage to property too. Lightning damage costs American power companies more than $100
million a year.
(4) _____ researchers in the United States and Japan are planning to hit back. Already in
laboratory trials they have tested strategies for neutralising the (5) _____ of thunderstorms,
and this winter they will brave real storms, equipped with an armoury of lasers that they will
be pointing towards the heavens to discharge thunderclouds before lightning can (6) _____.
The idea of forcing storm clouds to discharge their lightning on command is not new. In the
early 1960s, researchers tried firing rockets trailing wires into thunderclouds to (7) _____ up
an easy discharge path for the huge electric charges that these clouds generate. (8) _____
technique survives to this day at a test site in Florida run by the University of Florida, with
support from the Electrical Power Research Institute (EPRI), based in California. EPRI,
which is funded by power companies, is looking at (9) _____ to protect the United States’
power grid from lightning strikes. “We can cause the lightning to strike where we want it to
using rockets,” says Ralph Bernstein, manager of lightning projects at EPRI. The rocket site
is providing precise measurements of lightning voltages and allowing engineers to check how
(10) _____ equipment bears up.

1. 2. 3. 4. 5.
6. 7. 8. 9. 10.

Part 2: Read the following passage and choose the best answer (A, B, C or D) according
to the text. Write your answers (A, B, C or D) in the corresponding numbered boxes.
(10pts)
Undercover journalism
Journalism is too small or too distant a word to cover it. It is theatre; there are no second
takes. It is drama – it is improvisation, infiltration and psychological warfare. It can be
destructive in itself before any print has seen the light of day. It is exhilarating, dangerous and
stressful. It is the greatest job. It is my job.

I am an undercover reporter. For the past year or so, I have been a football hooligan, a care
worker, a bodyguard and a fashion photographer. It is a strange life and difficult one. In the
course of a day, I have assumed four different personalities, worn four different wardrobes
and spoken four different street dialects, and left a little of me behind in each of those worlds.
More important than this, though, are the experiences and emotions I’ve taken away with me.
It’s hard to put a label on them. They have seeped in and floated out of my psyche, but
somewhere in the backyard of my mind the footprints of this strange work are left behind.
I have as yet no real notion as to what, if any, long-term impact they will have. For the
moment, I relish the shooting gallery of challenges that this madness has offered me. In the
midst of all these acting roles and journalistic expeditions, I have endeavoured not to sacrifice
too much of my real self. I have not gone native and I am still sane. At least for the moment.

In the course of any one investigation, you reveal yourself in conversation and etiquette,
mannerism and delivery – of thousands of gesticulations and millions of words – and cover
yourself with the embroidery of many different disguises. If one stitch is loose or one word
misplaced, then everything could crash, and perhaps violently so.
Certainly, as a covert operator, the journalistic safe line is a difficult one to call. Every word
you utter is precious, every phrase, insinuation and gesture has to be measured and
considered in legal and ethical terms. Even the cadence of your voice has to be set to
appropriate rhythms according to the assumed role, the landscape and the terrain of your
undercover patch.
The golden rule is this: as an undercover reporter you must never be the catalyst for events
that would not otherwise have occurred, had you not been there. The strict guidelines within
broadcasting organisations about covert filming mean that, every time I go into the field, a
BBC committee or compliance officer has to grant permission first. It’s a strange but
necessary experience for someone like me, who operates on instinct and intuition, but it’s a
marriage that works well.

The undercover reporter is a strange breed. There is no blueprint that exists. It is your own
journalistic ethos and within those parameters you try to tread a safe line, both in terms of
your journalism and personal safe-keeping. And of course, there’s a high price you pay for
this kind work, home is now a BBC safe house. The only visitors to my bunker are work
colleagues. It’s not a pleasant lifestyle, but I have taken on all the stories in the full
knowledge of the risks involved.

Though I embarked upon my journey with enthusiasm and determination, the climate in
which we undertake this journalistic and documentary mission is an increasingly hostile one.
It is one in which covert filming has come under scrutiny because of concerns about fakery
and deception and the featuring of hoax witnesses. Issues concerning privacy, the use of
covert filming techniques across the media – from current affairs to the tabloid newspapers –
and the way journalists work with these tools have been rigorously appraised. I personally
welcome this scrutiny.
Hi-tech surveillance equipment allows me to tell the story as it unfolds, surrounded by its
own props, revealing its own scars and naked sinews, and delivered in its own dialect. There
is no distortion and only one editorial prism – mine. While the sophisticated technology
allows a visual and aural presentation of events, mentally I rely on the traditional method of
jotting things down to rationalise my thoughts and gain a coherent picture of all that I was
involved in. This is my delivery system – how I narrate.
Inevitably the spotlight has shone on me but those who have worked on either paper trail
investigation in newspapers or in television will know that it will fade. I am happy to return
to the career of a desk journalist because I recognise that the tools we have used are tools of
last resort. I’ll be returning to the more usual journalistic methods: telephone and computer
notebook rather than secret cameras and hidden microphones. But the aim will be the same:
to shed light into the darker corners of society where the vulnerable are most at risk.
1. Which of the following does the writer NOT suggest about his job in general?

A. Journalism is not truly a word to represent it.


B. Those involved hail from various occupations.
C. It entails people to act in different roles.

D. It has a miscellaneous collection of characteristics.


2. As implied by the author, what distinguishes undercover journalism from regular kinds?

A. the range of subjects it touches on


B. the effects of its destructive power
C. the degree of spontaneity in it
D. the harm that it can cause

3. What does the writer suggest about his attitude towards his job?
A. He has a recollection of most emotions triggered by it.

B. He attaches much of his own personality to the roles he assumes.


C. His interest in the job has been retained.

D. He abominates the ordeals involved in it.


4. What does the writer imply about undercover investigators?

A. They are required to be circumspect so as not to conceal their identities.


B. They have to be cautious in order not to cling to a preplanned set of actions.

C. They should try not to be factors causing changes in events.


D. They need to be observant to the reactions of other people while conducting tasks.
5. The writer suggests that undercover investigators:

A. have to live with the consequences of exposing themselves.


B. resent sticking to rules laid down by their employers.

C. tend to be a similar kind of person.


D. operate according to a similar code of conduct.

6. As indicated by the writer, tribulations arise within the profession because


A. the application of cutting-edge technologies is conducive to attempts to falsify
information.

B. covert filming has been put into question following worries about the effects of
documentaries.
C. deceptive testimonies have emerged to degrade the quality of covert filming.

D. how journalists make use of their equipment has come under scrutiny.
7. Regarding the harsher working environment in his profession, it can be implied that the
author:

A. harbours feelings of repulsion at it.


B. displays embrace of it.

C. finds it rather disconcerting.


D. expresses insouciance towards it.
8. What can be inferred about the method used by the author while working as a reporter?
A. He embellishes the stories with details not clearly reflecting what happened

B. He allows technology to cater for every stage of the process.


C. He uses writing as a way of brainstorming ideas and approaching what he would like to
include.

D. He lets the events speak for themselves with the aid of modern technology.
9. What does the passage suggest about the author’s intentions for the future?

A. He will adopt the more traditional work as a journalist.


B. He will make attempts to return to normalcy after all events.

C. He will have recourse to the tools used when there are no alternatives.
D. He will go to some lengths to direct the limelight away from him.

10. The writer sees the primary aim of journalism as:


A. combating the corruption within the society.

B. enlightening people about the disadvantage of the weak.


C. highlighting causes of present-day issues.
D. unraveling the mystery of criminal cases.

1. 2. 3. 4. 5.
6. 7. 8. 9. 10.

Part 3: Read the following passage and choose the best answer (A, B, C or D) according
to the text. Write your answers (A, B, C or D) in the corresponding numbered boxes. (13
pts)

Company innovation

A. In a scruffy office in midtown Manhattan, a team of 30 artificial-intelligence programmers


is trying to simulate the brains of an eminent sexologist, a well-known dietician, a celebrity
fitness trainer and several other experts. Umagic Systems is a young firm, setting up websites
that will allow clients to consult the virtual versions of these personalities. Subscribers will
feed in details about themselves and their goals; Umagic’s software will come up with the
advice that the star expert would give. Although few people have lost money betting on the
neuroses of the American consumer, Umagic’s prospects are hard to gauge (in ten years’
time, consulting a computer about your sex life might seem natural, or it might seem absurd).
But the company and others like it are beginning to spook large American firms, because they
see such half-barmy “innovative” ideas as the key to their own future success.
B. Innovation has become the buzz-word of American management. Firms have found that
most of the things that can be outsourced or re-engineered have been (worryingly, by their
competitors as well). The stars of American business tend today to be innovators such as
Dell, Amazon and Wal-Mart, which have produced ideas or products that changed their
industries

C. A new book by two consultants from Arthur D. Little records that, over the past 15 years,
the top 20% of firms in an annual innovation poll by Fortune magazine have achieved double
the shareholder returns of their peers. Much of today’s merger boom is driven by a desperate
search for new ideas. So is the fortune now spent on licensing and buying others’ intellectual
property. According to the Pasadena-based Patent & Licence Exchange, trading in intangible
assets in the United States has risen from $15 billion in 1990 to $100 billion in 1998, with an
increasing proportion of the rewards going to small firms and individuals.

D. And therein lies the terror for big companies: that innovation seems to work best outside
them. Several big established “ideas factories”, including 3M, Procter & Gamble and
Rubbermaid, have had dry spells recently. Gillette spent ten years and $1 billion developing
its new Mach 3 razor; it took a British supermarket only a year or so to produce a reasonable
imitation. “In the management of creativity, size is your enemy,” argues Peter Chemin, who
runs the Fox TV and film empire for News Corporation. One person managing 20 movies is
never going to be as involved as one doing five movies. He has thus tried to break down the
studio into smaller units—even at the risk of incurring higher costs.

E. It is easier for ideas to thrive outside big firms these days. In the past, if a clever scientist
had an idea he wanted to commercialise, he would take it first to a big company. Now, with
plenty of cheap venture capital, he is more likely to set up on his own. Umagic has already
raised $5m and is about to raise $25m more. Even in capital-intensive businesses such as
pharmaceuticals, entrepreneurs can conduct early-stage research, selling out to the big firms
when they reach expensive, risky clinical trials. Around a third of drug firms’ total revenue
now comes from licensed-in technology.

F. Some giants, including General Electric and Cisco, have been remarkably successful at
snapping up and integrating scores of small companies. But many others worry about the
prices they have to pay and the difficulty in hanging on to the talent that dreamt up the idea.
Everybody would like to develop more ideas in-house. Procter & Gamble is now shifting its
entire business focus from countries to products; one aim is to get innovations accepted
across the company. Elsewhere, the search for innovation has led to a craze for
“intrapreneurship”—devolving power and setting up internal ideas-factories and tracking
stocks so that talented staff will not leave.

G. Some people think that such restructuring is not enough. In a new book Clayton
Christensen argues that many things which established firms do well, such as looking after
their current customers, can hinder the sort of innovative behaviour needed to deal with
disruptive technologies. Hence the fashion for cannibalisation—setting up businesses that
will actually fight your existing ones. Bank One, for instance, has established Wingspan, an
Internet bank that competes with its real branches (see article). Jack Welch’s Internet
initiative at General Electric is called “Destroyyourbusiness.com”.

H. Nobody could doubt that innovation matters. But need large firms be quite so pessimistic?
A recent survey of the top 50 innovations in America, by Industry Week, a journal, suggested
that ideas are as likely to come from big firms as from small ones. Another skeptical note is
sounded by Amar Bhidé, a colleague of Mr Christensen’s at the Harvard Business School and
the author of another book on entrepreneurship. Rather than having to reinvent themselves,
big companies, he believes, should concentrate on projects with high costs and low
uncertainty, leaving those with low costs and high uncertainty to small entrepreneurs. As
ideas mature and the risks and rewards become more quantifiable, big companies can adopt
them.

I. At Kimberly-Clark, Mr Sanders had to discredit the view that jobs working on new
products were for “those who couldn’t hack it in the real business.” He has tried to change the
culture not just by preaching fuzzy concepts but also by introducing hard incentives, such as
increasing the rewards for those who come up with successful new ideas and, particularly, not
punishing those whose experiments fail. The genesis of one of the firm’s current hits,
Depend, a more dignified incontinence garment, lay in a previous miss, Kotex Personals, a
form of disposable underwear for menstruating women.

J. Will all this creative destruction, cannibalisation and culture tweaking make big firms
more creative? David Post, the founder of Umagic, is sceptical: “The only successful
intrapreneurs are ones who leave and become entrepreneurs.” He also recalls with glee the
looks of total incomprehension when he tried to hawk his “virtual experts” idea three years
ago to the idea labs of firms such as IBM though, as he cheerfully adds, “of course, they
could have been right.” Innovation unlike, apparently, sex, parenting and fitness is one area
where a computer cannot tell you what to do.

Questions 1 – 6. Which section contains the following information? Write the correct
number on the given boxes.
i The unpredictability of the public’s viewpoints about a certain topic in the future
ii A list of certain institutions that are having fewer business activities

iii A type of firms that are resorted to compulsive consumption for new ideas
iv The insatiable thirst for outstanding innovations being an impetus to big impacts on the
market.

v Some moguls which expressed financial concerns when investing in the acquisition of
smaller companies.
vi The reason why American business trends are highlighting the importance of initiatives

vii A company that is able to going through economic falters itself.


viii Small firms that can make certain impacts on bigger ones when the former possesses
more potential ideas.

ix Example of three famous American companies’ innovation


x A type of firms that are regarded fly-by-night when investing in entrepreneurship.

1. Section A 4. Section D

2. Section B 5. Section E

3. Section C 6. Section F

1. 2. 3. 4. 5. 6.

Questions 7-13. In boxes 7-13, write: Y (Yes), N (No), NG (Not Given)


Yes if the statement agrees with the claims of the writer
No if the statement contradicts the claims of the writer
Not Given if it is impossible to say what the writer thinks about this

7. Peter infers his unwillingness to invest more in restructuring his organization in return for
better creativity management.

8. Some small organizations have a craving for ideas that are regarded as an admixture of
“innovative” and “strange”.
9. Umagic is head and shoulders above other competitors in such a new field.

10. A new trend that has already superseded “entrepreneurship” in one area may directly
impact living organizations.
11. Big giants prioritize innovations with low certainty on the understanding that big risks are
parallel to big profits.

12. It takes many years for Mr Sanders to successfully ditch preconceived ideas in his
organization.
13. The author expressed a positive attitude towards the development of innovations at the
end of the passage.

7. 8. 9. 10. 11. 12. 13.

Part 4: In the passage below, seven paragraphs have been removed. For questions 1-7,
read the passage and choose from paragraphs A-H the one which fits each gap. There is
ONE extra paragraph which you do not need to use. Write your answers in the
corresponding numbered boxes provided. (7pts)
How satisfying to pull a chain again. Something went out of British plumbing with the arrival
of the integrated cistern, but everything that went out of British plumbing with the Victorians
has been reinstated in the stateliest form in the bathroom of our suite at the Pool House Hotel.
If only for that achievement alone, it deserves its AA accolade, awarded last Thursday, of
Scottish Hotel of the Year.

1.
In Wester Ross, the old parish of Gairloch - a glorious body of country clasped between the
long sea arms of Loch Torridon and Loch Broom - has all the classic components of the West
Highland landscape. It has the mighty Torridon range, the oldest rock in Britain; the moor-
and-mountain wilderness of Letterewe and the island-studded mirror of Loch Maree - an
inland loch more beautiful than any other, including Loch Lomond. It also has a lonely coast,
sandy bays, leafy glens, Hebridean vistas and numerous whitewashed villages.
2.

The Pool House building on the loch's foreshore, where the River Ewe enters the lake after a
short but vigorous journey from Loch Maree, doesn't look like a traditional Highland lodge.
But scrape away the white paint and roughcast and you will expose pink Torridonian
sandstone - the preferred building blocks of local lairds for three centuries.
3.

Osgood Mackenzie, who caused thin, acid layers of peat on a windswept headland to bloom
with the trees, shrubs and flowers of the temperate world from Chile to Tasmania, lived for a
time in Pool House while he worked obsessively on his horticultural masterpiece. Meanwhile,
his English wife whiled away the hours by carving a chain of Tudor roses in the banisters of
the central staircase.

4.
When the present owners, the Harrison family, made the decision to replace their 13
bedrooms with four themed suites it was Liz Miles who became the creative force. Liz
tracked down the extravagant wallpapers - putto friezes, Michelangelo ceilings, celestial
maps - and sourced most of the antique fittings and furniture.
5.

With some reluctance my husband and I jump ship - forcing ourselves out of the sumptuous
fantasy of Campania, with its 130-year-old cast iron and brass four-poster, to confront the
reality of the weather. As enthusiasts for the elemental challenge of the West Highland
seaboard, we have a busy programme: a rugged walk, a wildlife cruise and, as the softest
option locally available, a visit to Inverewe Garden.
6.

The headland couldn't be more exposed, but the squalls of rain beating in from the Atlantic
sail over our heads en route to the mountains. We flush grouse and snipe from the heather on
our three-hour walk and glimpse red deer. By the time we reach the great sea stacks of Stack
Dubh and Stac Buidhe, there are shafts of sunshine striking the wings of gannets, fulmars and
shags.
7.
We don't. But we do see grey seals, harbour porpoises, great skuas and - quite a spot for a
trainee birdwatcher like me - a huddle of rare, black-throated divers. Warblers and other
songbirds attend our visit to Inverewe Garden, now owned and maintained by the National
Trust for Scotland and not, perhaps, at their best on the cusp between summer and autumn.
But they are still remarkable.
Missing paragraph:

A. The Mackenzie country was dominant in this part of country. Pool House's golden age was
Victorian, when the Highlands became a sporting playground for the gentry. There was
salmon to pull from the River Ewe, deer to stalk and grouse to pot, but for a time the lodge
was home to one of the less predatory Mackenzies: a man who liked to let things grow rather
than cut them down.

B. There was only one willow tree on the promontory where Osgood Mackenzie began his
project in 1862. Now there is a prodigious stand of Scots pine and other native woodland,
planted to supply the windbreak for his exotic trees and shrubs. The contrast is beautiful.
C. Some visitors say they would be willing to pay merely to tour the rooms. Many are
especially fascinated with Diadem, which is modelled on the style of a first-class cabin on the
Titanic. Margaret Harrison's grandfather was a cousin of Captain Smith, the liner's master;
but Peter Harrison, who takes a keen interest in military history, has named all the suites after
warships as a tribute to Pool House's function during the Second World War, when it was the
Navy's headquarters for co-ordinating the North Atlantic and Murmansk convoys.

D. Not all the cottages in these old crofting townships are second homes or self-catering
units. The scattered "capital" of the parish, Gairloch, is something of a boom community,
with energetic young locals raising new houses on scenic building plots. At nearby Poolewe,
which has the botanical curiosity and tourist honeypot of Inverewe Garden, an old shooting
lodge is turning back the clock to find a future.
E. We usually do our own route finding but we want to investigate Rua Reidh Lighthouse,
where Fran Cree and Chris Barrett run residential walking and activity holidays on one of the
most remote headlands of the mainland. Just getting there is an adventure; and the airy nature
of the clifftop paths, with their views to the Outer Hebrides and dizzy drops into empty
beaches, makes us glad of the expert presence of Chris, who is a member of the local
mountain rescue team. "We get called out about a dozen times a year," she tells us.
F. Seldom has washing been such a treat. As I wallow beneath the cascading canopy of the
Shanks Independent Spray Bath (built in Glasgow in 1875 on a scale comparable to the boiler
of a Clyde steamship), I feel a certain kinship with the grey seal idling in the water outside.
From the bathroom window I can see the glassy surface of Loch Ewe - and much of its
wildlife.
G. Our tally of wildlife soars on the sturdy Starquest, which skipper Ian Birks steers to the
wide mouth of Loch Gairloch and the first tugging of the Atlantic. It's the whale-watching
season - the Minch is part of the minke's larder - and it's part of Ian's purpose to monitor their
movements for the Sea Watch Foundation. "But I never advertise these trips as whale-
watching cruises, otherwise people expect to see whales."

H. The carvings are still there, one of the few remnants of the 19th century to survive. How
can this be? From the outside Pool House may look like a made-over inn with 1960s add-ons;
step inside and you enter into the rich, decorative and occasionally camp interior of a
Victorian country house. Yet almost all its finest features, from huge, wood-panelled
bathrooms to marble and polished steel fireplaces, have been retrieved from architectural
salvage yards and put in place over the past three years.

1. 2. 3. 4. 5. 6. 7.

Part 5: You are going to read an article containing reviews of computer games. Choose
from the reviews (A-E). The reviews may be chosen more than once. Write your
answers in the corresponding numbered boxes provided. (15pts)
A. The retreat was a hasty as it was disorganized, and the fall of Galdor was completed. Had
they held out until the bitter end matching their foes blow for blow with conviction and
strength, they could at least have held their heads high, but Ruan knew all semblance of hope
had evaporated with the battle pressure, and that was what worried him the most, for Oldark
had yet to unleash his full wrath upon the people of Appleton. Though overcome with
frustration, Ruan knew in his heart of hearts that he couldn’t hold Redhorn and the others to
account for their wimpishness; after all, in much the same wat as Rome was not built in a
day, nor were farmers reshaped into warriors overnight. Weary, tired and near broken, Ruan
new the task ahead of him was gargantuan, yet, he told himself, again and again, that he must
succeed.
B. At least their numbers were little depleted, he thought. Such had been the haste to flee that
his men had largely escaped unscathed. Perhaps a handful, he estimated, had fallen, and
perhaps six dozen more had been wounded. Then, momentarily, he almost lost all his
composure as the realization hit him hard like a knife delivered heart-bound with deadly
accuracy. Orlach was not amongst the motley assemblage regrouping around him. What fate
had befallen him? His dearest brother – not kin, it’s true, but their bond to succeed bereft of
the courage Orlach imbued in him? He was, he thought, leader in but name only, for it was
Orlach’s fearlessness that had always driven him forth, sustained him and helped him keep
faith despite the faintest of hopes. Without Orlach, all was lost.

C. Ruan collected himself, eyeing its hapless followers and knowing he must deliver a
rousing message of hope with convincing, albeit false conviction. He held him arm aloft and
a hush descended over the gathering crowd. He gestured towards the woods, where his own
precious spouse and the other brave women of Appleton stood ready to defend their children
to the last; safe as yet, but for how long? He knew they would never abandon their post and
their responsibilities so pitifully as he and his men just had. There would beat no hasty retreat
if it came to it, though he hoped it never would, for that would surely be the end, and he and
his brethren would have utterly failed. So he told his weary listeners they were farmhands no
more and he gestured again towards the hidden dwelling in the woods.

D. Meanwhile, the plumes of smoke coming from what had once been their beloved village
were already evident on the distant horizon. Oldark had razed it to the ground. But Appleton
was alive, he told them, in their hearts and souls. And it would be resurrected. Oldark could
not destroy Appleton: not while a single Appletonian heart was yet beating. Indeed, so far, he
had not even come close. He had but peeled away the surface layter. Houses could be rebuilt.
Yes, the first layer of skin was gone, and it felt raw and bitter now, Ruan ceded. But there
were two more layers, were there not? The body of Appleton had not even been pierced the
once as yet. He stared at his men and thundered his words. They were the skin and bones; the
veins and the arteries; the muscle and sinew, he told them. Their brave spouses, the child
protectorate, the vital organs, the beating heart of Appleton. And the soul, what else could it
be? The younglings. It ran and played and skipped and screamed and smiled everyday; it
brought them love and happiness and completion. What else could it be indeed?
E. Suddenly, this hapless band of farmers realized that, really, the battle had barely begun.
What was lost was nothing, and all was yet to fight for. No less cowed than before, from
somewhere deep inside rumbled the warrior soul and their bellies fired with a primeval sense
of purpose. They would stand and fight yet, and do so with every last sinew of strength they
could muster from their bones, and Oldark would be dethroned. His reign of terror would and
as surely as the harshest winter must eventually retreat and give way to spring, and Appleton
would blossom again. And there would be running and playing and skipping and screaming
and smiling once more. Ruan felt Orlach’s strength within him as though he were still there
and his words no longer carried false conviction. He and his brethren were as men possessed,
and they would prevail.

In which extract…

does Ruan almost panic and lose control? 1. .............


is one of Ruan’s close relatives mentioned? 2. .............
is the nature of a defeat described? 3. .............
does the loss of the close friend leave Ruan feeling dejected? 4. .............
does Ruan compare the different genders of his people unfavourably? 5. .............
are children attributed a sort of spiritual significance? 6. .............
do we learn the fate of Appleton village? 7. .............
does the memory of someone give Ruan more resolve? 8. .............
does Ruan liken his people’s suffering to a wound? 9. .............
does Ruan find some relief in the fact that defeat was not heavy in 10. .............
casualities?

SECTION IV. WRITING (60pts)


Part 1: Read the following extract and use your own words to summarize it. Your
summary should be about 140 words. You MUST NOT copy the original. (15pts)
The Nature of Genius

There has always been an interest in geniuses and prodigies. The word ‘genius’, from the
Latin gens (=family) and the term ‘genius’, meaning ‘begetter’, comes from the early Roman
cult of a divinity as the head of the family. In its earliest form, genius was concerned with the
ability of the head of the family, the paterfamilias, to perpetuate himself. Gradually, genius
came to represent a person's characteristics and thence an individual's highest attributes
derived from his ‘genius’ or guiding spirit. Today, people still look to stars or genes,
astrology or genetics, in the hope of finding the source of exceptional abilities or personal
characteristics.

The concept of genius and of gifts has become part of our folk culture, and attitudes are
ambivalent towards them. We envy the gifted and mistrust them. In the mythology of
giftedness, it is popularly believed that if people are talented in one area, they must be
defective in another, that intellectuals are impractical, that prodigies burn too brightly too
soon and burn out, that gifted people are eccentric, that they are physical weaklings, that
there's a thin line between genius and madness, that genius runs in families, that the gifted are
so clever they don't need special help, that giftedness is the same as having a high IQ, that
some races are more intelligent or musical or mathematical than others, that genius goes
unrecognised and unrewarded, that adversity makes men wise or that people with gifts have a
responsibility to use them. Language has been enriched with such terms as ‘highbrow’,
‘egghead’, ‘blue-stocking’, ‘wiseacre’, ‘now-all’, ‘boffin’ and, for many, ‘intellectual’ is a
term of denigration.

The nineteenth century saw considerable interest in the nature of genius, and produced not a
few studies of famous prodigies. Perhaps for us today, two of the most significant aspects of
most of these studies of genius are the frequency with which early encouragement and
teaching by parents and tutors had beneficial effects on the intellectual, artistic or musical
development of the children but caused great difficulties of adjustment later in their lives, and
the frequency with which abilities went unrecognised by teachers and schools. However, the
difficulty with the evidence produced by these studies, fascinating as they are in collecting
together anecdotes and apparent similarities and exceptions, is that they are not what we
would today call norm-referenced. In other words, when, for instance, information is collated
about early illnesses, methods of upbringing, schooling, etc., we must also take into account
information from other historical sources about how common or exceptional these were at the
time. For instance, infant mortality was high and life expectancy much shorter than today,
home tutoring was common in the families of the nobility and wealthy, bullying and corporal
punishment were common at the best independent schools and, for the most part, the cases
studied were members of the privileged classes. It was only with the growth of paediatrics
and psychology in the twentieth century that studies could be carried out on a more objective,
if still not always very scientific, basis.

Part 2: CHART DESCRIPTION (15pts)


The bar chart below shows the percentage of government spending on roads and
transport in 4 countries in the years 1990, 1995, 2000, 2005.

Summarise the information by selecting and reporting the main features, and make
comparisons where relevant.

Percentage of government spending on road and transport (1990 -


2005)
30

25

20
Percentage

1990
15 1995
2000
10
2005
5

0
Italia Portugal UK USA

Part 3: ESSAY WRITING (30pts)


Some people believe that no one should do the same job for all their working life. Others
argue that doing the same job brings advantages for individuals, companies, and
society. Discuss both views and give your opinion.
Give reasons for your answer and include any relevant examples from your own
knowledge and experience. Write at least 250 words.

Người ra đề: Trần Thị Ánh Nguyệt - 0913310100


HỘI CÁC TRƯƠNG THPT CHUYÊN KÌ THI HỌC SINH GIỎI NĂM 2022
VÙNG DUYÊN HẢI VÀ ĐÔNG BẮC MÔN: TIẾNG ANH LỚP 11
BẮC BỘ Thời gian: 180 phút
TRƯỜNG THPT CHUYÊN HÙNG Đề thi gồm 18 trang
VƯƠNG
--------------------
ĐỀ THI ĐỀ XUẤT

SECTION I. LISTENING (50pts)

HƯỚNG DẪN PHẦN THI NGHE HIỂU

 Bài nghe gồm 4 phần; mỗi phần được nghe 2 lần, mỗi lần cách nhau 05 giây; mở đầu
và kết thúc mỗi phần nghe có tín hiệu. Thí sinh có 20 giây để đọc mỗi phần câu hỏi.

 Mở đầu và kết thúc bài nghe có tín hiệu nhạc. Thí sinh có 03 phút để hoàn chỉnh bài
trước tín hiệu nhạc kết thúc bài nghe.

 Mọi hướng dẫn cho thí sinh (bằng tiếng Anh) đã có trong bài nghe.

Part 1: For questions 1-5, listen to a talk about private education and decide whether
these statements are True (T), False (F), or Not Given (NG). Write your answers in the
corresponding numbered boxes provided. (10pts)
1. Enrollment in private primary schools has registered greater increase compared with

that in private secondary schools over the last 15 years.


2. The demand for private schools stems from profound socio-economic changes.

3. Private education plays an important role in solving illiteracy in several massive states
in Pakistan.

4. Inclusivity is one noticeable factor that many private schools lack.


5. High levels of tuition fee in the private sector are understandable considering the high
quality of education that private schools offer.

1. 2. 3. 4. 5.

Part 2: For questions 6-10, listen to a talk about Neptune and answer the questions.
Write NO MORE THAN FOUR WORDS taken from the recording for each answer in
the corresponding numbered boxes provided. (10pts)
6. What is the distance between Neptune and the Sun?
7. What is Neptune’s core made up of besides water ice?
8. What gives Neptune blue color?
9. What can strong winds recorded on Neptune do?
10. What is the name of the spacecraft that has visited Neptune?

6. 7. 8. 9. 10.

Part 3: For questions 11-15, listen to a radio interview in which two academics called
John Farrendale and Lois Granger, taking part in a discussion on the subject of
attitudes to work and choose the answer (A, B, C or D) which fits best according to what
you hear. Write your answers in the corresponding numbered boxes provided. (10pts)
11. Lois agrees with John's point that
A most people dread the prospect of unemployment.

B the psychological effects of unemployment can be overstated.


C some people are better equipped to deal with unemployment than others.

D problems arise when unemployment coincides with other traumatic events.


12. Lois agrees with the listener who suggested that

A work is only one aspect of a fulfilling life.


B voluntary work may be more rewarding than paid work.

C not everybody can expect a high level of job satisfaction.


D people should prepare for redundancy as they would for retirement.

13. What is John's attitude towards people who see work as a 'means to an end'?
A He doubts their level of commitment to the job.
B He accepts that they have made a valid choice.

C He fears it will lead to difficulties for them later.


D He feels they may be missing out on something important.

14. When asked about so-called 'slackers' at work, John points out that
A they accept the notion that work is a necessary evil.

B people often jump to unfair conclusions about them.


C their views are unacceptable in a free labour market.

D such an attitude has become increasingly unacceptable.


15. Lois quotes the psychologist Freud in order to

A show how intellectual ideas have shifted over time.


B provide a contrast to the ideas of Bertrand Russell.
C question the idea that a desire to work is a natural thing.
D lend weight to John's ideas about increased social mobility

11. 12. 13. 14. 15.

Part 4: For questions 16-25, listen to a recording about a medical robot named Grace
and complete the summary below using words taken from the recording. Write NO
MORE THAN THREE WORDS in each blank. (20pts)
- Both Grace and her sister – the well-known (16) __________ Sophia – was created by a
company called Hanson Robotics.
- Grace is only capable of determining your (17) __________ and also your temperature with
a(n) (18) __________.
- In terms of mental health treatment, Grace can socially stimulate patients’ mood, entertain
them or do (19) __________.

- According to David Hanson Grace’s (20) ___________, which gives her the ability to
socially interact by making natural engagement easier, was designed with the goal of
preventing (21) __________ from being overwhelmed.
- Grace is the result of a(n) (22) __________ between Hanson Robotics and another company
called Singularity Studio.

- Once Grace is mass produced, production costs – currently (23) ___________ the prices of
luxury cars – will begin to drop.
- In July or August, the beta version of Grace will be produced on the (24) _________ before
testing and deployment, all of which is part of the (25) ___________.

1. 2. 3. 4. 5.
6. 7. 8. 9. 10.

SECTION II. GRAMMAR & VOCABULARY (30pts)

Part 1: Choose the correct answer A, B, C or D to each of the following questions and
write your answers in the corresponding numbered boxes provided. (20 pts)
1. Rescuers cast a _____ to the drowning man and hauled him out of the sea.
A. lifeline B. lifeboat C. lifeguard D. lifesaver
2. Although citizen-centred schemes involve residents in debates, full political _____ is kept
by local councillors and MPs.
A. autonomy B. autarchy C. autocracy D. authority
3. He answered the teacher’s question so quickly that it seemed as though he had _____ the
answer out of the air.
A. grabbed B. snatched C. seized D. plucked
4. This curtain material _____ easily.
A. hangs itself B. makes itself C. creases D. bend
5. There is no need to get so _____ about being turned down. There are other advertising
agencies out there, you know.
A. destitute B. descendant C. despondent D. despicable
6. He likes nothing better than to spend his Sunday mornings _____ in the gardens.
A. pottering about B. hanging around C. whiling away D. winding down
7. Having lost her home, Lucy got _____ a gang of people who hang around causing trouble.
A. in with B. up to C. on with D. by on
8. Julie felt unfairly _____ when she spoke out against a company proposal and the entire staff
team turned against her.
A. prosecuted B. persecuted C. oppressed D. suppressed
9. It is impossible to miss the _____ of the Generation X in America.
A. manifestation B. propriety C. depreciation D. coalescence
10. Apart from one or two _____ of brilliance from Owen, England put on a rather poor
performance.
A. spells B. flashes C. storms D. spells
11. The way people store their emotions if more corporeal than _____.
A. telegenic B. asymmetric C. psychogenic D. telepathic
12. His fight to _____ four black men of the rape of a seventeen-year-old white girl two years
ago partially inspired the group to protest.
A. exculpate B. exonerate C. bereave D. misappropriate
13. After the hurricane, all that was left of our house was a pile of _____.
A. rabble B. rubble C. ramble D. rumble
14. Having sacked three employees, the boss was obliged to provide each with _____ pay.
A. retirement B. unemployment C. dismissal D. severance
15. Don’t get yourself _____ up over such a trivial matter
A. done B. worried C. whipped D. worked
16. I _____ with the performances but I got the flu the day before.
A. Was to have helped B. helped C. was to help D. had helped
17. She_____ fainted when she heard her father died.
A. Rather than B. nothing but C. all but D. near
18. Three candidates will be short-listed for the post but we do not know _____.
A. whom B. those C. which D. what ones
19. Stars differ differently from planets _____ they are self-luminous whereas planets shine
by reflected lights.
A. From which B. when C. and D. in that
20. Owning and living in a freestanding house is still a goal of young adults, _____ earlier
generations.
A. as did B. as it was of C. like that of D. so have

1. 2. 3. 4. 5. 6. 7. 8. 9. 10.
11. 12. 13. 14. 15. 16. 17. 18. 19. 20.

Part 2: Use the correct FORM of the word in capitals to fit each gap. Write your answer
in the numbered box. (10 pts)
1. A couple of victories would improve the team's _____ enormously. (MORAL)

2. What the _____ army lacked was not discipline, but numbers, and a coherent strategy.
(SUFFRAGE)

3. “People” is a _____ word. (SYLLABLE)


4. Questions were asked at the eye clinic but these are said to have brought merely a brisk
and _____ response. (OFFICIAL)

5. His _____ gains are all safely stashed away in a Swiss bank. (GET)
6. If a screen does not contain everything one wants, further lexicographic information can be
obtained by clicking on a _____. (LINK)

7. _____ is the transplantation of living cells, tissues or organs from one species to another.
(TRANSPLANTATION)

8. For major grain crops such as wheat, rice, jowar, and bajra, the _____ prices functioned as
the minimum support prices. (CURE)
9. The documented differences between men and women in scientific career paths do not
match what would be expected in a true _____ (MERIT)

10. In Paris, proud _____ never went out of fashion. (INTELLECT)

1. 2. 3. 4. 5.
6. 7. 8. 9. 10.

SECTION III. READING COMPREHENSION (60pts)


Part 1: Read the passage below and fill each of the following numbered spaces with
ONE suitable word. Write your answers in the corresponding boxes provided. (15pts)

Striking Back at Lightning With Lasers


Seldom is the weather more dramatic than when thunderstorms strike. Their electrical fury
inflicts death or serious injury on around 500 people each year in the United States (1) _____.
As the clouds roll in, a leisurely round of golf can become a terrifying dice with death - out in
the (2) _____, a lone golfer may be a lightning bolt’s most inviting target. And (3) _____ is
damage to property too. Lightning damage costs American power companies more than $100
million a year.
(4) _____ researchers in the United States and Japan are planning to hit back. Already in
laboratory trials they have tested strategies for neutralising the (5) _____ of thunderstorms,
and this winter they will brave real storms, equipped with an armoury of lasers that they will
be pointing towards the heavens to discharge thunderclouds before lightning can (6) _____.
The idea of forcing storm clouds to discharge their lightning on command is not new. In the
early 1960s, researchers tried firing rockets trailing wires into thunderclouds to (7) _____ up
an easy discharge path for the huge electric charges that these clouds generate. (8) _____
technique survives to this day at a test site in Florida run by the University of Florida, with
support from the Electrical Power Research Institute (EPRI), based in California. EPRI,
which is funded by power companies, is looking at (9) _____ to protect the United States’
power grid from lightning strikes. “We can cause the lightning to strike where we want it to
using rockets,” says Ralph Bernstein, manager of lightning projects at EPRI. The rocket site
is providing precise measurements of lightning voltages and allowing engineers to check how
(10) _____ equipment bears up.

1. 2. 3. 4. 5.
6. 7. 8. 9. 10.

Part 2: Read the following passage and choose the best answer (A, B, C or D) according
to the text. Write your answers (A, B, C or D) in the corresponding numbered boxes.
(10pts)
Undercover journalism
Journalism is too small or too distant a word to cover it. It is theatre; there are no second
takes. It is drama – it is improvisation, infiltration and psychological warfare. It can be
destructive in itself before any print has seen the light of day. It is exhilarating, dangerous and
stressful. It is the greatest job. It is my job.

I am an undercover reporter. For the past year or so, I have been a football hooligan, a care
worker, a bodyguard and a fashion photographer. It is a strange life and difficult one. In the
course of a day, I have assumed four different personalities, worn four different wardrobes
and spoken four different street dialects, and left a little of me behind in each of those worlds.
More important than this, though, are the experiences and emotions I’ve taken away with me.
It’s hard to put a label on them. They have seeped in and floated out of my psyche, but
somewhere in the backyard of my mind the footprints of this strange work are left behind.
I have as yet no real notion as to what, if any, long-term impact they will have. For the
moment, I relish the shooting gallery of challenges that this madness has offered me. In the
midst of all these acting roles and journalistic expeditions, I have endeavoured not to sacrifice
too much of my real self. I have not gone native and I am still sane. At least for the moment.

In the course of any one investigation, you reveal yourself in conversation and etiquette,
mannerism and delivery – of thousands of gesticulations and millions of words – and cover
yourself with the embroidery of many different disguises. If one stitch is loose or one word
misplaced, then everything could crash, and perhaps violently so.
Certainly, as a covert operator, the journalistic safe line is a difficult one to call. Every word
you utter is precious, every phrase, insinuation and gesture has to be measured and
considered in legal and ethical terms. Even the cadence of your voice has to be set to
appropriate rhythms according to the assumed role, the landscape and the terrain of your
undercover patch.
The golden rule is this: as an undercover reporter you must never be the catalyst for events
that would not otherwise have occurred, had you not been there. The strict guidelines within
broadcasting organisations about covert filming mean that, every time I go into the field, a
BBC committee or compliance officer has to grant permission first. It’s a strange but
necessary experience for someone like me, who operates on instinct and intuition, but it’s a
marriage that works well.

The undercover reporter is a strange breed. There is no blueprint that exists. It is your own
journalistic ethos and within those parameters you try to tread a safe line, both in terms of
your journalism and personal safe-keeping. And of course, there’s a high price you pay for
this kind work, home is now a BBC safe house. The only visitors to my bunker are work
colleagues. It’s not a pleasant lifestyle, but I have taken on all the stories in the full
knowledge of the risks involved.

Though I embarked upon my journey with enthusiasm and determination, the climate in
which we undertake this journalistic and documentary mission is an increasingly hostile one.
It is one in which covert filming has come under scrutiny because of concerns about fakery
and deception and the featuring of hoax witnesses. Issues concerning privacy, the use of
covert filming techniques across the media – from current affairs to the tabloid newspapers –
and the way journalists work with these tools have been rigorously appraised. I personally
welcome this scrutiny.
Hi-tech surveillance equipment allows me to tell the story as it unfolds, surrounded by its
own props, revealing its own scars and naked sinews, and delivered in its own dialect. There
is no distortion and only one editorial prism – mine. While the sophisticated technology
allows a visual and aural presentation of events, mentally I rely on the traditional method of
jotting things down to rationalise my thoughts and gain a coherent picture of all that I was
involved in. This is my delivery system – how I narrate.
Inevitably the spotlight has shone on me but those who have worked on either paper trail
investigation in newspapers or in television will know that it will fade. I am happy to return
to the career of a desk journalist because I recognise that the tools we have used are tools of
last resort. I’ll be returning to the more usual journalistic methods: telephone and computer
notebook rather than secret cameras and hidden microphones. But the aim will be the same:
to shed light into the darker corners of society where the vulnerable are most at risk.
1. Which of the following does the writer NOT suggest about his job in general?

A. Journalism is not truly a word to represent it.


B. Those involved hail from various occupations.
C. It entails people to act in different roles.

D. It has a miscellaneous collection of characteristics.


2. As implied by the author, what distinguishes undercover journalism from regular kinds?

A. the range of subjects it touches on


B. the effects of its destructive power
C. the degree of spontaneity in it
D. the harm that it can cause

3. What does the writer suggest about his attitude towards his job?
A. He has a recollection of most emotions triggered by it.

B. He attaches much of his own personality to the roles he assumes.


C. His interest in the job has been retained.

D. He abominates the ordeals involved in it.


4. What does the writer imply about undercover investigators?

A. They are required to be circumspect so as not to conceal their identities.


B. They have to be cautious in order not to cling to a preplanned set of actions.

C. They should try not to be factors causing changes in events.


D. They need to be observant to the reactions of other people while conducting tasks.
5. The writer suggests that undercover investigators:

A. have to live with the consequences of exposing themselves.


B. resent sticking to rules laid down by their employers.

C. tend to be a similar kind of person.


D. operate according to a similar code of conduct.

6. As indicated by the writer, tribulations arise within the profession because


A. the application of cutting-edge technologies is conducive to attempts to falsify
information.

B. covert filming has been put into question following worries about the effects of
documentaries.
C. deceptive testimonies have emerged to degrade the quality of covert filming.

D. how journalists make use of their equipment has come under scrutiny.
7. Regarding the harsher working environment in his profession, it can be implied that the
author:

A. harbours feelings of repulsion at it.


B. displays embrace of it.

C. finds it rather disconcerting.


D. expresses insouciance towards it.
8. What can be inferred about the method used by the author while working as a reporter?
A. He embellishes the stories with details not clearly reflecting what happened

B. He allows technology to cater for every stage of the process.


C. He uses writing as a way of brainstorming ideas and approaching what he would like to
include.

D. He lets the events speak for themselves with the aid of modern technology.
9. What does the passage suggest about the author’s intentions for the future?

A. He will adopt the more traditional work as a journalist.


B. He will make attempts to return to normalcy after all events.

C. He will have recourse to the tools used when there are no alternatives.
D. He will go to some lengths to direct the limelight away from him.

10. The writer sees the primary aim of journalism as:


A. combating the corruption within the society.

B. enlightening people about the disadvantage of the weak.


C. highlighting causes of present-day issues.
D. unraveling the mystery of criminal cases.

1. 2. 3. 4. 5.
6. 7. 8. 9. 10.

Part 3: Read the following passage and choose the best answer (A, B, C or D) according
to the text. Write your answers (A, B, C or D) in the corresponding numbered boxes. (13
pts)

Company innovation

A. In a scruffy office in midtown Manhattan, a team of 30 artificial-intelligence programmers


is trying to simulate the brains of an eminent sexologist, a well-known dietician, a celebrity
fitness trainer and several other experts. Umagic Systems is a young firm, setting up websites
that will allow clients to consult the virtual versions of these personalities. Subscribers will
feed in details about themselves and their goals; Umagic’s software will come up with the
advice that the star expert would give. Although few people have lost money betting on the
neuroses of the American consumer, Umagic’s prospects are hard to gauge (in ten years’
time, consulting a computer about your sex life might seem natural, or it might seem absurd).
But the company and others like it are beginning to spook large American firms, because they
see such half-barmy “innovative” ideas as the key to their own future success.
B. Innovation has become the buzz-word of American management. Firms have found that
most of the things that can be outsourced or re-engineered have been (worryingly, by their
competitors as well). The stars of American business tend today to be innovators such as
Dell, Amazon and Wal-Mart, which have produced ideas or products that changed their
industries

C. A new book by two consultants from Arthur D. Little records that, over the past 15 years,
the top 20% of firms in an annual innovation poll by Fortune magazine have achieved double
the shareholder returns of their peers. Much of today’s merger boom is driven by a desperate
search for new ideas. So is the fortune now spent on licensing and buying others’ intellectual
property. According to the Pasadena-based Patent & Licence Exchange, trading in intangible
assets in the United States has risen from $15 billion in 1990 to $100 billion in 1998, with an
increasing proportion of the rewards going to small firms and individuals.

D. And therein lies the terror for big companies: that innovation seems to work best outside
them. Several big established “ideas factories”, including 3M, Procter & Gamble and
Rubbermaid, have had dry spells recently. Gillette spent ten years and $1 billion developing
its new Mach 3 razor; it took a British supermarket only a year or so to produce a reasonable
imitation. “In the management of creativity, size is your enemy,” argues Peter Chemin, who
runs the Fox TV and film empire for News Corporation. One person managing 20 movies is
never going to be as involved as one doing five movies. He has thus tried to break down the
studio into smaller units—even at the risk of incurring higher costs.

E. It is easier for ideas to thrive outside big firms these days. In the past, if a clever scientist
had an idea he wanted to commercialise, he would take it first to a big company. Now, with
plenty of cheap venture capital, he is more likely to set up on his own. Umagic has already
raised $5m and is about to raise $25m more. Even in capital-intensive businesses such as
pharmaceuticals, entrepreneurs can conduct early-stage research, selling out to the big firms
when they reach expensive, risky clinical trials. Around a third of drug firms’ total revenue
now comes from licensed-in technology.

F. Some giants, including General Electric and Cisco, have been remarkably successful at
snapping up and integrating scores of small companies. But many others worry about the
prices they have to pay and the difficulty in hanging on to the talent that dreamt up the idea.
Everybody would like to develop more ideas in-house. Procter & Gamble is now shifting its
entire business focus from countries to products; one aim is to get innovations accepted
across the company. Elsewhere, the search for innovation has led to a craze for
“intrapreneurship”—devolving power and setting up internal ideas-factories and tracking
stocks so that talented staff will not leave.

G. Some people think that such restructuring is not enough. In a new book Clayton
Christensen argues that many things which established firms do well, such as looking after
their current customers, can hinder the sort of innovative behaviour needed to deal with
disruptive technologies. Hence the fashion for cannibalisation—setting up businesses that
will actually fight your existing ones. Bank One, for instance, has established Wingspan, an
Internet bank that competes with its real branches (see article). Jack Welch’s Internet
initiative at General Electric is called “Destroyyourbusiness.com”.

H. Nobody could doubt that innovation matters. But need large firms be quite so pessimistic?
A recent survey of the top 50 innovations in America, by Industry Week, a journal, suggested
that ideas are as likely to come from big firms as from small ones. Another skeptical note is
sounded by Amar Bhidé, a colleague of Mr Christensen’s at the Harvard Business School and
the author of another book on entrepreneurship. Rather than having to reinvent themselves,
big companies, he believes, should concentrate on projects with high costs and low
uncertainty, leaving those with low costs and high uncertainty to small entrepreneurs. As
ideas mature and the risks and rewards become more quantifiable, big companies can adopt
them.

I. At Kimberly-Clark, Mr Sanders had to discredit the view that jobs working on new
products were for “those who couldn’t hack it in the real business.” He has tried to change the
culture not just by preaching fuzzy concepts but also by introducing hard incentives, such as
increasing the rewards for those who come up with successful new ideas and, particularly, not
punishing those whose experiments fail. The genesis of one of the firm’s current hits,
Depend, a more dignified incontinence garment, lay in a previous miss, Kotex Personals, a
form of disposable underwear for menstruating women.

J. Will all this creative destruction, cannibalisation and culture tweaking make big firms
more creative? David Post, the founder of Umagic, is sceptical: “The only successful
intrapreneurs are ones who leave and become entrepreneurs.” He also recalls with glee the
looks of total incomprehension when he tried to hawk his “virtual experts” idea three years
ago to the idea labs of firms such as IBM though, as he cheerfully adds, “of course, they
could have been right.” Innovation unlike, apparently, sex, parenting and fitness is one area
where a computer cannot tell you what to do.

Questions 1 – 6. Which section contains the following information? Write the correct
number on the given boxes.
i The unpredictability of the public’s viewpoints about a certain topic in the future
ii A list of certain institutions that are having fewer business activities

iii A type of firms that are resorted to compulsive consumption for new ideas
iv The insatiable thirst for outstanding innovations being an impetus to big impacts on the
market.

v Some moguls which expressed financial concerns when investing in the acquisition of
smaller companies.
vi The reason why American business trends are highlighting the importance of initiatives

vii A company that is able to going through economic falters itself.


viii Small firms that can make certain impacts on bigger ones when the former possesses
more potential ideas.

ix Example of three famous American companies’ innovation


x A type of firms that are regarded fly-by-night when investing in entrepreneurship.

1. Section A 4. Section D

2. Section B 5. Section E

3. Section C 6. Section F

1. 2. 3. 4. 5. 6.

Questions 7-13. In boxes 7-13, write: Y (Yes), N (No), NG (Not Given)


Yes if the statement agrees with the claims of the writer
No if the statement contradicts the claims of the writer
Not Given if it is impossible to say what the writer thinks about this

7. Peter infers his unwillingness to invest more in restructuring his organization in return for
better creativity management.

8. Some small organizations have a craving for ideas that are regarded as an admixture of
“innovative” and “strange”.
9. Umagic is head and shoulders above other competitors in such a new field.

10. A new trend that has already superseded “entrepreneurship” in one area may directly
impact living organizations.
11. Big giants prioritize innovations with low certainty on the understanding that big risks are
parallel to big profits.

12. It takes many years for Mr Sanders to successfully ditch preconceived ideas in his
organization.
13. The author expressed a positive attitude towards the development of innovations at the
end of the passage.

7. 8. 9. 10. 11. 12. 13.

Part 4: In the passage below, seven paragraphs have been removed. For questions 1-7,
read the passage and choose from paragraphs A-H the one which fits each gap. There is
ONE extra paragraph which you do not need to use. Write your answers in the
corresponding numbered boxes provided. (7pts)
How satisfying to pull a chain again. Something went out of British plumbing with the arrival
of the integrated cistern, but everything that went out of British plumbing with the Victorians
has been reinstated in the stateliest form in the bathroom of our suite at the Pool House Hotel.
If only for that achievement alone, it deserves its AA accolade, awarded last Thursday, of
Scottish Hotel of the Year.

1.
In Wester Ross, the old parish of Gairloch - a glorious body of country clasped between the
long sea arms of Loch Torridon and Loch Broom - has all the classic components of the West
Highland landscape. It has the mighty Torridon range, the oldest rock in Britain; the moor-
and-mountain wilderness of Letterewe and the island-studded mirror of Loch Maree - an
inland loch more beautiful than any other, including Loch Lomond. It also has a lonely coast,
sandy bays, leafy glens, Hebridean vistas and numerous whitewashed villages.
2.

The Pool House building on the loch's foreshore, where the River Ewe enters the lake after a
short but vigorous journey from Loch Maree, doesn't look like a traditional Highland lodge.
But scrape away the white paint and roughcast and you will expose pink Torridonian
sandstone - the preferred building blocks of local lairds for three centuries.
3.

Osgood Mackenzie, who caused thin, acid layers of peat on a windswept headland to bloom
with the trees, shrubs and flowers of the temperate world from Chile to Tasmania, lived for a
time in Pool House while he worked obsessively on his horticultural masterpiece. Meanwhile,
his English wife whiled away the hours by carving a chain of Tudor roses in the banisters of
the central staircase.

4.
When the present owners, the Harrison family, made the decision to replace their 13
bedrooms with four themed suites it was Liz Miles who became the creative force. Liz
tracked down the extravagant wallpapers - putto friezes, Michelangelo ceilings, celestial
maps - and sourced most of the antique fittings and furniture.
5.

With some reluctance my husband and I jump ship - forcing ourselves out of the sumptuous
fantasy of Campania, with its 130-year-old cast iron and brass four-poster, to confront the
reality of the weather. As enthusiasts for the elemental challenge of the West Highland
seaboard, we have a busy programme: a rugged walk, a wildlife cruise and, as the softest
option locally available, a visit to Inverewe Garden.
6.

The headland couldn't be more exposed, but the squalls of rain beating in from the Atlantic
sail over our heads en route to the mountains. We flush grouse and snipe from the heather on
our three-hour walk and glimpse red deer. By the time we reach the great sea stacks of Stack
Dubh and Stac Buidhe, there are shafts of sunshine striking the wings of gannets, fulmars and
shags.
7.
We don't. But we do see grey seals, harbour porpoises, great skuas and - quite a spot for a
trainee birdwatcher like me - a huddle of rare, black-throated divers. Warblers and other
songbirds attend our visit to Inverewe Garden, now owned and maintained by the National
Trust for Scotland and not, perhaps, at their best on the cusp between summer and autumn.
But they are still remarkable.
Missing paragraph:

A. The Mackenzie country was dominant in this part of country. Pool House's golden age was
Victorian, when the Highlands became a sporting playground for the gentry. There was
salmon to pull from the River Ewe, deer to stalk and grouse to pot, but for a time the lodge
was home to one of the less predatory Mackenzies: a man who liked to let things grow rather
than cut them down.

B. There was only one willow tree on the promontory where Osgood Mackenzie began his
project in 1862. Now there is a prodigious stand of Scots pine and other native woodland,
planted to supply the windbreak for his exotic trees and shrubs. The contrast is beautiful.
C. Some visitors say they would be willing to pay merely to tour the rooms. Many are
especially fascinated with Diadem, which is modelled on the style of a first-class cabin on the
Titanic. Margaret Harrison's grandfather was a cousin of Captain Smith, the liner's master;
but Peter Harrison, who takes a keen interest in military history, has named all the suites after
warships as a tribute to Pool House's function during the Second World War, when it was the
Navy's headquarters for co-ordinating the North Atlantic and Murmansk convoys.

D. Not all the cottages in these old crofting townships are second homes or self-catering
units. The scattered "capital" of the parish, Gairloch, is something of a boom community,
with energetic young locals raising new houses on scenic building plots. At nearby Poolewe,
which has the botanical curiosity and tourist honeypot of Inverewe Garden, an old shooting
lodge is turning back the clock to find a future.
E. We usually do our own route finding but we want to investigate Rua Reidh Lighthouse,
where Fran Cree and Chris Barrett run residential walking and activity holidays on one of the
most remote headlands of the mainland. Just getting there is an adventure; and the airy nature
of the clifftop paths, with their views to the Outer Hebrides and dizzy drops into empty
beaches, makes us glad of the expert presence of Chris, who is a member of the local
mountain rescue team. "We get called out about a dozen times a year," she tells us.
F. Seldom has washing been such a treat. As I wallow beneath the cascading canopy of the
Shanks Independent Spray Bath (built in Glasgow in 1875 on a scale comparable to the boiler
of a Clyde steamship), I feel a certain kinship with the grey seal idling in the water outside.
From the bathroom window I can see the glassy surface of Loch Ewe - and much of its
wildlife.
G. Our tally of wildlife soars on the sturdy Starquest, which skipper Ian Birks steers to the
wide mouth of Loch Gairloch and the first tugging of the Atlantic. It's the whale-watching
season - the Minch is part of the minke's larder - and it's part of Ian's purpose to monitor their
movements for the Sea Watch Foundation. "But I never advertise these trips as whale-
watching cruises, otherwise people expect to see whales."

H. The carvings are still there, one of the few remnants of the 19th century to survive. How
can this be? From the outside Pool House may look like a made-over inn with 1960s add-ons;
step inside and you enter into the rich, decorative and occasionally camp interior of a
Victorian country house. Yet almost all its finest features, from huge, wood-panelled
bathrooms to marble and polished steel fireplaces, have been retrieved from architectural
salvage yards and put in place over the past three years.

1. 2. 3. 4. 5. 6. 7.

Part 5: You are going to read an article containing reviews of computer games. Choose
from the reviews (A-E). The reviews may be chosen more than once. Write your
answers in the corresponding numbered boxes provided. (15pts)
A. The retreat was a hasty as it was disorganized, and the fall of Galdor was completed. Had
they held out until the bitter end matching their foes blow for blow with conviction and
strength, they could at least have held their heads high, but Ruan knew all semblance of hope
had evaporated with the battle pressure, and that was what worried him the most, for Oldark
had yet to unleash his full wrath upon the people of Appleton. Though overcome with
frustration, Ruan knew in his heart of hearts that he couldn’t hold Redhorn and the others to
account for their wimpishness; after all, in much the same wat as Rome was not built in a
day, nor were farmers reshaped into warriors overnight. Weary, tired and near broken, Ruan
new the task ahead of him was gargantuan, yet, he told himself, again and again, that he must
succeed.
B. At least their numbers were little depleted, he thought. Such had been the haste to flee that
his men had largely escaped unscathed. Perhaps a handful, he estimated, had fallen, and
perhaps six dozen more had been wounded. Then, momentarily, he almost lost all his
composure as the realization hit him hard like a knife delivered heart-bound with deadly
accuracy. Orlach was not amongst the motley assemblage regrouping around him. What fate
had befallen him? His dearest brother – not kin, it’s true, but their bond to succeed bereft of
the courage Orlach imbued in him? He was, he thought, leader in but name only, for it was
Orlach’s fearlessness that had always driven him forth, sustained him and helped him keep
faith despite the faintest of hopes. Without Orlach, all was lost.

C. Ruan collected himself, eyeing its hapless followers and knowing he must deliver a
rousing message of hope with convincing, albeit false conviction. He held him arm aloft and
a hush descended over the gathering crowd. He gestured towards the woods, where his own
precious spouse and the other brave women of Appleton stood ready to defend their children
to the last; safe as yet, but for how long? He knew they would never abandon their post and
their responsibilities so pitifully as he and his men just had. There would beat no hasty retreat
if it came to it, though he hoped it never would, for that would surely be the end, and he and
his brethren would have utterly failed. So he told his weary listeners they were farmhands no
more and he gestured again towards the hidden dwelling in the woods.

D. Meanwhile, the plumes of smoke coming from what had once been their beloved village
were already evident on the distant horizon. Oldark had razed it to the ground. But Appleton
was alive, he told them, in their hearts and souls. And it would be resurrected. Oldark could
not destroy Appleton: not while a single Appletonian heart was yet beating. Indeed, so far, he
had not even come close. He had but peeled away the surface layter. Houses could be rebuilt.
Yes, the first layer of skin was gone, and it felt raw and bitter now, Ruan ceded. But there
were two more layers, were there not? The body of Appleton had not even been pierced the
once as yet. He stared at his men and thundered his words. They were the skin and bones; the
veins and the arteries; the muscle and sinew, he told them. Their brave spouses, the child
protectorate, the vital organs, the beating heart of Appleton. And the soul, what else could it
be? The younglings. It ran and played and skipped and screamed and smiled everyday; it
brought them love and happiness and completion. What else could it be indeed?
E. Suddenly, this hapless band of farmers realized that, really, the battle had barely begun.
What was lost was nothing, and all was yet to fight for. No less cowed than before, from
somewhere deep inside rumbled the warrior soul and their bellies fired with a primeval sense
of purpose. They would stand and fight yet, and do so with every last sinew of strength they
could muster from their bones, and Oldark would be dethroned. His reign of terror would and
as surely as the harshest winter must eventually retreat and give way to spring, and Appleton
would blossom again. And there would be running and playing and skipping and screaming
and smiling once more. Ruan felt Orlach’s strength within him as though he were still there
and his words no longer carried false conviction. He and his brethren were as men possessed,
and they would prevail.

In which extract…

does Ruan almost panic and lose control? 1. .............


is one of Ruan’s close relatives mentioned? 2. .............
is the nature of a defeat described? 3. .............
does the loss of the close friend leave Ruan feeling dejected? 4. .............
does Ruan compare the different genders of his people unfavourably? 5. .............
are children attributed a sort of spiritual significance? 6. .............
do we learn the fate of Appleton village? 7. .............
does the memory of someone give Ruan more resolve? 8. .............
does Ruan liken his people’s suffering to a wound? 9. .............
does Ruan find some relief in the fact that defeat was not heavy in 10. .............
casualities?

SECTION IV. WRITING (60pts)


Part 1: Read the following extract and use your own words to summarize it. Your
summary should be about 140 words. You MUST NOT copy the original. (15pts)
The Nature of Genius

There has always been an interest in geniuses and prodigies. The word ‘genius’, from the
Latin gens (=family) and the term ‘genius’, meaning ‘begetter’, comes from the early Roman
cult of a divinity as the head of the family. In its earliest form, genius was concerned with the
ability of the head of the family, the paterfamilias, to perpetuate himself. Gradually, genius
came to represent a person's characteristics and thence an individual's highest attributes
derived from his ‘genius’ or guiding spirit. Today, people still look to stars or genes,
astrology or genetics, in the hope of finding the source of exceptional abilities or personal
characteristics.

The concept of genius and of gifts has become part of our folk culture, and attitudes are
ambivalent towards them. We envy the gifted and mistrust them. In the mythology of
giftedness, it is popularly believed that if people are talented in one area, they must be
defective in another, that intellectuals are impractical, that prodigies burn too brightly too
soon and burn out, that gifted people are eccentric, that they are physical weaklings, that
there's a thin line between genius and madness, that genius runs in families, that the gifted are
so clever they don't need special help, that giftedness is the same as having a high IQ, that
some races are more intelligent or musical or mathematical than others, that genius goes
unrecognised and unrewarded, that adversity makes men wise or that people with gifts have a
responsibility to use them. Language has been enriched with such terms as ‘highbrow’,
‘egghead’, ‘blue-stocking’, ‘wiseacre’, ‘now-all’, ‘boffin’ and, for many, ‘intellectual’ is a
term of denigration.

The nineteenth century saw considerable interest in the nature of genius, and produced not a
few studies of famous prodigies. Perhaps for us today, two of the most significant aspects of
most of these studies of genius are the frequency with which early encouragement and
teaching by parents and tutors had beneficial effects on the intellectual, artistic or musical
development of the children but caused great difficulties of adjustment later in their lives, and
the frequency with which abilities went unrecognised by teachers and schools. However, the
difficulty with the evidence produced by these studies, fascinating as they are in collecting
together anecdotes and apparent similarities and exceptions, is that they are not what we
would today call norm-referenced. In other words, when, for instance, information is collated
about early illnesses, methods of upbringing, schooling, etc., we must also take into account
information from other historical sources about how common or exceptional these were at the
time. For instance, infant mortality was high and life expectancy much shorter than today,
home tutoring was common in the families of the nobility and wealthy, bullying and corporal
punishment were common at the best independent schools and, for the most part, the cases
studied were members of the privileged classes. It was only with the growth of paediatrics
and psychology in the twentieth century that studies could be carried out on a more objective,
if still not always very scientific, basis.

Part 2: CHART DESCRIPTION (15pts)


The bar chart below shows the percentage of government spending on roads and
transport in 4 countries in the years 1990, 1995, 2000, 2005.

Summarise the information by selecting and reporting the main features, and make
comparisons where relevant.

Percentage of government spending on road and transport (1990 -


2005)
30

25

20
Percentage

1990
15 1995
2000
10
2005
5

0
Italia Portugal UK USA

Part 3: ESSAY WRITING (30pts)


Some people believe that no one should do the same job for all their working life. Others
argue that doing the same job brings advantages for individuals, companies, and
society. Discuss both views and give your opinion.
Give reasons for your answer and include any relevant examples from your own
knowledge and experience. Write at least 250 words.

Người ra đề: Trần Thị Ánh Nguyệt - 0913310100


SỞ GIÁO DỤC & DÀO TẠO TỈNH KỲ KIỂM TRA CHẤT LƯỢNG CÁC
BÌNH DƯƠNG ĐỘI TUYỂN HSG
TRƯỜNG THPT CHUYÊN HÙNG VƯƠNG NĂM HỌC 2021-2022
MÔN: TIẾNG ANH – KHỐI 11
ĐỀ ĐỀ XUẤT Thời gian làm bài: 180 phút
(Đề thi gồm 18 trang) (không kể thời gian giao đề)

HƯỚNG DẪN CHẤM


I. LISTENING (25 x 0.2 = 5 points)
Part 1
1.T 2.F 3.F 4.T 5. T

Part 2
1. use bullet points
2. local wildlife, vegetation
3. 12.000
4. horses
5. the local caves

Part 3
1. C 2. A 3. C 4. B 5. C

Part 4
(1) interdisciplinary thinking
(2) minute, extreme detail
(3) the surgery suite
(4) image modalities
(5) inflammatory bowel disease
(6) medical avatar
(7) time-series data
(8) comprehensive information
(9) holistic digital healthcare
(10) sick-care/”sick-care”/sick care/”sick care” system

II. LEXICO - GRAMMAR (3 points)


Part 1 (20 x 0.1 = 2 points)
1. A 2. D 3. D 4. B 5. D 6. A 7. C 8. C 9. A 10. B
11. B 12. C 13. B 14. A 15. C 16. A 17. B 18. D 19. D 20. A
Part 2 (10 x 0.1 = 1 point)
1. governor 2. Incivility 3. scepticism 4. seemingly 5. deterrents
6. extremist 7. alliance 8. outweighs 9. assembly 10. relentlessly

III. READING (6 points)


Part 1 (10 x 0.1 = 1 point)
1. itself 2. than 3. Apart 4. at 5. up
6. Despite 7. capable 8. this/that 9. in 10. such

Part 2 (10 x 0.1 = 1 point)


1. A 2. C 3. B 4.D 5. A
6. B 7. A 8. D 9. A 10. D

Part 3 (13 x 0.1 = 1.3 points)


1. iii 2. vi 3. v 4. x 5. iv
6.viii 7. i 8. wheels 9. film 10. filter
11. waste 12. performance 13. servicing

Part 4 (0.7 x 0.1 = 0.7 point)


1. F 2. D 3.G 4. H 5. B
6. E 7. A

Part 5 (10 x 0.2 = 2 points)


1. B 2. D 3. D 4. B 5. C
6. E 7. B 8. C 9. A 10. B

WRITING (6 points)
PART 1. Writing summary (1.5 pts)
Contents (1 pt)
The summary MUST cover the following points:
+ problems of traditional methods of writing assessment
+ new alternative approach of writing assessment
Language use (0.5 pt)
The summary:
- should show attempts to convey the main ideas of the original text by means of paraphrasing
(structural and lexical use)
- should demonstrate correct use of grammatical structures, vocabulary, and mechanics (spelling,
punctuations, ...)
- should maintain coherence, cohesion, and unity throughout (by means of linkers and
transitional devices)

PART 2: Describing Chart (1.5 pts)


Contents (1.0 pt)
The report MUST cover the following points:
- Introduce the charts (0.2 pt) and state the overall trends and striking features (0.2 pt)
- Describe main features with relevant data from the charts and make relevant comparisons (0.6 pts)
Language use (0.5 pt)
The report:
- should demonstrate a wide variety of lexical and grammatical structures,
- should have correct use of words (verb tenses, word forms, voice...) and mechanics (spelling,
punctuations...)

PART 3: Writing essay (3 pts)


The mark given to part 3 is based on the following criteria:
1. Organization (0.5 pt)
a. Ideas are well organized and presented with coherence, cohesion and unity.
b. The essay is well-structured:
* Introduction is presented with clear thesis statement.
* Body paragraph are written with unity, coherence and cohesion.
Each body paragraph must have a topic sentence and supporting details and examples when
necessary.
* Conclusion summarizes the main points and offers personal opinions (prediction,
recommendation, consideration ...) on the issue.
2. Content (1.5 pts)
a. All requirements of the task are sufficiently addressed.
b. Ideas are adequately supported and elaborated with relevant and reliable explanations,
examples, evidence....
3. Language use (1 pt)
a. Demonstration of a variety of topic-related vocabulary.
b. Excellent use and control of grammatical structures (verb tenses, word forms, voice...) and
mechanics (spelling, punctuations...)
HỘI CÁC TRƯỜNG THPT CHUYÊN KỲ THI CHỌN HỌC SINH GIỎI LẦN THỨ XIII
KHU VỰC DUYÊN HẢI, ĐỒNG BẰNG BẮC BỘ MÔN THI: TIẾNG ANH – KHỐI 11
TRƯỜNG THPT CHUYÊN BIÊN HOÀ, T. HÀ NAM Ngày thi 14/07/2021
Thời gian làm bài 180 phút
(HDC gồm 07 trang)

HƯỚNG DẪN CHẤM ĐỀ THI ĐỀ XUẤT

SECTION A. LISTENING (50 points)


Part 1. You will hear an interview with Dr Lafford, a leading expert in the field of forensic science.
For questions 1-5, decide whether the following statements are True (T) or False (F) according to what
you hear. Write your answers in the corresponding numbered boxes provided. (10 points)
1. F 2. T 3. F 4. T 5. T

Part 2. Listen to a lecture about water and answer the questions. Write NO MORE THAN THREE
WORDS taken from the recording for each answer. (10 points)
1. safe and reliable
2. washing
3. creeks, lakes, dams
4. filtth and dirt
5. drinking

Part 3. You will hear part of a radio phone-in programme about consumer competitions that appear
in magazines or are run by shops, in which advice is given to people who regularly enter them. For
questions 1-5, choose the answer (A, B, C or D) which fits best according to what you hear. Write your
answers in the corresponding numbered boxes provided. (10 points)
1. A 2. D 3. C 4. C 5. B

Part 4. Listen to a news about dogs trained to detect Covid Infection at Miami Airport. Complete the
following summary with NO MORE THAN THREE WORDS taken from the recording for each gap.
Write your answers in the corresponding numbered boxes provided. (20 points)
1. deactivated 6. explosives
2. a pcr test 7. face masks
3. weapon 8. specific variant
4. shepherd 9. delta
5. 98.1 10.apocalyptic

SECTION B. GRAMMAR & VOCABULARY (30 ponits)


Part 1: Choose the answer A, B, C, or D that best completes each of the following sentences. Write
your answers in the corresponding numbered boxes. (20 points)
1. D 2. C 3. A 4. C 5. D 6. A 7. A 8. B 9. B 10. B

11. B 12. B 13. C 14. D 15. B 16. D 17. C 18. D 19. D 20. B
Part 2: Give the correct form of each bracketed words. Write your answers in the corresponding
numbered boxes. (10 points)
1. proofreading 6. benighted
2. werewolf 7. betokens
3. excommunicated 8. dietetics
4. aquaculture 9. burdensome
5. unsightly 10. anew

SECTION C. READING (60 points)


Part 1: Read the text below and think of the word which best fits each space. Use only ONE word in
each space. Write your answers in the corresponding numbered boxes. (15 points)
1. developed 2. led 3. treating 4. put 5. behind
6. this 7. into 8. not 9. remains 10. but

Part 2. Read the following passage and choose the answer (A, B, C or D) which fits best according to
the text. Write your answers in the corresponding numbered boxes. (10 points)
1. A 2. C 3. C 4. B 5. B 6. D 7. A 8. C 9. A 10. B

Part 3. Read the passage and do the tasks that follow. Write your answers in the corresponding
numbered boxes. (13points)
1. A/C 2. C/A
3. taxi drivers 4. detailed mental map 5. happocampus
6. games and puzzles 7. class attendance and behaviour 8. test scores
9. E 10. G 11. C 12. F 13. D
Part 4. You are going to read an extract from an article. Seven paragraphs have been removed from
the extract. Choose from the paragraphs A - H the one which fits each gap (1-7). There is one extra
paragraph which you do not need to use. Write your answers in the corresponding numbered boxes.
(7 points)
1. D 2. H 3. F 4. B 5. A 6. E 7. G

Part 5. You are going to read an article on travel adventures. For questions 1-10, select the travel
writer (A-E) using the separate answer sheet. Each travel writer may be selected more than once.
(15 points)
1. C 2. D 3. E 4. B 5. C 6. B 7. E 8. A 9. A 10. D

SECTION D. WRITING (60 points)


Part 1. (15 points)
Contents (10 points)
The summary should:
 introduce the topic of the passage,
 present the main ideas of the passage.
* lack of confidence
* too strictly held down or laxly handled
* parents too protective
* lack of trust
* lack of guidance and experience
* give child more responsibility, praise

Language use (5 points)


The summary should:
 demonstrate a wide variety of vocabulary and grammatical structures,
 have correct use of words and mechanics,
 maintain coherence, cohesion, and unity throughout.
Part 2. (15 points)
Contents (10 points)
The report should:
 introduce the table and state its striking features,
 summarise the main features with relevant data from the pie charts,
 make relevant comparisons.
Language use (5 points)
The report should
 demonstrate a wide variety of vocabulary and grammatical structures,
 have correct use of words and mechanics,
 maintain coherence, cohesion, and unity throughout.

Part 3. (30 points)


Task achievement (10 points)
The essay should:
 sufficiently address all requirements of the task,
 develop relevant supporting ideas with explanations, examples, evidence, etc.
Organization (10 points)
The essay should have
 an introduction presenting a clear thesis statement introducing the points to be developed,
 body paragraphs developing the points mentioned in the introduction,
 a conclusion summarising the main points discussed in the essay.
Language use (10 points)
The essay should:
 demonstrate a wide variety of vocabulary and grammatical structures,
 have correct use of words and mechanics,
 maintain coherence, cohesion, and unity throughout.

Tapescript:
Part 1.
Exam narrator: You will hear an interview with Dr Lafford, a leading expert in the field of forensic science.
For questions 1-5, choose the answer (A, B, C or D) which fits best according to what you
hear.
Interviewer: Today in the studio, we have Dr Lafford from the Forensic Science Service, who is here to tell
us a little more about forensic science ...
Dr Lafford: Good morning.
Interviewer: Now, I think most people have a fairly good general picture of the kind of work you do - in
many ways it's a development of Sherlock Holmes and his magnifying glass, isn't it?
Dr Lafford: Yes, you could put it that way, although we've come a long way since then of course, but
Holmes was very much a detective of the modern age - of course he was an intelligent man and
his medical knowledge often seemed better than that of his companion Dr Watson, and
certainly his observations were always very detailed... but what sets him apart is that (1)
Holmes provides us with a good model - a very good model in fact - of the kind of approach -
psychological approach - that a forensic scientist should have, and that is that the forensic
scientist is absolutely impartial.
Interviewer: So, you're not strictly speaking part of the police force ... ?
Dr Lafford: No, we are quite separate.
Interviewer: Could you tell us a little about your work and what it is you do?
Dr Lafford: (2) Well, the basic principle behind forensic science is that every contact leaves a trace.
Wherever we go, whatever we touch, material is transferred both ways. The shoe that leaves a
mark on the ground also picks up traces of dirt, and the hand that makes a fingerprint also
carries away particles from the object that was handled ... and things like hair or carpet fibres
are easily passed from one person to another. Interviewer I see, so it's not just that the suspect
leaves clues at the scene of the crime, the scene of the crime leaves clues on the suspect...
Dr Lafford: Absolutely, and that is normally the most compelling evidence - not just when you have a
fingerprint, for example, but also where you've found traces of, say, the carpet at the scene of a
crime on the suspect's clothing, and that, of course, can be very hard to explain away.
Interviewer: Is your work fairly straightforward - a question of seeing whether various samples match up?
Dr Lafford: No, no, no, not at all... (3) forensic science is often a very painstaking task - and very time-
consuming - much more so than previously, in fact, because the range of tests has mushroomed
... for example, we might be given some pieces of a headlight swept up after a hit-and-run
accident and asked to identify the car, so we would fit the thousands of pieces together like a
vast jigsaw, and might be able to identify the number embossed on the back of the glass... that
would help us identify the model of the car, the make, the age, and so on, making it easier to
search for the suspect... and then, once the car was located, we could tell whether the lights had
been on at the time of the accident by examining the light bulb, because there would be minute
pieces of molten glass on the metal filament in the bulb...
Interviewer: So you need some fairly sophisticated equipment as well as patience...
Dr Lafford: Indeed! Now, the electron microscope is invaluable in our line of work - and it basically does
two things - it gives us a fantastically detailed image, so, for example, you can tell whether a
piece of hair has been broken, or cut with a pair of scissors or a knife. (4) And it can also give
you the chemical composition of a sample, so that you can take, perhaps, a tiny flake of paint
on a suspect's clothing and match it with paint at the scene of the crime.
Interviewer: Are you at the stage then where you could say that the evidence that you provide is foolproof?
Dr Lafford: No, not foolproof. There is always room for human error and there's nothing that can be done
about that... forensics is not an absolute science - I mean, let me give you an example. Now,
Sherlock Holmes might have found a strand of hair at the scene of a crime, perhaps black and
two inches long, that would have helped him identify the murderer on the basis that the hair
matched the murderer's. Now, we also examine hair... but in the example I gave you, perhaps
one person in twenty has black hair two inches long, so we need to improve on this analysis
(5)to narrow down the number of suspects. So, we would analyze the strand of hair... using the
electron microscope, as I was saying earlier... look at its chemical composition, whether it came
from a man or a woman, see how it was cut, whether it showed any traces of particular
chemicals and so on, and in that way, (5) instead of saying that one person in twenty had hair
like that, it might be one person in 500,000 or a million. That would just about as far as we
could go. Now there's no such thing a certainty ... just a balance of probability, even with the
most advanced techniques of genetic fingerprinting. The evidence we provide is there to be
interpreted, and that's very important. Interviewer Well, Dr Lafford, thank you very much.
We'll take a break now, but stay with us and we'll be back ...

Part 2:
Good morning and welcome to yet another lecture in Environmental Science. I don’t think I am telling
you a secret when I mention that water is a big worry here in Australia. The stuff is scarce. Perhaps that’s
why we drink so much beer, eh?
Seriously, though, a safe and reliable source of water is one of the great concerns of people
everywhere. Moreover, as the world population grows, the pressure on existing water supplies grows greater
and greater. Think about it. Our economic system demands that there be more and more consumers. The
growing number of people has been tied to climate change, including droughts. So more people means less
water. But our economic system demands a high birth rate. Forget about oil. Soon enough you will see wars
for water. Mark my words.
But today, I’m going to confine my remarks to Australia. As noted already, here down under, the water
supply is extremely scarce. The only drier continent is Antarctica - and remember, no one really lives there
anyway. Moreover, in recent years, the wind patterns have changed. Rain that used to fall on the country
now falls out to sea hundreds of miles to the south.
Now, when I speak of people needing water, most of you probably think of drinking. Certainly everyone
needs water for drinking. But surprising as it may sound, drinking is not anywhere near being the main use
for water. Most water is actually used for washing. When you take a shower, you probably use well over a
hundred litres of water. Every time you flush your toilet, that’s about eight litres. But most people drink no
more than two litres or so per day.
So, where to get water? It could be obtained from rainwater, but often rainfall consists of other harmful
pollutants that evaporated with the water. In fact, acid rain, an intense example of this, causes harmful
effects on the wildlife of the habitat on which it falls. Water from underground could also be used, though it
is more difficult to contain and often must go through an extensive cleansing process. The purest water is
found in rivers, creeks, lakes, and dams. And, sad to say, Australia has precious few of these. Really, how
many of your home towns have rivers? Year-round rivers, I mean. The soil tends to be sandy, so water soaks
into the ground. Many places are rocky too, so 87% of the rainfall is lost to evaporation. That’s almost twice
the evaporation rate in my native Canada!
Speaking of rain, we already heard how rainfall is diminishing here in Oceania. The quantity itself isn’t
the only problem, either. Going back to the problems with obtaining rainwater, a further problem is that rain
is a useful source of water only if air pollution is fairly mild. Again, you’re in a situation where you can’t
win. You need water where most people live. People tend to build cities where rainfall is adequate. But then
modern cities tend to feature polluted air which renders the rain far less easily usable. OK, let's take a look at
the table here, you’ll see it showing the relative pollution of rainfall in the world’s cities. The more people,
the dirtier the rain.
This is becoming a huge concern for people in the West, who want their water to be pure and safe.
Though reliable drinking water is important everywhere, the concern in the West is reflected in all the
government regulations and political campaigns aimed at solving this problem. In contrast, there are not as
many demands made on the governments in Asian and African cultures to improve the water, as their focus
is on other issues.
Now, whatever the source of water, we can never afford to forget that all water is highly vulnerable to
contamination. Whether we’re getting it from the ground, from bodies of water, or rainfall, it is susceptible
to a variety of toxins. In fact, that’s why we clean it before using it: water carries with it filth and dirt. This
problem shows up in a number of different ways.
As humans and all other animals need water to survive, it's no surprise to us that one of the most
important domestic uses of water is for drinking. Yet if you have old-fashioned lead pipes, you may slowly
be poisoning yourself by drinking that nice clear water. The industrial pollution, farm chemicals, and leaky
landfills are well-known sources of contaminants as well.
So what is being done to ensure we Australians a safe and steady supply of drinking water? There are a
lot of initiatives that make admirable efforts to remedy this issue. We’ll be talking about this when we meet
again on Thursday. But, as a preview, I can tell you that so far the amount of real solutions that have been
produced is not nearly adequate. Traditionally, we’ve been very free in this country. That means that every
person and every province tend to go its own way. So the mechanisms for water management are, in a word,
insufficient.
To begin seeing how this is so, I want you to read something before our next class. Though a lot of
previous data on water usage and water management are inconclusive and have thus caused quite a concern,
we can learn a lot from the content of reports written on the subject.
The basis for the government’s water policy is the 1989 White Paper reporting on “Water Use: Present
and Future”. If you compare the numbers offered in the paper with those in the text, you’ll find that the
report is rather untrustworthy. Truth being told, I’m being too kind when I say that!

Part 3.
Presenter: OK, today I have with me Kathy Ford, winner of more than 500,000 worth of prizes in all sorts
of consumer competitions and dubbed ‘The Queen of Competitions’ by the British press. She’s
now editor of Competitor’s World magazine and as an expert on competitions has appeared
regularly on TV. Kathy, let’s go straight to our first caller, and that’s Diana. Diana, what’s your
query?
Diana: Yes, hello Kathy. Well, in order to send in two entries to a competition where only one entry per
person was allowed, I asked my best friend if I could submit an entry in her name. She agreed, and
the understanding was that, if ‘her’ entry won, I would receive the prize, but I would buy her a
small gift for allowing me to use her name. Well, the inevitable has happened - I’ve won a much-
needed new washing machine, but in my friend’s name, and she has now refused point blank to
hand the machine over. If I went to a lawyer, would I have any hope of getting my prize from her?
Kathy: Not even the faintest chance. I’m afraid that your efforts to evade the rules have not only cost you
the prize, but also your best friend as well, and legally you just don’t have a leg to stand on. Even
if you’d drawn up some sort of legal agreement with your erstwhile friend, I think you’d find that
the law would still take a very dim view of your case, since it was obviously done with
premeditated fraudulent intent. It’s not worth trying to evade the rules as you’ve just found out the
hard way.
Presenter: Next, it’s Ron. Ron, go ahead, you’re through to Kathy.
Ron: Someone told me that some firms that run competitions keep a blacklist of frequent prizewinners, and
that I should use a lot of different aliases in order to avoid being put on such a list. Is this true?
Kathy: No! Competitors can sometimes get a little paranoid, and if they start going through a winless spell
(and we all get them, from time to time!) they start to imagine that they’ve been blacklisted. No
reputable firm would even contemplate such a measure, and the only time there’s even a faint risk
of this sort of thing happening is with ‘in store’ competitions, where an individual store manager
might just conceivably think ‘Oh no, not him again’ and deliberately disregard your entry. For
mainstream competitions, however, such worries are groundless, and the use of aliases is not only
unnecessary but can even prove to be pretty stupid. Think about it for a moment - what would
happen if you won a holiday under a phoney name? Or were asked to prove your identity to
collect a prize at a presentation ceremony? My advice is to stick with your own name and if prizes
stop arriving, take a long, close look at the quality of your entries rather than trying to blame it on
blacklists.
Presenter: OK, next it’s Stan. Stan, what can Kathy help you with?
Stan: Well, Kathy, I recently entered a competition which asked you to estimate the distance between a store
in Newcastle and its London head office, using the shortest route. In order to make my entry as
accurate as possible, I used a Routemaster computer program to determine the shortest possible
way and calculate the distance, quite literally, from door to door. Imagine my astonishment,
therefore, when I sent for the results and found that the answer they had given as being ‘correct’
was fully 73 miles longer than mine. I know my answer was correct, so do I have grounds to make
a formal objection?
Kathy: I’m sorry, but no, you haven’t. As far as the promoter is concerned, the key word in the instructions,
here, is ‘estimate’ - they expect you to guess, not measure the distance accurately, and it’s likely
that their own answer will also be based purely on an estimate. As a result, judges will always be
right, even when they are wrong as in a case like this, and in entering the competition at all, you
have agreed to abide by the rule that states ‘the judges’ decision is final’. Distance estimation
competitions have always given rise to this sort of controversy, and although court cases have
been brought, the entrant very seldom succeeds in having the decision changed. You have only to
check the distance charts in road atlases to see how this type of problem occurs. No two ever
agree, yet as far as I know, towns simply don’t move around very much!
Presenter: OK, and now on to our next caller, who is ...

Part 4. https://www.youtube.com/watch?v=dhZlSxN162Y
At some very important level this is good specifically this is a good girl a good girl named Cobra because
because why not a dog looking for Covid is it even possible evidently it is and now you can find them at
Miami international airport four dogs trained to detect Covid 19 as part of a pilot program this is cobra one
of the dogs trained to detect the virus watch how she sits after she smells the inside of a mask that has
deactivated virus in it then watch how she just keeps walking when she smells the mask of someone who's
not infected the dogs are 97 percent accurate i mean it's the same as a pcr test so i mean it's a great it's a great
thing miami international airport is the first in the nation to have the dogs i think in this particular airport
now we're doing it with the employees but people uh will see the dogs and know that that's another weapon
that we're using to fight this pandemic that's coverage from wtvj um in in miami uh one of the dogs is named
cobra she's a seven-year-old belgian melanois the other dog is also a seven-year-old girl she is a dutch
shepherd her name is one betta as in I can do that one better you want to know how it breaks down dog by
dog of course you do uh one betta the the uh the dutch shepherd her accuracy rate is 98.1 percent and Cobra
the belgian melanois here uh she is 99.4 percent accurate as a covid test the dog is and that is a published
peer-reviewed double-blind study I mean we're familiar with dogs sniffing for drugs dog sniffing for
explosives in a custom setting some dogs are trained to sniff out large amounts of paper currency that might
be hidden in someone's luggage dogs can do amazing stuff these particular dogs before becoming kovid
experts they were agriculture detective dogs their specialty in customs was i kid you not sniffing out laurel
wilt which is apparently terrible for avocado Trees so it's their job to like sniff agricultural products and find
out if anybody was accidentally bringing laurel wilt into the country but now they're off laurel wilt the laurel
wilt beat leaving that to other pups and these dogs are retrained to fin kovid instead here's the provost at
florida international university's a chemistry professor this is where they trained the dogs explaining how it
works when you get Covid 19 you produce volatile organic compounds it's called on the chemist so vocs
and those vocs are what the dog detects researchers at fiu trained the dogs to detect cobit 19 by using face
masks from cobot patients we basically use ultraviolet light to kill the virus and the odor is still there so he
goes on to explain that these dogs are so good at sniffing out Covid 19 infections sniffing out the volatile
organic compounds produced by someone who's infected he goes on to explain that if you wanted to you
could actually train these dogs to key on one specific variant of the virus their noses are so sensitive you
could train them specifically to only sit down if they sniffed the delta variant of the virus or the move variant
of the virus or whatever they can be that sensitive we may have an apocalyptic mess on our hands in every
other possible way but as a civilization we have now successfully retrained the avocado tree laurel wilt
agricultural pest dog detectives into dogs who instead can tell you if you need to get a rapid covent test
before you come into work

- The end –
THPT CHUYÊN CAO BẰNG ĐỀ THI CHỌN HỌC SINH GIỎI KHU VỰC
ĐỀ THI ĐỀ XUẤT DUYÊN HẢI VÀ ĐB BẮC BỘ NĂM 2022
MÔN THI: ANH - LỚP: 11
Thời gian:180 phút (không kể thời gian giao đề)
Đề thi gồm 26 trang

A. LISTENING (50 pts)


Part 1. Listen to an interview with someone who consulted a 'life coach' to improve her
life and choose the answer (A, B, C or D) which fits best according to what you hear.
Write your answers in the corresponding numbered boxes provided. (10pts)
(Proficiency Test Builder 4th)
1. Brigid says that she consulted a life coach because
A she had read a great deal about them.
B both her work and home life were getting worse.
C other efforts to improve her life had failed.
D the changes she wanted to make were only small ones.
2. What did Brigid's coach tell her about money?
A It would be very easy for Brigid to get a lot of it.
B Brigid's attitude towards it was uncharacteristic of her.
C Brigid placed too much emphasis on it in her life.
D Few people have the right attitude towards it.
3. What does Brigid say about her reaction to her coach's advice on money?
A She felt silly repeating the words her coach gave her.
B She tried to hide the fact that she found it ridiculous.
C She felt a lot better as a result of following it.
D She found it difficult to understand at first.
4. What does Brigid say happened during the other sessions?
A She was told that most people's problems had the same cause.
B Her powers of concentration improved.
C Some things she was told to do proved harder than others.
D She began to wonder why her problems had arisen in the first place.

Page 1 of 26
5. What has Brigid concluded ?
A The benefits of coaching do not compensate for the effort required.
B She was too unselfish before she had coaching.
C She came to expect too much of her coach.
D It is best to limit the number of coaching sessions you have.
Your answers
1. 2. 3. 4. 5.

Part 2. Listen to a talk about ethical concerns with artificial intelligence and decide
whether these statements are True (T), False (F). Write your answers in the
corresponding numbered boxes provided.
(10 pts)- https://www.youtube.com/watch?v=1LyacmzB1Og
1. People tend to be oblivious to the moral issues associated with artificial intelligence in the
research stage.
2. Video tracking can be deployed to speed up recovery from injuries.
3. A handful of giants have become the dominant overlords in the realm of artificial
intelligence.
4. Artificial intelligence is intended to harm human society as it does not share human
values.
5. Machines are vulnerable to biases from the data introduced by their human makers.
Your answers
1. 2. 3. 4. 5.

Part 3. Listen to a talk about pandemic diseases and supply the blanks with the
missing information. Write NO MORE THAN THREE WORDS taken from the
recording for each answer in the space provided. (20 pts)
https://www.youtube.com/watch?v=nULJpyBbqpw

Page 2 of 26
TOP 5 DEADLIEST PANDEMIC DISEASES

1. Smallpox
• first emerged around 400BC
• caused 1.______________________ all over the body
• the only disease declared to be 2.______________________
2. Bubonic Plague
• also known as the 3.______________________
• caused by a bacterium spread by 4.______________________
• 5.______________________, called buboes, occurred in the body
3. The Spanish Flu
• began and ended in a 6.______________________
• filled the lungs of patients with 7.______________________
4. Malaria
• categorized as a blood disease
• caused by 8.______________________
• Anopheles mosquitoes would 9.______________________ infected blood and pass
it on to the next person they bite
5. HIV/AIDS
• often sexually transmitted
• HIV 10.______________________ the immune system
Part 4. Listen to the introduction about Manham Port and answer the questions. Write
NO MORE THAN THREE WORDS for each answer. (10pts)-
IELTS 10- Listening Test 4
1. What caused Manham’s sudden expansion during the Industrial Revolution?
…………………………………………………………………………………………………
2. Why did rocks have to be sent away from Manham to be processed?
…………………………………………………………………………………………………
3. What happened when the port declined in the twentieth century?
…………………………………………………………………………………………………
4. What did the Manham Trust hope to do?

Page 3 of 26
…………………………………………………………………………………………………
5. What is the name of the beautiful old sailing ketch near the school?
……………………………………………………………………………….
Your answers
1. 2. 3. 4. 5.

B. LEXICO AND GRAMMAR (30 pts)


Part 1: Choose the best answer to each of the following questions. Write your answers
in the corresponding numbered boxes (20 pts) (WORD PERFECT + OLYMPIC 30-4)
1. The science teacher asked the class to _____ the results of their experiment on a graph.
A. draw B. illustrate C. plot D. sketch
2. She's such a ______ of strength that everyone relies on her in a crisis.
A. tower B. pillar C. post D. support
3. Tom was able to ______ a pretty picture of the situation and impressed his manager.
A. paint B. draw C. present D. make
4. It is far too easy to lay the blame ______ on the shoulders of the management.
A. flatly B. willingly C. squarely D. perfectly
5. David has a(n) ______ to fainting at the sight of blood.
A. inclination B. predilection C. predisposition D. penchant
6. The school committee paid ______ to their famous former pupil by naming the new
gym after her.
A. esteem B. homage C. honour D. respect
7. After his long illness, the old man appeared so thin and ______ that a gust of wind
might have blown him away.
A. flimsy B. faint C. withered D. frail
8. You could tell that she wasn't happy about the news by the way she ______ her face in
disapproval.
A. came on B. brought round C. settled for D. screwed up
9. Going to the unemployment office and having to wait there for hours is a ______
experience.
A. soul-destroying B. heart-stopping C. power-sharing D. thought-provoking
Page 4 of 26
10. ______ a language family is a group of languages with a common origin and similar
vocabulary, grammar, and sound systems.
A. Linguists call it B. It is called by linguists
C. What linguists call D. What do linguists call
11. ________, the balcony chairs will be ruined in this weather.
A. Leaving uncovered B. Having left uncovered
C. Left uncovered D. Been left uncovered
12. The contemporary dialogue for me struck a slightly _______ note.
A. disembodied B. discordant C. dismissive D. disconcerting
13. Both of the jobs I’ve been offered are fantastic opportunities – I’m in such _______!
A. a constituency B. a deviation C. an arrhythmia D. a quandary
14. Sharon is such a positive person – she _______ her problems, whatever they are.
A. goes light on B. throws light to C. makes light of D. sheds light upon
15. The brother and sister were ________ over who would get to inherit the beach house.
A. at large B. at odds C. at a standstill D. at a loose end
16. ________ that Kim was getting married, we were sorry she’d be leaving home.
A. Delighted though we were B. As we were delighted
C. However delighted were we D. As we were so delighted
17. Peter’s so ________! I think he’d think things through a little more carefully.
A. impulsive B. repulsive C. compulsive D. expulsive
18. I’ve yet ________ a person as Theo.
A. to meet as infuriating B. to have met such infuriating
C. been meeting as infuriating D. been meeting such infuriating
19. Sniffer dogs are able to locate survivors beneath the rubble with ________ .
A. precision B. correctness C. meticulousness D. exactitude
20. The locks to the doors of the building are controlled ________ .
A. mainly B. centrally C. solidly D. completely
Part 2: Supply the correct form of the word provided in blankets in each sentence.
(10 pts)
1-2. Complaining consists of (SOCIAL) _______ moaning and groaning which leads to
(TRUST) _______ and unnecessary arguments within relationships.

Page 5 of 26
3. Thanks to plentiful fish, none of the islanders suffer from (NUTRIENT) _______.
4. I was at a difficult point of my life when I felt (ILLUSION) _________ by both work
and home life
5. I talked to various friends, most of whom soon got tired of my (WHIMSY) _________
rambling and indecisiveness.
6. It is estimated that between three to five million Britons suffer from such phobias, and the
majority of these people do not (GO) _________ form of treatment.
7. Men are less likely to suffer from such fears than women, but attempts by either men or
women simply to (REGARD) _______them can exacerbate the problem.
8. Behavioral therapy is decidedly a safe and lasting (ALTERNATE) ________to drug
treatment.
9. What contributes to her business success is that she always (WIT) her opponent
10. Many streets in Hanoi were flooded and (PASS) after it had rained for 2 hours
on end.
C. READING (60 pts)
Part 1: Read the text below and think of one word which best fits each space. Use only
ONE WORD for each space. Write your answers in the corresponding numbered
boxes. (15 pts) – Toward proficiency
SEASON AND BABY SIZE
People born in certain months of the year tend to be taller than those born in others.
This discovery was (1) __________ in a recent study carried out at the Danish
Epidemiology Science Centre in Copenhagen.
The researchers looked at the height and weight details (2) __________birth of over a
million Danes born in the 1970s, 1980s and 1990s, and found that a child born in April is,
on average, nearly two centimeters taller than one born in December. As (3) __________ as
being taller, the April baby is also likely to be heavier, and the odds (4) __________ that he
or she will remain bigger throughout life.
The study also shows that (5) __________ children born in December tend to be the
smallest, the size of those born in June and July is also well below (6) __________ of the
April babies. In these two midsummer months, however, the difference in height and weight
is only half as much as in midwinter.

Page 6 of 26
Similar results (7) __________ these emerged from a separate study in Australia.
There they found that by the time children reached 18, the gap between the April and the
December ones had increased to about five centimeters.
Another seasonal variation that (8) __________ to light concerned actual birth dates.
Winter babies, it was discovered, were born an average of one day earlier than spring,
summer or autumn babies.
In (9) __________ of these studies were scientists able to explain these differences.
In both cases they called (10) __________ further research into the subject.
Your answers:
1. 2. 3. 4. 5.
6. 7. 8. 9. 10.

Part 2. For questions 1-10, read an extract from an article and choose the answer A, B,
C or D that fits best according to the text. Write your answers in the corresponding
numbered boxes provided. (10 pts)- Toefl IBT Practice Test 2
Green Iceberg Icebergs are massive blocks of ice, irregular in shape; they float with only
about 12 per cent of their mass above the sea surface. They are formed by glaciers—large
rivers of ice that begin inland in the snows of Greenland, Antarctica, and Alaska—and move
slowly toward the sea. The forward movement, the melting at the base of the glacier where
it meets the ocean, and waves and tidal action cause blocks of ice to break off and float out
to sea.
Icebergs are ordinarily blue to white, although they sometimes appear dark or opaque
because they carry gravel and bits of rock. They may change colour with changing light
conditions and cloud cover, glowing pink or gold in the morning or evening light, but this
colour change is generally related to the low angle of the Sun above the horizon. However,
travellers to Antarctica have repeatedly reported seeing green icebergs in the Weddell Sea
and, more commonly, close to the Amery Ice Shelf in East Antarctica.
One explanation for green icebergs attributes their colour to an optical illusion when blue
ice is illuminated by a near-horizon red Sun, but green icebergs stand out among white and
blue icebergs under a great variety of light conditions. Another suggestion is that the colour
might be related to ice with high levels of metallic compounds, including copper and iron.

Page 7 of 26
Recent expeditions have taken ice samples from green icebergs and ice cores—vertical,
cylindrical ice samples reaching down to great depths—from the glacial ice shelves along
the Antarctic continent. Analyses of these cores and samples provide a different solution to
the problem.
The ice shelf cores, with a total length of 215 meters (705 feet), were long enough to
penetrate through glacial ice—which is formed from the compaction of snow and contains
air bubbles—and to continue into the clear, bubble-free ice formed from seawater that
freezes onto the bottom of the glacial ice. The properties of this clear sea ice were very
similar to the ice from the green iceberg. The scientists concluded that green icebergs form
when a two-layer block of shelf ice breaks away and capsizes (turns upside down), exposing
the bubble-free shelf ice that was formed from seawater.
A green iceberg that stranded just west of the Amery Ice Shelf showed two distinct layers:
bubbly bluewhite ice and bubble-free green ice separated by a onemeter-long ice layer
containing sediments. The green ice portion was textured by seawater erosion. Where cracks
were present, the colour was light green because of light scattering; where no cracks were
present, the colour was dark green. No air bubbles were present in the green ice, suggesting
that the ice was not formed from the compression of snow but instead from the freezing of
seawater. Large concentrations of single-celled organisms with green pigments (colouring
substances) occur along the edges of the ice shelves in this region, and the seawater is rich
in their decomposing organic material. The green iceberg did not contain large amounts of
particles from these organisms, but the ice had accumulated dissolved organic matter from
the seawater. It appears that unlike salt, dissolved organic substances are not excluded from
the ice in the freezing process. Analysis shows that the dissolved organic material absorbs
enough blue wavelengths from solar light to make the ice appear green.
Chemical evidence shows that platelets (minute flat portions) of ice form in the water and
then accrete and stick to the bottom of the ice shelf to form a slush (partially melted snow).
The slush is compacted by an unknown mechanism, and solid, bubble-free ice is formed
from water high in soluble organic substances. When an iceberg separates from the ice shelf
and capsizes, the green ice is exposed.
The Amery Ice Shelf appears to be uniquely suited to the production of green icebergs.
Once detached from the ice shelf, these bergs drift in the currents and wind systems

Page 8 of 26
surrounding Antarctica and can be found scattered among Antarctica’s less colourful
icebergs.
1. According to paragraph 1, all of the following are true of icebergs EXCEPT:
A. They do not have a regular shape.
B. They are formed where glaciers meet the ocean.
C. Most of their mass is above the sea surface.
D. Waves and tides cause them to break off glaciers.
2. According to paragraph 2, what causes icebergs to sometimes appear dark or
opaque?
A. A heavy cloud cover
B. The presence of gravel or bits of rock
C. The low angle of the Sun above the horizon
D. The presence of large cracks in their surface
3. Which of the sentences below best expresses the essential information in the the
bolded sentence in paragraph 3?
A. One explanation notes that green icebergs stand out among other icebergs under a great
variety of light conditions, but this is attributed to an optical illusion.
B. One explanation for the colour of green icebergs attributes their colour to an optical
illusion that occurs when the light from a near-horizon red Sun shines on a blue iceberg.
C. One explanation for green icebergs attributes their colour to a great variety of light
conditions, but green icebergs stand out best among other icebergs when illuminated by a
near-horizon red Sun.
D. One explanation attributes the colour of green icebergs to an optical illusion under
special light conditions, but green icebergs appear distinct from other icebergs under a great
variety of light conditions.
4. According to paragraph 4, how is glacial ice formed?
A. By the compaction of snow
B. By the freezing of seawater on the bottom of ice shelves
C. By breaking away from the ice shelf
D. By the capsizing of a two-layer block of shelf ice

Page 9 of 26
5. Ice shelf cores helped scientists explain the formation of green icebergs by showing
that
A. the ice at the bottom of green icebergs is bubble-free ice formed from frozen seawater
B. bubble-free ice is found at the top of the ice shelf
C. glacial ice is lighter and floats better than sea ice
D. the clear sea ice at the bottom of the ice shelf is similar to ice from a green iceberg
6. Why does the author mention that “The green ice portion was textured by seawater
erosion”?
A. To explain why cracks in the iceberg appeared light green instead of dark green
B. To suggest that green ice is more easily eroded by seawater than white ice is
C. To support the idea that the green ice had been the bottom layer before capsizing
D. To explain how the air bubbles had been removed from the green ice
7. Which of the following is NOT explained in the passage?
A. Why blocks of ice break off where glaciers meet the ocean
B. Why blocks of shelf ice sometimes capsize after breaking off
C. Why green icebergs are commonly produced in some parts of Antarctica
D. Why green icebergs contain large amounts of dissolved organic pigments
8. The passage supports which of the following statements about the Amery Ice Shelf?
A. The Amery Ice Shelf produces only green icebergs.
B. The Amery Ice Shelf produces green icebergs because its ice contains high levels of
metallic compounds such as copper and iron.
C. The Amery Ice Shelf produces green icebergs because the seawater is rich in a particular
kind of soluble organic material.
D. No green icebergs are found far from the Amery Ice Shelf.
9. Scientists have differed as to whether icebergs appear green as a result of light
conditions or because of something in the ice itself. Where would the sentence best fit?
A. Icebergs are ordinarily blue to white, although they sometimes appear dark or opaque
because they carry gravel and bits of rock. They may change colour with changing light
conditions and cloud cover, glowing pink or gold in the morning or evening light, but this
colour change is generally related to the low angle of the Sun above the horizon. Scientists
have differed as to whether icebergs appear green as a result of light conditions or because

Page 10 of 26
of something in the ice itself. However, travellers to Antarctica have repeatedly reported
seeing green icebergs in the Weddell Sea and, more commonly, close to the Amery Ice
Shelf in East Antarctica. One explanation for green icebergs attributes their colour to an
optical illusion when blue ice is illuminated by a near-horizon red Sun, but green icebergs
stand out among white and blue icebergs under a great variety of light conditions. Another
suggestion is that the colour might be related to ice with high levels of metallic compounds,
including copper and iron. Recent expeditions have taken ice samples from green icebergs
and ice cores—vertical, cylindrical ice samples reaching down to great depths—from the
glacial ice shelves along the Antarctic continent. Analyses of these cores and samples
provide a different solution to the problem.
B. Icebergs are ordinarily blue to white, although they sometimes appear dark or opaque
because they carry gravel and bits of rock. They may change colour with changing light
conditions and cloud cover, glowing pink or gold in the morning or evening light, but this
colour change is generally related to the low angle of the Sun above the horizon. However,
travellers to Antarctica have repeatedly reported seeing green icebergs in the Weddell Sea
and, more commonly, close to the Amery Ice Shelf in East Antarctica. Scientists have
differed as to whether icebergs appear green as a result of light conditions or because of
something in the ice itself. One explanation for green icebergs attributes their colour to an
optical illusion when blue ice is illuminated by a near-horizon red Sun, but green icebergs
stand out among white and blue icebergs under a great variety of light conditions. Another
suggestion is that the colour might be related to ice with high levels of metallic compounds,
including copper and iron. Recent expeditions have taken ice samples from green icebergs
and ice cores—vertical, cylindrical ice samples reaching down to great depths—from the
glacial ice shelves along the Antarctic continent. Analyses of these cores and samples
provide a different solution to the problem.
C. Icebergs are ordinarily blue to white, although they sometimes appear dark or opaque
because they carry gravel and bits of rock. They may change colour with changing light
conditions and cloud cover, glowing pink or gold in the morning or evening light, but this
colour change is generally related to the low angle of the Sun above the horizon. However,
travellers to Antarctica have repeatedly reported seeing green icebergs in the Weddell Sea
and, more commonly, close to the Amery Ice Shelf in East Antarctica. One explanation for

Page 11 of 26
green icebergs attributes their colour to an optical illusion when blue ice is illuminated by a
near-horizon red Sun, but green icebergs stand out among white and blue icebergs under a
great variety of light conditions. Scientists have differed as to whether icebergs appear green
as a result of light conditions or because of something in the ice itself. Another suggestion is
that the colour might be related to ice with high levels of metallic compounds, including
copper and iron. Recent expeditions have taken ice samples from green icebergs and ice
cores—vertical, cylindrical ice samples reaching down to great depths—from the glacial ice
shelves along the Antarctic continent. Analyses of these cores and samples provide a
different solution to the problem.
D. Icebergs are ordinarily blue to white, although they sometimes appear dark or opaque
because they carry gravel and bits of rock. They may change colour with changing light
conditions and cloud cover, glowing pink or gold in the morning or evening light, but this
colour change is generally related to the low angle of the Sun above the horizon. However,
travellers to Antarctica have repeatedly reported seeing green icebergs in the Weddell Sea
and, more commonly, close to the Amery Ice Shelf in East Antarctica. One explanation for
green icebergs attributes their colour to an optical illusion when blue ice is illuminated by a
near-horizon red Sun, but green icebergs stand out among white and blue icebergs under a
great variety of light conditions. Another suggestion is that the colour might be related to ice
with high levels of metallic compounds, including copper and iron. Scientists have differed
as to whether icebergs appear green as a result of light conditions or because of something
in the ice itself. Recent expeditions have taken ice samples from green icebergs and ice
cores—vertical, cylindrical ice samples reaching down to great depths—from the glacial ice
shelves along the Antarctic continent. Analyses of these cores and samples provide a
different solution to the problem.
10. Several logical suggestions have been offered to explain why some icebergs appear
green EXCEPT
A. Ice cores were used to determine that green icebergs were formed from the compaction
of metallic compounds, including copper and iron.
B. Green icebergs form when a two-layer block of ice breaks away from a glacier and
capsizes, exposing the bottom sea ice to view.

Page 12 of 26
C. Ice cores and samples revealed that both ice shelves and green icebergs contain a layer of
bubbly glacial ice and a layer of bubble-free sea ice.
D. In a green iceberg, the sea ice contains large concentrations of organic matter from the
seawater.
Your answers
1. 2. 3. 4. 5.

6. 7. 8. 9. 10.

Part 3. Read the following passage and do the tasks that follow. (13 pts)
https://collegedunia.com/news/e-482-does-water-have-memory-ielts-reading-sample-
with-explanation
Does water have memory?
The practice of homeopathy was first developed by the German physician Samuel
Hahnemann. During research in the 1790s, Hahnemann began experimenting with quinine,
an alkaloid derived from cinchona bark that was well known at the time to have a positive
effect on fever. Hahnemann started dosing himself with quinine while in a state of good
health, and reported in his journals that his extremities went cold, he experienced
palpitations, an “infinite anxiety”, a trembling and weakening of the limbs, reddening
cheeks and thirst – “in short”, he concluded, “all the symptoms of relapsing fever presented
themselves successively...” Hahnemann’s main observation was that things which create
problems for healthy people cure those problems in sick people, and this became his first
principle of homeopathy: simila similibus (with help from the same). While diverging from
the principle of apothecary practice at the time – which was contraria contrariis (with help
from the opposite) – the efficacy of simila similibus was reaffirmed by subsequent
developments in the field of vaccinations.
Hahnemann’s second principle was minimal dosing – treatments should be taken in the
most diluted form at which they remain effective. This negated any possible toxic effects of
simila similibus.
In 1988 the French immunologist Jacques Benveniste took minimal dosing to new extremes
when he published a paper in the prestigious scientific journal Nature in which he suggested
that very high dilutions of the anti-lgE antibody could affect human basophil granulocytes,

Page 13 of 26
the least common of the granulocytes that make up about 0.01% to 0.3% of white blood
cells. The point of controversy, however, was that the water in Benveniste’s test had been so
diluted that any molecular evidence of the antibodies no longer existed. Water molecules,
the researcher concluded, had a biologically active component that a journalist later termed
“water memory”. A number of efforts from scientists in Britain, France and the Netherlands
to duplicate Benveniste’s research were unsuccessful, however, and to this day no peer-
reviewed study under broadly accepted conditions has been able to confirm the validity of
“water memory”.
The third principle of homeopathy is “the single remedy.” Exponents of this principle
believe that it would be too difficult, if not impossible, to ascertain the potential effects of
multiple homeopathic remedies delivered simultaneously. If it did work, they suggest, one
could not know quite why it worked, turning homeopathy into an ambiguous guessing
game. If it did not work, neither patient nor practitioner would know whether the ingredients
were all ineffective, or whether they were only ineffective in combination with one another.
Combination remedies are gaining in popularity, but classical homeopaths who rely on the
single remedy approach warn these are not more potent, nor do they provide more treatment
options. The availability of combination remedies, these homeopaths suggest, has been led
by consumers wanting more options, not from homeopathic research indicating their
efficacy.
Homeopathy is an extremely contentious form of medicine, with strong assertions coming
from both critics and supporters of the practice. “Homeopathy: There’s nothing in it”
announces the tagline to 10:23, a major British anti-homeopathy campaign. At 10.23 a.m.
on 30 January 2010, over 400 supporters of the 10:23 stood outside Boots pharmacies and
swallowed an entire bottle each of homeopathic pills in an attempt to raise awareness about
the fact that these remedies are made of sugar and water, with no active components. This,
defenders of homeopathy say, is entirely the point. Homeopathic products do not rely on
ingredients that become toxic at high doses, because the water retains the “memory” that
allows the original treatment to function.
Critics also point out the fact that homeopathic preparations have no systematic design to
them, making it hard to monitor whether or not a particular treatment has been efficacious.
Homeopaths embrace this. While results may be less certain, they argue, the non-toxic

Page 14 of 26
nature of homeopathy means that practitioner and patient can experiment until they find
something that works without concern for side effects. Traditional medicine, they argue,
assaults the body with a cocktail of drugs that only tackles the symptoms of disease, while
homeopathy has its sights aimed on the causes. Homeopaths suggest this approach leads to
kinder, gentler, more effective treatment.
Finally, critics allege that when homeopathy has produced good results, these are
exceedingly dependent on the placebo effect, and cannot justify the resources, time and
expense that the homeopathic tradition absorbs. The placebo effect is a term that describes
beneficial outcomes from a treatment that can be attributed to the patient’s expectations
concerning the treatment rather than from the treatment itself. Basically, the patient “thinks”
himself into feeling better. Defenders suggest that homeopathy can go beyond this
psychological level. They point to the successful results of homeopathy on patients who are
unconscious at the time of treatment, as well as on animals.
For questions 1-7, decide whether the following statements are True (T), False (F) or
Not Given (NG). Write your answers in the corresponding numbered boxes provided.
1. Samuel Hahnemannn developed his principles based on an existent set of rules at his
time.
2. The existence of a biologically active part in water has yet to be conclusively proven.
3. The single remedy serves to preclude the unforeseeable outcomes of remedial
combinations.
4. It has been suggested that the practice of applying several treatments at the same time
becomes more common due to endorsements by scientists.
5. The uncertainty of preparations for homeopathy is perceived by both supporters and
opponents of it.
6. Patients’ feelings are affected by the outcomes of the treatments they receive.
7. Abortive attempts of homeopathic treatment are used to corroborate its opponents’
arguments.
Your answers.
1. 2. 3. 4. 5. 6. 7.

Page 15 of 26
For questions 8-13, complete the following paragraph with words taken from the
passage (NO MORE THAN THREE WORDS for each blank).
There are three principles behind the practice of homeopathy. The first one, simila
similibus, was developed by Samuel Hahnemann after experimentation in which he
observed that problem-inducing factors could become treatments for suffering people.
While marking a departure from that of (8) ________________, this principle of
homeopathy was substantiated by further advancements. The second principle, minimal
dosing, serves to avert (9) _______________ that can be caused by simila similibus. The
attempt for its furtherance was made by Jacques Benveniste, but controversy was sparked as
there was a lack of (10) _______________ in the used water. Moreover, a result from his
experiment termed “water memory” had received (11) _______________ to the moment of
writing. The third principle named “the single remedy” works on the ground that application
of multiple treatments at the same time can make homeopathy become a(n) (12)
_______________ even when the results are desirable.
Homeopathy is a controversial remedy. While there are arguments in favour of it,
critics have suggested weaknesses in the treatment including its components, lack of
systemic design and the reliance on (13) _______________ of its feasible positive effects.

Part 4. In the passage below, seven paragraphs have been removed. For questions 1-
10, read the passage and choose from the paragraphs A-H the one which fits each gap.
There is ONE extra paragraph which you do not need to use. Write your answers in
the corresponding numbered box provided. (7 pts)
Towards proficiency student’s book.
Page: 138-139

HIGH – TECH CRIME OF THE FUTURE

It begins with a beam of high-intensity radiation. Silent and invisible. It disables the
electronics of the alarm and telephone system of the bank manager's house, and then the
gang bursts in. Wearing identical clothes and caps to confuse the CCTV and communicating
by untraceable mobile phones, the gang of four London criminals incapacitate everyone in
the house with stun guns.

Page 16 of 26
1.

This, law enforcement experts believe, is the future of organised crime where villains will
use a combination of cutting-edge technology and old-fashioned brute force to defeat
increasingly sophisticated security systems. Rather than coshes and sawn-off shotguns, the
criminals of tomorrow will be skilled in crimes that produce high profits but with a fraction
of the risk of armed robbery or smuggling. It is not far away.

2.

Kidnapping is thought to be a particular growth area. This is because new security


technologies such as retina scans and fingerprint recognition require a living subject. Long
thought to be the stuff of science fiction, such devices are becoming increasingly common.
However, one negative result is that crimes in which staff are abducted and forced to help
criminals are also set to rise.

3.

Commodities traded by the organised criminals of tomorrow will also change. Increasing
profits will lead to a search for effective money-laundering solutions. Small, high-value
items such as computer chips, rare metals and even meteorite fragments will be traded
between gangs in an effort to avoid leaving electronic trails.

4.

Senior National Criminal Intelligence Service analysts are also warning of the threat of
digital stimulants, illegally transmitted across the internet. These could create new forms of
addiction far worse than that sometimes caused by interactive PC games.

Page 17 of 26
5.

Instead, users will hook up to machines which will directly stimulate the pleasure centres of
the brain. Some may go further and have silicon chips implanted in their bodies.

6.

A silicon chip implanted into the body will be able to pick up a signal transmitted across the
Internet, or using radio waves. It will give extremes of pleasure more addictive than the
most intense virtual-reality experiences – and be controlled by criminals.

7.

'Cyberspace offers criminals opportunities to create unprecedented and terrible new forms
of addiction,' said Professor Warwick. 'The question is not whether electronic stimulants can
be created, but how soon they can be put on the market.'
The missing paragraphs:
A. With the price of computer memory chips having risen 400 per cent recently, police
believe a new spate or robberies and factory break-ins is just around the corner.
B. 'The purpose of organised crime is to generate profits,' says one expert. 'To identity the
key areas of criminality in the future, you simply have to identify the areas that will be
generating the most money. Biotechnology and information technology are the two biggest
revolutions of this century, and we have to look at each of them and see where the potential
for money is from the criminal point of view. Organised crime will become more
sophisticated and more networked, and it is the challenge of law enforcement to look where
those networks may expand in the future.'
C. Last year Kevin Warwick, Professor of Cybernetics at Reading University, had a chip
implanted in his left elbow which allowed computers to communicate directly with his
body. He could switch machines on and off simply by scratching his head.
D. Murders of this sort, unfortunately, are likely to become less common. Increased
penalties for carrying firearms, along with an greatly increased likelihood of being shot dead
Page 18 of 26
by armed police, will lead to more criminals using non-lethal weapons to incapacitate
victims, such as stun guns and CS gas sprays.
E. Although the technology to deliver this kind of cyber-stimulant does not yet exist,
experts in the field admit that it is only a matter of time. Experiments on chimps and other
animals have already shown that when they are wired externally, via an electrode implanted
into the pleasure centres of the brain, they become so addicted to the sensations it produces
that they forget to eat and can die for lack of food.
F. A lap-top computer and portable palm reader is then produced and the hand of the
unconscious bank manager is placed on top. Within seconds, the gang have dialled into the
bank's computer and gained top-level access to thousands of accounts. After transferring
money the gang leaves, sealing the doors of the home with a foam which turns rock solid in
seconds, and makes its getaway in two identical vehicles equipped with anti-reflective
number plates to frustrate speed cameras.
G. At an FBI-sponsored gathering of international law enforcement agencies in Germany
last month, one NCIS officer claimed that criminals had begun to exploit areas such as
virtual banking and on-line gambling, and were now developing a new generation of
stimulants, the effects of which could be experienced without actually possessing them.
H. So,too are cases of blackmai and extortion. Although banks refuse to discuss the subject,
it is believed that there have been at least four instances of extortion in which criminals have
threatened to disrupt the systems of city institutions with high-intensity radiation devices.
Your answers
1. 2. 3. 4. 5. 6. 7.

Part 5. The passage below consists of four sections marked A, B, C and D. For
questions 1 - 10, read the passage and do the task that follows. Write your answers (A,
B, C or D) in the corresponding numbered boxes provided. (15 pts)
Source: Objective Proficiency Workbook.
Page: 68-69

CHEER UP: LIFE ONLY GETS BETTER

Page 19 of 26
Human’s capacity for solving problems has been improving out lot for 10,000 years, says
Matt Ridley

A. The human race has expanded in 10,000 years from less than 10 million people to around
7 billion. Some live in even worse conditions than those in the Stone Age. But the vast
majority are much better fed and sheltered, and much more likely to live to old age than
their ancestors have ever been. It is likely that by 2110 humanity will be much better off
than it is today and so will the ecology of our planet. This view, which I shall call rational
optimism, may not be fashionable but it is compelling. This belief holds that the world will
pull out of its economic and ecological crises because of the way that markets i goods,
services and ideas allow human beings to exchange and specialise for the betterment of all.
But a constant drumbeat of pessimism usually drowns out this sort of talk. Indeed, if you
dare to say the world is going to go on being better, you are considered embarassingly mad.

B. Let me make a square concession at the start: the pessimists are right when they say that
if the world continues as it is, it will end in disaster. If agriculture continues to depend on
irrigation and water stocks are depleted, then starvation will ensue. Notice the word “if”.
The world will not continue as it is. It is my proposition that the human race has become a
collective problem – solving machine which solves problems by changing its ways. It does
so through invention driven often by the marker: scarcity drives up price and that in turn
encourages the development of alternatives and efficiencies. History confirms this. When
whales grew scarce, for example, petroleum was used instead as a source of oil. The
pessimists’ mistake is extrapolating: in other words, assuming that the future is just a bigger
version of the past. In 1943 IBM’s founder Thomas Watson said there was a world market
for just five computers – his remarks were true enough at the time, when computers
weighed a ton and cost a fortune.

C. Many of today’s extreme environmentalists insist that the world has reached a ‘turning
point’ – quite unaware that their predecessors have been making the same claim for 200
years. They also maintain the only sustainable solution is to retreat – to halt economic
growth and enter progressive economic recession. This means not just that increasing your

Page 20 of 26
company’s sales would be a crime, but that the failure to shrink them would be too. But all
this takes no account of the magical thing called the collective human brain. There was a
time in human history when big-brained people began to exchange things with each other,
to become better off as a resut. Making and using tools saved time – and the state of being
‘better off’ is, at the end of the day, simply time saved. Forget dollars of gold. The true
measure of something’s worth is indeed the hours it takes to acquire it. The more humans
diversified as consumers and specified as producers, and the more they exchanged goods
and services, the better off they became. And the good news is there is no inevitable end to
this process.

D. I am aware that an enormous bubble of debt has burst around the world, with all that
entails. But is this the end of growth? Hardly. So long as somebody allocates sufficient
capital to innovation, then the credit crunch will not prevent the relentless upward march of
human living standards. Even the Great Depression of the 1930s, although an appalling
hardship for many, was just a dip in the slope of economic progress. All sorts of new
products and industries were born during the depression: by 1937, 40% of Dupont’s sales
came from products that had barely existed before 1929, such as enamels and cellulose film.
Growth will resume – unless it is stifled by the wrong policies. Somebody, somewhere, is
still tweaking a piece of software, testing a new material, of transferring a gene that will
enable new varieties of rice to be grown in African soils. The latter means some Africans
will soon be growing and and selling more food, so they will have more money to spend.
Some of them may then buy mobile phones from a western company. As a consequence of
higher sales, an employee of that western company may get a pay rise, which she may spend
on a pair of jeans made from cotton woven in an African factory. And so on. Forget wars,
famines and poems, This is history’s greatest theme: the metastasis of exchange and
specialisation.

In which section does the writer Your answers:


exemplify how short-term gloom tends to lift? 1.
mention a doom-laden prophecy that is obvious? 2.
express his hope that progress is not hindered by abominable 3.

Page 21 of 26
decisions?
acknowledge trying to find common ground with his potential 4.
adversaries?
identify unequivocally how money needs to be invested? 5.
suggest that his views are considered controversial? 6.
indicate an absurd scenario resulting from an opposing view to his 7.
own?
mention the deplorable consequences of taking a positive stance? 8.
define prosperity in life in an original approach? 9.
give an example of well-intentioned ongoing research? 10.

D. WRITING (60 pts)


Part 1: Read the following text and use your own words to summarize it. Your summary
should be about 120 words long. You MUST NOT copy the original. (15 pts)
PROMISING RESULTS FROM CANCER STUDY
A new experiment vaccine has shown promising results in the fight against lung
cancer. In a small Texas-based study, a vaccine developed by scientists at Baylor University
Medical Centre in Dallas, USA cured lung cancer in some patients and slowed the progress
of the disease in others. Researchers have reported encouraging findings from this small
study. Forty-three patients suffering from lung cancer were involved in these trials. Ten of
these patients were in the early stages and thirty-three in the advanced stages of the disease.
They were injected with the vaccine every two weeks for three months, and were carefully
monitored for three years. In three of the patients in the advanced stages of cancer, disease
disappeared and in the others, it did not spread for five to twenty-four months. However, no
great difference was seen in the patients in the early stages of the illness. This new vaccine
uses the patient’s own immune system. It is made specifically for each patient and is
injected into the arm or leg. It stimulates the body’s immune system, which then recognizes
that the cancer cells are harmful, and attacks and destroys them. The vaccine could be
effective against other forms of cancer. It offers great hope for the treatment of cancer in
general, although further studies are needed before such treatment can be widely used.
Source: Dakin, F.P. (2007) Promising results from cancer study. New Medical Journal,

Page 22 of 26
………………………………………………………………………………………………………………………………………………………

………………………………………………………………………………………………………………………………………………………

………………………………………………………………………………………………………………………………………………………

…………………………………………………………………………………………………………………………………………………………

………………………………………………………………………………………………………………………………………………………

………………………………………………………………………………………………………………………………………………………

………………………………………………………………………………………………………………………………………………………

………………………………………………………………………………………………………………………………………………………

………………………………………………………………………………………………………………………………………………………

………………………………………………………………………………………………………………………………………………………

………………………………………………………………………………………………………………………………………………………

………………………………………………………………………………………………………………………………………………………

………………………………………………………………………………………………………………………………………………………

………………………………………………………………………………………………………………………………………………………

………………………………………………………………………………………………………………………………………………………

Part 2. Chart description.(15 pts)


https://www.ielts-simon.com/ielts-help-and-english-pr/2017/09/ielts-writing-task-1-
two-charts-answer.html
The charts below show reasons for travel and the main issues for the travelling public in the US in 2009.

Page 23 of 26
Summarize the information by selecting and reporting the main features, and make
comparisons where relevant. Write at least 150 words.
 Your Answer:
………………………………………………………………………………………………………………………………………………………

………………………………………………………………………………………………………………………………………………………

………………………………………………………………………………………………………………………………………………………

………………………………………………………………………………………………………………………………………………………

………………………………………………………………………………………………………………………………………………………

………………………………………………………………………………………………………………………………………………………

……………………………………………………………………………………………………………………………………………………

………………………………………………………………………………………………………………………………………………………

………………………………………………………………………………………………………………………………………………………

………………………………………………………………………………………………………………………………………………………

………………………………………………………………………………………………………………………………………………………

………………………………………………………………………………………………………………………………………………………

………………………………………………………………………………………………………………………………………………………

………………………………………………………………………………………………………………………………………………………

………………………………………………………………………………………………………………………………………………………

Page 24 of 26
………………………………………………………………………………………………………………………………………………………

………………………………………………………………………………………………………………………………………………………

………………………………………………………………………………………………………………………………………………………

………………………………………………………………………………………………………………………………………………………

………………………………………………………………………………………………………………………………………………………

………………………………………………………………………………………………………………………………………………………

………………………………………………………………………………………………………………………………………………………

……………………………………………………………………………………………………………………………………………………

………………………………………………………………………………………………………………………………………………………

Part 3: Write an essay of at least 350 words to express your opinion on the following
topic: (30 pts)
Many countries face dilemma between pursuing economic growth and protecting the
environment. In your opinion should governments in theses countries give priority to
economic growth or environmental protection?
 Your Answer
………………………………………………………………………………………………………………………………………………………

………………………………………………………………………………………………………………………………………………………

………………………………………………………………………………………………………………………………………………………

……………………………………………………………………………………………………………………………………………

………………………………………………………………………………………………………………………………………………………

………………………………………………………………………………………………………………………………………………………

………………………………………………………………………………………………………………………………………………………

………………………………………………………………………………………………………………………………………………………

………………………………………………………………………………………………………………………………………………………

………………………………………………………………………………………………………………………………………………………

………………………………………………………………………………………………………………………………………………………

………………………………………………………………………………………………………………………………………………………

………………………………………………………………………………………………………………………………………………………

………………………………………………………………………………………………………………………………………………………

………………………………………………………………………………………………………………………………………………………

Page 25 of 26
………………………………………………………………………………………………………………………………………………………

………………………………………………………………………………………………………………………………………………………

………………………………………………………………………………………………………………………………………………………

………………………………………………………………………………………………………………………………………………………

………………………………………………………………………………………………………………………………………………………

………………………………………………………………………………………………………………………………………………………

………………………………………………………………………………………………………………………………………………………

………………………………………………………………………………………………………………………………………………………

………………………………………………………………………………………………………………………………………………………

The end

Người ra đề thi:

Hoàng Ngọc Linh (0961802339)

Page 26 of 26
TRƯỜNG THPT CHUYÊN BẮC NINH KỲ THI HỌC SINH GIỎI CÁC TRƯỜNG THPT CHUYÊN
TỔ TIẾNG ANH KHU VỰC DUYÊN HẢI VÀ ĐỒNG BẰNG BẮC BỘ

NĂM HỌC 2021-2022

(ĐỀ ĐỀ XUẤT) MÔN: TIẾNG ANH – KHỐI 11

(Thời gian làm bài: 180 phút)

Đề này gồm có 19 trang

_________________________________________

HƯỚNG DẪN PHẦN THI NGHE HIỂU


 Bài nghe gồm 4 phần, mỗi phần được nghe 2 lần, mỗi lần cách nhau 30 giây, mở đầu và kết thúc mỗi
phần nghe có tín hiệu.
 Mở đầu và kết thúc bài nghe có tín hiệu nhạc. Thí sinh có 3 phút để hoàn chỉnh bài trước tín hiệu
nhạc kết thúc bài nghe.
 Mọi hướng dẫn cho thí sinh (bằng tiếng Anh) đã có trong bài nghe.

I. LISTENING (50 points)


Part 1: You will hear a woman talking about caffeine. Listen and decide whether the following
sentences are true (T) or false (F). (10 points)
1. The desired effects of caffeine are brought by its role in enabling the proper function of specific
receptors in the brain.
2. Daily consumption of caffeine products has a long history, only at irregular intervals.
3. The local popularity Chinese tea enjoyed was preceded by its global reputation.
4. Caffeine consumption has become a part of the US beverage culture.
5. Caffeine has a risk warning, which led to a ban on the bulk sale of caffeine-infused beverages by the
US government.
Your answers
1. 2. 3. 4. 5.

Part 2: You will hear part of the lecture about the history of fireworks in Europe. Write NO
MORE THAN THREE WORDS AND/OR A NUMBER taken from the recording. (10 points))

1. What did the European emperors use fireworks to celebrate?


__________________________________________________________
2. In the late 17th century, what could be deployed to enable sailors to navigate at sea?
__________________________________________________________

1
3. Where was the contribution of fireworks specialists overlooked?
__________________________________________________________
4. What were featured in Grand displays in Paris, composed by Giovanni Servandoni, an architect and
stage designer?
__________________________________________________________
5. Whom did the Italian specialists desire to appeal during their journey around Europe?
__________________________________________________________

Part 3: Listen to a radio discussion about children who invent imaginary friends and choose the
correct answer A, B, C or D which fits best according to what you hear. Write your answers in the
corresponding numbered boxes provided. (10 points)
10. In the incident that Liz describes
A. her daughter asked her to stop the car.
B. she had to interrupt the journey twice.
C. she got angry with her daughter.
D. her daughter wanted to get out of the car.
11. What does the presenter say about the latest research into imaginary friends?
A. It contradicts other research on the subject.
B. It shows that the number of children who have them is increasing.
C. It indicates that negative attitudes towards them are wrong.
D. It focuses on the effect they have on parents.
12. How did Liz feel when her daughter had an imaginary friend?
A. always confident that it was only a temporary situation
B. occasionally worried about the friend's importance to her daughter
C. slightly confused as to how she should respond sometimes
D. highly impressed by her daughter's inventiveness
14. Karen says that the teenager who had invented a superhero is an example of
A. a very untypical teenager.
B. a problem that imaginary friends can cause.
C. something she had not expected to discover.
D. how children change as they get older.
15. According to Karen, how should parents react to imaginary friends?
A. They should pretend that they like the imaginary friend.
B. They shouldn't get involved in the child's relationship with the friend.
C. They should take action if the situation becomes annoying.
D. They shouldn't discuss the imaginary friend with their child.

2
Your answers
1. 2. 3. 4. 5.

Part 4: Listen to a recording about a growth mindset and complete the summary by writing NO
MORE THAN THREE words and/or a number in each gap. (20 points)
Growth mindset has acquired 1.________ these days with studies indicating greater success for those
who embrace efforts and learning than those who are merely supportive of 2. __________. Similarly,
deceptive behaviours are found in companies encouraging talent game while higher rates of innovative
developments, greater 3. _________ as well as more dedicated staff are reported in companies
supporting growth. However, a growth mindset can also be misinterpreted.

One common misconception is to dismiss the evolutionary nature of growth and to mistake features such
as a 4.________ or receptiveness for a growth mindset. Next, people might misinterpret endeavors as a
sure component of a growth mindset while not attaching enough significance to 5. _______ which
involve the lessons learnt, progress made and processes engaged in. Finally, organisations may solely
announce their goals to achieve 6. _________ without 7. ______ and _________.

All in all, organisations that encourage a growth zone value learning and support certain levels of 8.
________. In such environments, not 9._________ but ________ is espoused, and the concept of
“growth” must be truly delivered to employees through actions. The fixed mindset that triggers
10.__________ in response to negative feedback might be an obstacle to growth thinking; therefore, the
key is to get insight into what a growth mindset truly means and how to put it into practice.

Your answers:
1. 6.
2. 7.
3. 8.
4. 9.
5. 10.

II. LEXICO - GRAMMAR (3 points)


Part 1. Choose the best option to complete each of the following sentences. (20 points)
1. Beneath the streets of a modem city_______ of walls, columns, cables, pipes,and tunnels required to
satisfy the needs of its inhabitants.
A. where exists B. the existing network
C. the network’s existence D. exists the network

3
2. Everything looks very positive for the company, _______ the current investors do not default on their
agreements.
A. assuming that B. whether C. whereas D. as if
3. Probably no man had more effect on the daily lives of most people in the United States_______
A. as Henry Ford, a pioneer in automobile production.
B. rather than Henry Ford, a pioneer in automobile production.
C. than did Henry Ford, a pioneer in automobile production.
D. more than Henry Ford, a pioneer in automobile production.
4. I would rather _______ my holiday in Egypt than in Turkey. I really couldn’t stand the food.
A. take B had taken C. to have taken D. have taken
5. According to a recent report, the disease is rare, affecting _______ ten thousand adults.
A. out of every one B. the only one of
C. every one from D. only one out of every
6. I feel a bit wary that people seem so eager to make a _______ of faith about driverless cars being free
to drive in our street.
A. hop B. jerk C. leap D. twitch
7. He has secured a loan for a new building by ______ upon the resources of two existing tenants.
A. dwelling B. training C. playing D. drawing
8. They worked _______ all weekend, trying to get it finished on time.
A. ferociously B. furiously C. indignantly D. outrageously
9. Individuals are being arrested and detained for lengthy periods, often without trial, for disseminating
information judged to be ______ via the Internet.
A. frolicsome B. unpropitious C. pedestrian D. Seditious
10. The sun ______ down with merciless intensity on the six men adrift in the life boat.
A. shone B. scared C. glared D. flared
11. He was found to have provided the court with ______ evidence.
A. doctored B. tampered C. feigned D. shammed
12. The story _____ well, indicating that sports obsession is a universal quality.
A. travels B. carries C. sails D. crosses
13. Her legs were starting to grow tired and sore, and a _____ of pain was ______ in her side.
A. stitch / erupting B. crick / arousing C. prick / bursting D. throb / surging
14. Some find it hard to _____ themselves to the new working condition.
A. apply B. accommodate C. habituate D. conduct
15. She deliberately _____ the waters by constantly referring to other irrelevant cases.
A. sullied B. muddied C. blemished D. shrouded
16. The president claims the new media is _____ hostile to his initiatives.

4
A. impalpably B. implacably C. impeccably D. implausibly
17. Talking about the quality of your work behind your back was a _____ tactic on her part!
A. know-how B. look-out C. low-down D. show-down
18. Research in this field is of relatively recent _____; that’s why I’m not conversant with it.
A. progress B. caliber C. vantage D. vintage
19. He said my novel showed promise. He liked the style and the story-line, and he was ____ just to be
kind.
A. getting his bearings B. flexing his muscles C. pulling his punches D. spreading his wings
20. How am I expected to _____ a meal for six of his friends with almost nothing in the fridge?
A. conjure up B. eke out C. hack down D. roll in

Your answers:
1. 2. 3. 4. 5.
6. 7. 8. 9. 10.
11. 12. 13. 14. 15.
16. 17. 18. 19. 20.

Part 2: Write the correct form of word in each blank. (10 points)
1. Sometimes I'm really shocked by the (CONTEMP) behaviour of shop assistants.
2. The police (EXAMINE) the accusing man about how the accident had occurred.
3. The thing you have done makes a player in your team feel that he or she is being (LINE) . Give
everyone a chance to play.
4. The chairman of this company is merely a (HEAD), the Chief Executive is the one who is truly in
control.
5. The total (TON) of the bombs flopped in the Vietnam War increases every year as more bombs are
unearthed.
6. He looked at the policeman with (DEFY), as if he were asking for a fight.
7. She received a (PLENTY) of presents on her birthday - it would surely take the whole day to open
them.
8. The sides are bluish green above, violet in the middle, red beneath, (VARIANT) with oval spots of
brilliant silver.
9. When going on business trip, Henry enjoys staying in luxurious hotels that offer (POINT) suites.
10. He was born into a (BLOOD) family; therefore, he had an opportunity to attend the most prestigious
private school in the country.

5
Your answers:
1. 2. 3. 4. 5.
6. 7. 8. 9. 10.

III. READING (60 points)

Part 1: Fill in each blank with ONE word. (15 points)


We’ve all been there and come back with a T-shirt bearing a slogan about it – The Failed Holiday. The
kids are bored stiff you can’t find a decent restaurant for love nor 1.______, the cafes are all full and you
are fed up. Tempers fray and you are left wondering why you went away in the first place 2.______ a
good way to spend precious holiday time. But there is a way of escaping the stresses of being away from
home. Staying in an all-inclusive hotel 3.______ you don’t have to 4.______, meals, activities or night
life, as everything is catered for, leaving you to 5.______ back, relax and enjoy your holiday. Most
importantly, all-inclusive holidays 6.______ at all-inclusive prices, so you pay for everything before you
go away. And don’t think you’ll be 7.______ to one or two resorts. These days, holiday companies offer
all-inclusive deals in every main resort around the Mediterranean, as well as more exotic locations such
as the Caribbean or the Maldives. If you think this is the sort of holiday you are after, there are a few
things to keep in 8.______. Firstly, remember that you’ll be spending most of your time in the hotel, so
make sure you choose one which has the facilities you require. If you’re travelling as a family 9.______,
many hotels offer excellent children’s facilities such as kid’s clubs, playgrounds and crèches. For older
children, there are a number of hotels offering supervised activities for teens while others offer
10.______ sport facilities including team games, water sports and gym facilities.

Your answers:
1. 2. 3. 4. 5.
6. 7. 8. 9. 10.

Part 2: Read the following passage and choose the best answer to each question. (10 points)

I have been asked what I think about the idea of ‘Investing in People’. The best answer I can give is that
I think that what it tries to achieve - basically making the link between business improvement and
focusing on the needs of the people who work for an organisation - is great. My problem is with
organisations who subscribe to it as a way to help them 'get better', when they don't bother to
understand where they went wrong in the first place. They need to ask what explicit and implicit policies
and procedures they have in place that prevent their people from being able
to do the right thing for the right reasons.

6
I am sure that there are managers out there who don't know any better, and assume that to manage they
simply need to put pressure on their people to perform. [A] But people don't demonstrate high
performance because they are told to. [B] They do it because they see the need to do it, and make the
choice to do so. They do it because they are connected to the business goals and they see how their
contributions can help achieve them. [C] But simply putting ticks in boxes is no good if it doesn't reflect
reality. [D]
I know of a company that was so concerned that its people were doing the 'right thing' that it put in place
a series of metrics to measure their effectiveness. So far, so good. But one of the objectives - making
successful sales calls - manifested itself in the metric 'Number of potential customers seen in one day'.
The sales people obviously focused their efforts on going from one customer's office to another, and not
on closing deals. Instead of the employees becoming more effective,they focused on getting the boxes
ticked. Good intent; poor thinking.
Another company wanted to improve the speed with which it was able to introduce new products.
Competition was beating it to the market place, and consequently the company was losing market share.
Senior management sent out the message to reduce the time spent in getting products into customers'
hands, with the explanation that they couldn't afford delays. This was a relatively easy task,especially
since the time spent testing the products was cut in half to accomplish the time reduction. The result was
new products were introduced in less time than those of the competition - but soon rejected by customers
for poor quality. Good intent; reckless implementation.
A third company I know is trying hard to help employees see that they have some control over their
future. The company instituted a programme with a title like 'Creating our own future' or something like
that. A good idea; get the people involved in the future of the company. But instead of the employees
becoming motivated to contribute, they saw it as a hollow exercise on the part of senior management
who, in the past, had paid little attention to anything other than getting the job done so they could report
great earnings. Yes, the programme was a big 'tick the box' effort, but that was all it was in the minds of
the people that it was designed for.
A final example is of a company that brought in one of these 'Investing in People' programmes to change
the way the company was run. Assessors were running around like crazy, helping managers examine
how they managed. They told managers how they could manage better. And when the programme was
over, the company was able to say they had done it- it had invested in its people and life was now good.
But after all, the assessors were gone, and they again had targets to hit.
All these examples are representative of senior management who see the need to improve things in their
organisation, but don't see how to do it. For a start, a programme targeted at improving things is only as
good as management's ability to motivate their people. And when the employees simply see the
programme as a box-ticking exercise, then it's hopeless.
1. The writer thinks that putting the concept of 'Investing in People' into practice _______.

7
A. frequently results in confusion among the people it is supposed to help
B. involves more effort than some organizations are prepared to make
C. may create problems where previously there had not been any problems
D. is something that some organizations should attempt to do
2. What is the writer's attitude when using the phrase 'get better' in the first paragraph?
A. ironic B.humorous C. indignant D.practical
3. The writer's main point in the second paragraph is that the performance of employees _______.
A. may be very good even if management is poor
B. cannot be accurately measured by any box-ticking exercise
C. is related to their knowledge of the organization as a whole
D. is not as unpredictable as some managers believe it to be
4. Which of the following square brackets [A], [B], [C] or [D] best indicates where in the second
paragraph the sentence “Such managers may tell themselves they can put a 'tick' in the 'we care about
people' box.” can be inserted?
A. [A] B. [B] C. [C] D. [D]
5. What point does the writer make about the first company he describes?
A. It was not really interested in measuring the effectiveness of employees.
B. The targets that it set for staff were unrealistic.
C. It failed to understand the real needs of its employees.
D. The data that it collected did not measure what it was supposed to measure.
6. What point does the writer make about the second company he describes?
A. It made what should have been an easy task into a complicated one.
B. It failed to foresee the consequences of an instruction.
C. It misunderstood why a new approach was required.
D. It refused to take into account the views of employees.
7. What does the writer say about the programme introduced by the third company he mentions?
A. Employees did not believe that it had been introduced for their benefit.
B. Employees felt that it was in fact a way of making their jobs even harder.
C. The reason given for introducing it was not the real reason why it was introduced.
D. It was an inappropriate kind of programme for this particular organization.
8. The writer says that the programme in his final example.
A. was too demanding for managers to maintain long-term
B. was treated as a self-contained exercise by managers
C. involved some strange ideas on how managers could improve
D. caused managers to believe that their previous methods had been better
9. What does the writer mean by using the phrase 'they again had targets to hit'?

8
A. Setting targets and achieving them are a common thing in the mentioned company.
B. Despite the programme, the company did not deviate from conventional business practices.
C. The company were made up of goal-oriented managers and staff members.
D. Managers in the company were quite oblivious to change and innovation.-
10. What can be inferred from the passage?
A. Box ticking requires a coordination of efforts.
B. Box ticking is being misinterpreted by businesses.
C. Box ticking will need further revision along the way.
D. Box ticking has become obsolete for businesses.
Your answers:
1. 2. 3. 4. 5.
6. 7. 8. 9. 10.

Part 3: For questions 1-13, read the following passage and do the tasks that follow. (13 points)

Assessing the risk


A. As a title for a supposedly unprejudiced debate on scientific progress, “Panic attack: interrogating our
obsession with risk” did not bode well. Held last week at the Royal Institution in London, the event
brought together scientists from across the world to ask why society is so obsessed with risk and to call
for a “more rational” approach. “We seem to be organising society around the grandmotherly maxim of
‘better safe than sorry’,” exclaimed Spiked, the online publication that organised the event. “What are
the consequences of this overbearing concern with risks?”

B. The debate was preceded by a survey of 40 scientists who were invited to describe how awful our
lives would be if the “precautionary principle” had been allowed to prevail in the past. Their response
was: no heart surgery or antibiotics, and hardly any drugs at all; no aeroplanes, bicycles or high-voltage
power grids; no pasteurisation, pesticides or biotechnology; no quantum mechanics; no wheel; no
“discovery” of America. In short, their message was: no risk, no gain.

C. They have absolutely missed the point. The precautionary principle is a subtle idea. It has various
forms, but all of them generally include some notion of cost-effectiveness. Thus the point is not simply
to ban things that are not known to be absolutely safe. Rather, it says: “Of course you can make no
progress without risk. But if there is no obvious gain from taking the risk, then don’t take it.”

D. Clearly, all the technologies listed by the 40 well-chosen savants were innately risky at their
inception, as all technologies are. But all of them would have received the green light under the
9
precautionary principle because they all had the potential to offer tremendous benefits – the solutions to
very big problems – if only the snags could be overcome.

E. If the precautionary principle had been in place, the scientists tell us, we would not have antibiotics.
But of course, we would – if the version of the principle that sensible people now understand had been
applied. When penicillin was discovered in the 1920s, infective bacteria were laying waste to the world.
Children died from diphtheria and whooping cough, every open drain brought the threat of typhoid, and
any wound could lead to septicaemia and even gangrene.

F. Penicillin was turned into a practical drug during the Second World War when the many pestilences
that result from were threatened to kill more people than the bombs. Of course antibiotics were a
priority. Of course, the risks, such as they could be perceived, were worth taking.

G. And so with the other items on the scientists’ list: electric light bulbs, blood transfusions. CAT scans,
knives, the measles vaccine – the precautionary principle would have prevented all of them, they tell us.
But this is just plain wrong. If the precautionary principle had been applied properly, all these creations
would have passed muster, because all offered incomparable advantages compared to the risks perceived
at the time.

H. Another issue is at stake here. Statistics are not the only concept people use when weighing up risk.
Human beings, subtle and evolved creatures that we are, do not survive to three-score years and ten
simply by thinking like pocket calculators. A crucial issue is the consumer’s choice. In deciding whether
to pursue the development of new technology, the consumer’s right to choose should be considered
alongside considerations of risk and benefit. Clearly, skiing is more dangerous than genetically modified
tomatoes. But people who ski choose to do so; they do not have skiing thrust upon them by portentous
experts of the kind who now feel they have the right to reconstruct our crops. Even with skiing, there is
the matter of cost-effectiveness to consider: skiing, I am told, is exhilarating. Where is the exhilaration in
GM soya?

I. Indeed, in contrast to all the other items on Spiked’s list, GM crops stand out as an example of a
technology whose benefits are far from clear. Some of the risks can at least be defined. But in the present
economic climate, the benefits that might accrue from them seem dubious. Promoters of GM crops
believe that the future population of the world cannot be fed without them. That is untrue. The crops that
really matter are wheat and rice, and there is no GM research in the pipeline that will seriously affect the
yield of either. GM is used to make production cheaper and hence more profitable, which is an extremely
questionable ambition.

10
J. The precautionary principle provides the world with a very important safeguard. If it had been in
place in the past it might, for example, have prevented insouciant miners from polluting major rivers
with mercury. We have come to a sorry pass when scientists, who should above all be dispassionate
scholars, feel they should misrepresent such a principle for the purposes of commercial and political
propaganda. People at large continue to mistrust science and the high technologies it produces partly
because they doubt the wisdom of scientists. On such evidence as this, these doubts are fully justified.

Questions 1-6
Do the following statements agree with the information given in Reading Passage ?
TRUE if the statement agrees with the information
FALSE if the statement contradicts the information
NOT GIVEN if there is no information on this

1 The title of the debate is not unbiased.


2 All the scientists invited to the debate were from the field of medicine.
3 The message those scientists who conducted the survey were sending was people shouldn’t take risks.
4 All the 40 listed technologies are riskier than other technologies.
5 It was worth taking the risks to invent antibiotics.
6 All the other inventions on the list were also judged by the precautionary principle.

Questions 7-13: Complete the following summary of the paragraphs of Reading Passage Using NO
MORE THAN THREE WORDS from the Reading Passage for each answer.

When applying the precautionary principle to decide whether to invent a new technology, people should
also the consideration of the 7_________________, along with the usual consideration of
8_________________For example, though risky and dangerous enough, people still enjoy
9_________________for the excitement it provides. On the other hand, experts believe that future
population desperately needs 10_________________in spite of their undefined risks. However, the
researchers conducted so far have not been directed towards increasing the yield of
11_________________ , but to reduce the cost of 12_________________and to bring more profit out of
it. In the end, such selfish use of the precautionary principle for business and political gain has often led
people to 13_________________science for they believe scientists are not to be trusted.

11
Your answers:
1. 2. 3. 4. 5.
6. 7. 8. 9. 10.
11. 12. 13.

Part 4. You are going to read an article about graphology. Seven paragraphs have been removed
.from the extract. Choose from the paragraphs A-H the one which fits each gap (1-7). There is one
extra paragraph which you do not need to use (7 points)

Graphology is the study and analysis of handwriting in order to assess the personality and other traits of
the writer. As a means of providing in-depth details of personality, graphology is based on the principle
that most people write differently to the way they were taught at school. A unique pattern is formed on
the page in the way a writer combines characters from left to right and from top to bottom.
1______

Physical, mental and emotional characteristics are revealed. Handwriting is an exposure of a person's
current state of mind, feelings and body. An experienced graphologist can also detect past influences and
perhaps, future potential.
2______
When Michon published the results of his many years of research into handwriting movements, the
subject began to arouse public interest. His method was popular, owing to its simplicity. He studied
certain elements in handwriting, namely the 'stroke', the letters, words, paragraphs and free movements
(dots on i's and bars on t's).
3______
Crepieux-Jamin spent his life collating and improving upon Michon's observations and is credited with
founding the School of Isolated Signs, which linked specific handwriting elements to specific traits. He
defined the various elements of handwriting and divided them into seven categories: dimension, form ,
pressure, speed , direction , layout and continuity.
4______
At around the same time, in Germany, William T. Preyer recognised the fact that handwriting is, in fact,
brainwriting. He theorised that should a writer lose his writing arm, as did Nelson, for example, and have
to use the opposite hand, or even the foot or mouth in the case of paralysis, the same basic tendencies
will appear in the script, although obviously executed with less fluency at first.
5______

12
Dr Max Pulver, a renowned psychologist with a deep interest in graphology, was to make a further
contribution. He divided handwriting into three 'zones' - vertical movement, horizontal movement and
depth. This clarified aspects of handwriting previously misunderstood.
6______
At that time, many graphologists came to Britain hoping to continue their profession, only to find the
subject virtually unknown. In order to bring it to public attention, several graphologists produced books
for the layman. This was a breakthrough for graphology, and public reaction indicated there was a
demand for more information on the subject.
7______
Since graphology can give an all-round accurate specific handwriting elements to specific traits. He
profile, there is a deeper insight into a person's whole defined the various elements of handwriting and
character. There is also an elimination of the divided them into seven categories: dimension, form,
problem of premeditated candidate responses and any attempts at deception are detectable to the
graphologists. Strengths and weaknesses are emphasized, as well as many other features that are relevant
to the appointment.

Your answers:
1. 2. 3. 4. 5.
6. 7.

Part 5: The passage below consists of five sections marked A-E. Read the passage and do the task
that follows. Write your answers (A-E) in the corresponding numbered boxes provided. (15 points)

A. There are now five times as many cases of food poisoning as there were 20 years ago. It's partly down
to the fact that we eat out more often. Consider that one in men and one in three five women admit that
they don't wash their hands after going to the toilet or before preparing food; it's enough to put you off
popping out for a quick bite on a Friday night. And before you reach for one of those mints sitting on the
counter by the till, think how many non-hand-washers have dipped their fingers into that bowl. But the
problem isn't restricted to grubby eateries. More than half of food poisoning infections are acted in the
home. This may be because people are eating more pre-prepared food and shopping less often, so is
stored for longer. But most food poisoning is preventable if you know what you're doing.

B. Next time you have a headache, don't automatically reach for painkillers. Using them too often could
be more dangerous than you think. For a vulnerable minority, the route to addiction can be alarmingly
fast. It's thought that overusing painkillers leads to changes in the way the handles pain signals, so it
becomes oversensitive to stimuli that wouldn't normally cause pain. As the pain threshold lowers, people

13
seek out stronger medication and increase the dosage and soon they are hooked. Somewhere along the
line we have become blasé about the dangers of these medicines which are available in petrol stations,
news agents and supermarkets. People must not assume that over the counter drugs are safe because
clearly this is not the case.

C. The basic premise of detox is that we need to clear the toxic waste from our bodies every so often in
order to stay healthy. It sounds a reasonable idea, but nutrition experts are quick to point out that there's
no evidence to support it. Detoxing is a concept that underestimates the abilities of our liver and kidneys.
Any toxins that do get absorbed are very efficiently dealt with and secreted by our bodies. The concept
of detox diets is irrational and unscientific. Those promoting detox diets often claim that in order to
detoxify, we should avoid foods such as wheat and dairy products. The reality is that these foods provide
us with important nutrients, and it is unnecessary potentially harmful to exclude them from the diet.

D. There is a lot of publicity about the dangers of having 'high cholesterol' at the moment - mainly
coming from companies that make special foods that claim to lower it for you. It is a widespread
problem though: according to the British Heart Foundation around two thirds of British people have a
blood cholesterol level above the suggested healthy target figure. And as there are no symptoms until it
causes disease, many people with high cholesterol may not be aware that they have it. However we need
some cholesterol because the body wouldn't function without it! It is a key part of cell membranes and it
is also found in bile, which is important for digestion and absorption of fat.

E. Just like a real-time personal trainer (PT), an online coach discusses personal goals, tailors exercise
schedules and offers advice to clients of all abilities. Instead of face-to face guidance, however, the
virtual trainer gives feedback via emails and texts - perfect for the self-conscious exerciser! While critics
say that it is impossible to effectively rain clients without meeting them, online coaches offer a valuable
compromise when it comes to training. First they are a cost-effective alternative. Second, they represent
a more flexible option, especially if your work takes up much of your time or makes it difficult for you
to commit to regular sessions. And just because your coach is in cyberspace, it doesn't mean he won't be
keeping tabs on you. The mere thought of the next email should have you racing to put on your sports
kit.

F. Most of us are unclear as to where social drinking stops and alcohol dependency starts. A heavy-
drinking student is fairly normal, but a 40-year-old party animal? A sozzled 60- year-old? At what point
do the questions demand a serious answer? Recommendations from the government are that men should
consume no more than three to four units of alcohol a day and women only two to three units. Of course,
committed social drinkers of all ages often ignore units, taking comfort from the fact that they are not

14
reaching for the bottle before lunchtime, and their friends are drinking as much as they are so it must be
alright. But you can even test yourself online by looking at Alcohol Concern's website to determine if
alarm bells should be ringing. Most people can reduce their alcohol consumption without professional
help but it's not just about cutting down, it's about editing your life to remove the reasons you are
drinking too much.

In which extract is the following mentioned?


1. People may be unaware of something that is causing disease in their body.
2. Cutting back too much can be harmful.
3. If you feel self-conscious, this could be for you.
4. A shocking hygiene confession is made.
5. This could be a flexible way to monitor your activity and performance levels.
6. A change in body reaction could be dangerous.
7. Drastic measures may not achieve the expected results.
8. You should reassess your lifestyle to see if it is causing a bad habit.
9. People need to be aware of the dangers of something that seems safe.
10.A high-profile health problem boosts manufacturers' turnover.
Your answers:
1. 2. 3. 4. 5.
6. 7. 8. 9. 10.

IV. WRITING (60 points)


Part 1: Read the following extract and use your own words to summarise it. Your summary
should be 120 words long. (15 points)
The extraordinary system, language, allows us to communicate about anything whatsoever, whether it is
present, absent, or even non-existent. This is quite strange, compared with the communication systems of
other animals, which are mostly confined to messages about everyday events, such as food, danger,
mating and territorial rights. Humans are the odd ones out. We humans are strange, because language
with its fast and precise sounds has more in common with birdsong than with the vocal signals of our ape
relatives.

So how did it all begin, this powerful, weird communication system of ours? Frustratingly, we do not
know. Our earliest written records are around 5,000 years old, though most are more recent. Yet
language must have evolved at least 50,000 years ago, and most researchers propose a date around
100,000 years ago. Until recently, how it all began was an unfashionable question, a playground for
cranks.

15
Curious theories abounded. Take the views of Lord Monboddo, a Scottish aristocrat, who in 1773
published a book claiming that humans learned how to spin and weave from spiders, how to construct
dams from beavers, and how to sing and speak from birds. The cuckoo, the raven and the parrot, he
noted, produced almost alphabetical sounds. Therefore in his view human articulation was the result of
imitating such birds.

As absurd claims mushroomed, the question of language origin was shunned by serious scholars. In
1866, a ban on the topic was even imposed. Yet scholarly disapproval did not stop speculation. One
academic, in fact, has counted twenty-three 'principal theories' of language origin. Another acidly
commented: The very fact ... that human animals are ready to engage in a great "garrulity" over the
merits and demerits of essentially unprovable hypotheses, is an exciting testimony to the gap between
humans and other animals.'

The origin of language is like a juicy fruit dangling just out of reach. Human beings have a natural
curiosity about it seemingly built into their minds: 'Few questions in the study of human language have
attracted so much attention, provoked so much controversy, or resisted so resolutely their answers as that
of the origin of language,' noted a recent writer.

Source: Aitchison (1996) - Seeds of Speech (language origin and evolution)

_________________________________________________________________________________
_________________________________________________________________________________
_________________________________________________________________________________
_________________________________________________________________________________
_________________________________________________________________________________
_________________________________________________________________________________
_________________________________________________________________________________
_________________________________________________________________________________
_________________________________________________________________________________
_________________________________________________________________________________
_________________________________________________________________________________
_________________________________________________________________________________
_________________________________________________________________________________
_________________________________________________________________________________

Part 2: The chart below shows import and export values of crude oil in Vietnam and the table
shows fluctuations in world oil prices and average prices of products and services in Vietnam

16
between 2015 and 2020. Summarize the information by selecting and reporting the main features
and make comparisons where relevant. (15 points)

Increase/decrease in world oil prices and average prices of products and services over the years
(%)

2015 2016 2017 2018 2019 2020

Average world oil prices


 45.6  17.4  20.74  31.3  10.28  33.8%

Average price of products


 0.63  2.66  3.53  3.54  2.79  3.23
and services

_________________________________________________________________________________
_________________________________________________________________________________
_________________________________________________________________________________
_________________________________________________________________________________
_________________________________________________________________________________
_________________________________________________________________________________
_________________________________________________________________________________
_________________________________________________________________________________
_________________________________________________________________________________
_________________________________________________________________________________
_________________________________________________________________________________
_________________________________________________________________________________
_________________________________________________________________________________

17
_________________________________________________________________________________
_________________________________________________________________________________
_________________________________________________________________________________
_________________________________________________________________________________
_________________________________________________________________________________
_________________________________________________________________________________
_________________________________________________________________________________
_________________________________________________________________________________
_________________________________________________________________________________
_________________________________________________________________________________
Part 3: Write an essay of about 300 words to express your opinion on the following issue.
It is time to recognize the growing importance of teenagers’ mental well-being .Some people say
that so as to lead a healthy lifestyle, students should maintain a balanced diet and regular
workouts. Others, however, believe that courses to promote their mental condition are more
advisable.
Discuss both views and give your own opinion. (30 points)
_________________________________________________________________________________
_________________________________________________________________________________
_________________________________________________________________________________
_________________________________________________________________________________
_________________________________________________________________________________
_________________________________________________________________________________
_________________________________________________________________________________
_________________________________________________________________________________
_________________________________________________________________________________
_________________________________________________________________________________
_________________________________________________________________________________
_________________________________________________________________________________
_________________________________________________________________________________
_________________________________________________________________________________
_________________________________________________________________________________
_________________________________________________________________________________
_________________________________________________________________________________
_________________________________________________________________________________
_________________________________________________________________________________
_________________________________________________________________________________

18
_________________________________________________________________________________
_________________________________________________________________________________
_________________________________________________________________________________
_________________________________________________________________________________
_________________________________________________________________________________
_________________________________________________________________________________
_________________________________________________________________________________
_________________________________________________________________________________
_________________________________________________________________________________
_________________________________________________________________________________
_________________________________________________________________________________
_________________________________________________________________________________
_________________________________________________________________________________
_________________________________________________________________________________
_________________________________________________________________________________

- THE END -

Người ra đề:
1. Lê Ngọc Hân: 0987463619
2. Nguyễn Phương Ly Ly: 0985397666

19

You might also like